Vous êtes sur la page 1sur 405

Pathoma Lecture Notes

Chapter 1: Growth Adaptations,


Cellular Injury, and Cell Death
Growth adaptations

I. BASIC PRINCIPLES
A. An organ is in homeostasis with the physiologic stress placed on it.
B. An increase, decrease, or change in stress on an organ can result in growth adaptations.

Hyperplasia Metaplasia Dysplasia


- Increase in cell numbers - Change in cell type due to - Proliferation of pre cancerous
reprogramming of stem cell cells
- Reversible - Reversible - Reversible
- Pathologic hyperplasia - Can progress to dysplasia - Can progress to cancer
can progress to cancer and cancer
- BPH don't increase risk - Apocrine metaplasia don't
of cancer increase risk of breast cancer
- Etiology: - Etiology: - Etiology:
• Physiologic (ex- • Vit A deficiency • Long standing
pregnancy) (keratomalacia, pathologic hyperplasia
• Pathologic (ex - …) myositis ossificans) or metaplaisa
• Cellular stress (barret's
esophagus)

Extras – Vitamin A is necessary for maturation of immune system. 15-17 translocation in


pt genome causes acute promyelocytic leukemia. The translocation causes mutation in vit-
A receptor – retinoic acid receptor. Mutation of receptors cause immune cells to be
trapped in blast stage. Treatment for Promyelocytic leukemia is all trans retinoic acid
(drug) which can bind to mutated receptor. Cells can mature and be neutrophil.

• Fig – keratomalacia.
Processes Properties
Hyperplasia • Increase in stress leads to an increase in organ size.
and • Occurs via:
Hypertrophy - increase in the size (hypertrophy) of cell
- increase in the number (hyperplasia) of cells
• MOA:
- Hypertrophy involves - gene activation, protein synthesis, and
production of organelles
- Hyperplasia involves - production of new cells from stem cells.
• They Generally occur together (e.g., uterus during pregnancy).
• Permanent cell undergro hypertrophy only – cardiac myocytes (in
response to sys. HTN), skeletal muscle, and neurons.
• Pathologic hyperplasia can progress to dysplasia and eventually cancer:
- Ex- endometrial hyperplasia.
- Exception - benign prostatic hyperplasia (BPH is pathologic but does
not increase the risk for prostate cancer).

Atrophy • A decrease in stress leads to a decrease in organ size (atrophy)


[ex- decreased hormonal stimulation, disuse, or decreased
nutrients/blood supply]
• Occurs via a decrease in the size and number of cells.
• MOA:
- Decrease in cell number occurs - via apoptosis.
- Decrease in cell size occurs - ubiquitin-proteosome degradation of
cytoskeleton and autophagy of cellular components
• Ubiquitin-proteosome degradation pathway:
- intermediate filaments of the cytoskeleton are "tagged" with ubiquitin
and destroyed by proteosomes.
• Autophagy:
- Autophagy of cellular components involves generation of autophagic
vacuoles. These vacuoles fuse with lysosomes whose hydrolytic
enzymes breakdown cellular components.
Processes Properties
Metaplasia • A change in stress on an organ leads to a change in cell type.
• Mostly affects surface epithelium (squamous, columnar, or urothelial).
• Metaplastic cells are better able to handle the new stress.
• Barrett esophagus is a classic example.
- Esophagus is normally lined by nonkeratinizing squamous
epithelium (suited to handle friction of a food bolus). Acid reflux
from the stomach causes metaplasia to nonciliated, mucin-
producing columnar cells (better able to handle the stress of acid).
• MOA: occurs via reprogramming of stem cells to produce new cell type.
• Reversible [in theory, with removal of the driving stressor]. Ex-
treatment of gastroesophageal reflux may reverse Barrett esophagus.
• Under persistent stress, metaplasia can progress to dysplasia and
eventually result in cancer:
- Ex – Barrett esophagus may progress to adenocarcinoma of the
esophagus.
- Exception – Apocrine metaplasia of breast, which carries no
increased risk for cancer.
• Vitamin A deficiency can also result in metaplasia:
- Vitamin A is necessary for differentiation of specialized epithelial
surfaces such as the conjunctiva covering the eye. In vitamin A
deficiency, the thin squamous lining of the conjunctiva undergoes
metaplasia into stratified keratinizing squamous epithelium.
This change is called keratomalacia.
• Mesenchymal (connective) tissues can also undergo metaplasia:
- myositis ossificans - in which connective tissue within muscle
changes to bone during healing after trauma.

Fig – myositis ossificans


Processes Properties
Dysplasia • Disordered cellular growth
• Most often refers to proliferation of precancerous cells.
Ex- cervical intraepithelial neoplasia (CIN) represents dysplasia and is a
precursor to cervical cancer.
• Often arises from:
- longstanding pathologic hyperplasia (e.g., endometrial hyperplasia)
- metaplasia (e.g., Barrett esophagus)
• Reversible [in theory, with alleviation of inciting stress].
• Irreversible [If stress persists, dysplasia progresses to carcinoma].

Aplasia • Failure of cell production during embryogenesis.


• Ex – unilateral renal agenesis.

Hypoplasia • Decrease in cell production in embryogenesis


• Results in relatively small organ.
• Ex – streak ovary in Turner syndrome.
Cellular injury

1. What is cellular injury ?


- Cellular injury occurs when a stress exceeds the cell's ability to adapt.

2. What does cellular injury depend upon ?


- Type of stress
- Type of cell affected
=> Neurons are highly susceptible to ischemic injury whereas skeletal muscle is
relatively more resistant.
- Severity
=> Slowly developing ischemia (ex- renal artery atherosclerosis) results in
atrophy whereas acute ischemia (ex- renal artery embolus) results in injury.

1. What causes cellular injury ?


- Inflammation
- Nutritional deficiency or excess
- Hypoxia
- Trauma
- Genetic mutations.
HYPOXIA

3. What is hypoxia ?
- Low oxygen delivery to tissue
- Important cause of cellular injury
=> Oxygen is the final electron acceptor in the electron transport chain of oxidative
phosphorylation Decreased oxygen impairs oxidative phosphorylation, resulting in
decreased ATP production. Lack of ATP (essential energy source) leads to cellular injury.

14. What does low ATP disrupts key cellular functions ?


- Disrupted Na/K pump result in Na accumulation and water retention in cell.
- Disrupted Ca pump result in Ca2+ accumulation in cytosol and cellular enzyme activation.
- Anaerobic glycolysis leads to Lactic acidosis which denatures proteins and precipitates DNA.

3. What are causes of hypoxia ?


- ischemia - decreased blood flow through an organ.
- hypoxemia - low partial pressure of oxygen in the blood (Pao2 < 60 mm Hg, SaO2 < 90%).
- low O2 carrying capacity of blood - arises with hemoglobin (Hb) loss or dysfunction.

4. What causes Ischemia ?


- Decreased arterial perfusion (e.g., atherosclerosis)
- Decreased venous drainage (e.g., Budd-Chiari syndrome)
- Blockade of hepatic vein can result in hepatic infarction
- Cause of Blockade:
- Polycythemia vera
- Lupus
- Shock- generalized hypotension resulting in poor tissue perfusion.
- It occurs due to:
- cardiogenic causes
- hypovolemic causes
- neurogenic causes
- septic causes

3. What causes Hypoxemia ?


- High altitude: Decreased barometric pressure results in decreased PAO2
- Hypoventilation: Increased PACO2 results in decreased PAO2
- Diffusion defect: PAO2 not able to push as much O2 into the blood due to a thicker diffusion
barrier (ex- interstitial pulmonary fibrosis)
- V/Q mismatch:
- Blood bypasses oxygenated lung (circulation problem, ex- right-to-left shunt)
- Oxygenated air cannot reach blood (ventilation problem, ex- atelectasis).

3. What are some examples of low O2 carrying capacity of blood ?


- Anemia (decrease in RBC mass): PaO2 normal and SaO2 normal
- Carbon monoxide poisoning:
- CO binds hemoglobin more avidly than oxygen: PaO2 normal while SaO2 decreased
- Exposures include smoke from fires and exhaust from cars or gas heaters.
- Classic finding is cherry-red appearance of skin.
- Early sign of exposure is headache; significant exposure leads to coma and death.
- Methemoglobinemia
- Iron in heme is oxidized to Fe3+ which cannot bind oxygen: PaO2 normal while SaO2 decreased
- Seen with oxidant stress (ex- sulfa and nitrate drugs) or in newborns
- Classic finding is cyanosis with chocolate-colored blood.
- Treatment is intravenous methylene blue, which helps reduce Fe3+ back to Fe2+ state.
Reversible CELLULAR INJURY (initial phase)

15. What are the hallmarks of reversible injury ?


- Hallmark is cellular swelling.
- Cytosol swelling results in loss of microvilli and membrane blebbing.
- Swelling of the rough endoplasmic reticulum (RER) results in dissociation of ribosomes
and decreased protein synthesis.

Irreversible CELLULAR INJURY (eventual phase)

15. What are the hallmarks of irreversible injury ?


- The end result of irreversible injury is cell death.
- Hallmark is membrane damage.
=> Plasma membrane damage results in
- Cytosolic enzymes leaking into the serum (ex- cardiac troponin in MI & ALA in hepatitis)
- Additional calcium entering into the cell
=> Mitochondrial membrane damage results in
- Loss of the electron transport chain (inner mitochondrial membrane)
- Cytochrome c leaking into cytosol (activates apoptosis)
=> Lysosome membrane damage results in
- hydrolytic enzymes (activated by the high intracellular calcium) leaking into the cytosol
and digests cellular components
Cell death

1. What is morphologic hallmark of cell death?


- Loss of nucleus

2. What are the steps by which nucleus is lost?


• Pyknosis (nuclear condensation)
• Karyorrhexis (fragmentation)
• Karyolysis (dissolution)

2. What are the 2 mechanisms of cell death ?


• Necrosis
• Apoptosis
Necrosis

3. Explain necrosis
• Death of large groups of cells followed by acute inflammation.
• Due to some underlying pathologic process; never physiologic.
• Types [based on gross features]:
1. Coagulative necrosis
2. Liquefactive necrosis
3. Gangrenous necrosis - Wet and Dry gangrene.
4. Caseous necrosis
5. Fat necrosis
6. Fibrinoid necrosis

Coagulative necrosis

4. Describe coagulative necrosis.


- Features:
- Necrotic tissue that remains firm
- Cell shape and organ structure are preserved by coagulation of cellular proteins.
- Nucleus disappears
- Characteristic:
- Ischemic infarction of any organ (except brain)
- Area of infarcted tissue is often wedge-shaped (pointing to focus of vascular
occlusion) and pale.
- Red infarction arises if blood re-enters a loosely organized tissue (ex- pulmonary
or testicular infarction)

Liquefactive necrosis

6. Describe liquefactive necrosis.


- Featues:
- Necrotic tissue that becomes liquefied
- Enzymatic lysis of cells and protein results in liquefaction
- Characteristic:
- Brain infarction - Proteolytic enzymes from microglial cells liquefy the brain.
- Abscess - Proteolytic enzymes from neutrophils liquefy tissue.
- Pancreatitis - Proteolytic enzymes from pancreas liquefy parenchyma.

F. ig. Right is normal glomerulus and left is coaȁĀ


gulatiĀ
ve
necrosis of glomerulus ..
Gangrenous necrosis

7. Describe gangrenous necrosis.


- Feature:
- Dry gangrene: Coagulative necrosis that resembles mummified tissue.
- Wet gangrene: Liquefactive necrosis occurs in case of superimposed infection
of dead tissues.
- Characteristic:
- Ischemia of lower limb and GI tract

Caseous necrosis

8. Describe caseous necrosis


- Features:
- Soft and friable necrotic tissue ("cottage cheese-like" appearance)
- Combination of coagulative and liquefactive necrosis
- Characteristic:
- Granulomatous inflammation due to tuberculous or fungal infection

Fig - caseous necrosis Fig - fat necrosis around pancreas


Fat necrosis

9. Describe fat necrosis


- Features:
- Necrotic adipose tissue with chalky-white appearance due to deposition of calcium.
- MAO - dystrophic calcification.
- Characteristic:
- trauma to fat (e.g., breast)
- pancreatitis mediated damage of peripancreatic fat

Fibrinoid necrosis

10. What is fibrinoid necrosis


- Features:
- Necrotic damage to blood vessel wall
- Leaking of proteins (including fibrin) into vessel wall results in bright pink staining of
the wall microscopically in H&E stain.
- Characteristic:
- malignant hypertension [sudden super elevated BP] - medical emergency
- vasculitis
- Risk factor:
- Pre-eclampsia can cause fibrinoid necrosis of placenta
- Complictions include headache, renal failure, papilledema
Calcification

11. What is Pathological calcification.


- Abnormal tissue diposition of calcium salts together with smaller
amount of other (Fe,Mg) mineral salts.

11. What are the types of Pathological calcification ?


- Dystropic calcification
- Metastatic calcification

Dystropic calcification

12. What is dystropic calcification?


- Serum Ca and PO4 is normal [Normal Ca metabolism].
- Deposition of Ca on necrosed (dead) tissues
- Ex: In case of Fat necrosis - Fatty acids are released from the Adipocytes by either
trauma (e.g., to breast) or lipase (e.g., pancreatitis) binds with calcium via a process
called saponification.

Metastatic calcification

13. What is metastatic calcification?


- Serum Ca or PO4 is elevated [Abnormal Ca metabolism].
- Deposition of Ca on normal tissues
- Ex: hyperparathyroidism leading to nephrocalcinosis.
Apoptosis

14. Explain apoptosis.


- Energy (ATP)-dependent genetically programmed cell death.
- Involves single cells or small groups of cells.
- Apoptosis is not followed by inflammation.

15. Give few Examples of apoptosis ?


- Endometrial shedding during menstrual cycle
- Removal of cells during embryogenesis
- CD8+ T cell mediated killing of virally infected cells
- Negative selection of thymocytes in thymus.

15. What is the Morphology of apoptosis ?


- Dying cell shrinks, leading cytoplasm to become more eosinophilic (pink).
- Nucleus condenses and fragments in an organized manner.
- Apoptotic bodies fall from the cell and are removed by macrophages.

Fig - compare the pink apoptotic cell to it's surrounding neighbors

16. What is the Mechanism of apoptosis ?


- Caspase is a key enzyme which activates:
○ Proteases - breaks down cytoskeleton.
○ Endonucleases - breaks down DNA.

17. Mention some pathways that lead to caspase activation ?


- Intrinsic mitochondrial pathway:
○ Inactivation of Bcl2 is due to
=> Cellular injury
=> DNA damage
=> decreased hormonal stimulation
○ Lack of Bcl2 allows cytochrome c to leak from the inner mitochondrial matrix into the
cytoplasm and activate caspases [Bcl2 is a protein which stabilizes mito. membrane].
- Extrinsic receptor-ligand pathway:
○ FAS ligand binds FAS death receptor (CD95) on the target cell activating caspase.
○ Tumor necrosis factor (TNF) binds TNF receptor on the target cell activating caspase.
- Cytotoxic CD8+ T cell-mediated pathway:
○ Perforins secreted by CD8+ T cell create pores in membrane of target cell.
○ Granzyme from CD8+ T cell enters pores and activates caspases.
Free radical injury

1. What is free radical?


• Chemical species with an unpaired electron in their outer orbit.

2. Explain Physiologic generation of free radicals ?


• Occurs during oxidative phosphorylation.
- Cytochrome c oxidase (complex IV) transfers electrons to O2.
- Partial reduction of O2 yields:
=> superoxide radical (O2-) - receives 1 e-
=> hydrogen peroxide (H2O2) - receives 2 e-
=> hydroxyl radical (.OH) - receives 3e- [most damaging free radical]
- [Oxygen ----(accept 1 electron)-----> Superoxide (.O2) ----(accept 1
electron)-----> Hydrogen peroxide (H2O2) ----(accept 1 electron)----->
Hydroxide (.OH) ----(accept 1 electron)-----> Water (H2O)].

1. Explain Pathologic generation of free radicals ?


• It arises with:
- Ionizing radiation: water hydrolyzed to produce hydroxyl radical (.OH)
[Water (H2O)----(hv)-----> hydroxyl radical (.OH)].
- lnflammation: NADPH oxidase generates superoxide ions (.O2) during
oxygen dependent killing by neutrophils.
Fig - T represents fat in hepatocyte;
- Metals [copper and iron]:
a hepatocyte is shown in circle
Fe2+/Cu1+ generates hydroxyl free radicals - Fenton reaction.
[H202 + (Fe2+/Cu1+) -----> (Fe3+/Cu2+) + (.OH) + (-OH)].
- Drugs and chemicals: P450 system of liver metabolizes drugs (ex-
acetaminophen, CCL4), generating free radicals causing liver necrosis
- Hormones: Thyroid hormones increase free radical formation.

3. How do free radicals Free radicals cause cellular injury ?


• Peroxidation of lipids.
• Oxidation of DNA (implicated in aging and oncogenesis) and proteins.

4. Mechanisms of Elimination of free radicals.


• Antioxidants (ex- glutathione and vitamins A, C, and E)
• Metal carrier proteins (ex- transferrin, ferritin and ceruloplasmin)
• Enzymes:
○ Superoxide dismutase (in mitochondria) - Superoxide (O2-) -----> H2O2
○ Glutathione peroxidase (in mitochondria) - GSH + (.OH) -----> GSSH + H2O
○ Catalase (in peroxisome) - H2O2 -----> O2 + H20

5. Give some Examples of free radical injury ?


• Carbon tetrachloride (CCl4 - dry cleaning chemical):
- CCl4 is converted to CCl3 (free radical) by Cyt. P450 system of hepatocytes causing reversible damage.
- Results in cell injury with swelling of RER hence ribosomes detach impairing protein synthesis.
- Apolipoprotein synthesis decreases leading to fatty liver disease (liver can't repackage and send fat away)
• Reperfusion injury
- Return of blood to ischemic tissue results in production of O2 derived free radicals which further damage tissue
[immune cells will attack the dead tissue in presence of oxygen which will generate free radicals].
- Leads to a continued rise in cardiac enzymes (ex-troponin) after reperfusion of infarcted myocardial tissue.
Amyloidosis

1. What is amyloid ?
• Amyloid is a misfolded protein that deposits in the extracellular space, thereby damaging tissues.
• Deposition can be systemic or localized.
• Multiple proteins can deposit as amyloid

2. What are some features of amyloid ?


• Beta-pleated sheet configuration
• Congo red staining and apple-green birefringence when viewed microscopically under polarized light

Fig - apple-green appearing amyloid under polarized light


Systemic AMYLOIDOSIS

3. What is systemic amyloidosis ?


• Amyloid deposition in multiple organs.
• Amyloid cannot be removed hence damaged organs must be transplanted.
• It is classified into:
- Primary amyloidosis
- Secondary amyloidosis

3. Explain Primary amyloidosis.


• Primary amyloidosis is systemic deposition of AL (amyloid light chain protein) .
• It is derived from Ig light chain.
• Associated with plasma cell dyscrasias (ex- multiple myeloma).

3. Explain Secondary amyloidosis.


• SAA (Serum amyloid A protein) is an acute phase reactant which deposits as AA (amyloid A protein).
• It is increased in case of chronic inflammatory states.

5. What are classic findings of systemic amyloidosis?


• Kidney is most commonly involved - nephrotic syndrome
• Restrictive cardiomyopathy or arrhythmia
• Hepatosplenomegaly, Malabsorptionand Tongue enlargement

6. How do you diagnose Systemic amyloidosis ?


• Requires Tissue biopsy (needs to be congo-red positive).
• Abdominal fat pad and rectum are easily accessible biopsy targets.

11. Describe Hemodialysis associated amyloidosis.


• β2 microglobulin is a component of MHC class I molecules present on all nucleated cells.
• β2 microglobulin deposits as Aβ2m (amyloid β2 microglobulin protein) in the carpal
ligament resulting in carpal tunnel syndrome
• Patients on long term hemodialysis for renal failure can develop amyloidosis. This protein
is present in high conc. in patients with renal failure. It was retained in the circulation coz
it cannot be filtered through dialysis membrane.
Hereditary AMYLOIDOSIS

4. Explain the Pathophysiology of familial mediterranean fever ?


• Autosomal recessive mutation (gain of function) in the Mediterranean Fever (MEFV) gene on the
16p13 of leukocytes (ex- neutrophil) which codes for a protein called pyrin or marenostrin.
• Pyrin binds with the adaptor ASC and the pro form of the enzyme caspase-1 to generate a multiprotein
complex called inflammasome which converts Pro IL-18 and Pro IL-1β to IL-18 and IL-1β respectively.
• All the above steps take place in the Cytosol of Neutrophil.

6. What is the epidemology of familial mediterranean fever ?


• Occurs in persons of Mediterranean origin.

6. What is seen FMF ?


• High SAA during attacks deposits as AA amyloid in tissues.

6. What is the presentation of familial mediterranean fever ?


• Presents with episodes of fever and acute serosal inflammation.
• Acute serosal inflammation can mimic appendicitis, arthritis, or myocardial infarction.

8. Describe familial amyloid polyneuropathies.


• Mutated serum transthyretin (TTR) deposits as ATTR (amyloid TTR ptotein).
• Autosomal dominant neurodegenerative disease
• Deposition of amyloid predominantly in peripheral and autonomic nerves.

7. Describe Systemic senile amyloidosis.


• Accumulation of a wild-type (non-mutated) protein called transthyretin (TTR).
• Wild type TTR Deposits as ATTR predominantly in the heart
Local AMYLOIDOSIS

3. What is local amyloidosis ?


• Amyloid deposition usually localized to a single organ.

10. Explain Senile cerebral amyloidosis.


• Aβ (β amyloid protein) is derived from Amyloid precursor protein (APP).
• APP is an integral membrane proteins of neurons.
• Gene for APP is present on chromosome 21 of neurons.
• Increased deposition of Aβ (etiology is unclear) in the cerebral blood
vessels as amyloid plaques.
• Most individuals with Down syndrome (trisomy 21) develop Alzheimer
disease by the age of 40 (early-onset).

9. Explain Islets of Langerhans amyloisosis.


• Amylin or islet amyloid polypeptide (IAPP) deposits as AIAPP.
• AIAPP (amyloid islet amyloid polypeptide) deposits in the islets of the pancreas.
• Amylin is cosecreted with insulin from the pancreatic β-cells.
• Seen in individuals with Type 2 diabetes mellitis [insulin resistance].

12. Explain Medullary carcinoma of thyroid amyloidosis.


• Calcitonin deposits as A Cal (amyloid calcitonin protein).
• Medullary thyroid cancer (MTC) is a form of thyroid carcinoma which originates from
the parafollicular cells (C cells) which produce the hormone calcitonin.
• Calcitonin overproduction by tumor cells deposits within the tumor
• Fine Needle Aspiration Biopsy shows - 'tumor cells in an amyloid background'.
Chapter 2: Inflammation,
Inflammatory Disorders, and Wound
Healing
Acute Inflammation (Part 1)
1. What is inflammation?
• Process by which inflammatory cells, plasma proteins (e.g., complement), and fluid to exit
blood vessels and enter the interstitial space.
• Divided into acute and chronic inflammation
2. Explain acute inflammation
- Characterized by the presence of edema and neutrophils in tissue
- Arises in response to infection (to eliminate pathogen) or tissue necrosis (to clear necrotic debris)
- Immediate response with limited specificity (innate immunity)

MEDIATORS OF ACUTE INFLAMMATION

Toll-like receptors (TLRs)

3. How does TLR work?


- TLR is Present on:
- cells of innate immune system (macrophage and dendritic cells).
- cells of adaptive immunity (lymphocytes) hence mediates chronic inflammation. .
- TLR is activated by:
- PAMP (pathogen associated molecular patterns) - microbe biomolecule [Ex: LPS]
- DAMP (damage associated molecular pattern) - host biomolecule
- CD14 (a co-receptor for TLR4) on macrophages recognizes lipopolysaccharide (LPS) on the outer
membrane of gram-negative bacteria.
- TLR activation results in upregulation of NF-KB, a nuclear transcription factor that activates
immune response genes leading to production of multiple immune mediators in case of both
innate and adaptive immunity.
- Type 1 interferon (alpha and beta) - inhibit viral replication in cells

Arachidonic acid (AA) metabolites

4. Explain production and metabolism of arachidonic acid (AA).


- AA is released from the phospholipid cell membrane by phospholipase A2
- AA is then acted upon by cyclooxygenase or 5-lipooxygenase.

5. What are the products of cyclooxygenase pathway.


- Cyclooxygenase produces prostaglandins (PG) → PGI2, PGD2 and PGE2
- PGI2, PGD2 and PGE2 mediate vasodilation and increased vascular permeability.
- PGE2 also mediates fever and pain.

6. What are the products of lipooxygenase pathway.


- 5-Lipooxygenase produces leukotrienes (LT) → LTB4, LTC4, LTD4 and LTE4
- LTB4 attracts and activates neutrophils.
- LTC4, LTD4 and LTE4 (slow reacting substances of anaphylaxis) mediate:
- smooth muscle contraction (leading to vasoconstriction, bronchospasm), and
- pericytes contraction (leading to increased vascular permeability- opens up space
between endothelial cells).
Mast cells

7. What are the 3 ways that activate mast cells?


- Mast cells are found throughout connective tissue of body and are activated by -
○ Tissue trauma
○ Complement proteins C3a and C5a
○ Cross linking of cell surface IgE by antigen
8. What is acute response of mast cells activation?
- Histamine is released from preformed granules which causes vasodilation of arteriole
and increased vascular permeability (at the post capillary venule).
- Release proteolytic enzymes that kill bacteria or inactivate toxins
9. What is delayed response (after 4-5 hrs) of mast cells activation?
- Mast cells produces arachidonic acid metabolites, particularly leukotrienes.

Complement

9. What is complement protein ?


- Proinflammatory serum proteins that "complement" inflammation which Circulate as inactive precursors.

10. What are three pathways of complement activation?


- Classical pathway - C1 binds IgG or lgM that is bound to antigen.
- Alternative pathway - Microbial products directly activate complement.
- Mannose-binding lectin (MBL) pathway - MBL binds to mannose on microbes and activates complement.

11. How is membrane attack complex (MAC) formed?


- All pathways result in production of C3 convertase [mediates C3 → C3a and C3b] which in turn produces
C5 convertase [mediates C5 → C5a and C5b]. C5b complexes with C6-C9 to form the membrane attack
complex (MAC).

12. What are some key functions of complement proteins?


- C3a and C5a (anaphylatoxins) - trigger mast cell degranulation
- C5a - chemotactic for neutrophil
- C3b - opsonin for phagocytosis
- MAC lyses microbes by creating a hole in the cell membrane

Hageman factor (Factor XII)

13. What is hageman factor ? What systems does it activate ?


- It is an Inactive proinflammatory protein produced in liver that is activated by exposure to
subendothelial or tissue collagen.
- Plays an imp role in DIC (disseminated intravascular coagulation - pathologic activation of
coagulation cascade) and severe gram negative sepsis (bacterial products activates hageman
factor which in turn results in DIC).
- It activates
=> complement system
=> coagulation and fibrinolytic system
=> kinin system
14. Explain kinin system.
- Kinin cleaves high-molecular-weight kininogen (HMWK) to bradykinin.
- Bradykinin mediates vasodilation and increased vascular permeability, as well as pain.
Acute Inflammation (Part 2)

CARDINAL SIGNS OF INFLAMMATION

Redness (rubor) and warmth (calor)

15. What causes rubor (redness) and calor (warmth)?


- Due to vasodilation, which results in increased blood flow
- Occurs via relaxation of arteriolar smooth muscle
- Key mediators are:
- histamine.
- prostaglandins.
- bradykinin.

Swelling (tumor)

16. What causes tumor (swelling)?


- Due to leakage of fluid from postcapillary venules into the interstitial space (exudate)
- Key mediators are
- histamine (endothelial cell contraction).
- tissue damage (endothelial cell disruption).

Pain (dolor)

17. What causes dolor (pain)?


- Key mediators (that sensitize sensory nerve endings) are:
- Bradykinin.
- PGE2.

Fever

18. What causes fever?


- Pyrogens (e.g., LPS from bacteria) cause macrophages to release IL-l and TNF,
which increase cyclooxygenase activity in perivascular cells of the hypothalamus.
- COX causes increase in PGE2 in hypothalamus which increases temp set point.
Acute Inflammation (Part 3)

1. What are 3 phases of acute inflammation?


○ Fluid phase - edema, complement activation etc
○ Neutrophil phase - peaks about 24 hrs
○ Macrophage phase - peaks about 2-3 days
(Note that acute inflammation is not defined by time. It can happen for days or weeks. It
is defined by whether neutorphils are primary player. If pus is present, it's still acute.)

NEUTROPHIL

Step 1 - Margination

1. Vasodilation slows blood flow in postcapillary venules.


2. Cells marginate from center of flow to the periphery - Usually heavy particles in blood are in center of vessel lumen.
When blood vessel dilates, heavy particles come to edge of vessel.

Step 2- Rolling

1. Selectin "speed bumps" are upregulated on endothelial cells.


i. P-selectin release from Weibel-Palade bodies is mediated by histamine.
ii. E-selectin is induced by TNF and IL-1.
2. Selectins bind sialyl Lewis X on leukocytes.
3. Interaction results in rolling of leukocytes along vessel wall.

Step 3-Adhesion

1. Cellular adhesion molecules (ICAM and VCAM) are upregulated on endothelium by TNF and IL-l.
2. Integrins are upregulated on leukocytes by C5a and LTB4.
3. Interaction between CAMs and integrins results in firm adhesion of leukocytes to the vessel wall.

7. What is leukocyte adhesion deficiency?


- Autosomal recessive defect of integrins (CD18 subunit).
8. What are sign/symptoms of leukocyte adhesion deficiency (LAD)?
- Delayed seperation of umbilical cord - After childbirth, the umbilical cord shuts. Due to no
blood through it, it undergoes necrosis. Active inflammation ensues. Neutrophils come and
eat up umbilical cord. However, in LAD, neutrophils won't come much because adhesion
process is bad.
- Increased circulating neutrophils (due to impaired adhesion of marginated pool of leukocytes) -
normally, half of neutrophils are in circulation and remaining half are stuck to endothelium of
lung. Due to loss of adhesion, circulating neutrophils will increase.
- Recurrent bacterial infection that lack pus formation - Pus is dead neutorphil in fluid. If
neutrophils can't get into tissue, pus can't form.

Step 4-Transmigration and Chemotaxis

1. Leukocytes transmigrate across the endothelium of postcapillary venules


and move toward chemical attractants (chemotaxis).
2. Neutrophils are attracted by bacterial products, IL-8, C5a, and LTB4.
Step 5- Phagocytosis

1. Consumption of pathogens or necrotic tissue.


2. Phagocytosis is enhanced by opsonins (IgG and C3b).
2. Pseudopods extend from leukocytes to form phagosomes, which are
internalized and merge with lysosomes to produce phagolysosomes.

11. What is Chediak-Higashi syndrome ?


- It is a protein trafficking defect (autosomal recessive) characterized by impaired
phagolysosome formation.[microtubule railroad tracks in the cells are defective].
- Mutations in the CHS1 gene which encodes a large cytosolic protein called LYST
(Lysosomal trafficking regulator), which regulates lysosomal trafficking.

12. What are sign/symptoms of Chediak-Higashi syndrome ?


○ Immune fingings-
 Neutropenia (impaired cell division so low neutrophils) - intramedullary death of neutrophils.
 Giant granules in leukocytes (due to fusion of granules arising from the Golgi apparatus).
 Increased risk of pyogenic infections (phagosome merging with lysosome is impaired,
so immune cells are less efficient at killing pathogens).
○ Defective primary hemostasis (due to abnormal dense granules in platelets)
○ Albinism (a melanocyte supplies melanin to about 25 keratinocytes. As railroad
transport is defective, it's melanin won't be transported to other keratinocytes)
○ Peripheral neuropathy - stuff from cell body of neuron won't be transported to axon
endings well.
Step 6 - Destruction of phagocytosed material

1. What are two mechanism of phagocytosis? Which is more efficient


• Oxygen dependent killing (most effective mechanism: Oxidative burst)
• Oxygen independent killing [occurs via enzymes present in leukocyte secondary granules]
(ex- lysozyme in macrophages and major basic protein in eosinophils).

2. Explain O2 dependent mechanism of phagocytosis.


• It occurs in phagolysosome
• Oxygen (O2) is converted to superoxide radical (O2-) by NADPH oxidase.
• Superoxide radical (O2-) is coverted to hydrogen peroxide (H2O2) by superoxide dismutase.
• Hyrodgen peroxide (H2O2) is converted to bleach (HOCl) by myeloperoxidase.
• HOCl kills organism

3. What is mechanism of Chronic granulomatous disease (CGD)?


• Due to NADPH oxidase defect (X-linked or autosomal recessive), hence Poor O2 dependent
killing leading to aggregation of macrophage and neutrophils at the site resulting in recurrent
infection and granuloma formation.
• X-linked defect: Defect in the gene coding membrane component of NADPH/phagocyte oxidase.
• Autosomal recessive: Defect in the gene coding cytoplasmic component of NADPH/ phagocytic oxidase.

A. What are the 5 catalase positive organism that cause symptomatic CGD?
○ Staph. Aureus
○ Pseudo. cepacia
○ Serratia marcescens
○ Nocardia
○ Aspergillus

B. Why do only catalase positive organism cause CGD?


• Most (Aerobic) bacterias produce H2O2 (bacterial respiratory metabolism byproduct) which can
be converted into HOCL by myeloperoxidase in the phagolysosome, therefore most bacteria can't
cause CGD despite mutation in human NADPH oxidase. However, in case of catalase +ve
organisms the catalase produced destroys the H2O2 [ROS] into water (H20) and oxygen and
hence formation of bleach is reduced which results in Chronic granulomatous disease.

C. What is nitroblue tetrazolium (NBT) test?


• It is a screening test for CGD in which Leukocytes are incubated with NBT dye (colourless).
• The dye turns blue if NADPH oxidase is present (O2- reduces tetrazolium salt into formazan dye
that absorbs light).

7. What is myloperoxide (MPO) deficiency?


• Defective conversion of H2O2 to HOCL.
• NBT test is positive hence respiratory burst (O2 to H2O2) is intact.
• Increased risk for Candida infections
• Most MPO deficiency patients are asymptomatic

Step 7-Resolution

9. Describe the resolution phase of acute inflammation.


• Neutrophils undergo apoptosis and disappear within 24 hours after resolution of the inflammatory stimulus.
• Results in pus (dead Neutrophil and bacteria with tissue debris and serum) formation
MACROPHAGES

10. Describe macrophage phase of acute inflammation.


• Macrophages predominate after neutrophils and peaks 2-3 days after inflammation begins.
• When monocytes from blood get into tissue, they become macrophage.
• Arrive in tissue via the margination, rolling, adhesion, and transmigration sequence

11. Describe phagocytosis of macrophage.


• Ingest organisms via phagocytosis (augmented by opsonins) and Phagocytotic killing by macrophage
is mainly oxygen independent - via the enzymes (lysozyme) in secondary granules

12. Macrophage manage the next step of the inflammatory process. Outcomes include:
• Resolution and healing:
Anti-inflammatory cytokines (e.g., IL-10 and TGF-beta) are produced by MΦ.
• Continued acute inflammation-marked by persistent pus formation;
IL-8 from MΦ recruits additional neutrophils.
• Abscess - acute inflammation surrounded by fibrosis (walling off);
MΦ mediate fibrosis via fibrogenic growth factors and cytokines.
• Chronic inflammation:
MΦ presents antigen to MHC-II cells activating CD4+ helper Tcells, which secrete cytokines.
Chronic inflammation (part 1)

1. What is chronic inflammation?


• Characterized by the presence of lymphocytes and plasma cells in tissue
• Delayed response, but more specific (adaptive immunity) than acute inflammation

Fig - chronic inflammation. Note the absence of multilobed neutrophil. Cell on left is lymphocyte. Middle cell is plasma
cell (nucleus pushed to side, slight perinuclear halo). Cell on right is macrophage (clock face nucleus).

2. What are stimuli of chronic inflammation?


• Persistent infection (most common)
• Infection with virus, mycobacteria, parasites, fungi
• Autoimmune disease
• Foreign material
• Some cancers

B cells

9. Explain development and maturation of B-cells.


• Immature B cells are produced in the bone and undergo Ig rearrangements to become naive B cells
• Naive B cells express surface lgM and IgD.

10. Explain B cell activation.


• Binding of antigen to surface IgM or IgD; results in maturation to lgM or IgD secreting plasma cells.
• B cells presents phagocytosed antigen to CD4+ helper T cells via MHC class II.
• 2nd activation signal: CD40 receptor on B cell binds CD40L on helper T cell
• TH2 Helper T cell will secrete IL4 and IL5 mediating:
- isotype switching of B cells
- somatic hypermutation (aka affinity maturation)
- maturation to plasma cells

10. Memory B cell


• SECONDARY RESPONSE: Exposure to the same Ag --> Memory B cell Ab receptor binds to Ag --->
Memory B cell proliferates into long lived polyclonal plasma cell producing large amount of IgG,
IgA, IgE having improved affinity towards the antigen.
T cells

3. Explain development and maturation of T-cells.


• Produced in bone marrow as progenitor T cells
• Further develop in the thymus where the T-cell receptor (TCR) undergoes rearrangement and
progenitor cells become CD4+ helper T cells or CD8+ cytotoxic T cells
- T cells use TCR complex (TCR and CD3) for antigen surveillance
- TCR complex recognizes antigen presented on MHC molecules
- CD4+ T cells recognizes MHC class II
- CD8+ T cells recognizes MHC class I
Activation of T cells requires
- binding of antigen MHC complex
- an additional 2nd signal.

4. Explain CD4+ helper T-cell activation


• Extracellular antigen (e.g., foreign protein) is phagocytosed, processed, and presented on
MHC class II, which is expressed by antigen presenting cells (APCs).
• 2nd activation signals.
- B7 on APC binds CD28 on CD4+ helper T cells
- CD40 on B cells bind to CD40L on CD4+ helper T cells
• Activated CD4+ helper T cells secrete cytokines that is divided into two subsets:
- TH1 subset secretes:
○ IFN-y - activator of macrophage
- promotes B-cell class switching from lgM to IgG
- promotes TH1 phenotype
- inhibits TH2 phenotype.
○ IL-2 - activates CD8+ cytotoxic T-cell (T-cell growth factor)
- TH2 subset secretes:
○ IL-4 - facilitates B-cell class switching to IgE and IgA
○ IL-5 - eosinophil chemotaxis and activation
- B cell maturation to plasma cell and class switching to lgA
○ IL-13 - function similar to IL-4
○ IL-10 - inhibits TH1 phenotype (anti-inflammatory cytokine)

4. Explain CD8+ cytotoxic T-cell activation


• Intracellular antigen (derived from proteins in the cytoplasm) is processed and presented
on MHC class I, which is expressed by all nucleated cells and platelets.
• 2nd activation signal.
- IL-2 (T-cell growth factor) from CD4+ TH1 cell
• Cytotoxic T cells are activated for killing. Killing occurs via APOPTOSIS:
○ Secretion of perforin and granzyme; perforin creates pores that allow granzyme to
enter the target cell activating caspases.
○ Expression of Fas Ligand, which binds Fas receptor on target cells

B cells

9. Explain development and maturation of B-cells.


• Immature B cells are produced in the bone and undergo Ig rearrangements to become naive B cells
• Naive B cells express surface lgM and IgD.

10. Explain B cell activation.


• Binding of antigen to surface IgM or IgD; results in maturation to lgM or IgD secreting plasma cells.
• B cells presents phagocytosed antigen to CD4+ helper T cells via MHC class II.
• 2nd activation signal: CD40 receptor on B cell binds CD40L on helper T cell
• TH2 Helper T cell will secrete IL4 and IL5 mediating:
- isotype switching of B cells
- somatic hypermutation (aka affinity maturation)
- maturation to plasma cells
Chronic inflammation (part 2)

Granulomatous inflammation

11. What is granulomatous inflammation?


• It is a Subtype of chronic inflammation
• Characterized by granuloma, which is a collection of epithelioid histiocytes (macrophages with
abundant pink cytoplasm and elongated nuclei), usually surrounded by multinucleated giant cells and
a rim of lymphocytes
• Divided into noncaseating and caseating subtypes

12. Describe noncaseating granulomas ?


• Defining feature: lack of central necrosis.
• Causes:
○ Reaction to foreign material - (ex: leaking of breast implants)
○ Sarcoidosis - [hallmark]
○ Crohn disease - [hallmark]
○ Beryllium exposure
○ Cat scratch disease

13. Describe caseating granuloma ?


• Defining feature: exhibit central necrosis.
• Characteristic of tuberculosis [AFB stain] and fungal infections [GMS silver stain].

14. How are granulomas formed?


• Macrophages process and present antigen via MHC class II to CD4+ helper T cells.
• Interaction leads macrophages to secrete IL-12 which induces CD4+ helper T cells
to differentiate into TH1 subtype.
• TH1 cells secrete INF-y which converts macrophages to epithelioid histiocytes
and giant cells.

Fig - noncaseating granuloma. Not abundant Fig - caseating granuloma. Note the central necrosis
lymphocytes in periphery that indicates chronic
inflammation. Note multinucleated cell. Presence of
nucleus in epitheloid histiocytes means they are alive
- making this noncaseating granuloma.
Primary immunodeficiency

DIGEORGE SYNDROME

1. What is DiGeorge syndrome ?


• Developmental failure of the 3rd and 4th pharyngeal pouches
• Due to 22q11 microdeletion
2. What are presentation of DiGeorge syndrome?
• T-cell deficiency (lack of thymus)- problems fighting viral and fungal infection
• Hypocalcemia (lack of parathyroids) - no Ca2+ regulation
• Abnormalities of:
- heart
- great vessels
- face

SEVERE COMBINED IMMUNODEFICIENCY (SCID)

3. What is SCID (severe combined immune deficiency)?


• Defective cell-mediated and humoral immunity.

4. Etiology of SCID?
• Cytokine receptor defects - Cytokine signaling is necessary for proliferation and maturation of B and T cells.
• Adenosine deaminase deficiency - ADA is necessary to deaminate adenosine and deoxyadenosine for their
excretion as waste products thus preventing their toxic accumulation in lymphocytes.
• MHC class II deficiency -MHC class II is necessary for CD4+ helper T cell activation and cytokine production

5. Presentations of SCID?
• Lack of T cells result in - fungal and viral infections
• Lack of B cells result in - bacterial and protozoal infections
• Increased opportunistic infection and be wary to give live vaccines.

6. Treatment of SCID?
• Sterile isolation ('bubble baby')
• Stem cell transplant

X-LINKED AGAMMAGLOBULINEMIA

8. What is X-linked agammaglobulinemia ?


• Complete lack of immunoglobulin due to disordered B-cell maturation.
• Pre- and pro-B cells cannot mature - naïve B cells can't mature to plasma cells
• Due to X linked mutation of Bruton tyrosine kinase gene at Xq21 locus
• Bruton tyrosine kinase is a signaling receptor that helps in B cells maturation

9. What is presentation of X-linked agammaglobulinemia ?


• Presents after 6 months of life coz maternal Ab present during the first 6 months of life are protective.
• Recurrent infections:
- bacterial infection - No opsonization due to lack of IgG/IgM
- enterovirus (polio and coxsackievirus) infection - due to lack of IgA protecting GI mucosa
- Giardia Iamblia infection - due to lack of IgA protecting GI mucosa
• Live vaccines (e.g., polio) must be avoided.
COMMON VARIABLE IMMUNODEFICIENCY (CVID)

10. What is common variable immunodeficiency disease?


• Low Ig level due to B-cell or helper T-cell defects
11. Presentation?
• Increased risk for bacterial, enterovirus, and Giardia Lamblia infections, often in late childhood.
• Increased risk for:
- autoimmune disease
- lymphoma

IgA DEFICIENCY

12. What is IgA deficiency?


• MOST COMMON Ig deficiency
• Low serum and mucosal IgA

11. Presentation?
• Increased risk for mucosal infection, especially viral
• However, most patients are asymptomatic.

HYPER-IgM SYNDROME

13. What is Hyper-IgM syndrome?


• Characterized by elevated IgM level (and low IgA, IgG, and IgE levels).

14. What is pathophysiology of Hyper-IgM syndrome?


• Due to mutated CD40 Ligand (on helper T cells) or CD40 receptor (on B cells).
- Hence, Second signal cannot be delivered to helper T cells during B-cell activation.
- Consequently, cytokines necessary for Ig class switching are not produced.

11. Presentation?
• Recurrent pyogenic infections (due to poor opsonization), especially at mucosal sites.

WISKOTT-ALDRICH SYNDROME

15. What is Wiskott-Aldrich syndrome (WAS)?


• Defective humoral and cellular immunity
14. Pathophysiology ?
• X-linked mutation in the WASP (Wiskott-Aldrich syndrome protein) gene.

11. Presentation?
• Presents as:
- thrombocytopenia - bleeding is a major cause of death
- eczema
- recurrent infections

COMPLEMENT DEFICIENCIES

16. What are the consequences of the following complement deficiencies?


• C5-C9 deficiency - increased risk for Neisseria infection (N. gonorrhoeae and N. meningitidis)
• C1 inhibitor deficiency - results in hereditary angioedema which is characterized by edema of
the skin (especially periorbital area) and mucosal surfaces
Autoimmune Disorders

1. What is autoimmune disorders?


- Characterized by immune-mediate damage of self tissues.
- Autoimmune disorders are clinically progressive with relapses and remissions and often
show overlapping features; partially explained by epitope spreading.

2. What is epidemiology of autoimmune disorders.


- US prevalence is 1%-2%.
- More common in women; classically affects women of childbearing age
(Estrogen may reduce apoptosis of self-reactive B cells).

3. What is the cause of autoimmune disorders?


- Involves loss of self-tolerance
• Self-reactive lymphocytes are regularly generated but develop central (thymus and bone marrow)
or peripheral tolerance.
=> Central tolerance:
○ Central tolerance in thymus leads to T-cell (thymocyte) apoptosis or generation of regulatory T cells.
[AIRE mutations result in autoimmune polyendocrine syndrome].
○ Central tolerance in bone marrow leads to receptor editing or B-cell apoptosis.
=> Peripheral tolerance:
○ Peripheral tolerance leads to anergy or apoptosis of T and B cells.
[Fas apoptosis pathway mutations result in autoimmune lymphoproliferative syndrome (ALPS)].
• Regulatory T cells suppress autoimmunity by blocking T-cell activation and producing
anti-inflammatory cytokines (IL-10 and TGF-beta)
○ CD25 polymorphisms are associated with autoimmunity (MS and type 1 DM).
○ FOXP3 mutations lead to IPEX syndrome (Immune dysregulation, Polyendocrinopathy,
Enteropathy, X-linked).

1. What are the risk factors for autoimmune disorders?


- Etiology is likely an environmental trigger in genetically-susceptible individuals.
- Increased incidence in twins
- Association with certain HLA types (e.g., HLA-B27) and PTPN22 polymorphisms
- Environmental triggers lead to bystander activation or molecular mimicry.
SYSTEMIC LUPUS ERYTHEMATOSUS

4. What is Lupus?
- Chronic, systemic autoimmune disease in which autoantibodies can cause type II HSR (cytotoxic Ab binds
and kills Multiple cells) or type III HSR (Ag-Ab complex form and deposit in tissue causing damage).
- Almost any tissue can be involved.
- Flares and remissions are common.

2. What is epidemiology of lupus ?


- Classically arises in middle-aged females, especially African American and Hispanic women
- May also arise in children and older adults (less dramatic gender bias)

1. What are the causes of Lupus?


• Poorly-cleared apoptotic debris (e.g., from UV damage) activates self-reactive lymphocytes,
which then produce antibodies to host nuclear antigens.
• Antigen-antibody complexes are generated at low levels and taken up by dendritic cells.
• DNA antigens activate TLRs, amplifying immune response (IFN-alpha).
• Antigen-antibody complexes are subsequently generated at higher levels and deposit in
multiple tissues causing disease.
• Deficiency of early complement proteins (C1q, C4, and C2) is associated with SLE.

5. What are clinical features of lupus?


- Malar ‘butterfly’ rash or discoid rash, especially upon exposure to sunlight.
- Fever, weight loss, fatigue
- lymphadenopathy
- Raynaud phenomenon
- Oral or nasopharyngeal ulcers (usually painless)
- Arthritis (usually involving > 2 joints)
- Serositis (pleuritis and pericarditis) and Myocarditis
- Psychosis or seizures (inflammation of CNS)
- Renal damage (common cause of death)
- Diffuse proliferative glomerulonephritis is the most common and most severe form of injury.
- Other nephritic/nephrotic sydromes can also occur like Membranous glomerulonephritis
- Anemia, thrombocytopenia, or leukopenia (type II HSR - autoAb against blood cells)
- Libman-Sacks endocarditis (vegetations on both side of valve)

7. How do we diagnose lupus?


- ANA (anti-nuclear antibody) is used for screening test [sensitive but not specific].
- Anti-dsDNA or anti-Sm antibodies [highly specific].

9. What is antiphospholipid syndrome ?


- It is associated with SLE (one-third of patients).
- Autoantibody are directed against proteins bound to phospholipids
- Important antiphospholipid antibodies include:
- anticardiolipin (false-positive VDRL and RPR syphilis screening tests)
- anti-β2 glycoprotein I
- lupus anticoagulant (falsely-elevated PTT but induces hypercoagulable state)
=> Results in arterial and venous thrombosis including:
○ deep venous thrombosis
○ hepatic vein thrombosis
○ placental thrombosis (recurrent pregnancy loss)
○ cerebral thrombosis (stroke)
=> Requires lifelong anticoagulation.
=> More commonly occurs as a primary disorder.
9. What is Antihistone antibody ?
- It is characteristic of drug-induced lupus
- Drug that cause lupus:
- Procainamide
- Hydralazine
- Isoniazid
- ANA is positive by definition.
- CNS and renal involvement are rare.
- Removal of drug usually results in remission

9. What is the Treatment of lupus ?


- First-line treatment includes avoiding exposure to direct sunlight and glucocorticoids for flares.
- Other immunosuppressive agents are useful in severe or refractory disease.
- 5-year survival is > 90%; renal failure, infection, and accelerated coronary
atherosclerosis (occurs late) are common causes of death.

9. What are the complications of lupus ?


- 5-year survival is > 90%.
- Common causes of death are:
- renal failure
- infection
- accelerated coronary atherosclerosis (occurs late)
SJÖGREN SYNDROME

11. What is sjogren syndrome [SS] ?


- Autoimmune destruction of moisture-producing glands like lacrimal and salivary glands (parotids).
- Lymphocyte-mediated damage (type IV HSR) with fibrosis (leads to enlargement).
- Also presents as Extra-glandular disease.
- Classification of sjogren syndrome:
- Primary SS [occurs as an isolated disorder; Etiology unknown]
- Secondary SS [occurs as a result of another autoimmune CT disorder especially rheumatoid arthritis;
Rheumatoid factor is often present even when rheumatoid arthritis is absent]. It can also occur in
association with SLE, polymyositis,scleroderma, thyroiditis, etc.

12. What are clinical presentation of Sjogren syndrome?


- Sicca syndrome:
- dry eyes (keratoconjunctivitis sicca) - May progress to ulceration of corneal epithelium.
- dry mouth (xerostomia) - May progress to ulceration of oral mucosa.
- dry vagina
- dry skin
- dry nasal mucosa
Non Sicca syndrome:
- Recurrent dental caries (bacterial infection) in an older woman (50-60 years)
- joint pains (arthritis) and muscle pain
- numbness in the arms and legs
- urinary disorder
- abdominal pain

14. What is the risk factor for Sjogren syndrome ?


- Increased risk for B-cell (marginal zone) lymphoma - presents as unilateral enlargement of
the parotid gland late in disease course

13. How do you diagnose Sjogren syndrome ?


- Characterized by presence of:
=> ANA (anti-nuclear antibody) - sensitive
=> anti-ribonucleoprotein antibodies (anti-SS-A/Ro and anti-SS-B/La) - specific
- Anti-SSA and anti-SSB are associated with extraglandular manifestations (e.g., neuropathy).
- Pregnant women with anti-SSA are at risk for delivering babies with neonatal lupus and congenital HB.
- Anti-SSA and anti-SSB are also seen in a subset of patients with SLE (screen pregnant patients)
- Lymphocytic sialadenitis on lip biopsy (minor salivary glands) is an additional diagnostic criterion.
SYSTEMIC SCLEROSIS (SCLERODERMA)

15. What is scleroderma?


- Autoimmune disorder characterized by sclerosis of skin and visceral organs

17. What is the epidemology scleroderma ?


- Classically presents in middle-aged females (30-50 years)

15. Explain the Etiology of scleroderma.


- Fibroblast activation leads to deposition of collagen that thickens the tissue (also makes it tight and less mobile).
○ Autoimmune damage to mesenchyme is possible initiating event.
○ Endothelial dysfunction leads to:
- inflammation (increased adhesion molecules)
- vasoconstriction (increased endothelin and decreased NO)
- secretion of growth factors (TGF-beta and PDGF).
○ Fibrosis - initially perivascular which progresses and causes organ damage.

17. What are the CLASSIFICATION of systemic scleroderma ?


○ Limited type
- Skin involvement is limited (hands and face) with late visceral involvement.
- Prototype is CREST syndrome:
=> Calcinosis
=> Raynaud phenomenon
=> Esophageal dysmotility
=> Sclerodactyly
=> Telangiectasias of the skin
- Diagnosis:
○ anti-Centromere antibodies
○ Diffuse type
- Skin involvement is diffuse with early visceral involvement.
- Any organ can be involved
- Commonly involved organs include:
=> Vessels (Raynaud phenomenon)
=> GI tract (esophageal dysmotility and reflux)
=> Lungs (interstitial fibrosis and pulmonary hypertension)
=> Kidneys (scleroderma renal crisis)
- Diagnosis:
○ ANA (anti-nuclear antibody)
○ anti-DNA topoisomerase I antibody (aka SCL-70 antibody)

MIXED CONNECTIVE TISSUE DISEASE


18. What is mixed connective tissue disease?
- Autoimmune-mediated tissue damage with mixed features of:
- SLE
- systemic sclerosis Fig- sclerodermal hands
- polymyositis
18. What is the diagnosis of mixed connective tissue disease ?
- ANA (anti-nuclear antibody)
- Serum antibodies against U1 ribonucleoprotein.
Wound Healing

1. Basic principles.
- Healing is initiated when inflammation begins.
- Occurs via a combination of regeneration and repair

REGENERATION

1. What is regeneration?
- Replacement of damaged tissue with native tissue
- Dependent on regenerative capacity of tissue

3. What are 3 different types of tissue based on regenerative capacity?


- Labile tissue - possess stem cells that continuously cycle to regenerate the tissue.
○ Small and large bowel (stem cells in mucosal crypts)
○ Skin (stem cells in basal layer)
○ Bone marrow (hematopoietic stem cells)
○ Lungs (stem cell is type 2 pneumocyte)
- Stable tissue - comprised of cells that are quiescent (G0), but can reenter the cell
cycle to regenerate tissue when necessary.
○ Liver (compensatory hyperplasia after partial resection)
○ PCT of kidney (pt of acute tubular necrosis are put on dialysis until they regenerate)
- Permanent tissue - lack significant regenerative potential
○ Myocardium
○ Skeletal muscle
○ Neurons

Fig - circles are the crypts of bowel Fig - line showing basal layer of skin
Fig - granulation tissue showing blood vessels, collagen and fibroblast.
Contrast it with granuloma in chronic inflammation file.

REPAIR

2. What is repair?
- Replacement of damaged tissue with fibrous scar.
- Occurs when:
- regenerative stem cells are lost (e.g., deep skin cut)
- tissue lacks regenerative capacity (e.g., healing after a myocardial infarction)

4. What is granulation tissue formation ?


- It is the initial phase of repair.
- It consistis of:
○ fibroblasts (deposit type III collagen)
○ capillaries (provide nutrients)
○ myofibroblasts (contract wound)

5. Describe Scar formation.


- Granulation tissue eventually results in scar formation
- Type III collagen is replaced with Type I collagen
- Collagenase (zinc is a cofactor) removes type III collagen.

6. Where are 4 types of collagen found?


- Type 1 - skin, bone, tendons ;[very high tensile strength].
- Type 2 - cartilage.
- Type 3 - granulation tissue, embryonic tissue, uterus, and keloids ;[very pliable].
- Type 4 - basement membrane

7. What are the Mechanism of tissue repair ?


- Mediated by paracrine signaling via growth factors (ex- macrophages secrete
growth factors that target fibroblasts)
- Interaction of growth factors with receptors results in gene expression and
cellular growth (ex- epidermal growth factor with growth factor receptor).
- Examples of mediators include:
○ Fibroblast growth factor (FGF) - induces angiogenesis and skeletal development.
○ Vascular endothelial growth factor (VEGF) - induces angiogenesis.
○ Platelet derived growth factor (PDGF) - endothelium, smooth muscle and fibroblast GF.
○ TGF alpha - epithelial and fibroblast GF.
○ TGF beta - imp. fibroblast GF; inhibits inflammation.
HEALING

8. Describe Wound healing


- Cutaneous healing occurs via primary or secondary intention.
○ Primary intention: Wound edges are brought together (e.g., suturing of a surgical incision);
leads to minimal scar formation
○ Secondary intention: Edges are not approximated. Granulation tissue fills the defect;
myofibroblasts then contract the wound, forming a scar.

10. What causes Delayed wound healing ?


- Infection is the most common cause [S. aureus is the most common offender]
- Deficiency of:
○ Vitamin C deficiency - Vitamin C is an important cofactor for hydroxylase which aids
in hydroxylation of proline and lysine procollagen residues.
○ Copper deficiency - Copper is a cofactor for lysyl oxidase which cross-links lysine and
hydroxylysine to form stable collagen.
○ Zinc deficiency - Zinc is a cofactor for collagenase which replaces the type lll collagen
of granulation tissue with stronger type I collagen.
- Other causes include Foreign body, ischemia, diabetes, malnutrition etc .

11. What is wound Dehiscence ?


- Rupture of a wound.
- Most commonly seen after abdominal surgery

12. What is hypertrophic scar


- Excess production of scar tissue that is localized to the wound.
- Caused by excess type I collagen deposition

13. Describe keloid ?


- Excess production of scar tissue that is out of proportion to the wound.
- Characterized by excess type III collagen
- Genetic predisposition (more common in African Americans)
- Classically affects earlobes, face, and upper extremities. Fig- hypertrophic scar
Chapter 3: Principles of Neoplasia
3.1 Neoplasia

1. Define the following


Neoplasia - Neoplasia is cell growth that is monoclonal (arise from single cell),
unregulated, and irreversible.
- Contrast to hyperplasia (ex – hyperplasia of uterus during childbirth) which
is polyclonal and is regulated.
Desmoplasia - Process by which tumor becomes firm (it is because neoplastic cells induce
fibroblasts within the stroma to form abundant collagen)
Carcinoma in - (aka high-grade dysplasia) - when dysplastic changes involve entire
site thickness of epithelium (earliest form of epithelial malignancy)

2. What are two ways to determining monoclonality?


- G6PD or androgen receptor isoforms,
- Ig light chain phenotype for lymphomas.

3. How is monoclanity determined from G6PD?


- G6PD is a protein with multiple isoforms which is encoded in X-chromosome. Let’s say a
female has isoforms A and B in her 2 x chromosomes. Due to random silencing of X-
chromosomes, her A:B protein ratio in hyperplasia or normal condition will be 1:1. If the
ratio is different from 1:1, it strongly suggests monoclonality.

4. How is monoclonality determined from Ig light chain phenotype?


- The ratio of K to Lambda light chain is 3:1. If the ratio is different than this, it suggests
monoclonality

6. What are differences between benign and malignant neoplasm?


Benign neoplasia Malignant neoplasia
Remain localized and do not Invade locally and have potential to metastasize (don't
metastasize have to be already metastasized)
Slow-growing Rapid growing
Distinct Infiltrative
Mobile Fixed to surrounding tissue

7. Name the lineage of following benign and malignant tumors:


Cell lineage Benign Malignant (cancer)
Epithelium Adenoma Adenocarcinoma
Papilloma Papillary carcinoma
Mesenchyme Lipoma Liposarcoma
Angioma Angiosarcoma
Chondroma Chondrosarcoma
Osteoma Osteosarcoma
Lymphocyte doesn't exist Lymphoma
Melanocyte Nevus (mole) Melanoma (not melanosarcoma)
Adenoma = tumor that makes glands
Papilloma = tumor that makes papillary finger like structures
8. What are leading cause of death in adults in children? (HY)
Adults Children
1. Cardiovascular disease 1. Accidents
2. Cancer 2. Cancer
3. Cerebrovascular disease 3. Congenital defects

9. What are leading cancers by incidence and death toll in male and females?
Cancer by incidence Male 1. Prostrate 2. Lung 3. Colorectal
Female 1. Breast 2. Lung 3. Colorectal
Cancer by death tolls Male 1. Lung 2. Prostrate 3. Colorectal
Female 1. Lung 2. Breast 3. Colorectal
- Table excludes squamous cell and basal cell carcinoma of skin - very common and
malignant but rarely metastasize. Detected early and easily treatable.
- Lung cancer doesn't have easy screening test like others. It's usually detected late.

10. What are some properties of neoplasia?


- Approximately 30 divisions before earliest symptoms arise (2^30 cells)
- Subsequent divisions results in increased mutations
- Cancers that don't produce symptoms till late (ex-ovarian, pancreatic, lung cancer- because
lots of space to expand) will have accumulated tons of mutations and hence poor prognosis

11. What area goals of cancer screening?


- Catch dysplasia before it becomes carcinoma (dysplasia is reversible)
- Detect carcinoma before clinical symptoms
Tests Detection
Pap smear - Cervical intraepithelial neoplasia
Mammography - Breast cancer
- Ductal carcinoma in-situ
PSA (prostrate specific - Prostrate cancer (usually grow on rectal side and doesn't
antigen) and DRE (digital produce urinary syndrome until late; BPH grows centrally
rectal exam) and produce urinary syndrome)
Hemoccult test and - Colorectal cancer
colonoscopy - Detect blood in stool (hemoccult test)

30. What are 4 carcinomas that spread by blood instead of lymph? (HY)
- Renal cell carcinoma
- Follicular carcinoma of thyroid
- Hepatocellular carcinoma
- Corneal carcinoma
3.2 Carcinogenesis (Part 1)

1. What do carcinogens do? What are examples?


- They damage DNA. Ex - chemicals, viruses, radiation.

2. What are cancer associations of the following chemicals? (HY)


Chemicals Cancer Remarks
Aflatoxins - Hepatocellular carcinoma (most common cancer - Derived from Aspergillus
in some African countries) flavus;
- Usually contaminates
stored grains
Alkylating - Leukemia - Found in chemotherapy
agents - Lymphoma drugs
Alcohol - Squamous cell carcinoma (SCC)of oropharynx and
upper esophagus(another risk is tobacco)
- Pancreatic carinoma (EtOH causes chronic
pancreatiis that incrases risk of cancer)
- Hepatocellular carcinoma
Arsenic - SCC of skin (women used to apply Arsenic to be
fair-skinned) - test Arsenic poisoning from
fingernail and hair follicles
- Lung cancer (Arsenic present in cigarettes)
- Angiosarcoma of liver
Asbestos - Lung cancer (far more likely) and mesothelioma
Cigarette - SCC of oropharynx and esophagus - MOST COMMON
- Lung cancer CARCINOGENIC
- Kidney and bladder cancer (urithelial carcinoma - WORLDWIDE
cells that line urinary tubes; most IMP risk factor: - Polycyclic hydrocarbons
cigerrate - toxins from smoking in urine irritate main carcinogen
the cells)
Nitrosamine - Stomach carcinoma (intestinal type) (other - Found in smoked food;
stomach carcinoma is diffuse type but responsible for high
nitrosamine not associated) stomach cancer in Japan
Napthylaine - Urothelial carcinoma of bladder - Derived from cigarette
smoke (excreted by
urine)
Vinyl chloride - Angiosarcoma of liver - Occupational exposure
(used to make PVC pipes)
Nickel, - Lung cancers - Occupational exposure
chromium,
beryllium, or
silica
- 1 Billion people smoke worldwide
3. What are ongogenic association of the following viruses?
Virus Cancer type
EBV - Nasopharyngeal carcinoma (Classic pt - Chinese male, African person) -
metastasizes early; classic presentation is neck mass
- Burkitt lymphoma (Classic pt - African kids)
- CNS lymphoma in AIDS
HHV-8 - Kaposi sarcoma (tumor of endothelial cell) (Classic pt - 1. older Eastern
european males - take tumor out; 2. AIDS pt - treat HIV; 3. transplant pt -
reduce immunosuppression)
HBV and HCV - Hepatocellular carcinoma
HTLV-1 - Adult T-cell lukemia/lymphoma
HPV (type 16, - SCC of anogenital area (vagina, vulva, cervix, anus)
18, 31, 33) - Adenocarcinoma of cervix

4. What are cancer association of ionizing and non-ioninzing radiation?


Radiation type Cancer types MOA
Ionizing (nuclear - AML - Generation of hydroxyl free radical
reactor, - CML
radiotherapy) - Papillary thyroid carcinoma
(Cherbonyl kids have lots of
papillary cancer)
Nonionizing (UBV - Basal cell carcinoma of skin - DNA damage (formation of too much
from sun most - SCC of skin pyrimidine dimers to be excised by
common) - Melanoma of skin restriction endonuclease)
- In xeroderma pigmentosum, restriction endonuclease is bad and pt has high risk of BCC, SCC and
melanoma of skin

5. What is most common cause of ionizing radiation in USA?


- Radon (formed by decay of uranium) - present in soil and accumulates in closed space (basement)
○ 2nd most common cause of lung cancer in USA
3.2 Carcinogenesis (Part 2)

1. What are three systems to be disturbed in carcinogenesis? (HY)


- Proto-oncogene
- Tumor suppressor genes
- Regulator of apoptosis

Protooncogenes
2. What are protooncogenes and how can they cause cancer?
- Proto-oncogenes are essential for regular cell growth and differentiation
- Mutation produces oncogenes that leads to unregulated cell growth

3. What are examples of protooncogenes?


- Growth factors and their receptors
- Signal transducers
- Nuclear regulators (transcription factors)
- Cell cycle regulators (move cell division stuff around in cell)

- Normally, binding of growth factors to its receptor induces signal transduction. In


nucleus, cell cycle regulators are activated that induce cycling of cells through G1 - S -
G2 - M stages of cell division
4. What cancer are the following protooncogenes associated with and what's their
function and mechanism of causing cancer?
Cancer Function of MOA of cancer
protooncogene
Growth factor
PDGFB - Astrocytoma - Platelet derived Overexpression,
growth factor autocine loop
Growth factor
receptors
ERBB2 - Subset of breast Epidermal growth Amplification
(HER2/neu) cancer factor receptor
RET - MEN2A, MEN2B Neural growth factor Point mutation
- Sporadic MTC receptor
KIT - Gastrointestinal Stem cell growth Point mutation
stromal tumor factor receptor
Signal
Transducers
RAS gene family - Carcinoma GTP-binding protein Point mutation
- Melanoma
- Lymphoma
- (Found in ~70% of
all cancers)
ABL - CML Tyrosine kinase Translocation - t(9,22)
- Some ALL with BCR
Nuclear
regulators
C-MYC - Burkitt lymphoma Transcription factor T (8,14) involving IgH
(heavy chain)
N-MYC - Neuroblastoma Transcription factor Amplification
L-MYC - Lung carcinoma Transcription factor Amplification
(small cell)
Cell-cycle
regulators
CCND1 (cyclin Mantle cell Cyclin T(11, 14) involving IgH
D1) carcinoma
CDK4 Melanoma Cyclin dependent Melanoma
kinase
3.2 Carcinogenesis (Part 3), 3.4 Clinical Characteristics

1. Differentiate benign and malignant neoplasia.


Benign neoplasia Malignant neoplasia
Remain localized and do not Invade locally and have potential to metastasize (don't
metastasize have to be already metastasized)
Slow-growing (years) Rapid growing (weeks/months)
Distinct (well localized, ex - can isolate Infiltrative (can't distinguish from surrounding breast
by breast exam) tissue in physical exam)
Mobile (ex - you can grab a breast Fixed to surrounding tissue
tumor and move it around)
- Classification of benign vs malignant requires biopsy

2. What are histologic differences of benign and malignant tumor?


Benign neoplasia (well-differentiated) Malignant tumor (poor differentiation)
- Organized growth - Disoganized growth (doesn't look like tissue it's
growing in)
- Uniform nuclei - Nuclear pleomorphism with hyperchromasia (very
dark blue)
- Low nuclear to cytoplasm ratio (more - High nuclear to cytoplasm ratio (less cytoplasm, big
cytoplasm) nuclei)
- Minimum mitotic activity - High mitotic activity
- Lack of invasion - Invasion
- No metastatic potential

Fig: Follicular thyroid adenoma (left) - note organized cell growth with
colloid in between cells, uniform nuclei with lots of cytoplasm,
minimum mitotic activity. Anaplastic thryoid carcinoma (right) -
disorganized growth (cells look nothing like thyroid), large
nuclei:cytoplasm ratio, nuclear pleomorphism, mitotic figure (very
dark nuclei cell).

3. What's the absolute distinguishing feature between benign and malignant tumors?
- Potential to metastasize (benign tumors never metastasize, malignant tumors can metastasize)

3.5 What are some ways of identifying cell types in tumor?


- Immunohistochemistry
- Serum tumor markers
4. What is intermediate filament? How is immunohistochemistry of intermediate filament helpful in
grouping cancer? (HY)
- Intermediate filament are a type of cytoskeletal protein; different intermediate filament are
present in different cell types. -
- Immunohistochemistry of intermediate filament helps in identifying cell types in tumor.
Cell type Intermediate filament present
Epithelium Keratin
Mesenchyme (connective tissue) VImentin
Muscle Desmin
Neuroglia GFAP
Neurons Neurofilament

5. What immunohistochemical molecules help identify cancer of following cell types?


Cell type Immunohistochemistry
molecule
Prostrate PSA (prostrate surface antigen)
Breast epithelium ER (estrogen receptor)
Thyroid follicular cells Thyroglobulin
Neuroendocrine cells (small cell cancer of lung and carcinoid Chromogranin (HY)
tumor)
Melanoma S-100
- Well differentiated neuroendocrine tumor = carcinoid tumor. Poorly differentiated neuroendocrine
tumor = small cell cancer.

6. What are serum tumor markers and what's their utility?


- They are proteins released by tumors.
- Useful for
○ Screening (ex- high PSA) - still need biopsy for diagnosis
○ monitoring response to treatment (ex- PSA should drop after prostrate cancer surgery)
○ recurrence of tumor (ex - same pt returns 2 year later with bone pain; check for PSA to see if
it's prostrate cancer again)

7. What does tumor grading of well differentiated and poorly differentiated mean?
- Well differentiated - tumor resembles parent tissue - good prognosis
- Poorly differentiated - tumor doesn't resemble parent tissue - poor prognosis
- Look at cellular architecture and nuclear morphology to classify differentiation

8. What's staging of cancer? What's its value?


- Staging of cancer is based on size and spread
- It's no. 1 prognostic factor (more important than grade)
- Determined after final resection of tumor

9. What's TNM staging of cancer?


- T = tumor size and depth (size important for solid organ tumor, and depth important for tubular
organs like colon)
- N = spread to regional lymph nodes (2nd most important prognostic factor)
- M = metastasis (no. 1 prognostic factor)
3.3 Tumor progression

1. What are the steps in invasion and spread of tumor?


- Downregulation of e-kedherin (e-kedherin is an adhesion molecules that keeps epithelial cells
attached to each other)
- Tumor cells attach to laminin in basement membrane
- Tumor cells produce collagenase that destroys collagen 4 in basement mebrane. This helps tumor
pass through basement membrane
- Tumor cells bind to fibronectin in extracellular membrane and spread locally. It can now pass to
blood vessel or lymphatics.

2. How do metastatic cancer spread?


- Lymphatic spread is characteristic of carcinomas and less common for sarcoma.
- Hematogenous spread is characteristic of sarcoma and some carcinoma - usually follows venous
drainage and seeds at first capillary bed encountered
- Seeding of body cavities - most common is peritoneum (ex - by ovarian carcinoma (omental
caking)) but may also occur in the pleural or pericardial cavities, the subarachnoid space, and the
joint spaces

Fig: omental caking by ovarian carcinoma

3. What are carcinomas that spread hematogenously?


- Renal cell carcinoma - renal vein
- Hepatocellular carcinoma - hepatic vein
- Follicular thyroid carcinoma
- Choriocarcinoma (placental cancer - of trophoblast cells - function of placenta is to invade blood
vessels)
Chapter 4: Hemostasis and Related
Disorders
CMAP summary

HEMOSTASIS
A. Integrity of the blood vessel is necessary to carry blood to tissues.
1. Damage to the wall is repaired by hemostasis, which involves formation of a thrombus
(clot) at the site of vessel injury.
B. Hemostasis occurs in two stages: primary and secondary.
1. Primary hemostasis forms a weak platelet plug and is mediated by interaction between
platelets and the vessel wall.
2. Secondary hemostasis stabilizes the platelet plug and is mediated by the coagulation
cascade.
Primary Hemostasis and related bleeding disorders

PRIMARY HEMOSTASIS

Step 1 - Transient vasoconstriction of damaged vessel

2. What causes vasoconstriction?


- Vasoconstriction is mediated both neurally and chemically.
- Neural mediation: Reflex neural stimulation.
- Chemical mediation: endothelin release from endothelial cells while NO and
prostacyclin I2 production goes down.

Step 2 - Platelet adhesion to the surface of disrupted vessel

3. What happens in platelet adhesion?


- VWF (von willieband factor) binds to exposed subendothelial collagen in damaged tissue.
- Platelets bind to vWF using the GlycoProtein Ib surface receptor.
- vWF is derived from
- Weibel-Palade bodies of endothelial cells.
- Alpha-granules of platelets.

Step 3 - Platelet degranulation (aka platelet activation)

4. What happens in platelet activation (aka degranulation)?


- Platelet changes shape after adhesion and degranulates with release of multiple mediators:
○ Thromboxane A2 - synthesized by platelet COX.
- promotes platelet aggregation.
○ ADP - released from platelet dense or delta granules.
- promotes exposure of GPllb/GPllla receptor on platelets.

4. What are the components of granules ?


- Alpha granules - vWF, Fibrinogen, Fibronectin, PDGF, ILGF 1, TGFβ, PF 4, thrombospondin, fibronectin,
thrombospondin, fibronectin, factor V, PAI 1, etc.
- Delta granules - ADP, ATP, serotonin (for vasoconstriction), calcium (for secondary hemostasis), etc.

Step 4 - Platelet aggregation

5. What happens in platelet aggregation?


- Platelets aggregate at the site of injury via GPIIb/IIIa using fibrinogen (from plasma)
as a linking molecule [single fibrinogen molecule is linked to multiple platelets via
their glycoprotein IIB/IIIA receptor].
- Results in formation of platelet plug [Platelet plug is weak and coagulation cascade
(secondary hemostasis) stabilizes it].
DISORDERS OF PRIMARY HEMOSTASIS

6. What are 2 classification of platelet disorders?


- Quantitative disorders - Low platelet count - Thrombocytopenia.
- Qualitative disorders - Platelet dysfunciton - Thrombophilia.

6. What are some Clinical features of primary hemostatis ?


- Symptoms of mucosal bleeding:
- epistaxis (most common overall symptom)
- hemoptysis
- Gl bleeding
- hematuria
- menorrhagia (heavy menses)
- Intracranial bleeding (occurs with severe thrombocytopenia).
Symptoms of skin bleeding:
- petechiae (1-2 mm) - is a sign of thrombocytopenia
- purpura (> 3 mm) - bleeding spots
- ecchymoses (> I cm)
- easy bruising

7. What are some useful lab studies for platelet?


- Platelet count
=> normal count is 150-400 k/μl of blood
=> < 50 k/μl leads to symptoms
- Bleeding time
=> normal 2-7 minutes
=> prolonged with quantitative and qualitative platelet disorders
- Blood smear - used to assess number and size of platelets
- Bone marrow biopsy - used to assess megakaryocytes
Quantitative disorders

Idiopathic/Immune thrombocytopenic purpura (ITP)

8. What is ITP ?
- Autoimmune production of lgG against platelet antigens (ex- GPIIb/IIIa)
- Autoantibodies are produced by plasma cells in the spleen.
- Antibody-bound platelets are consumed by splenic macrophages, resulting
in thrombocytopenia.

9. Classify ITP [most common cause in kids and adults].


- Acute form (in children)
- arises weeks after a viral infection or immunization.
- Self-limited disorder - resolves within weeks of presentation
- Chronic form (in adults)
- arises usually in women of childbearing age.
- May be primary (idiopathic) or secondary (lupus)
- May cause short-lived thrombocytopenia in offspring since
antiplatelet IgG can cross the placenta.

11. What are the Lab findings.


- ↓ Platelet count often < 50 k/μl.
- Normal PT/PTT - Coagulation factors are not affected.
- ↑ megakaryocytes on bone marrow biopsy (Hyperplasia)

12. What are treatment choices of ITP?


Corticosteroids - Children respond well
(initial treatment) - Adults may show early response, but often relapse.
IVIG (Intravenous - used to raise the platelet count in symptomatic bleeding
immunoglobulins) - effect is short-lived (splenic φ phagocytose the IVIG).
Splenectomy (in - Eliminates the primary source of antibody production and the site of
refractory cases) platelet destruction
Microangiopathic hemolytic anemia

11. What is microangiopathic hemolytic anemia ?


- Pathologic formation of platelet microthrombi in small vessels.
- Platelets are consumed in the formation of microthrombi (leading to Thrombocytopenia).
- RBCs are "sheared" as they cross microthrombi, resulting in hemolytic anemia with
schistocytes (aka helmet cells).
- Seen in:
=> thrombotic thrombocytopenic purpura (TTP)
=> hemolytic uremic syndrome (HUS)

Fig - schistocyte - the two pointy things at the end is classic


appearance of cut RBC

15. What are clinical findings of microangiopathic hemolytic anemia ?


- Skin and mucosal bleeding
- Fever
- Renal insufficiency (more common in HUS) - Thrombi involve vessels of the kidney.
- CNS abnormality (more common in TTP) - Thrombi involve vessels of the CNS.

16. Labs findings of microangiopathic hemolytic anemia.


- Thrombocytopenia with ↑ bleeding time
- Normal PT/PTT (coagulation cascade is not activated)
- Anemia with schistocytes
- ↑ megakaryocytes on bone marrow biopsy

Thrombotic thrombocytopenic purpura (TTP)

12. What is thrombotic thrombocytopenic purpura (TTP) ?


- Cause: decreased ADAMTS13 (due to acquired autoantibodies), an enzyme that normally
cleaves vWF multimers into smaller monomers for eventual degradation.
- Large, uncleaved multimers lead to abnormal platelet adhesion, resulting in microthrombi.
- Most commonly seen in adult females
- Treatment: Plasmapheresis (removes Ab against ADAMTS13) and corticosteroids.

Hemolytic uremic syndrome (HUS)

13. What is hemolytic uremic syndrome (HUS) ?


- Cause: Endothelial damage [of Kidney] by drugs or infection (E coli verotoxin).
- Classically seen in children with E coli 0157:H7 dysentery (bloody diarrhea)
- Associated with foodborne illness or from a contaminated water supply
- This mainly leads to acute kidney failure (uremia) and effects CNS.
Qualitative disorders

Inherited [AR] disorder

Bernard-Soulier syndrome

18. What is bernard soulier syndrome ?


- GPIb deficiency hence platelet adhesion is impaired.
- Blood smear shows mild thrombocytopenia with enlarged platelets.

Glanzmann thrombasthenia

19. What is Glanzmann thrombasthenia?


- GPIIb/IIIa deficiency hence platelet aggregation is impaired.

Fig: arrow points to big platelets in Bernard


Soulier syndrome

Acquired disorder

Drug mediated disorder

19. What is Drug mediated disorder ?


- Aspirin irreversibly inactivates cyclooxygenase
- lack of TXA2 impairs platelet aggregation.

Chronic renal failure


20. What is the role of CRF in platelet disorder ?
- Uremia disrupts platelet function
- Both platelet adhesion and aggregation are impaired.
Secondary Hemostasis and related clotting disorders

1. What is secondary hemostasis ?


• Secondary hemostasis stabilizes the weak platelet plug via the coagulation cascade.
• Coagulation cascade generates thrombin (factor IIa). Thrombin converts fibrinogen in
the platelet plug to fibrin. Fibrin (factor Ia) is then cross-linked, yielding a stable
platelet-fibrin thrombus meshwork.

4. Describe Clotting factors.


• Factors of the coagulation cascade are produced by the liver in an inactive state.
• Activation of clotting factors requires:
- Exposure to an activating substance
i. Tissue thromboplastin activates factor VII (extrinsic pathway).
ii. Subendothelial collagen activates factor XII (intrinsic pathway).
- Phospholipid surface of platelets.
- Calcium (derived from platelet dense granules).

1. Describe clotting patways.


• Intrinsic pathway - Measured by PTT / APTT lab test.
- PTT is used to Monitor the treatment effects with heparin.
- Kaolin suspension is the key ingredient.
• Extrinsic pathway - Measured by PT lab test.
- PT is used to Monitor the treatment effects with warfarin.
- Thromboplastin powder is the key ingredient.

DISORDERS OF Secondary HEMOSTASIS

2. What causes disorders of secondary hemostasis?


• They occur due to factor abnormalities.

3. What are the clinical presentations ?


• Deep tissue bleeding into muscles and joints (hemarthrosis).
• Rebleeding after surgical procedures (e.g., circumcision and wisdom tooth extraction)

3. What are the Lab findings ?


• Prothrombin time (PT) - measures extrinsic (factor VII) and common (factors
II, V, X, and fibrinogen) pathways of the coagulation cascade.
• Partial thromboplastin time (PTT) - measures intrinsic (factors XII, XI, IX, VIII)
and common (factors II, V, X, and fibrinogen) pathways of the coagulation
cascade
Hemophilia A

9. What causes hemophilia A ?


• Factor 8 (FVIII) deficiency
- X-linked recessive disorder (predominantly affects males).
- Can arise from a new mutation (de novo) without any family history.

9. What are the clinical presentations ?


• Deep tissue, joint, and postsurgical bleeding
• Clinical severity depends on the degree of deficiency.

10. What are lab findings ?


• ↑ PTT (↓ FVIII)
• Normal PT
• Normal platelet count and bleeding time

11. How is the Treatment ?


• Administration of recombinant factor VIII.

Hemophilia B [Christmas disease]

11. Describe hemophilia B ?


• Factor 9 (FIX) deficiency
- X-linked recessive disorder
• Resembles [clinically and Lab values] hemophilia A.

Coagulation factor inhibitor disease [aka Acquired Hemophilia]

11. What is coagulation factor inhibitor disease ?


• Acquired antibody against a coagulation factor resulting in impaired factor function.
• Anti-Factor VIII antibody is most common.
• Clinical and lab findings are similar to hemophilia A.
• PTT does not corrected (as the antibodies will destroy new factor 8) upon mixing
normal plasma (pt gets some normal factor 8) with patient's plasma (mixing study)
due to inhibitor [PTT does correct in hemophilia A as there is no auto-Ab].
Von Willebrand disease

15. What is Von Willebrand disease ?


• Most common inherited coagulation disorder.
• Genetic deficiency of vWF factor.
• Low vWF impairs platelet adhesion.

15. Classify Von Willebrand disease.


• Multiple subtypes exist causing quantitative and qualitative defects
• Autosomal dominant disorder (most common type)

16. What are the clinical presentation of vWF disease ?


• Mild mucosal and skin bleeding.
• No secondary hemostasis disease symptoms are seen.

17. What are the lab values?


• ↑ bleeding time
• ↑ PTT - Decreased FVIII half-life (vWF normally stabilizes Factor 8)
• Normal PT
• Abnormal ristocetin test - Ristocetin induces platelet agglutination by
causing vWF to bind platelet GPIb receptor.

19. What is ristocetin test?


• The antibiotic ristocetin causes von Willebrand factor to bind the platelet receptor
glycoprotein Ib (GpIb), so when ristocetin is added to normal blood, it causes agglutination.
DISEASE Ristocetin drug + pt blood
vWF disease (vWF deficiency) - No agglutination
Bernard-Soulier (Gly Ib deficiency) - No agglutination
Glanzmann thrombasthenia - Yes agglutination
(Gly IIb/IIIa deficiency)

20. What is the treatment for vWF disease?


• Desmopressin (ADH analog) - it increases vWF release from
Weibei-Palade bodies (primary storage) of endothelial cells.


Vitamin K deficiency

21. What is the function of vit K in coagulation cascade?


• Vitamin K is activated by epoxide reductase in the liver.
• Warfarin [coumadin] blocks epoxide reductase - Vitamin K antagonist.
• Vitamin K is necessary for gamma carboxylation of:
- factor 2,7,9,10.
- protein c and s.
• IMPORTANT REACTIONS:
i. Oxidised Vit. K (Vit. K Epoxide) + epoxide reductase -----> Reduced Vit. K (Vit.K hydroquinone)
ii. Reduced Vit. K (Vit.K hydroquinone) + gamma gutamyl carboxylase + inactive clotting factors
------> Oxidised Vit. K (Vit. K Epoxide) + activated clotting factors

22. What causes vitamin K deficiency?


• Newborns - due to lack of Gl colonization by bacteria that normally synthesize
vitamin K. Vitamin K injection is given prophylactically to all newborns at birth
to prevent hemorrhagic disease of the newborn.
• Long-term antibiotic therapy - disrupts vitamin K-producing bacteria in the GI tract.
• Malabsorption - leads to deficiency of fat-soluble vitamins, including vitamin K.

Liver failure

23. Why does liver failure lead coagulation disease?


• Liver produces coagulation factors
• Epoxide reductase is found in liver.

24. How to check the coagulative functions in liver failure pt ?


• By measuring their PT time.

Large-volume transfusion

25. Why does large volume transfusion lead to secondary coagulation problems ?
• Because it dilutes the coagulation factors resulting in a relative deficiency.
Other Disorders of Hemostasis

Heparin induced thrombocytopenia [HIT]

1. How does heparin induce thrombocytopenia ?


- Heparin (hapten) binds to platelet factor 4 (cytokine) forming a complex.
- IgG Ab [HIT antibodies] binds this complex in the bloodstream.
- The tail of the Ab then binds to the FCGR2A receptor on platelets.
- This results in platelet activation and the formation of platelet microparticles
[Platelet destruction - Fragments of destroyed platelets] which activates
remaining platelets leading to thrombosis
- This results in thrombocytopenia (low platelet count).

2. What drug is risky to give to patients with heparin induced thrombocytopenia?


- Warfarin (thrombolytic drug) is contraindicated.
- MOA - Warfarin inhibits protein C [anti-clot forming factor] that has a shorter half-life.
- Therefore, Warfarin acts as procoagulant initially which leads to microthrombi in penis,
skin etc leading to tissue necrosis.
Disseminated intravascular coagulation

3. What is disseminated intravascular coagulation disorder (DIC) ?


- Pathologic activation of coagulation cascade.
- Results in Widespread microthrombi leading to Consumption of
platelets (thrombocytopenia) and factors.

3. What are the clinical presentation ?


- Ischemia and Infraction.
- Bleeding, especially from IV sites and mucosal surfaces (bleeding from
body orifices - Ex: gut, mouth).

4. What are some causes of disseminated intravascular coagulation disorder (DIC)?


- This disease is always secondary to another disease process:
○ Obstetric complication - Tissue thromboplastin in the amniotic fluid activates factor 7
in mother (A.fluid leakage) which leads to coagulation cascade activation.
○ Sepsis (especially with E Coli or N meningitidis) - Endotoxins from the bacterial wall and
cytokines (e.g., TNF and IL-l) produced by macrophages induce endothelial cells to make
tissue factor (Factor VII) thus activating coagulation cascade. Endotoxins also activate
○ Hageman factor (Factor XII) leading to coagulation cascade activation.
○ Adenocarcinoma - Mucin from adenocarcinoma can activate coagulation cascade. Acute
promyelocytic leukemia - Cells are loaded with primary granules that can coalesce to
make hour rods (classic histologic feature). Primary granules can enter circulation and
○ activate coagulation cascade.
Rattlesnake bite - Venom can activate coagulation cascade

5. What are the lab findings in DIC ?


- ↓ platelet count
- ↑ PT and PTT
- ↓ fibrinogen
- Microangiopathic hemolytic anemia
- ↑ D-dimer (Derived from splitting of cross-linked fibrin) - best screening test

6. What is the treatment for DIC ?


- Treating the underlying cause.
- Transfuse blood products and cryoprecipitate (contains coagulation factors).
Hyperactive plasmin [Disorder of Fibrinolysis]

9. Explain normal phyiology of plasmin ?


- Normal fibrinolysis removes thrombus after damaged vessel heals.
- Tissue plasminogen activator (tPA) converts plasminogen to plasmin.

7. Mention the functions of plasmin.


○ dissolves the cross-linked fibrin
○ cleaves and destroys serum fibrinogen
○ destroys coagulation factors
○ blocks platelet aggregation.

8. What are the causes of hyperactive plasmin ?


- Radical prostatectomy - Release of urokinase (activates plasminogen).
- Cirrhosis of liver - Reduced production of α2-antiplasmin (inhibits plasmin).

9. What is clinical presentation of hyperactive plasmin?


- Increased bleeding (resembles DIC)

10. What are the lab findings of hyperactive plasmin ?


- ↑ PT and PTT [Plasmin destroys coagulation factors].
- ↑ bleeding time with normal platelet count
- ↑ fibrinogen split products without D-dimers [Serum fibrinogen is lysed;
however, D-dimers are not formed because fibrin thrombi are absent
hence there is no fibrin to cleave]

11. How do you treat hyperactive plasmin conditions?


- Aminocaproic acid blocks the activation of plasminogen.
Thrombosis

1. What is thrombosis ?
- Pathologic intravascular blood clot (thrombus) that can form in any artery or vein.
- Most common site is deep veins (DVT) of leg below the knee.

2. What are the characteristics of thrombus (which distinguish it from postmortem clot) ?
- Line of zahn (alternating layers of platelets/fibrin and RBCs).
- Attachment to vessel wall.

Fig - lines of Zahn (the alternate layer of RBC and fibrin -


marked by R & f) is characteristic of thrombosis

3. What are the 3 main risk factor of thrombosis (Virchow's triad)?


- Disruption of blood flow
- Endothelial cell damage
- Hypercoagulable state

Disruption in normal blood flow

1. Explain Disruption in normal blood flow ?


- Stasis and turbulence of blood flow increases risk for thrombosis.
- Blood flow is normally continuous and laminar; keeps platelets and factors
dispersed and inactivated.

1. Give some examples.


- Immobilization - increased risk for deep venous thrombosis
- Cardiac wall dysfunction (e.g., arrhythmia or myocardial infarction)
- Aneurysm
Endothelial cell damage

1. Explain endothelial cell damage ?


- Endothelial damage disrupts the protective function of endothelial cells,
increasing the risk for thrombosis

4. How does intact endothelium prevents thrombosis ?


- Blocks exposure to subendothelial collagen and underlying tissue factor from
coagulation factors and platelets.
- Produce prostacyclin (PGI2) and NO - vasodilation and inhibition of platelet
aggregation
- Secrete heparin-like molecules (HLM) - augment antithrombin 3 (ATIII), which
inactivates thrombin and coagulation factors
- Secrete tissue plasminogen activator (tPA) - converts plasminogen to plasmin
- Secrete thrombomodulin - redirects thrombin to activate protein C which
inactivates factors V and Vlll

1. What causes endothelial cell damage ?


- Atherosclerosis.
- Vasculitis.
- High levels of homocysteine.

1. What causes Mildly elevated homocysteine levels ?


- Vitamin B12 and folate deficiency:
○ Folic acid (tetrahydrofolate, THF) circulates as methyl-THF in the serum.
○ Methyl is transferred to cobalamin (vitamin Bl2), allowing THF to participate in the
synthesis of DNA precursors.
○ Cobalamin transfers methyl to homocysteine resulting in methionine.
○ Lack of vitamin Bl2 or folate leads to decreased conversion of homocysteine to
methionine resulting in buildup of homocysteine.

1. What causes high homocysteine levels with homocystinuria ?


- Cystathionine beta synthase (CBS) deficiency:
○ CBS converts homocysteine to cystathionine; enzyme deficiency leads
to homocysteine buildup.
○ Characterized by
=> vessel thrombosis (one-fourth pt. die in young age due to thrombosis)
=> mental retardation
=> lens dislocation
=> long slender fingers
Hypercoagulable state

8. What causes hypercoagulable state ?


- Due to excessive defective anticoagulant proteins or procoagulant proteins.
- It may be inherited or acquired

8. What are the classical presentations ?


- Recurrent DVTs or DVT at a young age [Usually occurs in the deep veins of the leg].
- Other sites include hepatic and cerebral veins.

8. What causes excessive defective anticoagulant proteins ?


- Protein C or S deficiency (autosomal dominant) - decreases negative feedback on the
coagulation cascade:
○ Proteins C and S normally inactivate factors V and Vlll.
○ Pt are at Increased risk for warfarin skin necrosis.
=> Initial stage of warfarin therapy results in a temporary deficiency of proteins C & S
(due to shorter half-life) relative to factors II, VII, IX, and X.
=> In preexisting C or S deficiency, a severe deficiency is seen at the onset of warfarin
therapy increasing risk for thrombosis, especially in the skin.

11. What causes excessive procoagulant proteins ?


- Factor V Leiden (autosomal dominant) - mutated form of factor V that lacks the cleavage
site for deactivation by proteins C and S.
○ Most common inherited cause of hypercoagulable state.
- Prothrombin G20210A - inherited point mutation in prothrombin that results in increased
gene expression [The "G20210A" refers to the fact that the mutation is a guanine (G) to
adenine (A) substitution at position 20210 of the DNA of the prothrombin gene]:
○ Increased prothrombin results in increased thrombin, promoting thrombus formation.
- ATIII deficiency - decreases the protective effect of heparin-like molecules produced by the
endothelium, increasing the risk for thrombus.
○ Heparin-like molecules normally activate ATIII, which inactivates thrombin & coagulation
factors.
○ In ATIII deficiency, PTT does not rise with standard (normal) heparin dosing:
=> Pharmacologic heparin (anticoagulant) works by binding and activating ATIII.
=> High doses of heparin activate limited ATIII thus increasing PTT a bit.
=> Warfarin is then given to maintain an anticoagulated state.

14. Why are oral contraceptives associated with hypercoagulable state?


- Estrogen induces increased production of coagulation factors.
- Oral contraceptives usually work by increasing estrogen levels.
Embolism

1. What is embolism ?
- Intravascular mass that travels and occludes downstream vessels.
- Symptoms depend on the vessel involved.

1. What are the types of embolism ?


- Systemic embolism
- Pulmonary embolism
- Cholesterol embolism
- Fat and marrow embolism
- Air embolism
- Amniotic fluid embolism
- Septic embolism
- Tumor embolism
- Foreign body embolism

1. What is Systemic embolism ?


- Usually occurs (80%) due to thromboembolism
- Most commonly arise due to left ventricular wall infarct (mostly associated with
intracardiac mural thrombi) - thrombus dislodges.
- Travel down systemic circulation to occlude flow to organs, most commonly the
lower extremities (75%) or the brain (10%).

1. What is Pulmonary embolism ?


- Mostly occurs (90%) due to thromboembolism
- Most common source is deep venous thrombus (DVT) of the lower extremity, usually
involving the femoral, iliac, or popliteal veins.
- Most often clinically silent because
○ the lung has a dual blood supply (via pulmonary and bronchial arteries)
○ the embolus is usually small (self resolves)
- Presentations:
○ shortness of breath
○ hemoptysis
○ pleuritic chest pain
○ pleural effusion
- Diagnosis:
○ V/Q lung scan shows mismatch; perfusion is abnormal.
○ Spiral CT shows a vascular filling defect in the lung.
○ Lower extremity Doppler ultrasound is useful to detect DVT.
○ D-dimer is elevated.
○ Gross examination reveals a hemorrhagic, wedge-shaped infarct.
- Pulmonary infarction occurs if a large- or medium-sized artery is obstructed in patients
with pre-existing cardiopulmonary compromise [only 10% of PEs cause infarction].
- Sudden death occurs with a large saddle embolus that blocks both left and right
pulmonary arteries or with significant occlusion of a large pulmonary artery. Death is due
to electromechanical dissociation.
- Pulmonary hypertension may arise with chronic emboli that are reorganized over time.
2. What is Cholesterol embolism ?
- It occurs due to an atherosclerotic plaque that dislodges.
- Characterized by the presence of cholesterol clefts in the embolus

Fig - presence of cholesterol crystal (three white columns) indicates that this embolus is
artherosclerotic and not thrombotic.

3. What is fat embolus ?


- It is associated with bone fractures, particularly long bones, and soft tissue trauma.
- Develops while fracture is still present or shortly after repair
- Clinical presentation:
○ dyspnea (fat, often with bone marrow elements, is seen in pulmonary vessels)
○ petechiae on the skin overlying the chest

Fig - fat embolus

4. What is air embolus ?


- Causes:
○ Nitrogen gas precipitates out of blood due to rapid ascent by a diver.
○ during laparoscopic surgery (air is pumped into the abdomen).
- Clinical Presentation [Decompression sickness]:
○ Bends - joint and muscle pain
○ Chokes - respiratory distress
○ Caisson disease (Chronic form) - multi focal ischemic necrosis of bone.
6. What is Amniotic fluid embolism ?
- Amniotic fluid or fetal tissue (squamous cells and keratin debris from fetal skin)
enters maternal circulation during labor or delivery via tear in placental membrane
or rupture of uterine veins.
- Presentation:
○ shortness of breath
○ neurologic symptoms
○ DIC (amniotic fluid contains tissue thromboplastin)

Fig - amniotic fluid embolus in a blood vessel near alveoli. Note presence of keratin debris
(squiggly lines) inside the blood vessel.
Chapter 5: Red Blood Cell Disorders
CMAP summary

RBC disorders Page 1.1


5.1 Anemia

1. What is anemia? What’s its presentation?


- Reduction in total circulating RBC mass
- Presentation: signs and symptoms of hypoxia.
o Pale conjunctiva and skin
o Weakness, fatigue, dyspnea
o Headache and lightheadedness
o Angina, especially with preexisting CAD.

2. How is anemia measured?


- Hb level, hematocrit (Hct) and RBC count (but total RBC mass difficult to measure).
- All of these measures are concentration dependent so have problems. Ex – in pregnancy, blood volume
increases making Hb and Hct concentration low even though total amount might be same. Immediately
after gunshot wound and blood loss, Hb and Hct concentration might be normal even though pt might
have lost lots of blood.

3. What is practical definition of anemia?


- Males: Hb <13.5 g/dl [normal Hb is 13.5-17.5 g/dL].
- Females: Hb <12.5 g/dl [normal Hb is 12.5-16.0 g/dL].
[lower for females because of menstruation].

4. What are different types of anemia?


Based on mean corpuscular volume (MCV)
- Microcytic (MCV <80 μm3) – small RBC
- Normocytic (MCV = 80-100 μm3) – normal size RBC
- Macrocytic (MCV > 100 μm3) – big RBC

Laboratory measurements of iron status


1. Serum iron – measure of iron in the blood.
2. Total iron – binding capacity (TIBC) -measure of transferrin molecules in the blood.
3. %saturation – % of transferrin molecules that are bound by iron (normal is 33%
- 1 in every 3 transferrin in blood is bound to Fe).
4. Serum ferritin – reflects iron stores in macrophages and the liver.
5. RBC distribution width (RDW) – measures standard deviation of size of RBC
6. Free erythrocyte protoporphyrin (FEP) – unbound protoporphyrin conc. in RBC.

Normal level
Serum iron – 100 μg/dl
TIBC – 300 μg/dl

RBC disorders Page 2.1


5.2 Microcytic Anemia

1. What is pathophysiology of microcytic anemia?


- Decreased production of Hb [A decrease in any of the components of Hb leads
to microcytic anemia].
- A. RBC progenitor cells in the bone marrow are large and normally divide multiple
times to produce smaller mature cells.
B. Microcytosis is due to an "extra" division which occurs to maintain hemoglobin
concentration in RBC.
[RBC is produced from subsequent division of erythroblast (EB). During Hb deficiency, EB divides
too much. As a result, RBCs become small and microcytic anemia occurs. Think that by dividing
extra, RBC surface area exposed to blood increases and it can carry more O2 – not correct idea
but works for thinking] - There is variability in size of RBC and increased pallor in center of RBC.

2. What is hemoglobin made up of?


- Hemoglobin = heme + globin (protein).
- Heme = Iron + protoporphyrin.

3. What are etiologies/types of microcytic anemia ?


- Iron deficiency anemia – Fe deficiency.
- Anemia of chronic disease [In chronic inflammation, Fe is stored away in
macrophage and can’t be used].
- Sideroblastic anemia – protoprophyrin deficiency.
- Thalessemia – decreased production of globin.
5.2 Microcytic Anemia (Part 1) - Fe Deficiency

4. What is the Etiology of Fe deficiency anemia.


- Due to decreased levels of iron
↓ iron ---> ↓ heme ---> ↓ hemoglobin ---> microcytic anemia.

4. What is epidemiology Fe deficiency anemia.


- Fe deficiency is the most common nutritional deficiency in the world making this
the most common type of anemia (1/3rd of world is deficient in Fe).

5. Describe digestion and storage of Fe.


Iron is consumed in heme (meat-derived) and non-heme (vegetable-derived) forms.
1. Absorption occurs in the duodenum. Enterocytes have heme and non-heme
(DMT1) transporters; the heme form is more readily absorbed.
2. Enterocytes transport iron across the cell membrane into blood via ferroportin.
3. Transferrin transports iron in the blood and delivers it to liver and bone marrow
macrophages for storage.
4. Stored intracellular iron is bound to ferritin, which prevents iron from forming
free radicals via the Fenton reaction.
5. There is no real way to get rid of Iron from body. So absorption by enterocytes
is regulated (some lost during skin sloughing off and menstruation).

10. What are lab findings of Fe deficiency anemia?


- 1. Microcytic, hypochromic RBCs with ↑ red cell distribution width.
2. ↓ Serum Fe --> ↓ cellular Fe --> regulatory ↓ apo-ferritin + ↑ Transferrin
receptor --> ↓ ferritin.
3. ↓ Fe stores --> regulatory ↑ apo-transferrin --> ↑ TIBC; ↓ % saturation.
4. ↑ Free erythrocyte protoporphyrin (FEP).

7. What are some etiologies of Fe deficiency anemia?


- Malabsorption and Malnutrition
• Celiac disease.
• Gastrectomy – (Fe2+ is absorbed more readily than Fe3+. Acidic environment
promotes Fe2+ conformation. When stomach is resected, due to lost acidity,
more Fe will be in Fe3+ form and Fe won’t be absorbed well in DUODENUM).
- Dietary lack or blood loss
• Infants – breast feeding (human milk is low in iron)
• Children – poor diet
• Adults (20-50 years) –
▪ peptic ulcer disease in males.
▪ menorrhagia/pregnancy in females.
• Elderly –
▪ colon polyps/carcinoma in Developed world.
▪ hookworms in developing world.
--> Ancylostoma duodenale and Necator americanus.
8. What are stages of iron deficiency anemia?
1. Storage iron is depleted - ↓ ferritin, ↑ TIBC.
2. Serum iron is depleted - ↓ serum iron; ↓ % saturation
3. Normocytic anemia – Bone marrow makes fewer, but normal-sized, RBCs.
4. Microcytic, hypochromic anemia – Bone marrow makes smaller and fewer RBCs.

9. What are clinical presentation of iron deficiency anemia?


- Anemia.
- Koilonychia (spoon shaped nails).
- Pica (psychological drive to eat dirt – perhaps to get Fe).

12. What is blood smear finding in Fe deficiency anemia?


- Poikilocytosis (variable shapes)
- Anisocytosis (variable size)
- cigeratte shaped RBC (classic finding)
- tear drop shaped RBC
- Microcytic anemia (note RBC smaller than lymphocyte nucleus)
- central pallor in RBC is large.

14. What is Plummer-Vinson syndrome?


- Iron deficiency anemia with esophageal web + atrophic glossitis
Presents as: anemia, dysphagia, and beefy-red tongue (smooth
glossy tongue due to complete atrophy of the lingual papillae).

13. What is treatment of Fe deficiency anemia?


- Iron supplement – Ferrous sulfate
5.2 Microcytic Anemia (Part 2) - Anemia of Chronic Disease

1. What is epidemiology of anemia of chronic disease (ACD)?


- Most common anemia in hospitalized pt.
- ACD is seen in association with:
--> Chronic inflammation (endocarditis or autoimmune conditions).
--> Cancer.

2. What is pathophysiology of ACD?


- Chronic disease results in production of acute phase reactants from the liver (i.e hepcidin).
↑ Hepcidin causes Anemia by sequestering iron in storage sites - internalization of
ferroportin --> limiting iron transfer from MΦ to erythroid precursors --> ↓ Erythropoiesis.
- ↑ intracellular Fe --> degrades HIF --> ↓ Erythropoietin (EPO) production in kidney.
- ↓ available iron --> ↓ heme --> ↓ hemoglobin --> Microcytic anemia.
- Evolutionary advantage of hepcidin is that bacteria need Fe to grow and flourish.
presumably to keep iron away from pathogens that require it for their metabolism.
aim is to prevent bacteria from accessing iron, which is necessary for their survival.

3. What are lab findings in Anemia of chronic disease?


- 1. Inflammation (IL-6) --> stimulates HAMP gene ---> ↑ Hepcidin --> inhibits
Ferroportin in MΦ, Enterocyte, Hepatocyte --> ↓ release of Fe in circulation
from intracellular stores --> ↓ serum iron and ↑ ferritin.
2. ↑ Fe stores --> regulatory ↓ apo-transferrin --> ↓TIBC; ↓% saturation
3. ↑ FEP (free erythrocyte protoporphyrin).

6. What are lab difference between Fe deficiency anemia and ACD?


Fe deficiency Anemia of chronic Hemo-chromatosis / Pregnancy/ Oral
anemia disease Sideroblastic anemia contraceptive pill (OCP)
Serum Fe ↓ ↓ ↑ normal

Ferretin ↓ ↑ ↑ ↓

Transferrin ↑ ↓ ↓ ↑
/TIBC
% saturation ↓ ↓ ↑ ↓

Note:
i] Hemochromatosis — more serum Fe --> Iron overload in tissues
ii] Pregnancy — more Fe demand by baby --> decreases Fe stores in mom.
OCP — estrogen stimulates transferrin synthesis by liver.

4. What are stages of ACD?


- Normocytic anemia is followed by microcytic anemia (same as Fe deficiency anemia).

5. What is treatment of ACD?


- Treat underlying cause of chronic disease (to reduce hepcidin).
- give Exogenous erythropoietin (especially helpful in cancer pt).
5.2 Microcytic Anemia (Part 3) - Sideroblastic anemia

7. What is pathophysiology of sideroblastic anemia?


- Defective synthesis of protoporphyrin is main cause
↓ protoporphyrin --> ↓ heme --> ↓ hemoglobin --> microcytic anemia.
- Iron is transferred to erythroid precursors and enters the mitochondria to form heme.
If protoporphyrin is deficient, iron remains trapped in mitochondria.
A. Iron-laden mitochondria form a ring around the nucleus of erythroid precursors;
these cells are called ringed sideroblasts (hence, the term sideroblastic anemia)
B. Ringed sideroblasts can be seen in bone marrow biopsy (Prussian blue stain – marks Fe)

8. What are steps of heme synthesis?

- First and last three reactions take place inside mitochondria

Steps of heme synthesis.


Protoporphyrin is synthesized via a series of reactions.
1. Aminolevulinic acid synthetase (ALAS) converts succinyl CoA to
aminolevulinic acid (ALA) using vitamin B6 as a cofactor (rate-limiting step).
2. Aminolevulinic acid dehydratase (ALAD) converts ALA to porphobilinogen.
3. Additional reactions convert porphobilinogen to protoporphyrin.
4. Ferrochelatase attaches protoporphyrin to iron to make heme.

9. What are etiologies of sideorblastic anemia?


Congenital Acquired
X-linked Mutation of ALAS2 gene (most Alcoholism [mitochondrial poison: Et-0H (cytosol) -->
common cause of congenital sideroblastic Et-CHO (cytosol -> mito) --> Et-C00H (mito) -> ↓pH].
anemia) - rate limiting enzyme Lead poisioning (Pb inhibits ALAD and ferrocheletase)
Autosomal recessive defect of SLC25A38 Vit B6 deficiency (ALAS requires Vit B6 as cofactor)
protein (it transports glycine from cytosol – Isoniazid treatment for TB can cause Vit B6 deficiency.
into mito.)
10. What are clinical features of lead poisoning leading to sideroblastic anemia?
- Pt at old house with chipped paint at high risk
- Mnemonic LEAD:
o L - Lead lines on gingivae and metaphyses of long bones (aka Burton lines)
o E - Erythrocyte basophilic stippling and encephalopathy (lead inhibits rRNA degradation
causing RBCs to retain aggregates of rRNA seen as basophilic stippling)
o A - sideroblastic Anemia, Abdominal colic
o D - Dimercaprol and EDTA for treatment; Wrist and foot Drop
o Succimer used for chelation of Pb in kids - sucks to be kid that eats lead

Fig: From right to left - basophilic stippling; Burton lines on gum; metaphysis

11. What are lab findings in sideroblastic anemia?


- Lab findings based on Fe overload in erythroblast as they can’t attach to protoporphyrin
- Fe trapped in mito ---> becomes ringed sideroblast in BONE MARROW.
- As Fe increase in these RBC, it triggers free radical produced by Fenton reaction.
The damaged RBC is phagocytosed by bone marrow macrophages.
- Bone marrow MΦ (aka Monocyte) stores gets Iron overloaded --> ↑ferritin.
It also spills Fe in Circulation via ferroportin --> ↑serum Fe.
1. ↑ferritin; ↓TIBC.
2. ↑serum Fe; ↑% saturation.
It is an iron-overloaded state like Hemochromatosis.

11. What is the TXT of sideroblastic anemia?


- Pyrodoxine (Vit. B6 - cofactor for ALAS).
5.2 Microcytic Anemia (Part 4) - Thalassemia

1. What are normal globin molecule in hemoglobin?


Normal a- b- Developmental
Hb types subunit subunit stage

HbF 2α 2γ Fetal, persists for 6


(α2γ2) months after birth

HbA (α2β2) 2α 2β Adult (high level)

HbA2 (α2δ2) 2α 2δ Adult (low level)

1. What are the types of alpha-Thalassemias?


α-Thalassemias GENOTYPE Molecular genetics
Silent carrier −/α, α/α Gene deletion
(Genes are HBA1
α-Thalassemia trait −/−, α/α (Asian) - cis deletion
and HBA2 on Ch16)
−/α, −/α (black African) - trans deletion
'α' - gene intact
HbH disease −/− −/α '−' - gene deleted

Hydrops fetalis −/− −/−

1. What are the types of beta-Thalassemias?


β-Thalassemias GENOTYPE Molecular genetics
β-Thalassemia major Homozygous β-thalassemia Mainly Point mutations in β-globin gene (HBB gene) that
(β0/β0, β+/β+, β0/β+) lead to defects in the:
β-Thalassemia intermedia Variable 1) transcription of β-globin gene.
(β0/β+, β+/β+, β0/β, β+/β) 2) splicing of β-globin pre-mRNA
3) translation of β-globin mature-mRNA.
β-Thalassemia minor Heterozygous β-thalassemia
(β0/β, β+/β) 1. Promoter region mutations:
Reduce transcription by 75% to 80% ---> Leads to
β+ thalassemia.
2. Splicing mutations:
— removes normal splice site at intron ends --> leads to
β0 thalassemia.
— create ectopic splice site within an intron --> leads to
β+ thalassemia (most common cause).
3. Chain terminator mutations:
--> Blocks translation --> this leads to β0 thalassemia
(most common cause).
— creates new stop codon within an exon
β0 mutations - absent β-globin synthesis. — changes the reading frame of the exon
β+ mutations - reduced (but normal) β-globin synthesis.
2. How does thalassemia lead to microcytic anemia?
- Anemia is due to decreased synthesis of the globin chains of hemoglobin.
↓ globin ---> ↓ hemoglobin ---> ↓ microcytic anemia.

3. What are patients with thalassemia protected against?


- Autosomal recessive disorder — Inherited mutation
- carriers are protected against Plasmodium Falciparum malaria
(P. Falciparum requires host Hb for its nourishment).

5.2 Microcytic Anemia (Part 4) - alpha-Thalassemia

4. What causes alpha thalassemia? What chromosome is alpha gene located in?
- α thalassemia is is usually due to gene deletion.
- Normally, 4 alpha genes are present on chromosome 16.
- α-thalassemia = ↓ production of alpha globin chains.

5. What are subtypes of alpha thalassemia?


1 α-gene deleted 2 α-gene deleted 3 α-gene deleted 4 α-gene deleted

- Asymptomatic pt. - Mild anemia with slightly increased - Severe anemia - Lethal in utero
RBC count (Anemia---> Hypoxia --> (hydrops fetalis)
EPO release --> more RBC synthesis)

- Cis deletion classically seen in Asians - β-chains form tetramers - γ-chains form tetramers
(both deletions occur on the same (HbH - β4) that damage (Hb Barts - γ4) that damage
chromosome) RBCs. RBCs.
- Trans deletion classically seen in - HbH can be seen on - Hb Barts is seen on
Africans (one deletion occurs on electrophoresis. electrophoresis.
each chromosome) - HbH seen in adults. - Hb Barts seen in newborns.
- Cis deletion is worse than Trans - HbH has high affinity - Hb Barts has high
deletion; because cis deletion is for 02 --> no 02 supply affinity for 02 --> no 02
associated with increased risk of to tissue. supply to tissue.
severe thalassemia in offspring. - HbH is prone to oxidation
--> ppt. as intracellular
inclusions (Heinz body).
5.2 Microcytic Anemia (Part 4) - beta-Thalassemia

6. What causes beta thalassemia?


- β-Thalassemia is usually due to gene mutations (point mutations
in promoter or splicing sites).
- Two β genes are present on chromosome 11; mutations result in
absent (β0) or diminished (β+) production of the β-globin chain.
- β-thalassemia = ↓ production of beta globin chains.
- seen in individuals of African and Mediterranean descent .

7. What are subtypes of beta thalassemia?


β-Thalassemia minor (β/β+) β-Thalassemia major (β0/β0)
- The mildest form of disease. - The most severe form of disease.
- Pt is usually asymptomatic with ↑ RBC - Presents with severe anemia a few months after
count (Anemia---> Hypoxia --> EPO release birth [α tetramer (α4) is formed].
--> more RBC synthesis). - high HbF (α2γ2) at birth is temporarily protective.
- blood smear shows Microcytic, hypochromic - blood smear shows Microcytic, hypochromic RBC
RBC and target cells (central dark with target cells and nucleated RBC (Normoblast).
hemoglobinized area formed — the ratio of - α tetramer (α4) aggregate and damage RBCs --->
'RBC surface area' to 'RBC volume' is ineffective erythropoiesis and extravascular
increased disproportionally). hemolysis (removal of circulating RBCs by spleen).

- Hemoglobin electrophoresis findings: - Hemoglobin electrophoresis findings:


• Increased HbA2 to 5% (normal 2.5%). • Increased HbA2 and HbF.
• Increased HbF to 2% (normal 1%). • little or no HbA.
• Slightly decreased HbA.

8. What are target cells?


- Normally, hemoglobin is present mainly on edge of RBC giving it biconcave shape with
central pallor. In target cells, there are some hemoglobin in center giving central
darkness (like bull's eye target practice).
- MOA- due to reduced hemoglobin in edge of RBC, the membrane in center gets floppy
or loose and some hemoglobin comes to stay there.

9. Aplastic crisis with parvovirus B19 in β-Thalassemia major pts?


○ Risk of aplastic crisis with parvovirus B19 (parvovirus affects erythroid precursors and shuts
down RBC production. In normal person, shutting down of RBC production for a week or so won't
matter. For pt with beta thalassemia major, they can't afford even a single day of RBC production
loss. They depend on every drop of RBC)
9. What are presentation of beta thalassemia major?
○ Anemia due to Extravascular hemolysis ---> tissue hypoxia ---> ↑ EPO release from kidney
---> Increased erythropoisis ---> Massive erythroid hyperplasia (excessive count of erythroid
precursor cells in bone marrow) --->
1) marrow expansion in skull bones --> skull deformity ensues resulting in:
— the cranial bones (reactive bone formation leads to 'crewcut' appearance on x-ray)
— the facial bones ('chipmunk facies' -globular maxillae).
2) Bone marrow overcrowding --> no space left for hematopoiesis ---> Hematopoietic
stem cells enter the circulation --> migrate to liver and spleen.
— extramedullary hematopoiesis with hepatosplenomegaly.
○ Some additional points from robbins:
a) ineffective erythropoisis --> variation in size (anisocytosis) and shape (poikilocytosis).
Most erythrocyte die in bone marrow ---> contributes to iron overload.
b) hypoxia --> “stress” erythropoiesis --> The reticulocyte and Normoblast count is elevated
(they try to compensate for hypoxia by delivering 02 via their poor Hb if not RBC).
c) marrow expansion --> burgeoning marrow erodes existing cortical bone and induces new
bone formation --> “crewcut” appearanceon x-ray studies.
[elongated trabeculae in widened diploic space, seen as calcified spicules perpendicular
to bone surface, corresponding to a periosteal reaction to disturbed bone repair with
neoosteogenesis of the outer cranial table, marked calvarial thickening, external
displacement and thinning of the inner table].
d) ineffective erythropoiesis --> tissue hypoxia --> EPO act on bone marrow --> 'Stress'
erythropoiesis --> Erythroblasts release Erythroferrone --> inhibits expression of hepcidin
in liver --> Enterocyte ferroportin not inhibited --> increased dietary Fe absorption -->
leads toIron overload --> Fe deposits in heart & liver.

9. What are the Management of beta thalassemia major?


- TREATMENT:
1. Chronic blood transfusions are often necessary (healthy RBC improve anemia) ---> RBC gets
phagocytosed by Spleen after long time ---> increases Iron overload ---> risk of secondary
hemochromatosis [hence give Deferasirox - it chelates serum Fe].
2. Splenectomy for swollen spleen due to:
--> Extramedullary hematopoiesis in spleen.
--> Excess Extravascular hemolysis --> splenic Mφ hyperplasia.
3. Hematopoietic stem cell transplantation is the only therapy offering a cure.
NORMAL β-THALASSEMIA
Reduced β-globin synthesis,
with relative excess of α-globin Insoluble α-globin aggregate
HbA
(α2β2)
HbA

Normal erythroblast Abnormal erythroblast


α-globin
Few abnormal aggregate
red cells leave
Normal HbA

Hypochromic red cell


Normal red blood cells
Ineffective erythropoiesis
Most erythroblasts Extravascular hemolysis
die in bone marrow Destruction of
Dietary iron aggregate-containing
red cells in spleen

ANEMIA
Increased
iron Blood
absorption transfusions
Tissue hypoxia
Reduce
Erythropoietin
increase
Liver
Heart
Marrow expansion

Systemic iron overload


(secondary hemochromatosis)

Skeletal deformities

Figure 14-12  Pathogenesis of β-thalassemia major. Note that the aggregates of unpaired α-globin chains, a hallmark of the disease, are not visible in routinely 


stained blood smears. Blood transfusions are a double-edged sword, diminishing the anemia and its attendant complications, but also adding to the systemic 
iron overload. 
5.3 Macrocytic anemia

1. What is macrocytic anemia? What are it's causes?


- Anemia with MCV (mean corpuscle volume) >100 μm3.
- Causes:
○ Megaloblastic anemia: DNA synthesis is impaired.
 Folate deficiency  drugs (ex- MTX, 6-MP, Cytarabine)
 Vit B12 deficiency  drugs (ex- 5-FU)
 Orotic aciduria
○ non-Megaloblastic anemia: DNA synthesis unimpaired.
 Chronic Alcoholism
 Liver disease -↑ RBC membrane Ch
 Hypothyroidism
 RBC membrane disorder {target cells}
 Hypoxic conditions {Reticulocytes}

1. What causes anemia in Megaloblastic anemia?


- low red blood cell (RBC) count - RBC contain ample Hb and lack central pallor.
- Pancytopenia = anemia + thrombocytopenia + leukopenia [precursors undergo apoptsis]
EPO synthesis ---> Bone marrow is hypercellular with megaloblastic hyperplasia.
- Megaloblastic cells are also phagocytosed by bone marrow Mφ.
- Reticulocyte count is low in blood.
5.3 Macrocytic anemia - Megaloblastic anemia

2. Describe relationship between folate and Vit B12.


Folate and vitamin B12 are necessary for synthesis of DNA precursors.
1. Folate circulates in the serum as methyltetrahydrofolate (methyl THF); removal
of the methyl group allows for participation in the synthesis of DNA precursors.
2. methyl THF transfers Methyl group to vitamin B12 (cobalamin).
3. Vitamin B12 then transfers it to homocysteine, producing methionine.

3. What is presentation [CLASSIC FINDINGS] of Megaloblastic anemia?


Lack of folate or vitamin B12 impairs synthesis of DNA precursors.
1. Impaired division of Erythroid precursors leads to megaloblastic anemia.
Enlarged oval RBC (aka macroovalocyte) are also seen
- Megaloblastic changes are detected at all stages of erythroid development.
2. Impaired division of granulocytic precursors leads to hypersegmented neutrophils.
with >5 lobes (normal is 3-5 lobes).
3. Megakaryocytes have abnormally large and have bizarre multilobate nuclei.
4. Megaloblastic change is also seen in rapidly-dividing (e.g., intestinal epithelial cells).
Other non-specific findings:
5. Glossitis (inflammation of tongue - due to less turnover of tongue cells).
6. Cabot rings {remnants from a mitotic spindle} are seen on blood smear.

3. What happens in Megaloblastic anemia?


○ a) dUMP + methylene-THF ---> dTMP + dihydrofolate
b) dTMP + cellular kinases ---> dTTP
c) dTTP + DNA polymerase ----> DNA synthesis
○ dTMP cannot be generated in Megaloblastic anemia ---> DNA synthesis is
impaired ---> the cell cycle cannot progress from the G2 to the M stage --->
continuing cell growth without division (nuclear-cytoplasmic ratio drops due
to quick cytoplasmic development relative to nuclear development).

10. Describe absorbtion of Vit B12 in gut (HY)


- 1. Salivary gland enzymes (e.g., amylase) liberate vitamin B12, which is then bound
by R-binder (also from the salivary gland) and carried through the stomach.
2. Pancreatic proteases in the duodenum detach vitamin B12 from R-binder.
3. Vitamin B12 binds intrinsic factor (made by gastric parietal cells) in the small
bowel; the intrinsic factor-vitamin B12 complex is absorbed in the ileum.

Orotic aciduria
Defn - Inability to convert orotic acid to uridine monophosphate (UMP) that leads to
accumulation of orotic acid (defect in de-novo purine synthesis pathway)
- AR inheritence
Presentati - Megaloblastic anemia in children refractive to folate and vit B12
on - Failure to thrive, developmental delay
- Orotic acid in urine but no hyperammonemia
Treatment - UMP to pass the mutated enzyme
5. Compare dietary information of folate and vit B12.
Folate Vitamin B12
Diet green vegetables & some fruits Animal derived proteins
Absorption Jejunum Ileum
Deficiency Develops within months as body Takes years to develop due to large hepatic storage
stores are minimum (less common than folate deficiency)
Causes of - Age (commonly in elderly) Most common cause:
deficiency - Poor diet (alcoholics) - Pernicious anemia (autoimmune destruction of parietal cells
- Increased demand (pregnancy, in the body of stomach --> leads to intrinsic factor deficiency).
cancer, hemolytic anemia) Other causes:
- Folate antagonists
(methotrexate - inhibits DHFR) -
Pancreatic insufficiency: no Pancreatic protease.
- terminal ileum damage (Crohn disease): no IF receptor (cubulin)
- Diphyllobothrium latumm [fish tapeworm]: it absorbs ≥ 80% of
host dietary Vit. B12 intake in small gut.
- Using PPIs: ↓ HCL --> no pepsin --> unable to extract IF from
protein diet (i.e. meat).
- Vegans (dietary deficiency rare otherwise).

6. Compare lab findings of folate and Vit B12 deficiency.


Folate deficiency Vit B12 deficiency
- ↓ serum folate - ↓ serum vit B12
- ↑ serum homocysteine (high risk for thrombosis) - ↑ serum homocysteine (high risk for thrombosis)
a) m-THF + B12 --> THF + m-B12 a) m-THF + B12 --> THF + m-B12
b) m-B12 + homocysteine ---> methionine + B12 b) m-B12 + homocysteine ---> methionine + B12

- Normal methylmalonic acid - ↑ methylmalonic acid in Schwann cells.


a) m-Malonyl Coa + m-Malonyl Coa mutase --> succinyl coa
[enzyme co-factor — 5-deoxy-adenosyl-VitB12].
a) m-Malonyl Coa ---> m-malonate.
- Symptoms
 Subacute combined degeneration of the spinal cord
[m-malonate impairs spinal cord myelinization --> poor
proprioception and vibratory sensation (posterior column)
and spastic paresis (lateral corticospinal tract)].
 impairment of FA metabolism pathway
[odd chain FA --> propionyl Coa --> m-Malonyl Coa -->
succinyl Coa --> Krebs cycle].
5.4 Normocytic anemia

1. What is normocytic anemia? What are two types based on etiology?


 ↓ RBC count --> ↓ Hct; with normal-sized RBC (MCV = 80-100 μm3).
 Types: due to-
o Peripheral destruction of RBC (CRC >3% — good marrow response to EPO).
 Extravascular hemolysis — RBC destroyed by Spleen
 Intravascular hemolysis — RBC destroyed within blood vessel
o Underproduction of RBC (CRC <3% — suggests bone marrow defect).

2. What are reticulocytes?


 Seen as large cells with bluish cytoplasm (due to residual RNA) on blood smear.
 They are young (immature) RBC released from bone marrow to replace dead Erythrocyte
[Reticulocyte enters blood and circulate for 1 day before developing into mature RBC].
 Normal reticulocyte count (RC) is 1-2% of total RBC count
[RBC lifespan is 120 days; each day roughly 1-2% of RBCs are removed from circulation
and replaced by reticulocytes to compensate for 02 demand].
 Anemia --> hypoxia --> EPO --> Bone marrow (working) --> Reticulocytosis.
Bone marrow defects: 1) iron, folate, or B12 deficiency.
2) kidney failure (low EPO).

3. How can reticulocyte be falsely elevated in anemia? How is reticulocyte count corrected?
 In anemia, total RBC count goes down but Reticulocyte number remain same.
This falsely elevates reticulocyte count (RC) above normal range.
 It’s corrected by calculating Corrected reticulocyte count (CRC)
CRC = reticulocyte count x Hct/normal Hct.

Total RBC Total % of Hematocrit Corrected reticulocyte


Reticulocyte reticulocyte (Hct) count (CRC)
Normal pt. 100n 2n 2n/100n = 2% 45 2 x 45/45 = 2%

Anemic pt. 50n 2n 2n/50n = 4% 22.5 4 x 22.5/45 = 2%


(bad response)
Anemic pt. 50n 4n 4n/50n = 8% 22.5 8 x 22.5/45 = 4%
(good response)

2. What are Reticulocyte Index (RI)? How does it corrects CRC?


 No Anemia: normal reticulocyte loses their RNA within 1 day --> matures into RBC.
Severe Anemia: reticulocyte take 2-3 days to lose their RNA --> then matures in RBC.
 In Anemia, this has the effect of raising the reticulocyte count simply because reticulocytes
produced on any single day will spend more than 1 day in circulation as reticulocytes and,
therefore, will be counted for 2 or more days. This leads to overall increase in Reticulocyte
count and correspondingly decreased RBC count as Reticulocyte to RBC maturation is delayed.
 Correction of Reticulocyte count by finding Reticulocyte Index (RI)
RI = CRC/Maturation correction = CRC/2
 This is USED only when there is Polychromasia [high count of Immature marrow reticulocytes
(aka 'Shift' cell) in blood stream due to stress Erythropoiesis].
1. What happens in extravascular hemolysis?
• RBC destruction by the reticuloendothelial system (Mφ of the spleen, liver, and lymph nodes).
• Macrophages consume RBCs and break down hemoglobin.
a. Globin is broken down into amino acids.
b. Heme is broken down into iron and protoporphyrin; iron is recycled.
c. Protoporphyrin is broken down into unconjugated bilirubin, which is bound to
serum albumin and delivered to the liver for conjugation and excretion into bile.

2. What are lab and clinical finding of extravascular hemolysis?


• Anemia with splenomegaly.
• Jaundice due to ↑ unconjugated bilirubin.
• High risk for bilirubin gallstones
• Marrow hyperplasia with CRC >3%

3. What happens in intravascular hemolysis?


• RBC destruction within blood vessels ---> Hb and Heme leaks into blood.
• Haptoglobin binds Hb, while Hemopexin binds Heme.
[Hemoglobin/Heme is carried by serum Haptoglobin/Hemopexin and delivered to RE
system (mostly spleen). The Haptoglobin-Hb/Hemopexin-Heme complex will then be
completely removed by Mφs. There is no recycling of Haptoglobin/Hemopexin back
into circulation].
• Renal tubular cells (PCT) pick up some of the Hb that is filtered into the urine and break
it down into iron, which accumulates as hemosiderin; tubular cells are eventually shed
---> hemosiderin will be seen in urine ---> Hemosiderinuria.

2. What are lab and clinical finding of intravascular hemolysis?


• Presentation:
Immediate After few days
- Hemoglobin-emia - Hemosiderinuria
- Hemoglobin-uria
- ↓ serum haptoglobin
- ↓ serum hemopexin
5.5 Normocytic Anemia with Extravascular Hemolysis
Hereditary spherocytosis

1. What is the pathogenesis of hereditary spherocytosis?


- Inherited defect (AR/AD mutations) of RBC cytoskeleton-membrane tethering proteins-
integral proteins. Most commonly involves ankyrin, spectrin, or band 3.
[RBC cytoskeleton = Actin/Spectrin.
Membrane tethering protein = Ankyrin/Band 4.2/Band 4.1
Integral protein = Band 3/glycophorin A]
- RBC membrane fragility increase ---> blebs are formed and lost over time.
1. Loss of membrane renders cells round (spherocytes) instead of disc-shaped.
Loss of RBC volume leads to decreased Mean corpuscular volume (MCV).
[By basic geometry, a Sphere has least surface area to volume ratio].
2. Spherocytes are less able to maneuver through splenic sinusoids and are
consumed by splenic macrophages, resulting in anemia.

2. What are clinical and lab findings?


- Spherocytes with loss of central pallor.
- High RDW (the RBCs lose their membranes to different extent)
- ↑ MCHC (Hb conc. ≃ almost same while MCV decreases).
- Extravascular hemolysis findings
○ Anemia - RBC phagocytosis by splenic Mφ.
○ Splenomegaly (overworked spleen --> Mφ hyperplasia).
○ Jaundice (↑unconjugated bilirubin) w/ high risk for bilirubin gallstones.

3. What is one feared complication?


- Increased risk of aplastic crisis with parvovirus B19 infection of erythroid precursors
in the Bone marrow.

4. What is the diagnosis of hereditary spherocytosis?


- Osmotic fragility test - cells bursts in hypotonic solution very easily because cell
doesn't have much membrane to expand out as RBCs are already spherical.

5. What's it's treatment?


- Best TXT option is Splenectomy
1. Anemia resolves, but spherocytes persist.
2. Howell-Jolly bodies are seen on blood smear.
Fig: Howel Jolly bodies

5. What are Howell-Jolly bodies?


- Some RBC's are impefectly made with little nucleus or nuclear material left. It's job of
spleen to take them out or kill the defective RBC. Howell-Jolly bodies are RBC with
nuclear remnant (fragments of nuclear material). It indicates splenic dysfunction.
Howell–Jolly body {nuclear remnants - clusters of DNA} are seen inside RBCs.
Sickle cell disease
1. What causes sickle cell anemia?
• It’s caused due to mutation in B chain of hemoglobin that changes glutamic acid (hydrophilic) to
valine (hydrophobic). Think GingiVa - from Glutamic acid to Valine
• Disease is due to homozygous recessive mutation. Haterozygotes are protected against
plasmodium falciparum malaria
Phenotype Hemoglobin composition
Sickle cell disease (homozygous mutation) 90% HbS, 8% HbF, 2%HbA2, no HbA
Trait (one mutated and one normal B chain) 55% HbA, 43% HbS, 2% HbA2
• HbS – sickle cell hemoglobin (in α2β2 protein, both copies of β are mutated)

2. What is pathogenesis of sickle cell anemia?


• HbS polymerizes when deoxygenated (reversible). The polymers accumulate into needle shaped
structures and make RBC sickle cell.
• Sickling and de-sickling damages membrane leading to both intravascular and extravascular
hemolysis (spleen eats damaged RBC); sickled RBC cause vaso-occlusion; massive erythroid
hyperplasia to replace RBC.
• Sickling increases with hypoxemia, dehydration and acidosis.
• HbF protects against sickling. Kids protected for first few months of life.

3. What's treatment of sickle cell disease?


- Hydroxyurea - it increases level of HbF. It protects against sickling
4. What are presentations of sickle cell disease?
• Extravascular hemolysis – RBCs being sickle shaped and non-sickle cell repeatedly damages
membranes. Reticuloendothelial system removes these damaged RBC.
○ Anemia
○ Jaundice with unconjugated hyperbilirubinemia
○ Increased risk for bilirubin gallstones
• Intravascular hemolysis – due to membrane damage
○ Decreased haptoglobin
○ Hemoglobinemia, hemoglobinuria
○ Hemosiderinuria after few days
○ Target cells - hemoglobin leaks out due to membrane damage and extra membrane
produces target cells
• Massive erythroid hyperplasia (to compensate hemolysis and anemia):
○ Hematopoiesis in skull and facial bones (crewcut on X-ray and chipmunk face)
○ Extramedullary hematopoiesis (in liver, giving hepatomegaly - pt don't have spleen so don't
get splenomegaly)
○ Risk of aplastic crisis with parvo B19 infection

Fig: chipmunk face (left) and crewcut appearance


(right) seen in massive erythroid hyperplasia
• Extensive sickling leads to vaso-occlusion

5.What are some physical findings in sickle cell disease due to vaso-occlusion?
•All findings based on infraction
•Dactylitis – due to vasoocclusive infaracts in bones – common in infants
•Autosplenectomy – shrunken, fibrotic and calcified spleen
○ Increased risk of encapsulated organism infection (staph aureus, strep pneumo,
haemophilus influenza)
○ Salmonella paratyphi osteomyelitis (encapsulated) - most common cause of osteomyelitis
is staph aureus; in sickle cell, it's salmonella.
○ Howel-Jolly bodies on blood smear - nucleated RBC
• Acute chest syndrome (vaso-occlusion of pulmonary microcirculation)
○ Often precipitated by pneumonia
○ Presents with chest pain, SOB, lung infiltrates
• Pain crisis
• Renal papillary necrosis – presents as gross hematuria and proteinuria

Fig: vaso-occlusive complications of sickle cell disease - from left to right - autosplenectomy - small
calcified spleen; renal papillary necrosis; dactilytis; Howel-Jolly bodies
6. What’s the most common cause of death in sickle cell patients?
Kids Hemophilus influenza infection
Adults Acute chest syndrome

7. What is sickle cell trait?


• Haterozygote carriers of sickle cell mutation have sickle cell trait. They have one mutated and one
normal beta chain.
• HbS (both beta chain mutated) makes <50% of total hemoglobin because HbA is slightly more
efficiently made than HbS

8. What are presentations of sickle cell trait?


• Generally asymptomatic as RBC with <50% HbS don’t sickle
• Renal medulla problems:
○ Due to extreme hypoxia and hypertonicity in medulla, sickling occurs.
○ Presents as microscopic hematuria and decreased ability to concentrate urine due to
microinfraction of medulla.

9. What are lab findings in sickle cell disease and trait?


Sickle cell disease Sickle cell trait
Sickle cell and target cells seen in blood Don’t see sickle cell or target cells
smear
Metabisulfite screen +ve (cells with any Metabisulfite screen +ve
amount of HbS are sickled by the
screen)
• Confirm amount and presence of HbS with Hb electrophoresis
Sickle cell disease 90% HbS, 8% HbF, 2% HbA2, no HbA
Trait 55% HbA, 43% HbS, 2% HbA2
Hemoglobin C
1. What is hemoglobin C?
- Hemoglobin C is formed due to mutation in Beta chain of hemoglobin (autosomal recessive).
- Glutamic acid is changed to lysine (lyCne for hemoglobin C) - think Gingiva - Glutamic acid to
lyCine as gingiva is C shaped)
- Less common than sickle cell disease

2. What is presentation of hemoglobin C?


- Mild anemia due to predominant extravascular hemolysis
- HbC crystals on blood smear (HY)

Fig: HbC crystals characteristic of hemoglobin C (the rods)


5.6 Normocytic Anemia with Intravascular Hemolysis
Paroxymal nocturnal hemoglobinuria (PNH)

1. What causes paroxysmal nocturnal hemoglobinuria?


- Acquired defect in haematopoietic stem cells
- Mutation of PIG-A gene on X(p)-chromosome ---> defective PIG-A enzyme formed
[PIG-A = phosphatidyl-inositol glycan, class A] ---> no GPI anchor glycolipid made
[GPI = Glycosyl-phosphatidyl-inositol] ---> hence GPI-linked proteins cannot attach
to GPI anchor on lipid bilayer.

2. What is the normal protective mechanism?


- GPI-linked proteins block complement fixation on the surface of blood cells.
1] DAF (decay accelerating factor)/CD55 — prevents formation of C3 convertase.
2] MIRL (Membrane inhibitor of reactive lysis)/CD59 — prevents formation of MAC.
- In PNH, blood cells are susceptible to complement-mediated damage.
This leads to lysis of RBC, WBC and platelets --> they become deficient.

3. What PNH occurs is at night (nocturnal)?


- shallow breathing during sleep leads to Hypoventilation ---> accumulation of C02 in
blood leading to Hypercapnia ---> Mild respiratory acidosis develops ---> low pH
activates complement protein binding to blood cells ---> Intravascular hemolysis
occurs episodically more often at night ---> Dark urine seen early morning.

4. What is presentation of PNH ?


- Hemoglobin-uria and Hemoglobin-emia seen next morning after hemolysis.
- Hemosiderinuria is seen few days after hemolysis.

4. What is main cause of death in PNH?


- Thrombosis of the hepatic, portal, or cerebral veins — [MAO unclear]
1. destroyed platelets release cytoplasmic contents like CF-V, CF-XIII
---> Platelet activation ---> inducing thrombosis.
2. no CD59 ---> MAC complex on Platelet --> release of alpha-granules
---> Platelet activation ---> inducing thrombosis.

5. What are complications of PNH?


- Fe deficiency anemia (due to chronic loss of Hb in urine).
- Acute myeloid leukemia (seen in 10% of patients — in PNH there is a pool of mutated
Myeloid stem cell Clones in BM ---> ↑ risk of AML developemnt from these cells).

6. How is diagnosis of PNH made?


- Screening test: Sucrose Lysis test - RBC hemolysis in sucrose soln
(low-ionic-strength) containing serum.
- Confirmatory test:
a) Ham's acid hemolysis test - ↑ RBC fragility in acidified serum.
b) flow cytometry - detects lack of CD55 or CD59 on RBC.
Glucose-6-Phosphatase dehydrogenase (G6PD) deficiency

1. What is G6PD deficiency? What's it's pathophysiology


- X linked recessive disorder that results in defective G6PD enzyme.
- G6PD is first enzyme in pentose phosphate pathway.
- 1. RBCs are normally exposed to oxidative stress, in particular by H202.
2. Glutathione (an antioxidant) neutralizes H202, but becomes oxidized in the process.
3. NADPH (a by-product of G6PD) is needed to regenerate reduced glutathione.
4. ↓ G6PD --> ↓ NADPH --> ↓ reduced glutathione --> renders RBC susceptible to
oxidative stress by H202 --> intravascular hemolysis --> hemolytic anemia.

2. What is the normal Role of G6PD enzyme.


a) G-6-P + G6PD + (NADP+) ---> 6-P-gluconate + (NADPH + H+)
b) (NADPH + H+) + G-SS-G + Glutathione reductase ---> (NADP+) + 2G-SH
c) 2G-SH + H202 + Glutathione peroxidase ---> G-SS-G + 2H20

3. What are two major variants of G6PD deficiency?

African variant [Gd-A(-)] Mediterranean variant [Gd-Med]


- Mildly reduced t1/2 of G6PD. - Markedly reduced t1/2 of G6PD.
- Mild intravascular hemolysis with - Marked intravascular hemolysis
oxidative stress. with oxidative stress.
- prevalent in African-americans - prevalent in the Middle East.
- Mutation: band Xq28 (G→A) - Mutation: band Xq28 (C→T)

3. What is the protective role of G6PD deficiency?


- High carrier frequency in both populations is likely due to protective role
against falciparum malaria — Evolutionary advantage.
- P.falciparum cannot survive inside RBC due to oxidative stress leading to
death of the parasite.

4. What's histology finding of G6PD deficiency anemia?


- Heinz bodies - precipitation of denatured Hb due to oxidative stress.
e- from this Hb is transferred to 02 --> ROS generation --> intravascular hemolysis.
- Bite cells - removal of Heinz bodies from RBC margin by splenic Mφ.
denatured Hb were adhered to RBC membrane --> semicircular portion removed.
5. What are some causes of oxidative stress?
Non-genetic triggers of oxidative stress:
- bacterial or viral infection.
- drugs (e.g., primaquine, sulfa drugs, and dapsone).
- food: Fava beans.

6. What's presentation of G6PD deficiency anemia?


- Hemoglobinuria and lower back pain (Hb is nephrotoxic: Fe deposition in PCT
undergo fenton reaction --> ROS damage kidney --> referred pain) seen hours
after exposure to oxidative stress.

7. What is diagnosis of G6PD deficiency?


- Screening test: Heinz body preparation — ppt Hb seen w/ special Heinz stain.
- Confirmatory test: Enzyme studies — checks the deficiency of G6PD enzyme.
(performed weeks after hemolytic episode resolves - don’t do it during acute
phase because RBC lacking G6PD are already dead).

Pyruvate kinase (PK) deficiency

Genetics - Inherited mutation - autosomal recessive trait.

Pathophys - Mutation in the PKLR gene ---> deficiency in the PK enzyme ---> problem in
glycolysis ---> no ATP generation in RBC ---> Na/K pump can't work ---> K+ moves
out ---> intracellular hypotonicity ---> H20 moves out ---> RBC shrinks and cell
death occurs ---> intravascular hemolysis ---> hemolytic anemia.

Normal - PEP + Pyruvate kinase + ADP ---> Pyruvate + ATP.

Blood - Echinocyte ('echino' means hedgehog or sea urchin) - seen in PK deficiency.


smear [smaller and numerous, evenly spaced thorny/spiked RBC projections].
- Echinocyte are frequently confused with acanthocytes -DD
[acanthocytes have coarse, weirdly spaced, variably sized RBC crenations].
—Acanthocytes are seen in liver damage {hyperlipidemia or abetalipoproteinemia
---> high RBC membrane cholesterol}.

Fig: Echinocytosis Fig: Acanthocytosis


Immune hemolytic anemia (IHA)
1. What causes immune hemolytic anemia?
- IgG or IgM mediated destruction of RBC.

2. Differentiate between IgG vs IgM mediated IHA.


IgG mediated IHA IgM mediated IHA
- Hemolysis is usually extravascular - tagged RBC are eaten - Hemolysis is usually extravascular-
tagged RBC are eaten
- Warm agglutination - IgG binds to RBC in warm temp - Cold agglutination - IgM binds to
(central parts of body). RBC in cold temp (extremities).
- Splenic macrophage phagocytose tagged RBC leading to - RBC can inactivate complement, but
formation of spherocytes (when RBC are eaten only C3b acts as opsonin for splenic
halfway, remaining RBC makes sphere) macrophages - see spherocytes

- Associated with: - Assoiated with:


• Lupus - pt have anti-blood Ab • Mycoplasma pneumoniae
• CLL (chronic lymphocytic lukemia) - cause hemolytic (cold agglutination test)
anemia • infectious mononucleosis (+ve
• drugs (classically penicillin and cephalosporins) - drug haterophile agglutination - Ab
induces autoantibody production or Ab can bind to drug- made against sheep blood)
RBC complex • CLL
- Treatment
• Stop offending drug
• Steroids
• IVIG (distract spleen)
• Splenectomy - spleen is the one that eats RBC

3. How do you diagnose IHA?


Direct coombs test Indirect coombs test
- Confirms presence of Ab or complement coated - Confirm presence of anti-RBC Ab in
RBC patient's blood
- When anti-IgG or anti-complement Ab are added - Anti IgG and test RBC(normal RBC) are
to pt RBC, agglutination occurs only if RBC are mixed in patient serum (agglutination
already coated with IgG or complement occurs only if serum Ab are present)
- Most important test for IHA
Microangiopathic hemolytic anemia
1. What is microangiopathic hemolytic anemia (hemolysis in small blood vessel)?
- It's hemolysis that occurs due to vascular pathology (usually something in blood vessel breaks the
RBC)

2. What are some etiologies?


- Presence of microthrombi
○ TTP- thrombotic thrombocytopenic purpura
○ HU- hemolytic uremic syndrome
○ DIC
○ HELLP - hemolysis elevated liver enzyme and low platelet
- Prosthetic heart valves - crush RBC
- Aortic stenosis - crush RBC

3. What is blood smear finding?


- Schistocytes (broken RBC) - aka helmet cells

Fig - schistocytes (helmet cells) - has mostly two acute angle and loss of about 50% of RBC;
contrast bite cells that have usually >2 acute angles and almost entire volume of RBC is present.

Malaria
1. How does malaria cause anemia?
- Plasmodium infects and replicates in RBC. RBC ruptures as merozoites (a stage in their lifecycle)
- Plasmodium infects and replicates in RBC. RBC ruptures as merozoites (a stage in their lifecycle)
are released
- Spleen also consumes infected RBC causing some extravascular hemolysis

Erythroblastosis fetalis
Defn - Maternal IgG crossing placenta and attacking fetal RBC (ex - Rh -ve mother carrying
two consecutive Rh +ve babies)
Present - See extramedullary hematopoiesis (ex - in liver) because RBC are damaged
5.7 Normocytic Anemia due to Underproduction

1. What is anemia due to underproduction?


- It's anemia caused due to low RBC production by bone marrow.
- Characterized by low corrected reticulocyte (<3%)

3. What are some etiologies of anemia due to underproduction?


- Renal failure - decreased erythropoietin production by peritubular interstitial cells
- Anything that causes microcytic and macrocytic anemia
- Damage to bone marrow precursor cells - ex parvovirus B19

4. Describe how parvovirus B19 infection leads to anemia.


- Parvovirus B19 infects progenitor RBC and temporarily halts erythropoiesis.
- It causes significant anemia in setting of preexisting marrow stress (ex - sickle cell)
- Treatment is supportive (infection is self-limited)

Aplastic anemia
1. What is aplastic anemia?
- Aplastic anemia is damage to hematopoietic stem cell resulting in pancytopenia (anemia,
leukopenia, thrombocytopenia)

2. What are etiologies of aplastic anemia?


- Etiologies:
○ Drugs or chemicals, radiation
○ Viral infection - parvo B19, HIV, EBV, HCV
○ Autoimmune damage
○ Fanconi anemia (inherited DNA repair defect that causes bone marrow failure) - high risk of
leukemia later

3. What are biopsy finding in aplastic anemia?


- Empty fatty marrow

Fig: Aplastic anemia (left) vs normal bone marrow on right. Note the depletion of marrow and
replacement by fat globules on left.

4. What's treatment for aplastic anemia?


- Immunosuppression for cases with abnormal T cell activation
- Stop causative drugs
- Blood transfusion and marrow stimulating factors (erythropoietin, GM-CSF, G-CSF)
- May need bone marrow transplant

Myelophthisic process
1. What are melophthisic process?
- Pathologic processes that replace bone marrow (ex - cancer)
- Hematopoiesis is impaired resulting in pancytopenia
Chapter 6: White Blood Cell Disorders

Lymphoid tissue anatomy

1. Spleen
- PALS (periarteriolar lymphatic sheath) - has T cells (drink tea with pals) - in white pulp
- Germinal center - has B cells - in white pulp
CMAP summary

WBC disorders Page 1.1


Chromosome with genes

1. Ch 8 - c-myc ( protein in Ras- MAPK pathway)


2. Ch 11 - B globulin of Hb (1 allele per chromosome), Cyclin D1
3. Ch 14 - Ig heavy chain
4. Ch 15 - retinoic acid receptor
5. Ch 16 - alpha globulin of Hb (2 alleles per chromosome)
6. Ch 18 - Bcl2 (anti-apoptotic protein)

WBC disorders Page 2.1


6.1 Leukopenia and Leukocytosis

1. Draw out the development of different blood cell lines.

2. What happens in leukopenia or leukocytosis?


- Leukopenia - low WBC (penia = lack of)
- Leukocytosis - high WBC
- Usually, increase or decrease is in one particular cell line

Leukopenia
1. List the differences between neutropenia and lymphopenia.
Neutropenia Lymphopenia
Decreased circulating neutrophils Decreased circulating lymphocytes
Causes: Causes:
- Drug toxicity- Chemotherapy - Damage - Immunodeficiency - HIV, DiGeorge syndrome
stem cell and decreased production of - High cortisol - Induces apoptosis of lymphocytes
WBC, specially neutrophil - Autoimmune destruction - Lupus
- Severe infection- Neutrophils move to - Whole body radiation - Lymphocytes are highly
tissue sensitive to radiation; lyphopenia is earliest change to
- Lupus emerge after whole body radiation

Treatment: GM-CSF or G-CSF


*Corticosteroids cause neutrophilia but lymphopenia and eosinopenia. They cause sequestering of eosinophils
in lymph nodes, induce apoptosis of lymphocytes, and impair adhesion of neutrophils to vessel wall.
Leukocytosis
1. What are the causes of leukocytosis.
- Leukocytosis is increased WBC count.
Causes
Neutrophilic - Bacterial infection or tissue necrosis (also see left shift which have decreased Fc receptors
leukocytosis (CD16)
- High cortisol state - impairs neutrophil adhesion; marginated pool is released to blood
(marginated pool - leukocytes that are attached to vessel wall and hang there like bats)
Monocytosis - Chronic inflammatory state (autoimmune and infectious)
- Malignancy
Eosinophila - Allergic reaction (type I hypersensitivity)
- Parasitic infections
- Hodgkin lymphoma (increased IL5)
Basophilia - Classically seen in chronic myeloid leukemia (CML) - (HY)
Lymphocytic - Viral infection
leukocytosis - Bordetella pertussis (whopping cough)- (exception because bacteria usually increase
neutrophil)

Infectious mononucleosis
1. What causes mono?
- EBV (most common), CMV (less common)

2. What's presentation of EBV infection?


- Mono has classic triad of lymphadenopathy, fever and exudative pharyngitis
- Other:
○ T cell hyperplasia
 Lymphadenopathy due to hyperplasia in paracortex (location IMP to know)
 Splenomegaly due to hyperplasia in periarterial lymphatic sheath (PALS)
○ Hepatitis

3. Describe the screening test for mono (monospot test)?


- Test detects haterophile antibodies (haterophile = loves others) - detects IgM Ab that cross-react with horse or
sheep RBC
- Test turns positive after 1 week of infection

4. What are two cases where person has mono but monospot test is negative?
- Mono is caused by CMV (only EBV mono will have positive monospot test)
- Test is done within 1 week of infection

5. What's definitive diagnosis for EBV infection?


- Serologic test for EBV viral capsid Ag.

6. What are complications of EBV infection?


- Splenic rupture (due to splenomegaly) - avoid contact sports for 1 year
- Rash is exposed to ampicillin (pharyngitis of group A strep is treated by penicillin)
- Virus dormant in B cell:
○ Risk for B cell lymphoma, esp in immunodeficiency
○ Recurrence of mono
6.2 Acute Leukemia

1. What defines acute leukemia? What are its types?


- Acute leukemia is presence of >20% blasts in the bone marrow (normal is 1-2%). Blasts indicate
myeloid or lymphoid stem cell (undifferentiated cells).

- Types:
○ Acute lymphoblastic leukemia (ALL) - presence of >20% lymphoblast stem cell
○ Acute myeloid leukemia (LML) - presence of >20% myeloid stem cell

2. What is presentation and clinical findings in acute leukemia?


- Normal hematopoiesis is disturbed:
○ Anemia (tired)
○ Thrombocytopenia (bleeding)
○ Neutropenia (infection)
- Note that WBC count is high due to presence of lots of blasts in blood
- Blast cell appear as large immature cells with punched out nucleoli.

Fig: blast cells with punched out nucleolus (as if punched via punching machine) - notice lots of
them (gives high WBC count)
Acute lymphoblastic leukemia
1. How do you distinguish if acute leukemia is from myeloblast of lymphoblast?
Acute lymphoblatsic leukemia +ve nuclear staining for TdT (a DNA polymerase)
Acute myelogenous leukemia +ve cytoplasmic staining for myeloperoxidase (MPO)

2. What is epidemiology of ALL?


- Most commonly seen in children
- Associated with down syndrome (usually >5 years)

3. Distinguish the two types of acute lymphoblastic leukemia.


B-ALL (B-acute lymphoblastic leukemia) T-ALL (acute lymphoblastic lymphoma)
Characterized by lymphoblast (TdT+) that Characterized by lymphoblasts (TdT+) that express
express CD10, CD19, and CD20 (markers of CD2-CD8; lack CD10
B cells)
Excellent response to chemotherapy (give Excellent response to chemotherapy (give
prophylaxis dose in scrotum and CSF) prophylaxis dose in scrotum and CSF)
- Usually present in teenagers as mediastinal
(thymic) mass - cancer called lymphoma because of
cells make a mass

4. What are two types of B-ALL?


- T (12;21) - good prognosis - usually seen in children
- T (9;22) (aka Philadelphia + ALL) - poor prognosis - usually seen in adults

Picmonic for ALL

Pt popn - Affects <15 years (quincenera scene)


Prognosis - t(12;21) has better prognosis (girl with 1 wand and tutu looking at her reflection)
- Excellent response to chemotherapy - chemo girl kissing on the last figure
- Give prophylactic chemo to testes and CNS - bottom left figure guy hitting enemy
on head and testes
Markers - TDT +ve (explode with TNT)
- CALLA +ve (villan with collar - bottom middle pic)
Presentatio - Thymic mass (medistinal arrow) for T-ALL
n
Acute Myeloid leukemia
1. What is acute myeloid leukemia?
- Presence of immature myeloid cells >20% in bone marrow
- Myeloid stem cell characterized by +ve cytoplasmic staining for myeloperoxidase (MPO)
- Crystal aggregate of MPO may be seen as Auer rods

Fig: Aure rods inidicating MPO presence (diagnosis for AML)

2. What is epidemiology of AML?


- Mostly affects old people - (50-60 years)

3. What are different types of AML?

Acute Cause - t(15;17) translocates retinoic acid receptor (RAR) to


promyelocytic chromosome 17. Bad receptor blocks maturation of
leukemia WBC and promyelocytes (blasts) accumulate.
Treatmen - All trans retinoid acid (ATRA - vit A derivative) - it
t can bind to bad retinoic acid receptor. (this signaling
important for development of myeloblasts)
Complica - DIC (promyelocytes contain numerous primary
tion granules that increase risk for DIC).

Acute monocytic ○ Blasts characterstically infiltrate gums


leukemia (MPO -
ve)

Acute - Associated with Down syndrome (usually seen <5 years)


megakaryoblastic
leukemia (MPO -
ve)
Myelodysplastic syndrome
7. What is myelodysplastic syndrome?
- Dysplasia of myeloid stem cells. It increases risk of AML.
- Presents as
○ > 3% but <20% blasts (if it's > 20%, It's acute leukemia) abnormal maturation of
cells and increased because cells can't fully differentiate-

○ abnormal maturation of cells and increased blasts because cells can't fully differentiate-
that's why increased risk of AML
○ Most pt die from infection or bleeding (some progress to AML)
○ Cytopenia and hypercellular bone marrow (cells are stuck in bone marrow and not going
out)

- Ex - Pseudo-Pegler-Huet anomaly - neutrophils with bilobed nuclei typically seen after


chemotherapy

Fig: Pseudo-Pegler-Huet anomaly


6.3 Chronic Leukemia

1. What is chronic leukemia? What are it's types?


- Chronic leukemia is proliferation of MATURE circulating lymphocytes.
- It's usually seen in adults and have insidious onset.
- Types:

Cancer name Cell type high Remarks


Chronic lymphocytic Naïve B cells (CD5 - Most common leukemia overall
leukemia (CLL) and CD20 +ve) - - See smudge cell (like someone smashed a cell in slide) -
naïve kids smudge - See generalized lymphadenopathy
their food - Small lymphocytic lymphoma is a type of CLL that affects tissue

Complicati • Transformation to diffuse large B-cell lymphoma (richter


ons transformation) - see enlarging lymph nodes or spleen
• Hypogammaglobulinemia - INFECTION MOST COMMON
CAUSE OF DEATH
Associatio • Autoimmune hemolytic anemia
n

Picmonic:

CaLL - CLL
Epid - Most common leukemia in adults (old people in pic)
Presentati - Often asymptomatic - both elderly are happy and dancing
on - Smudge cell - cell smudged on glass of grandpa
Associatio - Autoimmune hemolytic anemia - both IgM and IgG - cold
n and warm sea anemone
Hairy cell leukemia Mature B cells (TRAP - Lymphadenopathy usually absent
(rare leukemia) +ve (tartarate - Marrow fibrosis (dry bone marrow tap)
resistant acid
phosphatase) and
hairy cytoplasm) -
hairs trap; mature
people are hairy

Picmonic:

Hairy guy with Basketball - hairy cell leukemia


Presentati - Splenomegaly (can be the only finding) - hairy cells
on accumulate in spleen - spleen baloon
- Lymphadenopathy usually absent
Treatment - Excelent response to 2-CDA (cladribine) - cloud ribbons

Adult T cell Mature CD4+ cells - Associated with HTLV-1; common in Japan and caribbean
leukemia/lymphoma - Skin rash, generalized lymphadenopathy, hepatosplenomegaly,
- lytic (punched out) bone lesion with hypercalcemia

- Excellent treatment to chemotherapy


Mycosis Fungoides Mature CD4+ cells - See sezary cells (cerebriform nuclei) in blood smear
(cutaneous T-cell that infiltrate skin - CD4 accumulation in skin produces rash, plaques and nodules called
lymphoma) pautrier microabscess

Fig - Sezary cells, pautrier microabscss - microcospic and gross


6.4 Myeloproliferative Disorders

1. What is myeloproliferative disorders? What's its presentation?


- Myeloproliferative disorders is neoplastic proliferation of mature cells of myeloid lineage
- Disease presents in late adulthood (50-60 years)
- Presentation:
○ Hypercellular bone marrow with increased WBC
○ All cells in myeloid lineage are increased, classification is based on what cell is dominantly
present

2. What are types of myeloproliferative disorders? What is characteristic mutation?


Disorder Dominant cell Mutation
Chronic myeloid Granulocytes (neutrophil, t(9;22) aka philadelphia
leukemia (CML) eosinophil, basophil); increased chromosome - present in 95% of pt
basophil (HY)
Polycythemia vera RBC JAK2 kinase mutation
(PV)
Essential Platelets JAK2 kinase mutation
thrombocythemia
(ET)
Myelofibrosis Megakaryocytes JAK2 kinase mutation (50% of cases)
Chronic myelogenous leukemia (CML)
1. What is the mutation that causes CML? What's treatment?
- T(9;22) produces philadelphia chromosome. It creates fusion of BCR on ch 22 with ABL receptor
tyrosine kinase protein. It leads to increased tyrosine kinase activitiy.
- Treatment
○ Imatinab - blocks tyrosine kinase activity (HY)
○ Hydroxyurea - inhibits ribonucleotide reductase (DNA synthesis) - prevent cell dividion
○ IFN alpha - increased destruction of cells
○ Hydroxyurea and IFN alpha can be used in many disease where cell divides too much

2. What is presentation and complication?


- Splenomegaly is common
- Complication:
○ Transformation to AML (2/3rd of cases)
○ Transformation to ALL (1/3rd of cases)

3. How do you diagnose CML?


- Peripheral blood smear shows leukocytosis - too much WBC
- FISH can be used to see philadelphia chromosome (seen in 95% of cases); however gene
translocation is seen in 100% of cases.

4. How do you distinguish CML from lukemoidreaction (reactive neutrophilic leukocytosis)?


CML Lukemoid reaction
Negative leukocyte alkaline phosphatase Positive leukocyte alkaline phosphatase (LAP) stain
(LAP) stain (LAP present in normal
neutrophils but absent in malignant
neutrophils)

INCREASED BASOPHILS (HY) No increase in basophil


t(9;22) present t(9;22) absent
5. Picmonic for CML

CML Camel
Mutation - t(9;22) - ninja and double tutu
- Makes philadelphia chromosome (philadelphia cream cheese)
- BCR-ABL fusion protein created - VCR - Abraham lincoln
Treatment - Imatinib - eye-mat
Polycythemia vera (PV)

1. What is PV?
- Increase in RBC, WBC and platelets but main presentation is based on too much RBC.

1. What are presentation of PV?


- Presentation based on increased viscosity of blood:
○ Flushed face due to congestion (plethora)
○ Intense itching, especially after bathing (due to histamine release from increased mast cells
due to heat)
○ Erythromelalgia - severe burning pain, and red-blue discoloration due to blood clots in
vessels of extremities; melalgia = buring pain in limbs
○ Hepatosplenomegaly (splenomegaly because there's too much RBC so amount of bad RBC
will be high and spleen has to eat them; hepatomegaly because liver makes RBC due to
mutation in JAK2 kinase).
○ Gout - proerythroblasts (RBC witn nucleus) will leak out and be eaten by spleen. Metabolism
of nucleus increases uric acid.
○ Venous thrombosis
 No. 1 cause of budd-chiari syndrome
 Dural sinus thrombosis
 Retinal vein thrombosis

Fig: polycythemia presentation - from left to right: dural sinus thrombosis, Erythromelalgia, facial
plethora, itching, hepatosplenomegaly, gout

2. How do you treat PV?


- Phlebotomy (first line)
- Hydroxyurea - second line (inhibits ribonucleotide reductase - DNA synthesis)
- Interferon alpha - increased destruction of proerythroblasts (can induce gout)

3. What's prognosis of PV?


- Death within a year without treatment

4. How do you distinguish PV from reactive polycythemia?


PV Reactive polycythemia
Normal SaO2, low erythropoietin - If due to lung disease or high altitude- Low SaO2, high
(EPO) EPO
- If due to ectopic EPO production, SaO2 normal, high EPO
Essential thrombocythemia vs Myelofibrosis
1. Differentiate essential thrombocythemia vs myelofibrosis
Essential thrombocythemia (ET) Myelofibrosis
Neoplastic proliferation of platelets - Neoplastic proliferation of megakaryocytes
megakaryocytes divide too much. (megakaryocyte = cell with big nucleus)
Thrombocyte = platelet
Associated with JAK2 kinase mutation Associated with JAK2 kinase mutation
Presentation Presentation:
- increased risk of thrombosis and/or - Increased risk of thrombosis and/or bleeding,
bleeding (low platelet due to too infection
much thrombosis) - Hepatosplenomegaly - they will produce blood
- Splenomegaly - spleen has to eat cells as marrow is fibrosed (MOST COMMON
too much platelets CAUSE OF SPLENOMEGALY IN PT >55 YEAR
- Rarely progress to marrow fibrosis OLD)
or acute lukemia - Marrow fibrosis (due to PDGF production from
- Rare risk for hyperuricemia or gout megakaryocytes) will impinge on bone and
(only platelets are increased which cause bone pain
don't have nucleus) - Splenomegaly due to extramedullary
hematopoiesis (due to marrow fibrosis)
- Leukoerythroblastic smear (tear drop RBC -
RBC squeezing through fibrosed marrow,
nucleated RBC, immature granulocytes)
• Enlarged megakaryocytes in bone
marrow biopsy

Fig: leukoerythroblastic smear, marrow fibrosis


- Diagnosis: - Diagnosis:
• Diagnosis of exclusion • Tear drop RBC
• See platelets >450k for >2 months • Bone marrow tap will be dry; see fibrosis in
biopsy
- Treatment: - Treatment:
• Asprin for low risk pt to prevent • Stem cell transplant - replace fibrosis
clot • Transfusion to give blood cells that’s missing
• Hydroxyurea , interferon alpha to
reduce megakaryocyte formation
• Plateletpheresis - take excess
platelet from circulation in
emergency condition
- Leukoerythroblastic anemia is anemia due to space occupying lesion in marrow
6.5 Lymphadenopathy (LAD)

1. When do you see painful vs painless lymphadenopathy?


- Lymphadenopathy is enlarged lymph nodes.
- Painful:
○ Lymph nodes draining a region of acute infection (acute lymphadenitis)
- Painless:
○ Chronic inflammation (chronic lymphadenitis)
○ Metastatic carcinoma
○ Lymphoma

2. What are indications depending on what part of lymph node is enlarged?


- Follicular enlargement (B cell region):
○ Rheumatoid arthritis
○ Early stages of HIV
- Paracortex enlargement (T cell region):
○ Viral infection (ex - EBV)
- Sinus histiocytes enlargement:
○ Lymph nodes draining with cancer

WBC disorders Page 7.1


6.6 Lymphoma, 6.7 Hodgkin Lymphoma

1. What is lymphoma?
- Neoplastic proliferaion of lymphoid cells that makes a mass; may arise in lymph node or extranodal tissue.

2. Distinguish Hodgkin lymphoma (HL) and non-Hodgkin lymphoma (NHL).

Hodgkin lymphoma Non-Hodgkin lymphoma


Frequency 40% 60%
Malignant cells Reed-Sternberg cells (owl eye cells) - B cells - Lymphoid cells - many subtypes - most are B cell
induce inflammation and fibrosis than T cells

Mass Inflammatory cells and fibrosis (reactive cells) Lymphoid cells


composition
Clinical - Painless lymphadenopathy - Painless lymphadenopathy
- B symptoms - Classically seen in late adults
- Bimodal: classically seen in young adult
and >55 yrs
Spread Contiguous (touching); rarely extranodal Diffuse; often extranodal
Staging Guides therapy Limited importance
Radiation mainstray treatment
Leukemic phase Doesn't occur Occurs
Prognosis Much better prognosis than non-Hodgkins
Association EBV HIV and autoimmune diseases
Hodgkin lymphoma (HL)
1. What are Reed-sternberg cells?
- Large B cells (CD 15 and CD 30 +ve) with multilobed nuclei and prominent nucleoli that looks like Owl eye
- They produce cytokines which attract inflammatory cells and cause fibrosis (can be seen in biopsy)

2. What are presentation of Hodgkin lymphoma?


- Due to cytokine release:
○ B symptoms - fever, chills, wt loss, night cells
○ Presence of reactive inflammatory cells that make bulk of tumor
○ Fibrosis

3. What are different subtypes of Hodgkin lymhoma and what are their characters?
Subtype Characters
Nodular sclerosis - Most common type of HL
- Classic pt is young adult female
- Presents as enlarged cervical or mediastinal lymph nodes
- Biopsy:
• Lymph node is divided by band of fibrosis
• Reed sternberg cells are present in lake like spaces (lacunar cells)

Fig: band of fibrosis (left), lacunar cells (a variant of Reed sternberg cells - right)
Mixed cellularity - Associated with abundant eosinophils (reed sternberg cells produce IL-5)
Lymphocyte rich - Best prognosis of all types
Lymphocyte depleted - Most aggressive of all types
- Usually seen in HIV pt or old pt

Non-Hodgkin lymphoma
1. What are types of non-hodgkin lymphoma?
- Non-hodgkin lymphoma is classified based on cell size, cell type (B or T), pattern of cell growth, cell surface markers,
translocations.
- Types:
○ Small B cells
 Follicular lymphoma - enlarged and too many follicles
 Mantle cell lymphoma - cancer of mantle (area around follicle)
 Marginal zone lymphoma - cancer of marginal zone (area around mantle)
 Small lymphocytic lymphoma (CLL that involve tissue)
○ Intermediate B cell lymphoma
 Burkitt lymphoma
○ Large B cells
 Diffuse large B-cell lymphoma
2. What are the causes of the different NH lymphoma?

NH lymphoma Cause Remarks


Follicular t(14;18) - BCL2 on ch 18 moves to Ig heavy chain locus on ch14 - Can progress to diffuse large B
lymphoma (BCL2 inhibits apoptosis by blocking Bax and bac from making cell lymphoma (presents as
channel in mitrochondrial membrane so cytochrome C can leak enlarging lymph node)
out)

Mantle cell t(11;14) - Cyclin D1 on ch 11 goes to Ig heavy chain locus on ch14 - Often present in late
lymphoma adulthood with painless
lymphadenopathy

Marginal zone - Chronic inflammatory state - Hashimoto thyroiditis, Sjogren - Marginal zone made of post-
lymphoma syndrome, H pylori gastritis (causes MALToma - a type of germinal B cells
marginal zone lymphoma)
Small - Chronic lymphocytic leukemia (naïve B cells; smudge cell)
lymphocytic affecting tissue
lymphoma
Burkitt - t(8;14) - c-myc on ch 8 goes to Ig heavy chain locus on ch14 (myc - Often present in child or young
lymphoma is a protein in RAS-MAPK pathway - cellular proliferation and adult as extranodal mass:
increased cyclin D production) • African form - in jaws
- Associated with BV infection • Sporadic form - in
- Biopsy finding abdomen

Fig: starry sky appearance (night sky made of dark lymphocytes;

Fig: starry sky appearance (night sky made of dark lymphocytes;


bright stars made up of mitotic figures)- left; right - african form
appearing on jaws

Diffuse large - Arise sporadically or transformation of low grade lymphoma (ex- - Most common NHL
B-cell follicular lymphoma, chronic lymphocytic leukemia) - High grade (aggressive)
lymphoma - Present in late adulthood as
extranodal mass or enlarging
lymph nodes
3. How do you distinguish follicular lymphoma from reactive follicular hyperplasia?
Follicular lymphoma Reactive follicular hyperplasia
- Cells are monoclonal - Cells are polyclonal
- Bcl2 expression in follicles (Bcl2 - anti-apoptotic - No Bcl2 expression in follicles
protein)
- Disruption of lymph node architecture - Preservation of lymph node architecture
- Lack of tingible body marcophage in germinal - Presence of tingible body marcophage (they eat bad B cells
center produced during somatic hypermutation)

Fig: bad lymph node architecture (follicles


present not only in cortex)

Fig: left (follulicular lymphoma lacking tingible body macohpage);


right (follicular hyperplasia with tingible body macrophage)

4. What's treatment of follicular lymphoma?


- Rituximab (anti-CD20 antibody) or low dose chemotherapy
6.8 Plasma Cell Disorders (dyscrasias)
Multiple myeloma

1. What is multiple myeloma? What is molecular mechanism?


- MM is malignant proliferation of plasma cells in bone marrow. It's most common primary
malignancy of bone.
- Serum IL-6 (aka RANK-L) may be increased that induces plasma cell growth and Ig production.

2. What are clinical features of MM?


Bone pain with - Plasma cell produce IL-6 (RANK-L) that binds to RANK on
hypercalcemia osteoclasts (bone breaks down) - creates punched out lesions
mainly on skull and vertebra

Elevated serum protein (M - Classically due too much IgG (55% of cases) or IgA (25%)
spike present in serum production
protein electrophoresis -
SPEP)

Increased risk of infection - Even though too much Ig produced, lack of Ig specificity
Rouleaux formation on - Increased serum protein decreases charge between RBC and RBC
blood smear makes stacks
(rouleau = cylindrical
packet of coins)

Anemia • Plasma cells crowd out bone marrow so little RBC synthesis
Primary AL amyloidosis - Due to too much Ig light chain (they deposit in tissue)
Proteinurea and renal - Free light chain protein excreted in urine as Bence Jones protein
failure - Light chain deposition in kidney can cause renal failure

Fig: Bence Jones crystal (see in Waldenstrom macroglobulinemia


too)

• Mnemonic CRAB
○ HyperCalcemia
○ Renal involvement / Roulex formatin
○ Anemia / Amylodosis
○ Lytic Bone lesions

Monoclonal Gammopathy of undetermined significance (MGUS)


1. What is MGUS? What's its significance?
- Increased serum protein with M spike on serum protein electrophoresis but no other feature of
MM (- no CRAB - no lytic bone lesion, no hypercalcemia, no rouleaux, no Bence Jones proteinura
etc)
- Significance:
○ Common in elderly (5% of >70 year old people)
○ 1% of pt with MGUS progress to multiple myeloma each year

Waldenstrom Macroglobulinemia
1. What is Waldenstrom macroglobulinemia? What are clinical features? What is treatment?
- B cell lymphoma with monoclonal IgM production (M for Macroblobulinemia)
- Clinical features:
○ M spike present - due to hyper IgM
○ Presentation based on hyperviscosicity of blood due to IgM pentamer (no CRAB findings
present)
▪ Retinal hemorrhage and stroke
▪ Bleeding - high blood viscosity leads to defective platelet aggregation
▪ Raynaud phenomena
○ Generalized lymphadenopathy
- Treatment:
○ Plasmapheresis to remove IgM from serum
6.9 Langerhans Cell Histiocytosis

1. What is langerhans cell histiocytosis? What is histology finding?


- It is neoplastic proliferation of Langerhans cells (dendritic cells in skin and mucus membrane derived from monocytes).
Histiocyte = tissue macrophage
- Histology finding:
○ Characteristic Birbeck (tennis racket) granules on electron microscopy
○ Cells are CD1a and S100 +ve by imunohistochemistry

Fig: Birbeck granules characteristic of Langerhans cell histiocytosis


2. Describe clinical features of different types of langerhans cell histiocytosis.
Disease Clinical presentations
Letterer-Swie disease (acute disseminated Malignant - Rapidly fatal due to multiple organ involvement
langerhans cell histiocytosis) proliferation of - Classic pt is <2 year old infant
Langerhans cells - Presentation:
• Skin rash
• Cystic skeletal defect

Eosinophilic granuloma Benign - Classic presentation is pathologic fracture in


proliferation of adolescents
Langerhans cells - Skin is not involved
in bone - Biopsy shows langerhans cells with mixed inflammatory
cells and lots of eosinophils
Hand-Schuller-Christian disease Malignant - Classic pt is >3 year old
proliferation of - Presentation:
Langerhans cells • Skull rash, Lytic skull defect
• Diabetes insipidus
• Exopthalmos

- Classic features:
○ If you see person's name, it's malignant and you see rash
○ if disease has 2 person's name, pt is <2 year old
○ If disease has 3 person's name, pt is >3 years
Chapter 7: Vascular Pathology
Vasculitis

1. What is vasculitis?
• Inflammation of blood vessel wall - can be both artery or vein
• Initially, the inflammation weakens the blood vessel wall and can cause aneurysm.
Later on, it causes fibrosis of vessel wall reducing the lumen size thus leading to ischemia of tissues.

2. What are the layers of blood vessel wall?


• Intima - endothelial cell
• Media - smooth muscle
• Adventitia - connective tissue

3. What is etiology of vasculitis? Is it infectious?


• Etiology mostly unknown. Not infectious.

4. What are clinical features of vasculitis?


• Non-specific symptoms of inflammation - fever, fatigue, weight loss, myalgia
• Symptoms of organ ischemia due to -
○ Thrombus formation - damage of endothelium exposes tissue factor and collagen
that stimulates clot formation
○ Reduced lumen size - Injury and healing of vessel wall causes fibrosis that reduces
lumen size of the vessel
○ Aneurysm - inflammation and necrosis of vessel wall weakens it leading to aneurysm

4.5. What are the types of arteritis?


• Large vessel arteritis - affects aorta and its major branches
• Medium vessel arteritis - affects muscular arteries that supplies organs
(ex - renal artery, coronary arteries)
• Small vessel arteritis - affects arterioles, capillaries and venules

Large vessel vasculitis Medium vessel vasculitis Small vessel vasculitis


Temporal (Giant Cell) Arteritis Polyarteritis nodosa Wegner granulomatosis
Takayasu Arteritis (aka pulseless disease) Kawasaki disease Microscopic polyangiitis
Buerger disease Churg-Strauss
Henoch-Schonlein
Large vessel vasculitis

5. What are clinical features of temporal (giant cell) arteritis


• Vasculitis of old people (avg age 72) - usually affect female
• Classic location is branches of carotid artery-
○ Temporal artery - new onset headache (classic involvement)
○ Opthalmic artery - visual disturbance
○ Arteries of jaw - jaw claudication
• Associations - Polymyalgia rheumatica (joint and muscle pain) with flu like symptoms
• ESR is elevated (>100) due to C-reactive proteins

6. How do you diagnose temporal arteritis? What's a caution point?


• Lesions are segmental hence diagnosis requires biopsy of a long segment of
vessel and a negative biopsy does not exclude disease.
• Biopsy revelas - inflammed vessel wall with giant cells and intimal fibrosis
• Granulomatous vasculitis

7. What is the treatment of temporal arteritis ?


• Corticosteroid.

7. What is the Risk factor of temporal arteritis ?


• high risk of blindness without treatment due to involvement of ophthalmic artery.

Fig - temporal arteritis biopsy. M is the media, I is the intima, F is the area of fibrosis between media and
intima that has narrowed the lumen, circle above M shows giant multinucleated cells, circle left of F
shows inflammation.

8. Contrast symptoms of Takayasu arteritis (aka pulseless disease)


• Takayasu arteritis is same as temporal arteritis with a couple of exceptions-
○ Affects mainly adults <50. Classic pt is young asian female
○ Granulomatous vasculitis
○ ESR is elevated
○ Treat with corticosteroids
○ Affects aortic arch at branch point
▪ Visual and neurologic symptoms
▪ Weak or absent pulse in upper extremity
Medium vessel arteritis

9. What is polyarteritis nodosa ?


• Necrotizing vasculitis that affects vessels in many organs but spares the lungs

10. What is clinical presentation of polyarteritis nodosa? What chemical is it associated with?
• Classic pt is young adults
• Symptoms depend on vessel involved-
○ Renal artery - hypertension
○ Mesenteric arteries - abdominal pain with melena
○ CNS arteries - neurologic disturbances
○ Cutaneous arteries - Skin lesions

10. What chemical is it associated with?


• Associated with serum HBsAG (hepatitis B surface antigen)

11. Correlate pathophysiology of polyarteritis nodosa with imaging.


• The disease has lesion of varying stage in the same artery.
Early stage - transmural inflammation with fibrinoid necrosis leading to Aneurysm
Late stage - fibrosis narrowing vessel lumen
• 'String-of-pearls' appearance is seen on imaging with alternate thick and thin sections.

Fig - the red marking showing fibrinoid


Fig - 'pearl on string' appearance of
necrosis seen in polyarteritis nodosa.
polyarteritis nodosa

12. What is treatment of polyarteritis nodosa?


• Corticosteroids and cyclophosphamide
• Fatal if not treated
13. What are clinical presentation of Kawasaki disease ?
• Classic pt is asian child under 4yrs of age
• Presentation (very nonspecific - mimics viral infection)
○ Fever (>5 days)
○ Enlarged cervical lymph nodes
○ Conjuctivitis sparring area right next to iris
○ Erythematous rash of palms, soles & any mucocutaenous membrane (ex - strawberry tongue)
○ Shrieking irritability - extremely irritable kid

14. What is preferentially involved in Kawasaki disease?


• Coronary artery involvement is a MAJOR risk factor for complication
• Complication -
○ thrombosis with MI - due to endothelial damage
○ aneurysm with rupture

13. What is treatment of Kawasaki disease (self limited disease) ?


• Asprin to prevent thrombosis (antiplatelet drug - prevents formation of thromboxane A2)
• IVIG (intravenous immunoglobulins)

Fig - Kawasaki disease

15. What is Buerger disease ?


• Necrotizing vasculitis involving digits.
• Presents with ulceration, gangrene, and autoamputation of fingers and toes.
Raynaud phenomenon [spasm of arteries cause episodes of reduced blood flow] is often present -
pale to blue to red color (return of circulation) change in digits due to exposure to cold.
• Disease is associated with smoking
Treatment is smoking cessation.
Small vessel vasculitis

Wegner Granulomatosis Microscopic polyangiitis


Necrotizing vasculitis Necrotizing vasculitis
Necrotizing granuloma present Granulomas absent
C-ANCA positive P-ANCA positive
Affects nasopharynx, lungs, and kidney Affects lungs and kidney and other organs
Treatment - cyclophosphamide, corticosteroids. Treatment - cyclophosphamide, corticosteroid.
Relapse common Relapse common

Churg-Strauss Henoch-Schonlein purpura


Necrotizing vasculitis Most common vasculitis in children
Necrotizing Granulomas present Palpable purpura in buttocks pathognomic
P-ANCA positive Vasculitis due to IgA immune deposition. Can lead to
IgA nephropathy
See asthma and eosinophila - pt Disease usually follows upper respiratory tract
misdiagnosed with allergies often infection
Affects multiple organs including lung and GI involvement can cause GI pain bleeding
heart
Treat by steroids if necessary. Disease self limited.

22. Differentiate Churg-Strauss and microscopic polyangiitis.


Churg-Strauss Microscopic Polyangiitis
• Necrotizing granulomas with eosinophils present • No granulomas
• Peripheral eosinophila and asthma present • No eosinophila or asthma
• Involves heart and lung • Involve lung and kidney
• P-ANCA correlate with disease activity • P-ANCA correlate with disease activity
16. What is Wegner Granulomatosis ?
• Necrotizing granulomatous vasculitis involving nasopharynx, lungs, and kidneys.
• Usually affect middle-aged males

17. What is presentation of Wegner Granulomatosis?


• Sinusitis or nasopharyngeal ulceration
• Hemoptysis with bilateral nodular lung infiltrates
• Hematuria due to RPGN

18. How do you diagnose Wegner Granulomatosis?


• Pt are C-ANCA positive (Serum C-ANCA level correlate with disease activity).
• Biopsy shows large necrotizing granuloma with adjacent necrotizing vasculitis.

18. How do you Treat Wegner Granulomatosis?


• Cyclophosphamide and corticosteroids.
• Relapse is common.

Fig - large necrotizing granuloma seen in Wegner granulomatosis. Granulomas are


absent in microscopic polyangiitis.
19. What is Microscopic polyangiitis ?
• Necrotizing vasculitis involving blood vessels of multiple organs, especially lung and kidney
• Serum P-ANCA levels correlate with disease activity
• Treatment - corticosteroids and cyclophosphamide
• Relapses are common

Fig - microscopic polyangiitis

21. What is Churg-Strauss syndrome ?


• Necrotizing granulomatous vasculitis with eosinophilic infiltrate
• Involves blood vessel of multiple organs, especially lungs and heart.
• Asthma and peripheral eosinophila often presesnt.
• Serum p-ANCA levels correlate with disease activity.

23. What is Henoch-Schonlein purpura ?


• Vasculitis due to IgA immune complex deposition
• Most common vasculitis in children

24 What is presentation of Henoch-Schonlein purpura?


• Palpable purpura on buttocks and legs.
• GI involvement - GI pain and bleeding
• IgA nephropathy leads to hematuria
• Usually occurs following an upper respiratory tract infection

25. How do you treat Henoch-Schonlein purpura?


• Disease is self-limited but may recur.
Treated with steroids if severe.
Hypertension

1. What are two types of hypertension?


• Pulomnary HTN
• Systemic HTN (systolic or diastolic BP >140/90) - divided into primary & secondary types .

3. What are risk factors for primary HTN ?


- Etiology unknown - It represents 95% of cases
• Age
• Race - high risk for African Americans, low risk for asians
• Obesity
• Stress
• High salt diet [Hypernatremia increases blood volume]
• Lack of physical activity

5. What is the cause of secondary HTN ?


• Etiology known - It represents 5% of cases
• Renal artery stenosis is a common cause (renovascular hypertension).
Important causes of stenosis include :
- Atherosclerosis (elderly males)
- fibromuscular dysplasia (young females) - it is a developmental defect of the blood vessel wall
resulting in irregular thickening of large- and medium-sized arteries, especially the renal artery.

6. ....
Describe renal artery stenosis .
• Stenosis decreases blood flow to glomerulus → JGA apparatus activated by baroreceptors
→ RAAS system stimulated → AGT II increases BP by 2 ways:
○ Powerful arterioconstrictor → increases TPR → diastolic BP increased
○ Stimulate aldosterone production → increases sodium reabsorption from DCT
→ Plasma volume expanded → increases systolic BP.

6. What is clinical presentation of renal artery stenosis (RAS)?


• Unilateral atrophy of Stenosed kidney (due to low blood flow)
• Increased blood renin leading to systemic HTN

8. Distinguish benign and malignant HTN.


Benign HTN Malignant HTN
Mild or moderate increase in BP Severe increase in BP (200/120 mm Hg); comprises < 5% of cases
Clinically silent May arise from preexisting benign HTN or de novo
Vessels and organs are damaged Pt presents with acute end organ damage - acute renal failure,
over time headache, papilledema
Most cases of HTN are benign MEDICAL EMERGENCY - pt can have fibrinoid necrosis of blood
vessel wall
Arteriosclerosis

1. What is Arteriosclerosis? What are it's 3 patterns?


- Arteriosclerosis is hardening of arterial wall.
- Patterns-
○ Atherosclerosis - thickening of intima due to plaque(occurs in medium and large sized vessels)
○ Arteriolosclerosis - thickening of small vessels. Wall thickens due to protein deposition
(hyaline arteriolosclerosis) or hyperplasia of smooth muscle (hyperplastic arteriolosclerosis)
○ Monckeberg medial sclerosis - calcification of media. Not very clinically significant
Atherosclerosis

2. Describe histology of atherosclerosis.


- Intimal plaque that obstructs blood flow
- Consists of a necrotic lipid core (mostly cholesterol) with a fibromuscular cap
- Often undergoes dystrophic calcification

Fig - artherosclerosis. M is the media, I is the intima, the circle represents the necrotic lipid core with
cholesterol crystals. The necrotic core is surrounded by a fibromuscular wall.

3. What are 4 most common sites for artherosclerosis?


Artherosclerosis occurs in large and medium sized arteries
○ Abdominal aorta
○ Coronary arteries
○ Popliteal artery
○ Internal carotid artery

4. What are key risk factors for artherosclerosis?


Modifiable risk factors Non modifiable risk factors
HTN Age (number and severity oflesions increase with age)
Hypercholesteremia (LDL Gender - male and postmenopausal females at high risk coz
increases risk and HDL reduces) [Estrogen --> stimulates eNOS --> NO mediated vasodialtion]
Smoking Genetics (multifactorial, but family history is highly predictive of risk)
Diabetes
5. What is pathogenesis of atherosclerosis?
- Damage to endothelium --> lipids leak between intima and media --> lipid is oxidized and then
phagocytozed by macrophages via scavenger receptor --> resulting in foam cells --> Inflammation and
healing occurs --> leads to deposition of extracellular matrix and proliferation of smooth muscle along
with fibromuscular cap formation.

6. Morphological changes in atherosclerosis ?


- Begins as fatty streaks (flat yellow lesions of the intima consisting of lipid-laden macrophages);
arise early in life (present in most teenagers)
- Progresses to atherosclerotic plaque

6. What are some complication of atherosclerosis?


- It's complications account for > 50% of disease in western world
- Artherosclerosis is asymptomatic until occlusion is > 70%
a) Stenosis of medium sized vessels results in impaired blood flow and ischemia leading to :
○ Peripheral vascular disease (popliteal A)
○ Angina (coronary A)
○ Ischemia bowel disease (mesenteric A)
b) Plaque rupture with thrombosis results in :
○ MI (coronary A)
○ Stroke (middle cerebral A)
c) Plaque rupture with embolization results in :
○ Atherosclerotic emboli (hallmark is cholesterol crystals within the embolus )
d) Weakening of vessel wall results in :
○ Aneurysm (abdominal aorta): MOA - As artherosclerotic plaque increases, it creates
diffusion (O2 & nutrients) barrrier resulting in atrophy of media and adventitia.

Fig - presence of cholesterol cleft in embolus is hallmark of artherosclerotic embolus


Arteriolosclerosis

7. What is arteriolosclerosis?
- Narrowing of small arterioles.
- TYPES
- Hyaline arteriolosclerosis.
- Hyperplastic arteriolosclerosis.

Fig - pink hyaline in vessel wall characterstic of hyaline arteriolosclerosis

8. What is hyaline arteriolosclerosis?


- Proteins leaking into vessel wall causes vascular thickening
- Proteins are seen as pink hyaline on microscopy

9. What are 2 causes of hyaline arteriolosclerosis? (VHY)


- Long standing Benign HTN - high hydrostatic pressure forces protein into the vessel wall
- Diabetes - non-enzymatic glycosilation of basement membrane makes vessel wall leaky
and protein leaks in.

10. What is key consequence of hyaline arteriolosclerosis? (HY)


- Reduced vessel diameter leads to end organ ischemia
- Classically produces glomerular scarring (arteriolonephrosclerosis) which slowly progress
to chronic renal failure

Fig - glomerular scarring seen in arteriolonephrosclerosis


11. What is hyperplastic arteriolosclerosis?
- Vessel wall thickens due to hyperplasia of smooth muscle
- 'Onion-skin' like appearance

Fig - onion skin like appearance of blood vessel in hyperplastic arteriolosclerosis

12. What causes hyperplastic arteriosloclerosis?


- Malignant HTN

13. What are consequences of hyperplastic arteriosclerosis?


- Reduced vessel diameter leads to end organ ischemia
- May lead to fibrinoid necrosis of the vessel wall with hemorrhagic necrosis
- Classically causes acute renal failure (ARF) with 'flea-bitten' appearance

Fig - flea bitten kidney classic of hyperplastic arteriolosclerosis due to pin point hemorrhage -
leads to ARF. Contrast glomerular scarring in chronic renal failure due to arterionephrosclerosis
Monckeberg medial calcific sclerosis

14. Describe Monckeberg medial sclerosis.


- Calcification of the media of muscular (medium-sized) arteries
- Does not obstruct lumen, therefore not clinically significant
- Seen as incidental finding in X-ray or mammography

Fig- Enclosure shows monckeberg medial sclerosis. Note how it's limited to media and doesn't
have much effect on lumen diameter.

Fig - mammography showing Monckelberg medial calcification of blood vessel


Aortic Dissection

Aortic Dissection

1. What is aortic dissection?


- Intimal tear with dissection of blood through media of the aortic wall

Fig - aortic dissection

2. What are causes of aortic dissection?


- Occurs in the proximal 10 cm of the aorta (high stress region) with preexisting weakness of the media

3. What is the role of Hypertension ?


- Most common cause of aortic dissection in older adults
- Hypertension results in hyaline arteriosclerosis of the vasa vasorum (blood vessel supplying
the aortic wall) resulting in decreased flow of blood to the media leading to its atropy.

5. What is the role of INHERTED connective tissue disorders ?


- Most common cause of aortic dissection in young people
- Classically lead to weakness of the connective tissue in the media (cystic medial necrosis).
- Marfan syndrome - mutation in fibrillin-1 protein which is the base on which elastic fibers are
laid down in the Media.
- Ehlers-Danlos syndrome - mutation in collagen or proteins that interact with collagen.

6. What is clinical picture of aortic dissection?


- Presents with sharp tearing chest pain that radiates to the back

6. What are the complications of aortic dissection?


- Cardiac tamponade (most common cause of death)
- Rupture with fatal hemorrhage
- Obstruction of branching arteries (e.g. coronary or renal) with resultant end-organ ischemia.
Aneurysm
7. What are two classic location of aneurysm [Balloon-like dilation of AORTA] ?
- Thoracic aneurysm
- Abdominal aneurysm

Thoracic Aneurysms

8. What is a classic cause of thoracic aneurysm?


- Tertiary syphilis- obliterative endarteritis of vasa vasorum ----> hyperplasia of the vasa vasorum ----> luminal
narrowing of vasa vasorum ---> decreased blood flow to aortic wall ----> ischemic injury of aortic wall ---->
necrotic changes and immune reaction is seen ---> fibrosis and scarring of Aortic wall ("Tree bark" appearance)
----> Aortic wall WEAKNESS ----> Aneurysm.
9. What are complications of thoracic aneurysm?
- Aortic valve insufficiency - dilation of the aortic valve root
This leads to other complications like :
- Compression of mediastinal structure (airway or esophagus)
- Thrombosis/Embolism due to disruption in laminar blood flow.

Fig - normal aorta (left); tree bark like aorta in tertiary


syphilis (right)
Abdominal Aneurysms

10. What is the most common site of abdominal aortic aneurysm ?


- Below the renal arteries but above the aortic bifurcation
- Vasa vasorum stops at the level of renal artery bifurcation hence atherosclerosis of aorta
below the renal arteries increases the diffusion barrier to the media resulting in poor
diffusion of nutrient and gases leading to atrophy and weakness of the vessel wall.

Fig: normal abdominal aorta (left); AAA (right) - note calcification of vessel wall and thrombosis

11. What are the presentation ?


- Classically seen in male smokers> 60 years old with hypertension
- Presents as a pulsatile abdominal mass that grows with time

12. What are the complication ?


- Rupture, especially when > 5cm in diameter - MAJOR COMPLICATION.
- Presents with triad of :
- hypotension
- pulsatile abdominal mass
- flank pain.
- Compression of local structures (e.g., ureter)
- Thrombosis/embolism
VASCULAR TUMORS

HEMANGIOMA
A. Benign tumor comprised of blood vessels
B. Commonly present at birth
C. Often regresses during childhood
C. Most often involves:
- skin
- liver

ANGIOSARCOMA
A. Malignant proliferation of endothelial cells
B. Highly aggressive
B. Common sites include:
- skin
- breast
- liver - Liver angiosarcoma is associated with exposure to:
→ PVC
→ Arsenic
→ Thorotrast

KAPOSI SARCOMA
A. Low-grade malignant proliferation of endothelial cells
B. Associated with HHV-8
B. Presents on SKIN as:
- purple patches
- plaques
- nodules
C. It may also involve visceral organs
C. Classically seen in:
1. Older Eastern European males
- tumor remains localized to skin
- treatment involves surgical removal.
2. AIDS pt.
- tumor spreads early
- treatment is antiretroviral agents (to boost immunity)
3. Transplant recipients
- tumor spreads early
- treatment involves decreasing immunosuppression.
Chapter 8: Cardiac Pathology
8.1 Ischemic Heart Disease

1. Describe Ischemic Heart Disease.


- Group of syndromes related to myocardial ischemia
- IHD is the leading cause of death in the US

2. What is most common cause of ischemic heart disease?


- Artherosclerosis of coronary arteries
- Decreased blood flow is not able to meet the metabolic demands of the myocardium

3. What are the Risk factors of IHD ?


- Same as risk factor for artherosclerosis
- age
- sex
- race
- smoking
- HTN
- diabetes
4. How long does a ischemia last before irreversible injury to cardiac myocytes occur?
- 20 minutes

5. Some basic terms.


-Infarction - tissue necrosis due to lack of oxygen
-Ischemia - inadequate blood supply to an organ
-Hypoxemia - (PaO2 < 60mm Hg)
Angina

3. Describe Stable angina and its causes.


- Chest pain that arises with exertion or emotional stress.
- Caused due to atherosclerosis of coronary arteries with > 70% stenosis
- Myocyte undergoes reversible injury (no necrosis)

4. What is presentation of stable angina?


- Chest pain (lasting < 20 minutes) that radiates to the left arm or jaw
- Diaphoresis
- Shortness of breath
- EKG shows ST segment depression due to subendocardial ischemia (blood vessels travel in
epicardium and endocardium is last part to receive blood).

5. How is stable angina relieved ?


- Pain relieved by rest or nitroglycerin

5. Describe Unstable angina and its causes.


- Chest pain THAT occurs at rest
- Usually due to rupture of an artherosclerotic plaque with thrombosis and incomplete occlusion
of a coronary artery (rupture usually occurs at neck of plaque)
- Myocyte undergoes reversible injury (no necrosis)

6. What is presentation of unstable angina?


- EKG shows ST segment depression due to because subendocardial ischemia (blood vessels
travel in epicardium and endocardium is last part to receive blood).
- High risk of progression to myocardial infarction (thrombus can grow)

7. How is unstable angina relieved ?


- Relieved by nitroglycerin (venodilation reduces the Heart load)

Fig - unstable angina in coronary artery. Note the dark thrombus.


The thrombus has high chance of growing and leading to MI.

7. Describe Prinzmetal angina and its causes.


- Due to coronary artery vasospasm
- Episodic chest pain unrelated to exertion.
- Myocyte undergoes reversible injury (no necrosis)

8. What is presentation of prinzmetal angina?


- EKG shows ST-segment elevation due to transmural ischemia (as coronary artery completely
clamps down).

9. How is Prinzmetal angina relieved ?


- Relieved by nitroglycerin or calcium channel blockers
Myocardial infarction

9. What is MI and its causes ?


- Necrosis of cardiac myocytes (irreversible injury)
- Infarction usually involves the LV but RV and both atria are generally spared.
- Main cause: rupture of artherosclerotic plaque with thrombosis and complete occlusion of coronary artery
- Other causes:
- coronary artery vasospasm (due to Prinzmetal angina or cocaine use)
- emboli
- vasculitis (e.g., Kawasaki disease).
- Symptoms are not relieved by nitroglycerin.
- Pain with MI is usually described as sharp Pleuritic chest pain
- ST depression in AVR lead is more specific for pericarditis
- Most common cause of chest pain in outpatient setting: GERD, anxiety, costochondral tenderness

10. What are clinical features of MI ?


- Severe crushing chest pain (lasting> 20 minutes) that radiates to the left arm or jaw
- Diaphoresis
- Dyspnea

11. What are key arteries involved in MI ?


- Left anterior descending artery (most common) - Occlusion of LAD leads to infarction of the :
- anterior wall of the LV.
- anterior septum of the LV.
- Right coronary artery (2nd most common) - Occlusion of RCA leads to infarction of the :
- posterior wall of LV.
- posterior septum of LV.
- papillary muscles of the LV.
- Left circumflex artery (other cause) - Occlusion of LCA leads to infarction of the :
- lateral wall of the LV.

12. Describe phases of MI.


- EARLY PHASE [Initial phase] ; <55 year for men and <45 yrs for women
- Subendocardial necrosis involving< 50% of the myocardial thickness
- EKG shows ST-segment depression (due to subendocardial infarction)
- LATE PHASE [Continued or severe ischemia]
- Transmural necrosis involving most of the myocardial wall
- EKG shows ST-segment elevation (due to transmural infarction)

13. Laboratory tests detect elevated cardiac enzymes [necrosis → cell membranes leaky → enzymes come in blood]
- Troponin I - most sensitive and specific marker (gold standard) for MI.
a. Rises 2-4 hrs post infarction
b. Peak at 24 hrs
c. Returns to normal by 7-10 days
- Creatine kinase MB [CK-MB] - useful for detecting reinfarction (that occurs days after an initial Ml).
a. Rises 4-6 hrs post infarction
b. Peaks at 24 hrs
c. Returns to normal by 72 hrs
9. Treatment.
- Aspirin and/or heparin- limits thrombosis
- Supplemental O2 - minimizes ischemia
- Nitrates - vasodilate veins and coronary arteries
- ACEi - decreases LV dilation (reduced blood volume and Afterload)
- Beta-blocker: slows heart rate => decreasing O2 demand
=> reduces risk for arrhythmia
- Fibrinolysis or angioplasty - opens blocked vessel

10. What are the Complications of myocardial infarction ?


- closely related to gross and microscopic changes
- Contraction band necrosis: Reperfusion of irreversibly-damaged cells results in calcium influx, leading
to hypercontraction of myofibrils
- Reperfusion injury: Return of oxygen and inflammatory cells may lead to free radical generation,
further damaging myocytes
15. Describe time frame of MI
Time Microscopic Gross change Complication
change
<4 hrs none none - Cardiogenic shock (massive infarction)
- CHF
- Arrhythmia
4-24 hrs Coagulative Dark discoloration - Arrhythmia (it doesn't happen post 24 hrs
necrosis because the conduction system is already
(no nuclei damaged and necrosis occurs)
with very
little
structural
damage)

1-3 days Neutrophils Yellow pallor - Fibrinous pericarditis (presents with chest
(early infla.) pain with friction rub)- only occurs with
transmural infraction

4 day - 1 Macrophages Yellow pallor - Rupture of Ventricular free wall (=> cardiac tamponade)
week (late inflam.) - Rupture of Interventricular septum (=> shunt)
- Rupture of papillary muscle (=> mitral regurgitation)
1-3 Granulation Red border emerges as
weeks tissue with granulation tissue enters from
plump edge of infarct.
fibroblasts,
collagen(I),
and blood
vessels

Months Fibrosis White scar - Aneurysm


- Mural thrombosis
- Dressler syndrome (autoimmune pericarditis -
is the inflammation of pericardium and
exposure of pericardial antigen which can
cause autoimmune attack to pericardium)

<1 day 1 day - 1 week 1 week- 1 month >1 month


Coagulative necrosis First neutrophils and then Granulation tissue Scar tissue
macrophage
• Dark discoloration of • Yellow pallor • Red border surrounding yellow • White
heart pallor scar
Fig - coagulative necrosis of heart
showing dark discoloration (<1 day)
Fig - 1-3 weeks post MI. The yellow pallor is central area of necrosis
(granulation tissue?) surrounded by emerging blood vessels from
edge of infaract

Fig- the box shows necrotic myocytes (no nuclei) and


the circles show contraction band necrosis
Fig - Months after MI (white scar tissue)

Fig- yellow pallor of heart post MI (1 day - 1 week)

Fig - fibrinous pericarditis (fibrin exudate during neutrophil rich stage (day 1-3 post MI) rubs when hear
contracts producing and characteristic friction rub). Only see during transmural infraction
Sudden cardiac death

16. What is sudden cardiac death? What is it's etiology


- Unexpected death due to cardiac disease
- Occurs without symptoms or < I hour after symptoms arise usually due to Fatal ventricular arrhythmia
- Etiology:
○ MOST COMMON- acute ischemia; 90% of patients have preexisting severe atherosclerosis.
○ LESS COMMON:
- mitral valve prolapse
- cardiomyopathy
- cocaine abuse

Chronic ischemic heart disease

17. What is chronic ischemic heart disease?


- Poor myocardial function due to chronic ischemic damage (with or without infraction)
- Can progress to congestive heart failure [CHF]
Congestive Heart Failure (CHF)

1. What is CHF and its divisions ?


- Pump failure
- Divided into:
- Left sided heart failure
- Right sided heart faiure

2. Differentiate right and left sided CHF.


Left sided HF Right sided HF
Causes - ischemia - Most commonly due to LHF
- HTN - Left-to-right shunt
- MI - Chronic lung disease
- Dilated cardiomyopathy (cor pulmonale).
- Restrictive cardiomyopathy
Present. ○ Pulmonary congestion leads to pulmonary edema. ○ Clinical features are due to
- Dyspnea congestion.
- Paroxysmal nocturnal dyspnea (↑VR while lying flat) - Jugular venous distension
- Orthopnea (dyspnea that occurs when lying flat) - 'Nutmeg' liver due to painful
- Crackles made by lung hepatosplenomegaly
- Hemosiderin laden macrophage (aka HF cells) due to - Cardiac cirrhosis
bursting of small congested p. capillaries leading to - Dependent pitting edema
intraalveolar haemorrhages & phagocytosis of RBCs. (due to ↑ H. pressure)
○ Decreased Renal perfusion activates RAAS system.
- Fluid retention exacerbates CHF
Treat. ACE inhibitors
Work load of Heart is determined by Cardiac Contractility
Cardiac contraction --> Actin-Myosin interaction --> requires ATP --> need 02.
[Laplace law: P = 2Tw/R] ; w - wall thickness, R = chamber radius.
a) ↑ Afterload --> Pressure Overload --> Concentric hypertrophy --> ↑w.
b) ↑ Preload --> Volume Overload --> Eccentric Hypertrophy --> ↑R.

Low output Cardiac failure


--> LV/RV failure
--> CHF

Severe Anemia ---> High-output heart failure ---> CHF ?


1] Anemia --> less 02 --> less ATP --> open KATP channel --> Vasodilation --> ↓ TPR.
2] ↓ TPR --> ↓ BP --> Baroreceptor activation --> Sympathetic drive --> SA node
--> ↑ HR --> ↑ CO (normal systolic function).
3] Sympathetic drive --> LV myocardium --> ↑Contractility (T) --> ↑LV pressure (P)
--> LV cannot fill (disatolic failure) --> Pul. edema
4] P.edema --> RV failure --> systemic pooling of blood --> Systemic edema
--> LV cannot pump blood --> systolic failure.

1] Anemia --> less 02 --> less ATP --> open KATP channel --> Vasodilation --> ↓ TPR.
2] ↓ TPR --> ↓ BP --> Baroreceptor activation --> Sympathetic drive --> ↑ HR --> ↑ CO.
3] Sympathetic drive --> Vasoconstriction -->
↑ Afterload (↑P) --> heart has to pump harder
[compensatory ↑ Contractility (T)] --> ↑ workload
--> mechanical receptor activated on Myocardium
--> LV Hypertrophy --> D.failure --> P.edema.
Congenital Defects

1. What is epidemiology of congenital heart defect?


- Arise during embryogenesis [usually b/w 3 to 8 weeks (organogenesis)]
- Seen in 1% of live births
- Most defects are sporadic

2. Describe basic principles of CHD.


- Often result in shunting between left (systemic) and right (pulmonary) circulations.
- Defects with left-to-right shunting may be relatively asymptomatic at birth,
but the shunt can eventually reverse due to :
- Increased flow through the pulmonary circulation => hypertrophy of pulmonary vessels
=> pulmonary hypertension => Increased pulmonary resistance => reversal of shunt =>
Eisenmenger syndrome
- Defects with right-to-left shunting usually present as cyanosis shortly after birth.

3. What is Eisenmenger syndrome (aka tardive cyanosis) ?


- Serious hypoxemia caused by reversal of left-to-right shunt is called Eisenmenger syndrome.
- It can be present during
- Atrial septal defect
- Ventricular septal defect
- patent ductus arteriosus
- Presentation:
- Late cyanosis
- Right ventricular hypertrophy
- Polycythemia vera (increased RBC to combat hypoxemia)
- Clubbing
VSD [VENTRICULAR SEPTAL DEFECT]

1. Describe VSD.
- Most common congenital heart defect
- Defect in the Interventrivular septum
- Associated with fetal alcohol syndrome
- Presentation:
- Results in left-to-right shunt
- Size of defect determines extent of shunting and age at presentation
- Small defects are often asymptomatic
- Large defects can lead to Eisenmenger syndrome. :
- Treatment:
- surgical closure
- small defects may close spontaneously.

ASD [ATRIAL SEPTAL DEFECT]

1. Describe ASD ?
- Defect in the interatrial septum
- Types
○ Ostium secundum (most common) - 90% of cases
○ Ostium primum (aka endocardial cushion defect) - ASD + valvular defect + IV septum defect
- Associated with Down syndrome (Sinus venosus)
- Presentation:
- Results in left-to-right shunt
- Split S2 on auscultation (increased blood in right heart delays closure of pulmonary valve)
- Complication:
- Paradoxical emboli (ex - DVT emboli will go to systemic organs instead of lungs)

Fig: osteum primum (left), osteum secundum (middle and right)


PDA [PATENT DUCTUS ARTERIOSUS]

1. Describe PDA.
- Failure of ductus arteriosus to close after birth.
- Associated with congenital rubella
- Presentation:
- Results in left-to-right shunt between the aorta and the pulmonary artery
(During development, the ductus arteriosus normally shunts blood from the
pulmonary artery to the aorta, bypassing the lungs).
- Asymptomatic at birth with continuous (holosystolic) 'machine-like' murmur
- May lead to Eisenmenger syndrome, resulting in lower extremity cyanosis
(because ductus arteriosus arises after upper extremity branching).
- Treatment:
- Indomethacin, which decreases PGE, resulting in PDA closure
(PGE maintains patency of the ductus arteriosus).
Tetralogy of fallot

1. Describe Tetralogy of fallot ?


- Characteristics:
- Stenosis of the right ventricular outflow tract
- RV hypertrophy
- VSD
- Aorta that overrides the VSD
- Presentation:
- Right-to-left shunt leads to early cyanosis
(Degree of stenosis determines the extent of shunting and cyanosis).
- Patients usually after exercise learn to squat (increases arterial resistance) in
response to a cyanotic spell which decreases shunting thus increasing pulmonary
blood flow
- 'Boot shaped' heart on X-ray

Fig: boot shaped heart indicating tetralogy of falot


Transposition of the great vessels

1. Describe Transposition of the great vessels.


- Characterized by:
- pulmonary artery arising from the left ventricle and
- aorta arising from the right ventricle
- Associated with maternal diabetes
- Presentation:
- Early cyanosis (right and left sided blood do not mix)
- RV hypertrophy and
- LV atrophy
- Treatment:
- Creation of shunt (allowing blood to mix) after birth is required for survival.
- Administer PGE to maintain a PDA until definitive surgical repair is performed.
Truncus arteriosus

1. Describe truncus arteriosus?


- Truncus fails to divide into aorta and pulmonary artery.
- Characterized by a single large vessel arising from both ventricles
- Presentation:
○ Early cyanosis (deoxygenated blood from right ventricle mixes with oxygenated blood
from left ventricle before pulmonary and aortic circulations separate).

Tricuspid atresia
1. Describe Tricuspid atresia ?
- Tricuspid valve orifice fails to develop
- Often associated with ASD
- Presentation:
○ Hypoplastic RV
○ Right-to-left shunt
○ Early cyanosis.
Coarctation of aorta

1. What is coarctation of aorta?


- Narrowing of the aorta
- classical Types:
- infantile form
- adult form

Infantile type Adult type


Anatomy coarctation lies after (distal to) the aortic arch, coarctation lies after (distal to) the
but before (proximal to) the PDA. aortic arch.

Associa. - PDA bicuspid aortic valve


- Turner syndrome
Present. lower extremity cyanosis in infants, often at - HTN in upper extremities and
birth (lower extremity isn't supplied by LV but - Hypotension with weak pulse in
by RV. Upper extremities are fine because LV lower extremities
supplies there) - Collateral circulation develops
across the intercostal arteries
Engorged arteries cause 'notching'
of ribs on x-ray
Valvular Disorders

Acute rheumatic fever

1. What is actue rheumatic fever?


- Systemic complication of pharyngitis due to group A beta hemolytic streptococci
- Affects children 2-3 weeks after an episode of streptococcal pharyngitis ("strep throat")
- Bacterial M protein mimics human protein and autoantibodies are generated.

2. How is acute rheumatic fever diagnosed?


Diagnosis is based on Jones criteria.
- Evidence of a group A beta hemolytic streptococcal infection (AR fever) is made by :
(elevated ASO or anti-DNase B titers) + the presence of few major & minor criteria.
- Minor criteria (non-specific)
○ Fever
○ elevated ESR
- Major criteria
○ Joint (migratory polyarthritis) - swelling in pain in large joints (wrist, knee, ankle) that
resolve in days and migrates to involve another large joint
○ O (pancarditis)
 Endocarditis - Mitral valve is involved more commonly than the aortic valve.
See Small vegetations along lines of closure that lead to regurgitation
 Myocarditis (most common cause of death) - Aschoff bodies are seen [focal area of chronic
inflammation with the presence of Anitschkow cells (reactive histiocytes with slender, wavy
nucleus) along with giant cells and fibrinoid material].
 Pericarditis - leads to friction rub and chest pain
○ Subcutaneous Nodules
○ Erythema marginatum - annular, nonpruritic rash with erythematous borders, commonly involving
trunk and limbs
○ Sydenham chorea (rapid, involuntary muscle movements)

4. What is prognosis of acute rheumatic fever?


- Acute attack usually resolvs but may progress to chronic rheumatic heart disease
- Repeat exposure to group A beta hemolytic strep results in relapse of the acute
phase and increases risk for chronic disease.
Aschoff body: It is spheroidal or fusiform characteristic lesion seen in rheumatic disease.
– Size: About to 1 to 2 mm.
– Site: During acute RF, may be found in any of the three layers, in the perivascular interstitial region
of the heart.
– Formation of Aschoff body consists of 3 phases:
i. Early/exudative phase: It consists of a perivascular focus of swollen eosinophilic collagen
surrounded by lymphocytes, plasma cells, and macrophages.
ii. Intermediate/granulomatous phase: It is pathognomonic of rheumatic myocarditis. It is a
granulomatous lesion, with a central fibrinoid focus associated with lymphocytes (primarily
T cells), macrophages, Anitschkow cells (pathognomonic for RF) and occasional plasma cells.
Anitschkow cells are plump activated macrophages, which have abundant cytoplasm. They
have round-to-ovoid nuclei in which the chromatin is disposed in a central, slender, wavy ribbon.
These nuclei resemble a caterpillar when cut longitudinally (hence known as “caterpillar
cells”). On cross section, these nuclei have an owl eye appearance. Few Anitschkow cells may
become multinucleated (with 2 to 4 nuclei) in which case, they are termed Aschoff giant cells.
iii. Late/healed /final phase: Aschoff body is replaced by a nodule of scar tissue.
Chronic rheumatic heart disease
1. What's presentation of chronic rheumatic fever?
- Valve scarring that arises as a consequence of rheumatic fever
- Results in stenosis with a classic 'fish-mouth' appearance
- Mitral valve (mainly): thickening of chordae tendineae and cusps.
- Aortic valve (occasionally): fusion of the commissures.
2. What's a complication of chronic rheumatic fever?
- Infective endocarditis
Aortic stenosis

1. What is aortic stenosis?


- Narrowing of the aortic valve orifice

1. What are some causes of aortic stenosis?


- Fibrosis and calcification from "wear and tear"
- Bicuspid aortic valve (increases risk and hastens disease onset)
- Chronic rheumatic valve disease [coexisting mitral stenosis and fusion of the aortic valve
commissures distinguish rheumatic disease from "wear and tear"]

2. What is its presentation?


- Presents in late adulthood (>60 years)
- Cardiac compensation leads to a prolonged asymptomatic stage during which a systolic
ejection dick followed by a crescendo-decrescendo murmur is heard.

4. What are complications of aortic stenosis?


- Concentric left ventricular hypertrophy - may progress to cardiac failure
- Angina and syncope with exercise - Limited ability to increase blood flow across the
stenotic valve leads to decreased perfusion of the myocardium and brain.
- Microangiopathic hemolytic anemia - RBCs are damaged (producing schistocytes)
while crossing the calcified valve.

5. How do you treat aortic stenosis?


- Valve replacement after the onset of complication

Aortic regurgitation

1. What is aortic regurgitation?


- Backflow of blood from the aorta into the left ventricle during diastole

1. What are causes of aortic regurgitation?


- Isolated aortic root dilation - due to syphilitic aneurysm and aortic dissection
- Valve damage - due to infective endocarditis

2. What are clinical features of aortic regurgitation?


- Early, blowing diastolic murmur
- Hyperdynamic circulation due to increased pulse pressure
- Diastolic pressure decreases due to regurgitation,
while Systolic pressure increases due to increased stroke volume
- Presents with
=> bounding pulse (water-hammer pulse)
=> pulsating nail bed (Quincke pulse)
=> head bobbing
- LV dilation and eccentric hypertrophy due to volume overload

3. What is treatment of aortic regurgitation?


- Valve replacement once LV dysfunction develops
Mitral valve prolapse

1. What is mitral valve prolapse? What are some etiologies?


- Ballooning of mitral valve into left atrium during systole

1. What are the causes of mitral valve prolapse?


- Myxoid degeneration (accumulation of ground substance) of the valve, making it floppy.
- Etiology is unknown; may be seen in:
○ Marfan syndrome
○ Ehlers-Danlos syndrome

2. What is presentation of mitral valve prolapse?


- Seen in 2- 3% of US adults
- Mostly asymptomatic
- Mid-systolic click followed by regurgitation murmur
- Click and murmur become softer with squatting (increased s. resistance decreases LV emptying)

3. What are complications?


- Complications are Rare but may include:
- infective endocarditis,
- arrhythmia
- severe mitral regurgitation

4. What is treatment of mitral valve prolapse?


- Valve replacement

Mitral regurgitation

1. What is mitral regurgitation ?


- Reflux of blood from the left ventricle into the left atrium during systole

1. What are some causes of mitral regurgitation?


- Arises as a complication of mitral valve prolapse
- LV dilatation (due to left-sided cardiac failure)
- Infective endocarditis
- Acute rheumatic Heart disease
- Papillary muscle rupture (due to MI)

2. What is presentation of mitral regurgitation?


- Holosystolic "blowing" murmur which is louder when :
- squatting (increased s. resistance decreases LV emptying) and
- expiration (increases blood return to LA)
- Volume overload and left sided heart failure

Mitral stenosis

1. What is mitral stenosis?


- Narrowing of the mitral valve orifice
1. What is the causes of mitral stenosis?
- Chronic rheumatic valve disease

2. What are presentations of mitral stenosis?


- Opening snap followed by diastolic rumble
- Volume overload leads to dilation of left atrium resulting in:
○ Pulmonary congestion with edema and alveolar hemorrhage
○ Pulmonary HTN and eventual right sided heart failure
○ A-fib with associated risk for mural thrombi
Endocarditis

1. What is endocarditis?
- Inflammation of endocardium that lines the surface of cardiac valves
- Usually due to bacterial infection

Pathogen Causes
1 Strep. viridans Most common cause of
endocarditis due to
dental procedure
2 Staph. aureus Most common cause of
endocarditis in IV drug
users
3 Staph. epidermidis Key organism to cause
endocarditis of prosthetic
valves
4 Strep. bovis Cause endocarditis in pt
with underlying
colorectal carcinoma
5 Coxiella burnetti Most common cause of
endocarditis with
negative blood culture

6 HACEK organisms Cause endocarditis with


- Hemophilus negative blood culture
- Actinobacillus
- Cardiobacterium
- Eikenella
- Kingella

1. Describe endocarditis due to S. viridans.


- Most common cause of endocarditis.
- Low virulence pathogen therefore mainly infects previously damaged valve in case of
chronic rheumatic heart disease and mitral valve prolapse.
- Results in small vegetations that that do not destroy the valves hence cause subacute endocarditis
- Pathogenesis of endocarditis due to S. viridans
- Damaged endocardial surface develops thrombotic vegetations (platelets and fibrin).
- During transient bacteremia (ex - dental procedure), bacteria can be trapped in these vegetations.
- Prophylaxis: prophylactic antibiotics decrease risk of endocarditis.

4. Describe endocarditis due to S. aureus.


- Most common cause of endocarditis in IV drug abusers
- High-virulence organism that infects normal valves (most commonly the tricuspid valve)
- Results in large vegetations that destroy the valve causing acute endocarditis
- Endocarditis may lead to glomerulonephritis and positive Rheumatic factor.
- If valve vegetation is >1cm, consider surgery of valves

Fig - staph aureus endocarditis

5. Describe clinical presentation of endocarditis.


- Fever - due to bacteremia
- Murmur - due to vegetations on heart valve
- Janeway lesion - erythematous non tender lesions on palms and soles
- Osler nodes - tender lesions on fingers or toes
- Roth spots
- Nailbed splinter hemorrhage - due to embolization of septic vegetations

9. Describe Laboratory findings.


- Positive blood cultures
- Hemolytic anemia (usually microcytic) - due to chronic
inflammation presents as ↑Hepcidin, ↑Hb, ↓MCV, ↑ferritin,
↓TIBC, ↓serum iron and ↓ %saturation
- Echocardiogram [useful for detecting lesions on valves]:
- Surface Echocardiogram - 60% sensitive
- Transesophageal echocardiogram - 90% sensitive

6. How does endocarditis lead to low blood iron.


- Acute phase reactant proteins are made (hepsidin being one major
one). Hepsidin traps iron in storage site. This leads to high ferritin. Fig - roth nodules (retinal hemorrhage);
Also, bone marrow takes iron from blood because hepsidin is osler and roth nodules are immunologic.
trapping iron in storage site. That’s how serum iron decreases.

9. Describe Nonbacterial thrombotic endocarditis.


- Sterile vegetation seen on mitral valve along lines of closure
- Leads to mitral valve regurgitation
- Occurs if pt with :
- hypercoagulable state
- underlying adenocarcinoma

8. Describe Libman-Sacks endocarditis.


- Sterile vegetations on both side of mitral valve
- Leads to mitral valve regurgitation
- Associated with lupus [SLE]
Cardiomyopathy

1. What is cardiomyopathy ?
- Group of myocardial diseases that result in cardiac dysfunction

Dialated cardiomyopathy

1. Describe dialated cardiomyopathy.


- Most common type of cardiomyopathy
- Dilation of all four chambers of the heart
- Leads to systolic dysfunction leading to biventricular CHF
- Complications:
○ mitral and tricuspid valve regurgitation
○ Arrhythmia

2. What are some causes of dialated cardiomyopathy?


- Idiopathic in most cases
- Genetic Mutation - usually autosomal dominant
- Myocarditis - due to coxsackie virus A or B
- Alcohol abuse
- Drugs - ex: doxorubicin, cocaine
- Pregnancy - occurs during late pregnancy (3rd trimester)
or soon (weeks to months) after birth

3. Explain the pathogenesis of Coxsackie infection.


- Characterized by a lymphocytic infiltrate in the myocardium.
- Results in chest pain, arrhythmia with sudden death, or heart failure.
- Dilated cardiomyopathy is a late complication.

Fig - myocarditis; notice the presence of lymphocytes. Most common cause is coxcakie virus;
acutely, it can cause death; in chronic cases, it can cause dialated cardiomyopathy

3. What is treatment for dilated cardiomyopathy?


- Heart transplant
Hypertrophic cardiomyopathy
4. Describe hypertrophic cardiomyopathy.
- Massive hypertrophy of left ventricle

5. What are some causes of hypertrophic cardiomyopathy ?


- genetic mutations in sarcomere proteins
- most common form is autosomal dominant.

5. What are its clinical presentation?


- Decreased cardiac output- Left ventricular hypertrophy leads to diastolic dysfunction
- Sudden death in young athletes due to ventricular arrhythmias
- Syncope with exercise - Subaortic hypertrophy of the ventricular septum results in
functional aortic stenosis
- Biopsy : Myofiber hypertrophy with disarray

Fig - myofiber hypertrophy and disarray (fibers oriented in different directions)common in


hypertrophic cardiomyopathy

Restrictive cardiomyopathy

6. What is restrictive cardiomyopathy and it's causes?


- Diastolic dysfunction - Decreased compliance of the ventricular endomyocardium
- Causes
○ Amyloidosis
○ Sarcoidosis
○ Hemochromatosis
○ Endocardial fibroelastosis (in children)
○ Loeffler syndrome - endomyocardial fibrosis with an eosinophilic infiltrate &
eosinophilia

7. What is the presentation of restrictive cardiomyopathy?


- Congestive heart failure
- EKG findings shows:
- Low voltage EKG
- Diminished QRS amplitudes
Cardiac Tumors

Myxoma Rhabdomyoma Metastasis


Most common Primary cardiac Most common cardiac Most common cardiac tumor
tumor in adults tumor in children
Benign mesenchymal tumor with Benign hamartoma of
abundant ground substance in cardiac muscle
histology & gelatinous appearance
Pedunculated mass in left atrium Usually seen in ventricle Most commonly affects pericardium
and seen as pericardial effusion

Associated with tuberous Breast, lung carcinoma, melanoma and


sclerosis lymphoma common source of
metastasis

Myxoma

1. What is most common primary cardiac tumor in adults?


- Myxoma

2. What type of tumor is myxoma?


- Benign mesenchymal tumor with a gelatinous appearance and abundant ground substance on histology

2. What is its presentation?


- Pedunculated mass in the left atrium that causes syncope (temporary loss of
consciousness caused by a fall in blood pressure) due to obstruction of the mitral valve

Fig: biopsy (abundant ground substance) and autopsy of myxoma


Rhabomyoma
1. What is most common primary cardiac tumor in children?
- Rhabdomyoma

2. What kind of tumor is rhabdomyoma?


- Benign hamartoma of cardiac muscle
- Usually seen in ventricle

2. What is it associated with?


- Tuberous sclerosis
- A genetic disorder that causes non-malignant tumors in many different organs, primarily in the
brain, eyes, heart, kidney, skin and lungs.
- Common presentation: seizures, developmental delay, intellectual disability and autism

Fig: rhabdomyoma

Metastasis

1. What is most common type of cardiac tumor?


- Metastasis (more common than primary tumors)

2. What are the most Common metastases to the heart ?


- Breast carcinoma
- lung carcinoma
- Melanoma
- lymphoma

3. What is its most common presentation?


- Most commonly affects pericardium resulting in pericardial effusion
Chapter 9: Respiratory Tract
Pathology
9.1 Nasopharynx
Nasopharynx

1. Differentiate angiofibroma and nasopharyngeal carcinoma.


Angiofibroma Nasopharyngeal carcinoma
Defn Benign tumor of nasal mucosa Malignant tumor of nasopharyngeal epithelium
made up of large blood vessels
and fibrous tissue
Demograp Classically seen in adolescent - Classically seen in Chinese young adults and
hics males - very rare in females african kids

Presentati Presents with profuse epistaxis - Associated with EBV


on (nose bleed). - Often involves cervical lymph nodes
Biopsy Biopsy- pleomorphic keratin positive epithelial
cells (poorly differentiated Sq.Cell Carcinoma)
in background of lymphocytes.

2. Differentiate Rhinitis and Nasal polyp.


Rhinitis Nasal polyp
Cause Rhinovirus - Secondary to repeated rhinitis
Adenovirus - Associated with common cold in children.
- Occurs in cystic fibrosis (in kids) and aspirin-
intolerant asthma (in adults)
Inflammation of the nasal
mucosa.

Presentatio Runny nose (common cold), Protrusion of edematous, inflamed nasal mucosa
n sneezing & congestion

3. What is allergic rhinitis? What's it's associated with?


- A subtype of rhinitis caused due to type 1 hypersensitivity reaction (ex - pollen)
- Association: - Presentation:
○ Asthma ○ Eosinophilic infiltrate
○ Eczema

4. What is aspirin intolerant asthma?


- Seen in 10% of asthma patients
- Presents as triad of :
→ asthma
→ aspirin induced bronchospasm
→ nasal polyps.
9.2 Larynx
Larynx
1. Differentiate laryngeal papilloma and laryngeal carcinoma
Laryngeal papilloma Laryngeal carcinoma
Defn Benign papillary tumor of vocal cord Sq.cell carcinoma of vocal cord epithelium.
Cause HPV 6 and 11 alcohol and tobacco (papilloma rarely
progress to carcinoma)
Presentatio - Hoarseness of voice - Hoarseness of voice
n - Usually single in adults and multiple - Cough and stridor
in children.

2. Differentiate acute epiglottitis and laryngotracheobronchitis (croup).


Acute epiglottitis laryngotracheobronchitis (croup)
Cause H. Influenzae type b - most common cause Parainfluenza virus - most common
(especially in non immunized children). cause.
Present high fever, sore throat, drooling w/ dysphagia, hoarse voice, barking cough and
ation muffled voice, inspiratory stridor. inspiratory stridor
xray Thumb sign on xray Steeple sign on Xray

Risk of actue airway obstruction (medical Inflammation of the upper airway


emergency) - Inflammation of the epiglottis
3. What is vocal cord nodule (singer's nodule) ? what is its cause ?
- Nodule on true vocal cord
- Caused due to excessive use of vocal cord; usually bilateral
- Composed on degenerative (myxoid) connective tissue
- Presents with hoarseness.
- Resolves with resting of voice.

Fig: vocal cord nodule (usually bilateral and seen on true vocal cord)

Pulmonary Page 1.3


9.3 Pulmonary Infections
Pneumonia
1. What is pneumonia? What causes it?
- Infection of lung parenchyma (functional tissue) and stroma (supporting tissue).
- Causes [occurs when normal defenses are impaired] :
○ Lack of cough reflex
○ Damage to mucociliary escalator
○ Mucus plugging

2. What are presentation of pneumonia?


- Fevers and chills (organism usually leak out to blood).
- Productive cough with yellow-green (pus) or rusty (blood) sputum.
- Tachypnea (to compensate for decreased gas exchange in lungs) with pleuritic chest pain.
(inflammation produces bradykinin and PGE2 which causes pain).
- Decreased breath sounds and dullness to percussion (due to loss of air volume due to exudates).
- Elevated WBC count (due to infection).

3. How do you diagnose pneumonia?


- Chest X-ray
- Sputum gram stain and culture.
- Blood culture

4. What are patterns of pneumonia seen on Xray?


Lobar pneumonia Bronchopneumonia Interstitial pneumonia (aka atypical pneumonia)
- consolidation - scattered patchy - Diffuse interstitial infiltrates (of immune cells) seen on Xray.
(accumulation of consolidation. - Presents with relatively mild upper respiratory symptoms:
pus --> hardning of - often multifocal and - minimal sputum
soft aerated lungs) bilateral - low fever
of an entire lobe. - Patchy infiltrates on - 'atypical' presentation.
CXray
- affects the alveoli
surrounding the
bronchioles

Cause is mostly Cause is mostly - Caused by bacteria or viruses.


bacterial bacterial

Treat: ceftriaxone

Fig - lobar pneumonia (right), bronchopneumonia (middle), interstitial pneumonia (left). Note lack of
alveolar exudate in interstitial pneumonia.
Lobar pneumonia Association
Strep pneumo [95%] • Most common cause of ca. & s. pneumonia; seen in middle aged adults & elderly.
Klebsiella pneumoniae • Enteric flora that is aspirated.
• Affects malnourished individuals & elderly in nursing homes, alcoholics, and diabetics.
• Bacterial thick mucoid capsule results in gelatinous sputum (currant jelly)
• Often complicated by lung abscess

Bronchopneumonia Association

Staph aureus • Second Most common cause of secondary pneumonia (bacterial pneumonia
superimposed on viral upper respiratory tract infection)
• Often complicated by abscess or empyema (free pus in pleural space)
Haemophilus • Common cause of secondary (s) pneumonia and pneumonia superimposed on
influenzae COPD (leads to exacerbation of COPD)
Pseduomonas • Pneumonia in cystic fibrosis patients
aeruginosa
Moxarella catarrhalis • Community acquired (ca) pneumonia
• Pneumonia superimposed on COPD (leads to exacerbation of COPD)
Legionella • Community-acquired pneumonia
pneumophila • Pneumonia superimposed on COPD or immunocompromised states
• Transmitted from water source [Pt. Presents with pneumonia, diarrhea & hyponatremia]
• Intracellular organism is best visualized by silver stain

Interstitial Association
pneumonia
Mycoplasma • Most common cause of atypical pneumonia; not visible on gram stain due to lack of cell wall.
pneumoniae • Affects young adults living in close quarters (military recruits, college students)
• Complications: autoimmune hemolytic anemia (IgM against I antigen on RBC
causes cold hemolytic anemia) and erythema multiforme.
Chlamydia • 2nd most common cause of atypical pneumonia in young adults
pneumoniae
Respiratory • Most common cause of atypical pneumonia in infants
syncytial virus (RSV)
Cytomegalovirus • Atypical pneumonia with posttransplant immunosuppressive therapy
(CMV)
Influenza virus • Atypical pneumonia commonly seen in elderly, immunocompromised, or people
with preexisting lung disease
• Increases risk for superimposed S aureus or H influenza secondary pneumonia
Coxiella burnetti • Atypical Pneumonia with high fever (Q fever)
(rickettsial • Seen in farmers and veterinarians (Coxiella spores are deposited on cattle by ticks or
organism) are present in cattle placentas)
• Coxiella is distinct from most rickettsiae in 3 ways:
- causes pneumonia
- does not require an arthropod for transmission (highly heat-resistant endospores)
- does not produce a skin rash
5. What are Classical gross phases of lobar pneumonia?
- Congestion - distended vessels --> edema (intra-alveolar) --> consolidation.
- Red hepatization : due to Massive haemorrhagic exudation: neutrophils, RBC, Fibrin fill the
alveolar air space giving the normally spongy lung a solid consistency --> consolidation.
- Gray hepatization - due to degradation of RBC and fibrosuppurative exudate.
- Resolution - Exudate is broken down by enzymes and ingested by macrophages. Lung tissue is
regenerated by type II pneumocyte.

6. What is aspiration pneumonia?


• Seen in patients at risk for aspiration (ex - comatose pt, alcoholics)
• Causes: anaerobic bacteria of oropharynx:
○ Bacteroides
○ Fusobacterium
○ Peptococcus
• Classic presentation:
○ Right lower lobe abscess (coz the right main stem bronchus branches
at a less acute angle than the left).

Tuberculosis

1. Describe presentation of primary TB.


• Caused due to inhalation of aerosolized Mycobacterium tuberculosis
• Presentation:
○ Focal caseating necrosis classically in lower lobe and hilar lymph nodes
○ The foci undergo fibrosis and calcification resulting in Ghon complex formation
○ Mostly asymptomatic
○ Leads to positive PPD Skin Test.

Fig: Ghon complex (calcified and fibrosed lung): classic location is subpleural region near hilar nodes

1. Describe presentation of secondary TB (aka reactivated TB).


• Caused due to reactivation of Mycobacterium tuberculosis seen in AIDS pt or old people.
• Presentation:
○ Occurs at apex of lung (relatively poor lymphatic drainage and high oxygen tension)
○ Forms cavitary foci of caseous necrosis
○ Symptoms:
▪ Fevers and night sweats
▪ Cough with hemoptysis [Erosion of bronchial vessels]
▪ Weight loss
• Biopsy - reveals caseating granulomas and AFB stain reveals acid-fast bacilli
• Secondary TB may lead to miliary TB or tuberculous bronchopneumonia

9. What are classic locations for spread of miliary TB? Fig: Cavitary lesion /
• Kidney - most common organ to be involved in Systemic spread (sterile pyuria) Cavitary pneumonia
• Meninges (classical location is in the base of brain)
• Cervical lymph nodes
• Lumbar vertebrae (pott's disease)
9.4 Chronic Obstructive Pulmonary Disease

1. What is COPD? What are some findings?


• Group of diseases characterized by airway obstruction; lung does not empty, and air is trapped.
• Findings:
○ ↓ FEV1:FVC ratio
- ↓ FVC - Forced vital capacity
- ↓↓ FEV1 - Volume of air that can be forcefully expired during the first second of expiration.
- Normal FEV1:FVC ratio is 80% (4/5)
○ ↑ TLC (total lung capacity) due to air trapping
- Normal TLC = 7 L

2. What are some causes of COPD?


- Chronic bronchitis (blue bloaters)
- Emphysema (pink puffers)
- Asthma
- Bronchiectasis
Chronic bronchitis

3. What is chronic bronchitis? What is the mechanism?


- Chronic productive cough lasting at least 3 months over a minimum of 2 years
- Hypertrophy & Hyperplasia of bronchial mucinous glands

8. What is the Histology of airway ?


- Below lamina propria are serous glands (secrete water to humidify air) and
mucous glands (secrete mucus to trap pollutants).
- Epithelium is pseudostratified columnar
- Lamina propria has venules that warm the cold air coming from outside

5. With what is chronic bronchitis highly associated?


- Smoking

Fig: Cross section of chronic bronchitis. Top right portion has


respiratory epithelium, bottom left has cartilage. It's clearly visible
that mucus glands make >50% thickness of airway.

6. What are the clinical features of chronic bronchitis?


- Productive cough due to excessive mucous production
- Hypoxemia : Excessive mucus plugs the airway causing obstruction
- Cyanosis ('blue bloaters') : low PaO2 - inducing Vasoconstriction leading to Cor pulmonale (pulm HTN)
- Mucus plugs trap CO2 : high PaCO2 - increased risk of infection

7. What is Reid index?


- It's the ratio of the thickness of bronchial mucus glands to bronchial wall thickness
- Reid index Increases to > 50% from its normal < 40% value
Emphysema

8. What is emphysema?
• Proteases digests elastin of the acinus wall (no fibrosis) thus leading to:
i) degrading the elasticity of the lungs - Loss of elastic recoil leads to collapse
of terminal bronchioles during exhalation results in obstruction and air
trapping [Bernoulli's principle - high velocity → low pressure].
ii) making it more compliant - irreversible Overinflation of any portion of
the pulmonary acinus.
iii) breaking down alveolar septa (ABM) - Multiple alveoli coalesce to become
one large air space [destruction of alveolar sac].

Fig: Histology of emphysema

9. What is pathophysiology of emphysema?


• Imbalance between protease and antiprotease.
• Inflammation in the lung normally leads to release of proteases by neutrophils and macrophages.

10. Differentiate two classic causes of emphysema (destruction of alveolar air sacs).
Smoking (no. 1 cause) A1AT deficiency (rare cause)
Pollutants cause inflammation that induce A1AT is an important antiprotease that
protease mediated damage to alveoli inhibits protease damage to alveoli
Centriacinar emphysema seen mainly in upper Panacinar emphysema seen mainly in lower
lobes (smoke is lighter than air) lobes
Can cause liver cirrhosis too
Complications: hypoxemia and cor pulmonale Complications: hypoxemia and cor pulmonale
(pulm HTN) (pulm HTN)

11. What is Pulmonary acinus?


• Functional unit of lung - the airspaces distal to a terminal bronchiole comprising of:
- Respiratory bronchiole
- Alveolar duct
- Alveolar sac
- Alveoli
12. Why does A1AT (alpha 1 antitrypsin) deficiency cause cirrhosis?
- A1AT deficiency is due to misfolding of mutated protein which
accumulates in ER of hepatocytes resulting in liver damage.
- A1AT is produced by hepatocytes and exported to blood.

13. What does liver biopsy in A1AT deficiency show?


- Pink-purple, PAS positive globules in hepatocytes.

14. Describe the genetics of A1AT transmission.


- PiM - normal allele
- PiZ - mutant allele
PIMM • Norman healthy person
PiMZ • Heterozygoes mutant; usually asymptomatic w/ ↓ level of
circulating A1AT - Significant emphysema risk with smoking.
PiZZ • Homozygous mutant - Significant risk for panacinar
emphysema and liver cirrhosis.
[Pi = Protease inhibitor]

15. What are clinical presentation of emphysema?


- Dyspnea and cough
- Minimum sputum production
- Prolonged expiration with pursed lips (pink puffer) - creating back pressure to prevent airway collapse.
- Barrel chest - increased anterior-posterior diameter of lung
- Weight loss - using muscles to breathe
- Late complication:
○ Cor pulmonale
○ Hypoxemia (due to destruction of capillaries in the alveolar sac).
Asthma

16. What is asthma?


- Reversible airway bronchoconstriction, most often due to allergic stimuli (atopic asthma)
- Associated with :
- allergic rhinits,
- eczema (atopic dermatitis)
- family history of atopy (tendency of type I hypersensitivity reactions)
- Commonly seen in kids

17. Pathogenesis [type I hypersensitivity reactions] of asthma.


- Allergens induce TH2 phenotype in CD4+ T cells of genetically susceptible patient. TH2 secretes:
IL-4 Induces class switching to IgE
IL-5 Attracts eosinophils
IL-10 Stimulates TH2 and inhibits TH1

18. What is early phase reaction in asthma?


- Reexposure to allergen leads to lgE-mediated activation of mast cell degranulation releasing
preformed histamine and new generation of leukotrienes C4, D4, and E4 leading to
bronchoconstriction & inflammation (increased vascular permeability → Edema).

18. What is late phase reaction in asthma?


- Eosinophils release MAJOR BASIC PROTEIN that damages cells and induces bronchoconstriction

19. What are clinical features of asthma?


- Productive cough, classically with Curschmann spirals admixed with Charcot-leyden crystals.
- Dyspnea & Wheezing in response to allergen exposure (episodic).
- Severe, unrelenting attack can result in status asthmaticus and death.

20. What are biopsy findings in sputum of asthmic pts ?


- Curschmann spirals [mucus plug from subepithelial mucous gland ducts].
- Charcot-leyden crystals [eosinophil lysophospholipase (galectin-10)].
- Creola bodies [ciliated columnar cells sloughed from the bronchial mucosa].

Fig: Carcot leyden crystal on left and curschmann spiral on right

21. What are the causes of Nonallergic asthma (non-atopic asthma) ?


- Exercise
- Viral infection
- Asprin
- Occupational exposure

22. What is presentation of asprin intolerant asthma?


- Nasal polyps [highly associated with cystic fibrosis (CF)]
- Bronchospasm
Bronchiectasis

1. What is bronchieatasis?
- Chronic Necrotizing inflammation with damage to airway walls [smooth muscle & elastic tissue] leading to
Permanent dilatation of bronchioles and bronchi (loss of airway tone results in air trapping).

3. What are come cauases of bronchiectasis?


Causes:
○ Cystic Fibrosis (classic pt) - primary defect in ion transport leading to defective mucociliary action.
---> airway obstruction by thick mucus secretion ---> chronic bacterial infection occurs.
○ Tumor or foreign body that blocks airway (infection behind block can cause necrosis)
Necrotizing infections
○ Kartagener syndrome - inherited defect of the dynein arm, which is necessary for ciliary movement.
Loss of muco-ciliary clearance system leads to Mucus accumulation followed by bacterial overgrowth
resulting in pus filled infection.
○ Allergic bronchopulmonary aspergillosis [ABPA] - Hypersensitivity reaction to Aspergillus leads to
chronic inflammatory damage. Classically seen in asthamic and CF pt.

4. What is presentation of Kartagener syndrome?


- Sinusitis (cilia in nasal sinus not working well)
- Infertility (poor motility of sperm)
- Situs inversus (position of major organs is reversed, e.g., heart is on right side of thorax)
- Bronchiectasis

3. What is presentation and complication of bronchiectasis?


- Cough, dyspnea and foul smelling sputum
- Complication:
○ Secondary systemic amyloidosis - systemic increase in SAA (an acute phase reactant)
produced due to chronic inflammation. SAA is converted to AA which is deposited.
○ Hypoxemia with cor pulmonale

Fig: large dilated structures are airway, not coeleaced alveoli


9.5 Restrictive Diseases
1. What are the BASIC PRINCIPLES of restrictive diseases ?
• Findings :
○ ↑ FEV1:FVC ratio
- ↓FEV1 and ↓↓ FVC
○ ↓ TLC (restricted filling of the lung)
• Most commonly arises due to interstitial diseases of the lung; may also arise with chest
wall abnormalities (e.g., massive obesity)

1. What are 4 examples of restrictive lung disease?


- Idiopathic pulmonary fibrosis (IPF)-----
- Pneumoconioses
- Sarcoidosis
- Hypersensitivity pneumonitis

Idiopathic pulmonary fibrosis

2. What are etiologies (unknown) of Idiopathic pulmonary fibrosis?


Primary Cyclical lung injury → injured pneumocytes release TGF beta → induce fibrosis.
Secondary Drugs (bleomycin & amiodarone) and radiation therapy
Fibrosis of lung interstitium - Treatment is lung transplantation.
2. What are the Clinical features of IPF ?
- Progressive dyspnea and cough
- Fibrosis on lung CT; initially seen in subpleural patches, but eventually results in diffuse
fibrosis with end-stage 'honeycomb' lung
Pneumoconiosis

3. What is pneumoconiosis? What's its pathophysiology?


- Interstitial fibrosis due to occupational exposure; requires chronic exposure to small particles that are fibrogenic
- Pathophysiology - alveolar macrophages ingest fibrogenic material and induce fibrosis

4. What are some examples of pneumoconiosis?


Example Risk group Complication Presentation
Silicosis -sandblasters - High risk of TB (inhibits - Fibrotic nodules in upper lobes of lung
-Silica -silica miners phagolysosome formation)
- High risk for lung
carcinoma
Berylliosis -Beryllium High risk for lung cancer Non caseating granuloma in
-Beryllium miners - lung
-Aerospace - hilar lymph nodes
ind. workers - systemic organs
Asbestosis -Construction -High risk of lung carcinoma - Asbestos body (ferruginous body) seen in
-Asbestos workers (more) and mesothelioma biopsy - ferritin and hemosiderin coat
fibres -Plumbers -Fibrosis of lung and pleura asbestos
-Shipyard (plaques) - Lesions contain long, golden brown
workers fibers with associated iron
Coal -coal miners -Mild exposure results in -Massive exposure leads to diffuse
Workers' anthracosis (collections of fibrosis ('black lung')
Pneumo carbon-laden macrophages) -associated with rheumatoid arthritis
coniosis -not clinically significant (Caplan syndrome)
- Carbon
dust

Figure:
A: Asbestos body (ferritin and hemosiderin coated
asbestos particle);
B: pleural plaque on diaphragmatic pleura;
C: Silicotic nodule, notice lots of collagen with
minimum inflammation
Sarcoidosis

5. What is sarcoidosis?
- Systemic disease seen as Noncaseating granulomas in multiple organs
- Classically seen in African American females
- Most commonly involved tissue
- hilar lymph nodes
- lung
- Other commonly involved tissues include
- uvea (uveitis),
- skin (cutaneous nodules or erythema nodosum)
- salivary and lacrimal glands (mimics Sjogren syndrome)
- heart (cardiac sarcoidosis)
- CNS (neurosarcoidosis)

6. Biopsy findings in sarcoidosis.


- Characteristic stellate inclusions ('asteroid bodies') are often seen within
giant cells of the granulomas - non specific findings.

7. Etiology of sarcoidosis.
- Etiology is unknown;
- Likely due to CD4+ helper T-cell response to an unknown antigen

8. Clinical features of sarcoidosis.


- Dyspnea or cough (most common presenting symptom)
- Elevated serum ACE
- Hypercalcemia (HY)
○ 1-alpha hydroxylase activity of epithelioid histiocytes converts vit. D to its active form
- Fatigue and weight loss w/ Arthritis and joint pain [non-specific findings]

9. Treatment of sarcoidosis.
- Treatment is steroids; often resolves spontaneously without treatment.

Fig: asteroid bodies seen in giant cells of granulomas (present in sarcoidosis)


Hypersensitivity Pneumonitis

6. What is hypersensitivity pneumonitis (aka extrinsic allergic alveolitis)?


- Granulomatous reaction to inhaled organic antigens (aka pigeon breeder's lung).
- Chronic exposure leads to interstitial fibrosis.
- Pathophys:
○ Ab-Ag complex forms in lung that activates neutrophils and eventually lymphocytes
which mediate most damages to tissue
- Presentation:
○ fever, cough, and dyspnea hours after exposure
○ resolves with removal of the exposure

7. How is iron stored in body?


- Free Fe produces ROS by Fenten reaction so cells store free Fe in ferretin protein or hemosiderin
Ferretin Hemosiderin
Intracellular protein that acts as buffer against Fe overload or Intracellular complex made of
shortage (protein can be secreted too) Ferretin and other stuff
Fe in Ferretin can be given out when needed Plasma Ferretin Fe here is poor source to supply
correlates well with total Fe in body; so serum ferretin a to body
common test to access anemia.
9.6 Pulmonary Hypertension

1. What is pulmonary hypertension?


- Mean arterial pressure in pulmonary circulation >25 mm Hg (normal is 10 mm Hg)

2. What are causes of pulmonary HTN?


- Causes can be divided to primary and secondary.
Primary pulmonary HTN Secondary pulmonary HTN
- Etiology is unknown - Due to Hypoxemia (e.g., COPD & interstital lung
- Some familial forms related to BMPR2 disease) or increased volume in the pulmonary
inactivating mutation leading to circuit (e.g., congenital heart disease)
proliferation of vascular smooth muscle. - May also arise with recurrent pulmonary embolism
- Classically seen in young adult females

3. What are clinical characters of pulmonary hypertension?


- Characterized by atherosclerosis of pulmonary trunk (HTN leads to artherosclerosis)
- Smooth muscle hypertrophy of pulmonary arteries and intimal fibrosis
- See plexiform lesions with severe long standing disease

4. Features of Pul. HTN.


- Leads to Right ventricular hypertrophy with eventual cor pulmonale
- Presents with Exertional dyspnea or Right heart failure

Fig: plexiform changes - tuft of capillaries that arise as a complication of HTN


9.7 Respiratory Distress Syndrome (RDS)

Acute respiratory distress syndrome

1. What is ARDS?
- Damage to alveolar-capillary interface (diffuse alveolar damage).
- Leakage of protein-rich fluid leads to edema that combines with necrotic epithelial cells to form
hyaline membranes in alveoli.
- This results in :
○ Impaired gas exchange (thickned diffusion barrier)
○ Collapse of air sacs (increased Surface Tension)

Fig: hyaline membrane seen on alveoli represents ARDS (right); diffuse whiteout seen in Xray in ARDS (left)

2. What are causes of ARDS?


- Mechanism is activation of neutrophils that induces protease and free radical mediated damage of
type I and II pneumocytes.
- It's occurs secondary to (100's of causes)
○ Sepsis
○ Infection
○ Shock
○ Trauma
○ Aspiration
○ Pancreatitis
○ DIC
○ Hypersensitivity reactions
○ Drugs

3. What's presentation of ARDS?


- Hypoxemia and cyanosis
- "White out" on chest x-ray
- Hyaline membrane in alveoli on biopsy

4. How do you treat ARDS? What makes recovery complicated?


- Address underlying cause
- Ventilation with positive end-expiratory pressure (PEEP) - every time patient breathes out, the
lung collapses a little more due to hyaline membrane. Positive pressure at end of respiration
prevents this.
- Recovery may be complicated due to interstitial fibrosis. Type II pneumocytes (stem cells) are
damaged and new cells can't be generated leading to scarring and fibrosis.
Neonatal respiratory distress syndrome

1. What is neonatal respiratory distress syndrome?


- Respiratory distress in newborn due to inadequate surfactant level.
Alveolar collapse and hyaline membranes are seen.

2. Etiology of NRDS?
- Surfactant is made by type II pneumocytes; phosphatidylcholine (lecithin) is the major component.
- Surfactant decreases surface tension in the lung, preventing collapse of alveolar air sacs after expiration.
- Lack of surfactant leads to collapse of air sacs and formation of hyaline membranes.

3. Association of NRDS?
- Prematurity: Surfactant production begins at 28 weeks; adequate levels are not reached until 34 weeks.
(childbirth occurs in 40th week)
○ Amniotic fluid lecithin to sphingomyelin ratio is used to screen for lung maturity.
○ Phosphatidylcholine (lecithin) levels increase as surfactant is produced; sphingomyelin remains constant.
○ A ratio > 2 indicates adequate surfactant production.
- Caesarian section delivery: coz child lacks stress-induced steroids; steroids increase surfactant synthesis & release
- Maternal diabetes: Insulin decreases surfactant production (high blood sugar from mom causes high insulin
production in baby).

4. What is presentation of neonatal respiratory distress syndrome?


- Hypoxemia and cyanosis
- Diffuse granularity of the lung ('ground-glass' appearance) on x-ray
- Increasing respiratory effort after birth; tachypnea with use of accessory muscles and grunting

5. What are it's complication?


- Hypoxemia increases the risk for
○ persistence of patent ductus arteriosus
○ necrotizing enterocolitis.
- Supplemental oxygen used for treatment increases risk of free radical injury
○ retinal injury leads to blindness.
○ lung damage leads to bronchopulmonary dysplasia

Pulmonary Page 6.2


9.8 Lung Cancer
1. What are key risk factors for lung cancer?
- Cigarette smoking
○ Risk directly linked to duration and amount of smoking (pack years)
○ Cigarette smoke Contains > 60 carcinogens
○ Polycyclic aromatic hydrocarbons and arsenic are particularly mutagenic.
○ 85% of lung cancer occurs in smokers
- Radon
○ Colorless, odorless gas
○ Decay product of uranium
○ Found in soil and accumulates in closed spaces (basement)
○ 2nd most frequent cause of lung carcinoma in US
○ Responsible for most of the public exposure to ionizing radiation
○ Increased risk of lung cancer is also seen in uranium miners.
- Asbestos

2. What is presentation of lung cancer?


- Average age at presentation is 60.
- Most common cause of cancer death in USA
- Nonspecific presentation -
○ Cough
○ weight loss
○ hemoptysis
○ post obstructive pneumonia

3. How do you diagnose lung cancer?


- Diagnosis requires biopsy
- Imaging reveals solitary nodule (coin-lesion)
- Benign lesions, which often occur in younger patients
- Presents as
○ Granulomas - due to TB or fungus (especially histoplasma in the midwest)
○ Bronchial hamartoma - benign tumor composed of lung tissue + cartilage; often calcified on imaging.

4. Describe the TNM staging of lung cancer.


- T - Tumor size and local extension
i. Pleural involvement classically seen in adenocarcinoma
ii. Obstruction of SVC (superior vena cava syndrome) - distended head and neck veins with edema and blue
discoloration of arms and face
iii. Involvement of recurrent laryngeal nerve (hoarseness) or phrenic nerve (diaphragmatic paralysis)
iv. Horner's syndrome - due to compression of sympathetic chain by an apical (Pancoast) tumor leads to
- ptosis (drooping eyelid), miosis (pinpoint pupil), anhidrosis (no sweating).
- N - spread to regional lymph nodes (hilar and mediastinal)
- M - unique site of distant metastasis is the adrenal gland (Others - brain, bone, liver).

5. What's prognosis of lung cancer?


- Poor prognosis - often presents late (no effective screening method)
- 5 year survival rate is 15%
6. What are two categories of lung cancer?
Small cell carcinoma (15% of all lung carcinoma) Non-small cell carcinoma (85% of all lung carcinoma)
No subtypes - Adenocarcinoma (40%) - glands or mucus production
- Squamous cell carcinoma (30%) - keratin pearls or intercellular
bridges
- Large cell carcinoma (10%) - none of above features seen
- Carcinoid tumor (5%)
- Usually not amenable to surgical resection - Treated upfront with surgical resection
- treated with chemotherapy - doesn't respond well to chemotherapy

7. What are different types of lung cancer?


S.N Cancer type Remarks
1 Small Cell carcinoma - Treat with chemotherapy
2 Non-small cell carcinoma Subtype
Squamous cell carcinoma
Adenocarcinoma
Broncheoalveolar carcinoma (Adenocarcinoma in situ)
Large cell carcinoma
(Bronchial) carcinoid tumor

3 Mesothelioma - related to asbestos


4 Metastasis - Common origin of metastasis - breast, colon

8. Classify the different types of lung cancer.


Neuroendocrine Adenocarcinoma Related to smoking Paraneoplastic Undifferentiat Excellent
(NE) tumor syndrome ed and poor prognosis
prognosis
Small cell Adenocarcinoma Squamous cell carcinoma Squamous cell Small cell Bronchio
carcinoma (most common in male carcinoma carcinoma alveolar
(poorly smokers (PTHrp) carcinoma
differentiated
NE cells)
(Bronchial) Bronchioalveolar Small cell carcinoma (male Small cell Large cell
carcinoid tumor carcinoma smokers) carcinoma carcinoma
(well (adenocarcinoma (ADH, ACTH, Ab
differentiated in situ) for Ca channel)
NE cells)
-Adenocarcinoma (most Large cell
common in female smokers carcinoma
and non-smokers) - (B-HCG)
-Broncheoalveolar carcinoma
(not associated with smoking)
Large cell carcinoma
9. Describe the following types of cancer.

Cancer Histology Association Location Remarks


Small cell - Poorly differentiated small cells - Male smokers Central - MOST AGGRESSIVE TYPE OF
carcinoma and very aggressive (99% cases) LUNG CANCER (rapid growth
- Arise from neuroendocrine and early metastatis)
cells (Kulchitsky cells) - mainly metastatise to BRAIN :
- Chromogranin +ve (less +ve) hence,prophylactic cranial
irraditation is done
- due to Amplification of myc
oncogene

PARANEOPLASTIC SYNDROME
- Produces ACTH or ADH
- Cause LE syndrome (auto Ab
against presyn. Ca+2 channel)
- Anti-neuronal antibody
syndrome (limbic encephalitis,
cerebellar degeneration,
opsoclonus, GI dysmotility,
poly radiculopathy)
Adeno - Glands or mucin - Most common Peripheral
carcinoma tumor in
nonsmokers and
female smokers

Fig: glandular structure in


adenocarcinoma
Squamous - Keratin pearls or intercellular - Most common Central PARANEOPLASTIC SYNDROME
cell bridges tumor in male - May produce PTHrp
carcinoma smokers - Hilar mass from bronchus
- Associated with double C
• HyperCalcemia- due to
PTHrp
• Cavitation

Fig-
intracellular
Fig: keratin pearl bridge
Large cell - Poorly differentiated large and - Smoking Central or - Poor prognosis
carcinoma highly anaplastic cells peripheral - Poor response to
(no keratin pearls, intercellular chemotherapy; remove
bridges, glands or mucin) surgically
- Paraneoplastic syndrome -
may secrete B-HCG
(Bronchial) - Well differentiated - Not related to Central or - MOST COMMON PRIMARY
carcinoid neuroendocrine cells; smoking peripheral LUNG CANCER IN CHILDREN
tumor chromogranin positive (when central, - Low grade malignancy; rarely,
makes polyp can cause carcinoid
like mass in syndrome - caused due to
bronchus) release of vasoactive
substance (mainly serotonin):
- flushing,
- diarrhea,
- restrictive cardiomyopathy
Fig: chromogranin positivity
(due to fibrosis)

Bronchiolo - Columnar cells that grow along - Not related to Peripheral - Excellent prognosis
alveolar preexisting bronchioles and smoking - Pneumonia like consolidation
carcinoma alveoli; arise from clara cells on imaging

Fig: normal alveoli top right;


columnar cells on rest
Metastasis - Most common source are Multiple - More common than primary
to lung breast and colon carcinoma Canon-ball tumors
nodules on
imaging

Mesotheli - See psammoma bodies in - Highly associated - Malignant tumor of


oma biopsy (concentric with asbestos mesothelial cells
calcifications - other HY exposure (lung (mesothelium is a membrane
cancer - papillary thyroid, cancer more of simple squamous cells that
meningioma, papillary serous common in lines body cavities: pleura ,
ovarian) asbestos peritoneum, mediastinum and
exposure) pericardium)
- Tumor encases the lung
9.9 Pleura
1. Describe anatomy of pleura?
- It is lined by mesothelial cells
- mesothelium produces pleural fluid

2. What are differences between spontaneous and tension pneumothorax?


Spontaneous pneumothorax Tension pneumothorax
- Due to rupture of - Due to penetrating chest wall injury (Air enters the pleural
emphysematous bleb space, but cannot exit)
- Often seen in young adults
- Results in collapse of a portion
of the lung
- Xray: trachea shifts to side of - X-ray: trachea is pushed opposite to the side of injury.
collapse. - Medical emergency is treated with chest tube insertion.

4. What is Pneumothorax
- Accumulation of air in the pleural space.

Fig: spontaneous pneumothorax (no tracheal shift)

Fig: mesothelioma (tumor encasing the lung)

3. Describe mesothelioma (malignant neoplasm of mesothelial cells).


- Presentation:
○ Recurrent pleural effusion, dyspnea, and chest pain.
○ Tumor encases the lung
- Risk Factor
○ Occupational exposure to asbestos (lung cancer far more likely).
Chapter 10: Gastrointestinal
Pathology
10.1 Oral Cavity

1. Describe the following oral cavity pathologies


Presentation Cause
Cleft palate - Full-thickness defect of lip or palate Facial prominences (one from superior, two
and lip - Both usually occur together from the sides, and two from inferior) grow
otherwise less common. but don't fuse together - early pregnancy.
Apthous - Painful superficial ulceration of oral - Arise in relation to stress but resolves
ulcer mucosa - grayish base (granuloma) spontaneously
surrounded by erythema - Often recur
Behcet - Recurrent apthous ulcer, genital - Viral infection but etiology is unknown.
syndrome ulcer and uveitis - I.complex vasculitis involving small vessels
Oral herpes - Vesicles involving oral mucosa that - HSV1: Primary infection occurs in childhood
rupture, resulting in shallow, - Stress and sunlight can reactivate virus
painful, red ulcers. leading to vesicles that often arise
- Lesions heal after P.infection. on the lips (cold sores)

2. Describe the squamous cell carcinoma of oral cavity.


Description - Malignant neoplasm of squamous epithelia of oral cavity
Risk factor - alcohol and tobacco
Common site - Floor of mouth
Precursor - Oral Leukoplakia
lesions • White plaque that can't be removed [unlike oral candidiasis] -
represents squamous cell dysplasia - increases risk of cancer.
- Erythroplakia (red plaque)
• Vascularized leukoplakia and is highly suggestive of squamous cell
dysplasia

3. Describe hairy leukoplakia


- White, rough ('hairy') patch that arises on the lateral tongue.
- EBV induced squamous cell hyperplasia and is often seen in AIDS pts.
- It is not pre-malignant.

Fig: erythroplakia (right), leukoplakia (middle), hairy leukoplakia (left - not precancerous)

GI Page 1.1
10.2 Salivary Gland

1. Describe the following pathologies of the salivary gland.


Presentation Cause Labs Risk
Mumps - Bilateral inflammation Mumps virus Serum amylase is Orchitis carries
of parotid glands increased due to risk of sterility,
- Orchitis salivary gland or especially in
- Pancreatitis pancreatic teenagers.
- Aseptic meningitis involvement.
Siala - Mostly unilateral - Mostly due to obstructive
denitis inflammation of stone (sialolithiasis) leading
salivary gland to staph Aureus infection.

2. Describe the following cancers of salivary gland (SG)


Cancer Epidemiology Location Composition Presentation
Pleomorphic Most common Mostly in Stromal (cartilage) and -Mobile painless circumscribed
adenoma benign tumor of parotids epithelial cells - mass at the angle of jaw.
SG biphasic tumor -High recurrence; irregular margins
& extension through tumor
capsule makes resection hard.
-Rarely becomes carcinoma - if it
does, it will damage CN 7.
Warthin 2nd most Mostly in Cystic tumor with
tumor common benign parotids abundant lymphocytes
tumor of SG and germinal centers.
Muco Most common Mostly in Mucinous and - Involves CN 7 (facial nerve)
epidermoid malignant tumor parotids squamous cell
carcinoma of SG

GI Page 2.1
10.3 Esophagus

1. Talk about features of following pathologies.

Pathology Description Presentation


Tracheo - Congenital defect - connection b/w trachea and esophagus. - Vomiting
Esophageal - Most common variant has proximal esophageal atresia with - polyhydramnios
Fistula distal esophagus arising from trachea. [inability to
swallow amniotic
fluid]
-aspiration
- abdominal
distension
[inhaled air
reaches stomach]

Esophageal - Thin protrusion of esophageal mucosa, most often in - Dysphagia for


web upper esophagus - partial obstruction. poorly chewed
- Increased risk for esophageal squamous cell carcinoma. food

Plummer ○ Severe iron deficiency anemia


venson ○ Esophageal web
syndrome ○ Beefy red tongue due to atrophic glossitis (inadequate cell
turnover)
○ Usually seen in post-menopausal woman
Zenker - Acquired defect - Outpouching of pharyngeal mucosa in - Halitosis (rotting
diverticulum the muscular wall (false diverticulum). food in
- Arises above the upper esophageal sphincter at the junction diverticulum
of the esophagus and pharynx . leading to bad
breadth)
- Dysphagia
- Obstruction
Mallory - Longitudinal laceration of mucosa at the gastroesophageal - Painful
Weiss (GE) junction. hematemesis
syndrome - Cause is vomiting : usually alcoholism or bulimia
- Risk for Boerhaave syndrome

Boerhaave - Esophageal rupture leading to air in mediastinum and


syndrome subcutaneous emphysema (crepitis).

Esophageal varices
Definition - Dilated submucosal veins (varices) in lower esophagus due to portal
hypertension.

Anatomy - Esogphageal vein is connected to portal vein via left gastric vein
Epidemiology - Most common cause of death in cirrosis pt
Presentation - Asymptomatic or presents as painless hematemesis w/ bleeding disorder

Achalasia
Definition - Disordered esophageal motility with inability to relax lower esophageal sphincter
(LES) due to damage of ganglion cells in myenteric plexus
Etiologies - Idiopathic or
- Trypanosoma cruzi infection in Chagas disease.
Clinical - Dysphagia of solid and liquid food
features - Putrid breath
- High LES pressure on esophageal manometry
- 'Bird beak' sign on barium swallow study.

Complication - Increased risk for squamous cell carcinoma


Gastroesophageal reflux disease (GERD)

Definition - Reflux of stomach acid due to reduced LES tone


Risk factor - Hiatal hernia [sliding or rolling type]
- alcohol, tobacco
- Obesity
- fat rich diet & Caffeine
Clinical features - Heartburn (mimics cardiac chest pain)
- Asthma (adult-onset) and cough
- Damage to enamel of teeth
Complications - Barrett esophagus [in 10% pt] and Ulcerated strictures.

Barrett's esophagus

Definition - Metaplasia in LE mucosa from non keratinized st. sq. ep. to nonciliated cl. ep.
- with goblet cells. Response of lower esophageal stem cells to acidic stress.
Complication - May progress to Dysplasia and adenocarcinoma.

Histology

Fig: squamous epithelia on left, columnar epithelia with goblet cells (right) - barret's
esophagus
Esophageal carcinoma

Adenocarcinoma Squamous cell carcinoma


Epidemiolo - Most common esophageal cancer in western - Most common esophageal cancer
gy world (involves lower 1/3rd of esophagus) worldwide (arises from Upper or
due to Malignant proliferation of glands. Middle 3rd of esophagus)
Risk factor - Barrett esophagus - Alcohol and Tobacco (the most
common cause)
- Very hot tea
- Achalasia
- Esophageal web
(ex : Plummer-Vinson synd).
- Esophageal injury
(ex : lye ingestion).

Prognosis - Poor (both type presents late)


Presentation - Progressive dysphagia of solid and liquid food
- Weight loss, pain
- Hematemesis
- Squamous cell carcinoma may additionally present with hoarse voice
(recurrent laryngeal nerve involvement) and cough (tracheal involvement).
Lymph node Upper 1/3rd E.carcinoma Cervical nodes
involvement

Middle 1/3rd E.carcinoma Mediastinal or tracheobronchial


nodes
Lower 1/3rd E.carcinoma Celiac and gastric nodes

GI Page 3.4
10.4 Stomach

Sections of stomach

Gastroschisis and Omphalocele (schisis = break; omphalo = navel; cele = swelling)


Gastroschisis - Congenital malformation of the anterior abdominal wall leading to exposure of abdominal contents.
Omphalocele Etiology - Persistent herniation of bowel into umbilical cord.
- Due to failure of herniated intestines to return to the body cavity during development.
→ Contents are covered by peritoneum and amnion of umbilical cord.

Fig: omphalocele (middle), gastroschisis (right)

Pyloric stenosis
Definition - Congenital hypertrophy of pyloric smooth muscle
Epidemology - More common in males
- Presents after 2 weeks of birth
Presentation - Visible peristalsis and Projectile non-bilious vomiting after eating.
- Olive-like mass in the abdomen.

Fig: visible peristalsis after eating in a kid with pyloric stenosis


Treatment - Myotomy (muscles of pyloric sphincter are cut open)
Acute and chronic gastritis

Acute gastritis Chronic gastritis


Definition - Acidic damage to stomach mucosa - Chronic inflammation of stomach mucosa that leads to intestinal
metaplasia
Defense - Mucin produced by foveolar cells
against - HCO3- secretion by surface epithelium.
acid - Normal blood supply (provides nutrition
and picks up leaked acid).

nb - Acid damage results in :


- superficial inflammation
- erosion (loss of superficial ep.)
- ulcer (loss of mucosal layer).

Fig: intestinal metaplasia seen in chronic gastritis


Etiologies - Severe burn (aka Curling ulcer) - - Chronic autoimmune gastritis
Hypovolemia → decreased blood supply. - Chronic H. Pylori gastritis
- Increased intracranial pressure (aka
Cushing ulcer) - ↑ stimulation of vagus
nerve leads to ↑ acid production.
- NSAIDs : decreased PGE2 leading to :
↑ acid production
↓ mucus production
↓ HCO3- production
↓ blood flow
- Chemotherapy
- Shock - Multiple (stress) ulcers in ICU pt.
- Heavy alcohol consumption

Differentiate chronic autoimmune gastritis and chronic H pylori gastritis.

Chronic autoimmune gastritis Chornic H pylori gastritis


Defn - T cell mediated damage [type 4 hypersensitivity - - H pylori produces urease (to create basic environment)
antibodies against gastric parietal cells (body & and protease (to damage mucosal defense) - acidic
fundus) and/or intrinsic factor]. damage to stomach.
Epi. - Most common cause of gastritis (90%)
- Antrum is most common site.
Cli. - Atrophy of mucosa with intestinal metaplasia (leads to - Intestinal metaplasia - high risk of adenocarcinoma
Feat. high risk of stomach adenocarcinoma). (intestinal type).
- Achlorhydria (absence of HCL) with increased gastrin - Epigastric abdominal pain
levels due to antral G cell hyperplasia. - Increased risk of ulceration (peptic ulcer disease)
- Megaloblastic anemia (pernicious anemia) - due to - MALT lymphoma.
lack of intrinsic factor.

Lab - Ab against parietal cells and/or intrinsic factor (IF). - Positive urea breath test.
- Stool antigen detection.
Treat. - Triple therapy:
[PPI + marcolide (clarithromycin) + amoxicillin].

Comp. Increased risk of gastric adenocarcinoma Increased risk of adenocarcinoma (intestinal type)
(intestinal type) and MALToma
Peptic ulcer disease

Definition - Solitary mucosal ulcer involving proximal duodenum (90% of cases) or distal stomach (10%)
Differenc Duodenal ulcer Gastric ulcer
es
Cause - 95% due to H. pylori - 75% due to H. pylori
- Rarely due to Zolinger Ellision (ZE) syndrome - NSAID use
- Bile reflux
Present. - Epigastric pain that improves with meal - Epigastric pain that worsens with
meal
Location - Mostly in anterior duodenum. - Ulcer is located on the
lesser curvature of antrum
Risk - If ulcer in posterior duodenum, rupture may lead to bleeding - Rupture carries risk of bleeding
from gastroduodenal artery or acute pancreatitis from left gastric artery
Biopsy - Ulcer with hypertrophy of Brunner glands
Risk of - Duodenal ulcers almost never malignant (duodenal carcinoma - May be caused by gastric
carcinoma extremely rare) carcinoma (intestinal subtype)

How do you differentiate if gastric ulcer is due to gastric carcinoma or not?

Benign gastric peptic ulcer Malignant gastric peptic ulcers (intestinal type)
- Usually small (<3cm), sharply demarcated (punched out), and - Usually large and irregular with heaped up margins.
surrounded by radiating folds of mucosa (biopsy is needed for (biopsy is needed for definitive diagnosis).
definitive diagnosis).
Gastric carcinoma (adenocarcinoma) - Malignant proliferation of surface epithelial cells

Classes - Intestinal type (more common)


- Diffuse type
Presentatio - Presents late with weight loss, abdominal pain, anemia and early satiety
n - Rarely presents as acanthosis nigricans (hyperplasia and darkening of epidermis) or Leser-Trelat sign (lots of
seborrheic keratosis)
Metastasis - Liver (most common) and Spreads to left supraclavicular lymph node (Virchow node).
- Periumbilical region (Sister Mary Joseph nodule) Intestinal type

- Bilateral ovaries (Krukenberg tumor) Diffuse type


Differentiate intestinal and diffuse type of gastric carcinoma

Intestinal type Diffuse type


Epid More common
Presentati - Large irregular ulcer with - Desmoplasia results in thickening of stomach wall (linitis plastica).
on heaped up margin

Location - Lesser curvature of gastric


antrum.
Risk - Intestinal metaplasia ( H. pylori - Not associated with any factors
factor and autoimmune gastritis)
- Nitrosamine (present in
smoked food- japan has high
rates)
- Blood type A
Histology - Signet ring cells that diffusely infiltrate the gastric mucosa - nucleus pushed
to edge as tumor cell produces lots of mucin - looks like ring.

Mets - Periumbilical region (Sister - Bilateral ovaries (Krukenberg tumor)


Mary Joseph nodule)
10.5 Small Bowel

Duodenal atresia

Defn - Congenital Failure of duodenum to canalize


Association - Down syndrome
Clinical features - Bilious vomiting
- Polyhydramnios (unable to digest a.fluid)
- Distension of stomach and blind loop of duodenum (double bubble sign)

Fig: bilious vomit (left), double bubble sign (right)

Merkel diverticulum (MOST COMMON CONGENITAL GI ANOMALY)


Defn - Outpouching of all three layers of bowel wall (true diverticulum) due to failure of vitelline
duct to involute (partial involution)
Anatomy - Developing midgut receives nutrients through vitelline duct from yolk sack
- Vitelline duct forms in 4th week and involutes in 7th week
Partial involution of vitelline duct Merkel diverticulum
No involution Meconium comes out through umbilicus

Rule of 2 - Seen in 2% of population (most common congenital anomaly of the GI tract).


- 2 inches long
- Located 2 feet away from ileocecal valve.
- Can present during the first 2 years of life with bleeding.
Presentation - Most cases are asymptomatic
- Bleeding is due to heterotopic gastric mucosa.
- Volvulus, intussuception (telescoping) or obstruction (MIMICS APPENDICITIS).
Volvulus and intussuception

Volvulus Intussusception
Defn - Twisting of bowel along its - Telescoping of proximal segment of bowel forward into distal
mesentery segment.
- Results in obstruction and - Telescoped segment is pulled forward by peristalsis, Resulting
infarction [b.supply disruption]. in obstruction and infraction [b.supply disruption].
Cause - Associated with leading edge (focus of traction)
• In kids, most common cause is rotavirus related
lymphoid hyperplasia - mostly occurs in terminal ileum
leading to intussuception into cecum
• In adults most common cause is tumor
Pres - Most common location are - See currant jelly stool
sigmoid colon (elderly),
cecum (young adult)

Small bowel infraction

Feature - Small bowel highly susceptible to ischemic injury .


- Abdominal pain, bloody diarrhea and decreased bowel sounds
Types Cause
Transmural - Thrombosis/embolism of superior mesenteric artery (a-fib, vasculitis) or
infarction - Thrombosis of mesenteric vein (polycythemia vera, lupus)
Mucosal - marked Hypotension
infarction

Lactose intolerance

Defn - Hypofunction of lactase enzyme found in brush-border of enterocytes. Enzyme breaks


lactose into glucose and galactose
Presentation - Lactose osmotically active (abdominal distension and diarrhea after eating milk products)
Cause - Congenital : rare autosomal recessive disorder
- Acquired : develops in late childhood
- Temporary deficiency : after infection (lactase is highly susceptible to injury)
Celiac disease

Defn - Immune mediated damage (hypersensitivity) to small bowel villi due to gluten exposure.
- Damage most prominent in duodenum (leading to Fe deficiency).
Genetics - Related to HLA DQ2 and HLA DQ8
Pathophys - Gliadin is the most pathogenic component of gluten present in wheat and grains.
After absorption, it is deamidated by tissue transglutaminase (tTG). Deamidated gliadin is
presented by APC via MHC II followed by CD4+ T cells mediated tissue damage.
Presentati - Chlidren present with failure to thrive, abdominal distention & diarrhea
on - Adults present with chronic diarrhea and bloating
- Dermatitis herpetiformis : Small, herpes-like vesicles may arise on skin due to IgA
deposition at the tips of dermal papillae.
- IgA deficiency may be seen
- All symptoms resolve with gluten free diet
Lab - IgA Ab is formed against endomysium, tTG, or gliadin.
findings - IgG Ab are diagnosed against these antigens during IgA deficiency.
Biopsy - Flattening of villi, hyperplasia of crypts, increased intraepithelial lymphocytes [Damage is
most prominent in the duodenum; jejunum and ileum are less involved].

Fig - Flattened villi with crypt hyperplasia in Celiac (left) and normal villi (right)
Complicati - Small bowel carcinoma and Enteropathy-associated T-cell lymphoma (EATL) are present as
on refractory disease despite good dietary control.

Tropical sprue

Defn - Damage to small bowel villi due to unknown organism (disease responds to
antibiotics) resulting in malabsorption

Difference from - Damage most prominent in jejunum and ileum (secondary Vit B12 and
celiac folate deficiency may ensue) - duodenum is less commonly involved.
- Arises after infectious diarrhea and responds to antibiotics
- Occurs in tropical regions : ex-Caribbean
Whipple disease

Defn - Systemic tissue damage characterised by macrophages loaded with Tropheryma whippelii.
Presentati - Classic site of involvement is the small bowel lamina propria.
on • Macrophages compress lacteal and interfere with chylomicron transport from
enterocytes to lymphatics - fat malabsorption and steatorrhea
- Other sites include synovium of joints (arthritis), cardiac valves, lymph nodes and CNS
Biopsy - Foamy macrophages in lamina propria of small intestine
- Positive PAS stain - partially destroyed T.whippelii is present in macrophage lysosomes
(mucin is also PAS +ve).

Fig - PAS stain in whipple disease (mucin in goblet cells and bacteria inside macrophages
are staining pink)

Abetalipoproteinemia

Defn - Autosomal recessive deficiency of apolipoprotein B-48 and B-100.


Presenta - Fat malabsorption due to defective chylomicron formation (requires B-48)
- Absent plasma VLDL and LDL (require B-100)
Carcinoid tumor

Defn - Malignant proliferation of neuroendocrine cells; low grade malignancy


Clinical - Can arise anywhere in gut; small bowel is the most common site.
features - Grows as submucosal polyp-like nodule.
- Often secretes serotonin which is released into the portal circulation and metabolized
by liver monoamine oxidase (MAO) into 5-HIAA which is then excreted in urine.

nb -Tumor cells contain neurosecretory granules


that are positive for chromogranin.

Carcinoid Etiology - Carcinoid tumor metastatize to liver allowing serotonin to bypass liver
syndrome metabolism (it is released into h.vein → s. circulation via hepatic shunt).
Trigger - Alcohol or emotional stress that triggers serotonin release from tumor
Presentati - Bronchospasm
on - Facial flushing
- Diarrhea
- Carcinoid heart disease
→ right sided valvular fibrosis (increased collagen) leading to tricuspid
valve regurgitation and pulmonary valve stenosis
→ left-sided valvular lesions are not seen due to presence of MAO
(metabolizes serotonin) in the lung.
10.6 Appendix

Acute appendicitis (MOST COMMON CAUSE OF ACUTE ABDOMEN)

Defn - Acute inflammation of appendix


Cause - Obstruction of appendix by lymphoid hyperplasia (common in kids) and
fecalith (common in adults)
Presen - Periumbilical pain, fever, nausea
- Pain eventually localizes to RLQ (McBurney point)
- Rupture results in peritonitis that presents with guarding and rebound tenderness
Complic. - Periappendicular abscess.
10.7 Inflammatory Bowel Disease (IBD)

BASIC PRINCIPLES
A. Chronic, relapsing inflammation of bowel.
B. Possibly due to abnormal immune response to enteric flora.
C. Classically presents in young women (teens to 30s) as recurrent bouts of bloody diarrhea and abdominal pain.
1. More prevalent in the West, particularly in Caucasians and Eastern European Jews.
D. Diagnosis of exclusion; symptoms mimic other causes of bowel inflammation (e.g.,infection).
E. Subclassified as ulcerative colitis or Crohn disease.
Ulcerative colitis Crohn disease
Wall - Mucosal and submucosal layers - Full thickness with knife like fissures
involve
ment
Locatio - Begins in rectum and can - Anywhere from mouth to anus with skip lesions;
n extend proximally up - Terminal ileum most common - Vit B12 deficiency
to the cecum (involvement is - Rectum is least common
continuous).
- Remainder of the GI tract is
unaffected.

Fig: Crohn (left); UC (right)

Sympto - LLQ pain (rectum) with bloody - RLQ pain (ileum) with non-bloody diarrhea
ms diarrhea
Inflam - Crypt abscess with neutrophils - Lymphoid aggregated with noncaseating granuloma in
mation 40% of cases

Gross - Psudopolyps - Cobblestone mucosa


appear - Loss of haustra ['lead pipe' sign - Creeping fat
ance on imaging]. - Strictures ['string-
sign' on imaging].

Complic - Toxic megacolon - Malabsorption (coz terminal ileum mostly affected)


ation - Carcinoma [depends on extent - Calcium oxalate nephrolithiasis
of colonic involvement & - Fistula formation
duration of disease (≥ 10 yrs)]. - Carcinoma (if colonic disease present)
Smokin - Protects against UC - Increases chance of Crohn disease
g
Associa - Primary sclerosing cholangitis - Ankylosing spondylitis
tion (P-ANCA +ve) - Sacroilitis
- Migratory polyarthritis
- Erythema nodosum
- Uveitis
10.8 Colon

Hirschprung disease

Definition Congenital Failure of Ganglion cell (neural crest-derived) to descend into Myenteric
& Submucosal plexus of distal sigmoid colon and rectum.
Association Down's syndrome
Presentati - Presentation based on Defective relaxation and peristalsis → Obstruction:
on • Failure to pass meconium
• Empty rectal vault on digital rectal exam (DRE)
• Megacolon proximal to obstruction with risk for rupture

nb -
1. Myenteric (Auerbach) plexus is located between the inner
circular and outer longitudinal muscle layers of the
muscularis propria and regulates motility.
2. Submucosal (Meissner) plexus is located in the
submucosa and regulates blood flow, secretions, and
absorption.

Fig: normally, rectum is bigger than colon


Diagnosis Rectal Suction biopsy reveals lack of ganglion cells
Treatment Resection of the involved bowel - ganglion cells are present in the bowel proximal
to the diseased segment.

Colonic diverticula
Definition Outpouching of colonic mucosa and submucosa through muscularis propria (false
diverticulum)

Cause - Increased wall stress due to:


• Constipation, low fiber diet (increased risk with old age) leads to straining.
• Arise where vasa recta traverse the muscularis propria (weak point in colonic wall)
• Most often found in sigmoid colon

Complica. - Mostly asymptomatic


- Rectal bleeding (hematochezia)
- Diverticulitis - due to obstructing fecal material (presents with appendicitis like
symptom in the LLQ).
- Fistula - inflamed diverticulum ruptures and attaches to a local structure
• Colovesicular fistula - presents with air (or stool) in urine.
Angiodysplasia and Hereditary hemorrhagic telangiectasia

Angiodysplasia Hereditary hemorrhagic telangiectasia


Acquired malformation of mucosal and Autosomal dominant disorder resulting thin walled
submucosal capillary beds blood vessels
Mostly seen in cecum and right colon Mostly seen in oral cavity and GI tract.
due to high wall tension
Rupture of capillary beds presents Rupture of blood vessels presents as bleeding
as hematochezia in old adults

Ischemic colitis

Definition Ischemic damage of colon at the splenic flexure : watershed area of SMA.
Cause Atherosclerosis of SMA (superior mesenteric artery)
Presentation - Postprandial pain and wt loss
- Infarction results in pain and bloody diarrhea

IBS (Irritable bowel syndrome)

Presentation - Relapsing abdominal pain with bloating, flatulence and change in bowel
habits (diarrhea or constipation).
- IMPROVES WITH DEFECATION and Increased dietary fiber intake.
Prevalence - Classic pt is middle aged women
Cause - Disturbed intestinal motility (no identifiable pathologic change)
Colonic polyps - Raised protrusions of colonic mucosa
1. Differentiate hyperplastic and adenomatous polyps.
Hyperplastic polyp Adenomatous polyp
Most common colon polyp; usually arise in the Second most common colon polyp
left colon (rectosigmoid section)
Occurs due to hyperplasia of glands (polyclonal) Occur due to neoplastic proliferation of
-classically shows a 'serrated appearance' on glands (monoclonal)
microscopy.
Benign, with no malignant potential Benign but Pre-malignant - may progress
to adenocarcinoma via adenoma-
carcinoma sequence: [hence REMOVED]

2. What is adenoma-carcinoma sequence and what are the steps in it?


- Adenoma-carcinoma sequence describes molecular progression from normal colonic
mucosa → adenomatous polyp → carcinoma.
- Steps:
Mutation Presentation Gene function
APC (adenomatous increases risk of APC inactivates B-catenin (b-catenin
polyposis coli gene) - muta. polyp formation increases Cyclin D transcription and
sporadic or germline progresses cell cycle)
K-ras mutation leads to formation K-ras works in RAS pathway. One function
of polyp in increase Cyclin D transcription
P53 mutation and increased Progression of P53 transcription factor of p21; prolonged
COX expression (aspirin adenoma to activation causes transcription of Bax and
inhibits COX) carcinoma lead to apoptosis

3. How are screening of polyps done?


- Colonoscopy - both hyperplastic and adenomatous polyp look the same;
Hence, all polyps are removed and examined microscopically.
- Fecal occult blood - polyps are usually clinically silent, but can bleed.

4. What types of polyps have greatest risk of progression to cancer [adenoma → carcinoma] ?
- Sessile growth
- Size >2 cm
- Villous histology

5. Age of presentation
- Common age 60-70
Familial adenomatous polyposis (FAP)
Presentation - Presence of 100 or 1000's of adenomatous colonic polyps throughout colon & rectum (↑ propensity)
- Will progress to carcinoma by 40 y of age (hence remove colon & rectum prophylactically)
Mutation - Autosomal dominant disorder
- Inherited APC mutation (chromosome 5)

1. Describe the following syndromes associated with FAP.


Gardner - FAP + fibromatosis + osteoma
syndrome - Fibromatosis - usually presents as desmoplasia in the retroperitoneum and
destroys local tissue (non-neoplastic proliferation of fibroblasts)
- Osteoma - benign tumor of bone; usually present in skull

Turcot FAP + CNS tumor (medulloblastoma and glial tumor)


syndrome

Juvenile polyp, juvenile polyposis and Peutz-Jeghers syndrome


1. Differentiate between the above three.
Juvenile polyp Juvenile polyposis Peutz-Jeghers syndrome
(aka heriditary intestinal
polyposis)
Sporadic, in children <5 years Sporadic or Autosomal dominant Inherited - autosomal
(mutation of SMAD genes) dominant
Usually presents as solitary Multiple juvenile polyp in - Multiple hamartomatous
rectal hamartomatous stomach and colon (benign) polyp in GI tract.
(benign) polyp that prolapses - Mucocutaneous Hyper-
and bleeds pigmentation (freckle like
spots) on lips, oral mucosa
and genital skin.
nb - juvenile refers to type of
polyp, not age of pt.

Large number of Juvenile Increased risk of colorectal,


polyps increase risk of breast and gynecologic
progression to carcinoma cancer
10.9 Colorectal Carcinoma

Colorectal Carcinoma - Carcinoma arising from colonic or rectal mucosa

1. What is colorectal cancer and what's its epidemiology?


- 3rd most common site of cancer and 3rd most common cause of cancer-related death .
- Peak incidence in 60-70 year olds (both men and women)

2. What are the two molecular pathways that cause CRC ?


- Adenoma-carcinoma sequence (most common)
- Microsatellite instability (MSI).

3. Describe microsatellite instability's role in CRC ?


- Microsatellites are repeating sequences of noncoding DNA;
integrity of sequence (stability) is maintained during cell division.
- Instability indicates defective DNA copy mechanisms.
- Cause - Hereditary nonpolyposis colorectal cancer (HNPCC) aka Lynch syndrome
→ Inherited (AD) Mutation of DNA mismatch repair (MMR) enzymes.
→ Increased risk for colorectal, ovarian, and endometrial carcinoma
→ Cancer arises de-novo (not from adenomatous polyps)
→ Seen at an early stage
→ Cancer is usually right sided

4. Screening for colorectal carcinoma .


- Colonoscopy and fecal occult blood.
- Testing begins at 50 years of age.
- GOAL: - remove adenomatous polyps before carcinoma develops
- to detect cancer early (before clinical symptoms arise).

5. Describe the following features of colorectal cancer.


Location - Right sided - Left sided
& presen. • raised lesion • napkin ring lesion:
(anywhere • vague pain. • decreased stool caliber,
along the • iron deficiency anemia (due to occult bleeding) • LLQ pain
entire • Older adult with Fe deficiency anemia has CRC
• blood streaked stool
length of until proven otherwise
colon)
Association : increased risk for Streptococcus bovis endocarditis.

Staging - T - depth of invasion; tumors limited to the mucosa generally do not spread due
to lack of lymphatics in the mucosa.
- N - spread to regional lymph nodes
- M - distant spread; most commonly metastatize to liver
Serum - CEA (serum tumor marker) - useful for assessing treatment response and
marker detecting recurrence; not useful for screening
Chapter 11: Exocrine Pancreas,
Gallbladder, and Liver Pathology
Exocrine Pancreas

Annular pancreas

0. What is annular pancreas ?


- Developmental malformation in which.
- Pancreas forms a ring around the duodenum.
- There is risk of duodenal obstruction.

Acute pancreatitis

1. Describe the Pathophysiology of acute pancreatitis.


- Due to reversible autodigestion of pancreatic parenchyma by pancreatic enzymes.
- Premature activation of trypsin leads to activation of other pancreatic enzymes.
- Results in liquefactive hemorrhagic necrosis (pancreas has lots of blood supply) of
the pancreas and fat necrosis of the peripancreatic fat followed by Inflammation.

1. What are the Etiologies ?.


○ Major cause [>75% of cases]
 EtOh and Gallstones [EtOh causes contraction of sphincter of oddi which slows drainage
of pancreatic enzymes ----> risking premature activation of enzymes in pancreas].
○ Other minor causes
 Trauma (ex - automobile accident)
 Hypercalcemia (Ca is activator of enzymes)
 Hyperlipidemia
 Cystic fibrosis [In CF, low bicarb and water drainage to pancreatic ducts; slow flow and
thick enzyme --> enzyme activation]
 Drugs
 Scorpion venom
 Mumps
 Rupture of posterior duodenal ulcer

1. What are the Clinical features.


- Epigastric abdominal pain that radiates to the back (pancreas is secondary
retroperitoneal organ)
- Periumbilical and flank hemorrhage (necrosis spreads into the periumbilical soft
tissue and retroperitoneum)
- Nausea and vomiting

1. How do you diagnose acute pancreatitis ?


- ↑ serum lipase [more specific test - 3 fold increase in level].
- ↑ serum amylase [also seen in salivary gland damage]
- Hypocalcemia (calcium is consumed during saponification in fat necrosis - poor
prognotic indicator)
Fig - liquifactive necrosis of pancreas with fat necrosis of surrounding fat in pancreatitis

3. What are the complication of acute pancreatitis ?


- Shock - due to peripancreatic hemorrhage and fluid sequestration
- Pancreatic pseudocyst
○ Formed by fibrous tissue surrounding liquefactive necrosis and pancreatic enzymes
○ Presents as an abdominal mass with persistently elevated serum amylase
○ Rupture is associated with release of enzymes into the abdominal cavity leading to
digestion and hemorrhage of gut.
- Pancreatic abscess
○ Usually due to E.Coli.
○ Presents with:
i. abdominal pain
ii. high fever
iii. persistently elevated amylase
- DIC - enzymes enters the circulation activating coagulation factors.
- ARDS - enzymes enters the circulation and migrates to the lungs where it
damages alveoli-capillary interface.
Chronic pancreatitis

4. What causes chronic pancreatitis?


- Fibrosis of pancreatic parenchyma.
- Most often secondary to recurrent acute pancreatitis.

4. What are the Etiologies of chronic pancreatitis?


○ EtOH (most common in adults)
○ Cystic fibrosis (most common in kids)
○ Idiopathic [many cases]

5. What are clinical features of chronic pancreatitis?


- Epigastric abdominal pain that radiates to the back
- Pancreatic insufficiency (small fibrosed pancreas):
○ Leads to malabsorption with steatorrhea
○ Fat soluble vitamin deficiency
- Dystrophic calcification of pancreatic parenchyma on imaging [contrast studies reveal a
'chain of lakes' pattern due to dilatation of pancreatic ducts]
- Secondary diabetes mellitus in late phase - due to damage of islet cells

6. What is the complication of chronic pancreatitis ?


- Increased risk for pancreatic carcinoma.

6. Why are serum lipase and amylase are not good marker for chronic pancreatitis ?
- Because in chronic pancreatitis, pancreas don't produce much enzyme so enzymes
aren't elevated in blood.
Pancreatic carcinoma

7. Describe pancreatic adenocarcinoma


- Adenocarcinoma arising from the pancreatic ducts.
- Most commonly seen in the elderly (average age is 70 years).

8. What are the Major Risk factors ?


○ Smoking
○ Chronic pancreatitis

8. What are the presentations ?


- Epigastric abdominal pain
- Weight loss
- If tumors arise in the head of the pancreas:
=> Obstructive jaundice (common bile duct blockage) with:
i. pale stools
ii. palpable gallbladder
- If tumors arise in the body or tail of the pancreas:
=> Secondary diabetes mellitus (islet damage by tumor)
- Acute pancreatitis - due to obstruction of pancretic duct.
- Migratory thrombophlebitis which presents as (Trousseau sign):
i. swelling
ii. erythema
iii. tenderness in the extremities (seen in 10% of patients)

8. How to diagnose pancreatic adenocarcinoma ?


- Detecting the Serum tumor marker CA 19-9.

8. What is the prognosis ?


- Very poor prognosis - 1 year survival is < 10%.

8. What is the treatment ?


- Whipple procedure - Surgical resection involves en bloc removal of:
○ the head and neck of pancreas
○ proximal duodenum
○ gallbladder
Gallbladder and Biliary Tract

Biliary atresia

1. What is biliary atresia ?


- Failure to form (Agenesis) or early destruction or hypoplastic narrowing of
extrahepatic biliary tree leading to biliary obstruction.
- It can be:
○ Congenital disease - associated with certain gene (CFC1) mutation
○ Acquired disease - autoimmune inflammatory response due to some (viral) infection.
- There are three main types of extra-hepatic biliary atresia:
○ Type I: Atresia is restricted to the common bile duct.
○ Type II: Atresia of the common hepatic duct.
○ Type III: Atresia involves the most proximal part of the bile ducts.

1. What is the presentation of biliary atresia ?


- Presents with neonatal jaundice (conjugated bilirubinemia)
- Progresses to Liver cirrhosis
Cholelithiasis [gallstones]

2. What is Cholelithiasis ?
- Solid, round stones in the gallbladder.
- Gallstones are usually asymptomatic.

2. Describe causes of gall bladder stones.


Gall-stone Etiology
Cholesterol - Very high cholesterol level --> solid monohydrate crystals.
stones (yellow) - Low bile acids and lecithin (bile acids and phospholipids
dissolves cholestrol thus increasing its solubility preventing
its supersaturation and further precipitation in bile).
- Statis (hypomotility) and mucus hypersecretion of g.bladder.
Bilirubin stones - Some defects (ex- Chronic H.anemia / Crohn's disease)
(pigmented or Chronic H.anemia --> ↑ Conjugated bilirubin in bile.
Black stones) Crohn's disease --> ↑ unconjugated bilirubin in bile.
- About 1% of bilirubin glucuronides are deconjugated in
the biliary tree while the rest is deconjugated in intestines
by brush border β-glucuronidase for the Enterohepatic
circulation [Found in sterile gallbladder bile].
- UB forms insoluble Ca2+ salts --> stones formed.+

Mixed stones - It is mainly found in infected large bile ducts.


(Brown stones) - It comprises of:
--> Insoluble UB-Ca2+ salts.
--> Ca-salts of Palmitate & stearate (Ca soaps).
--> 20–80% Cholesterol.
- Microbial enzymatic (β-glucouronidase) breakdown of
conjugated bilirubin glucuronides [stasis can cause
bacterial growth in gallbladder causing biliary tract
infection by E coli, Ascaris lumbricoides or Clonorchis
sinensis] into Unconjugated bilirubin.
3. Describe cholestrol stones.
- Most common type (90%), especially in the West
- Usually radiolucent (10% are radiopaque due to associated calcium)

3. What are the risk factors of cholestrol stones.


- Age (40s)
- Estrogen [Estrogen stimulates HMG CoA reductase activity and increase the
expression of HDL receptor on hepatocytes].
=> female gender
=> obesity
=> multiple pregnancies
=> oral contraceptives
- Fibrates [Clofibrate] - It is a lipid lowering drug (by increasing lipoprotein lipase
activity dissolving triglycerides in VLDL) that prevents conversion of cholesterol to
bile acids (by blocking 7alpha-hydroxylase activity).
- Native American ethnicity
- Crohn disease [terminal ileum is affected which is the site of
reuptake of bile salts and bile acid]
- Liver cirrhosis

3. Describe Bilirubin stones.


- Composed of bilirubin
- Usually 50 -75% Black stones are radiopaque while most Brown stones
are radiolucent.

3. What are the risk factors of bilirubin stones.


- Ascaris lumbricoides is a common roundworm that infects 25% of the
world 's population, especially in areas with poor sanitation (fecal-oral
transmission).
- Clonorchis sinensis is endemic in China, Korea, and Vietnam (Chinese
liver fluke); also increases the risk for cholangitis and cholangiocarcinoma.
Cholangiocarcinoma
- Cancer of bile duct

Choledocholithiasis
- Presence of gallstones in the common bile duct.
- 90-95% of pt with choledocolithiasis have cholelithiasis but
15% of pt with cholelithiasis have choledocolithiasis.
- Lab finding:
○ Increased serum alk. phosphatase
○ Increased serum bilirubin

6. What are some complications of gallstones ?


- Biliary colic
- Acute cholecystitis
- Chronic cholecystitis
- Ascending cholangitis
- gallstone ileus
- gallbladder carcinoma

Biliary colic
- Waxing and Waning RUQ pain.
- Cause: gallbladder contracting against a stone lodged in the cystic duct
- Symptoms are relieved if the stone passes.
- Common bile duct obstruction may result in:
○ acute pancreatitis
○ obstructive jaundice

Acute cholecystitis
- Acute inflammation of the gallbladder wall
- Cause: Impacted stone in the cystic duct results in:
○ dilatation with pressure ischemia (squeezing of vessels).
○ bacterial overgrowth (E coli) and inflammation
- Presentation:
○ RUQ pain, often radiating to right scapula
○ fever, nausea, vomiting
○ ↑ WBC count
○ ↑ serum alkaline phosphatase (from duct ep. damage)
- Risk of rupture if left untreated
- Detection: Murphy's sign.
Chronic cholecystisis.
- Chronic inflammation of the gallbladder.
- Cause: Due to chemical irritation from longstanding cholelithiasis, with or
without superimposed bouts of acute cholecystitis.
- Characteristic: herniation of gallbladder mucosa into the muscular wall
(Rokitansky-Aschoff sinus).
- Presentation: vague RUQ pain, especially after eating.
- Late complication: Porcelain gallbladder [Shrunken, hard gallbladder due to
chronic inflammation, fibrosis, and dystrophic calcification of necrotic wall].
- Increased risk: gallbladder carcinoma.
- Treatment: cholecystectomy (gall bladder removal), especially if porcelain
gallbladder is present.

Fig - procelian gallbladder.

Fig - M resembles gall bladder mucosa. The red circles on bottom indicates mucosa inside
muscular layer (Rokitansky-Aschoff sinus) and it's not supposed to be there.

Ascending cholangitis.
- Bacterial infection of the bile ducts
- Cause: ascending infection of biliary tree with enteric gram (-) bacteria.
- Presents as:
i. Charcot's triad
=> sepsis (high fever and chills)
=> jaundice
=> RUQ abdominal pain
ii. Reynolds' pentad
=> Charcot's triad
=> Shock (hypotension)
=> Altered mental status
- Association with Choledocholithiasis (stone in biliary ducts) - > 80% of cases.
[stone prevents washing of bacteria into the duodenum]

Gallstone ileus
- Gallstone enters and obstructs the small bowel
- Cause: due to cholecystitis with fistula formation between the
gallbladder and small bowel

Gallbladder carcinoma
- Adenocarcinoma arising from the glandular epithelium that lines the
gallbladder wall
- Major risk factor: porcelain gallbladder.
- Classically presentation: cholecystitis in an elderly woman (40-50yr) Fig - gallbladder adenocarcinoma
- Poor prognosis.
Liver

Liver function tests


Liver: Jaundice

1. Explain the pathway of Bilirubin metabolism.


RBCs are consumed by macrophages of the reticuloendothelial system

Protoporphyrin (from heme) is converted to unconjugated bilirubin (UCB)

Albumin carries UCB to the liver.

Uridine glucuronyl transferase (UGT) in hepatocytes conjugates bilirubin.

Conjugated bilirubin (CB) is transferred to bile canaliculi to form bile, which is stored in the gallbladder.

Bile is released into the small bowel to aid in digestion.

Intestinal flora convert CB to urobilinogen [UB], which is oxidized to:
- stercobilin (in the intestines making stool brown) and
- urobilin (in the kidney tubules making urine yellow).

2. What is significance of bile?


- Bile is the only way of cholesterol excretion
- Bile is the main way of copper and iron excretion.
3. What is presentation of jaundice?
- Earliest sign - scleral icterus (yellow discoloration of the sclera).
- Later sign - Yellow discoloration of the skin.
- Occurs due to ↑ serum bilirubin, usually > 2.5 mg/dL
- Arises with disturbances in bilirubin metabolism Entero-hepatic circulation

Disease Lab values Etiology (of JAUNDICE) Clinical features


Extravascular hemolysis / -↑ UCB High levels of UCB overwhelm the - Dark urine (↑ urine UB)
ineffective hemolysis -↑ urine UB conjugating ability of the liver - High risk for pigmented bilirubin gallstones

Physiologic jaundice of ↑ UCB Newborn liver has transiently low - Kernicterus: UCB is fat soluble and can
newborn UGT activity. deposit in basal ganglia leading to
neurological deficits and death.
- Treatment: phototherapy (makes UCB
water soluble hence excreted by kidney).

Gilbert syndrome [AR] ↑ UCB Mildly low UGT activity - Jaundice during stress (ex - severe infection)
otherwise clinically insignificant
Crigler-Najjar syndrome ↑ UCB Absence of UGT - Kernicterus, usually fatal
[AR]

Dubin-Johnson ↑ CB Defect in bilirubin canalicular - Liver is dark due to polymerized epinephrine


syndrome [AR] transport protein (MRP2) metabolites (otherwise clinically silent).
Rotor syndrome [AR] ↑ CB Defect in bilirubin canalicular - Lacks liver discoloration;
transport protein (MRP2) (similar to Dubin-Johnson syndrome).
Biliary tract obstruction -↑CB Associated with: - Dark urine (↑urine CB - bilirubinuria)
(obstructive jaundice) -↑alkaline - gallstones - Pale stool (↓ gut UB hence ↓stercobilin)
---> Cholestasis [bile phosphatase - pancreatic carcinoma - Pruritus (↑ serum bile acids).
cannot flow from liver -↓urine UB - cholangiocarcinoma - Hypercholesterolemia (leading to Xanthoma)
to duodenum] -↑bile acid - parasites (Clonorchis sinensis) - Steatorrhea with malabsorption of fat
-↑cholestrol soluble vitamin (bile acid not breaking fat)
Viral hepatitis -↑ CB Inflammation disrupts hepatocytes - Dark urine (↑ urine CB)
-↑ UCB and small bile ductules
-↓ / normal
urine UB
Liver: Viral Hepatitis

1. What is viral hepatitis?


- Inflammation of liver parenchyma - CD8+ cell mediated cytotoxic damage.

1. What are some causes of viral hepatitis?


- Hepatitis viruses
- EBV
- CMV

2. Describe Acute hepatitis.


- Inflammation involves lobules of the liver and portal tracts and is
characterized by apoptosis of hepatocytes
- Clinical pressentation:
• jaundice (mixed CB and UCB) with dark urine (due to CB)
• Fever, malaise and nausea
- Diagnosis:
• elevated liver enzymes (ALT > AST).
- Symptoms:
• Some cases may be asymptomatic
• Symptoms last < 6 months

2. Describe Chronic hepatitis.


- Inflammation predominantly involves portal tract.
- Symptoms: last > 6 months.
- Risk: progression to cirrhosis.

Fig - L is liver parenchyma, P is portal tract. Note inflammation is mainly in portal tract only.
Virus Transmission Comment
Hepatitis A (HAV) - Fecal-oral transmission - Acute hepatitis; no chronic state.
- commonly acquired by travelers. - Serum markers:
○ Anti-virus IgM marks active infection.
○ Anti-virus IgG is protective, and its
presence indicates prior infection or
immunization (immunization is
available for HAV)
Hepatitis E (HEV) - Fecal-oral transmission
- HEV infection in pregnant women is
- commonly acquired from contaminated
associated with fulminant hepatitis
water or undercooked seafood.
(liver failure with massive liver necrosis).
Hepatitis B (HBV) Parenteral transmission: - Results in acute hepatitis; chronic disease
- childbirth occurs in 20% of cases.
- unprotected intercourse - Hepatitis B vaccine is available
- intravenous drug abuse (IVDA)

Hepatitis C (HCV) Parenteral transmission: - Results in acute hepatitis; chronic disease


- IVDA occurs in most (50%) cases.
- unprotected intercourse - Diagnosis:
- needle stick ○ HCV-RNA test confirms infection
[risk from transfusion is almost  decreased RNA levels indicate recovery
nonexistent due to screening of the blood  persistant RNA levels indicate chronic
supply]. disease
Hepatitis D (HDV) Dependent on HBV for infection; superinfection
(HDV infects pt with preexisting HBV) is more
severe than coinfection (infection with HBV and
HDV at the same time).

5. Serologic Markers of Hepatitis B Virus


Stage HBsAG HBeAG & HBV dna HBcAB HBsAB
Acute + (first serologic + IgM -
marker to rise)
Window - - IgM -

Resolved - - IgG IgG (protective)


Chronic + (presence > 6 +/- (presence indicates IgG -
months) infectivity).
Immunization - - - IgG (protective)
Liver: Cirrhosis

1. Describe cirrhosis.
- It is an End-stage liver damage.
- Disruption of the normal hepatic parenchyma by:
○ bands of fibrosis [bridging fibrosis - fibrosis between portal tracts].
○ regenerative nodules of hepatocytes.

Fig - note the regenerative nodules with fibrosis in between -


characterstic of cirrhosis. (normal liver surface is smooth)

2. What mediates fibrosis ?


- Stellate cells (which lie beneath the endothelial cells that line the
sinusoids) secrete TGF-beta which mediates the fibrosis.

2. What are the causes of Cirrhosis [Chronic liver disease]?


- Alcohol related liver disease
- Nonalcoholic fatty liver disease
- Hemochromatosis
- Wilson disease
- α1-Antitrypsin deficiency
- Chronic hepatitis C
- Chronic hepatitis B
- Primary biliary cholangitis
- Primary sclerosing cholangitis
- Autoimmune hepatitis
- Hepatotoxic drugs or toxins
- Cystic fibrosis

3. What is the result of cirrhosis ?


- Portal HTN
- Decreased detoxification (-----> Acute liver failure).
- Decreased protein synthesis (-----> Acute liver failure).
3. What are the presentations of portal HTN ?
○ Ascites - fluid in the peritoneal cavity [underfill theory: RAAS activation ---> renal Na avidity ---> high blood volume]
 If SAAG is > 1.1 g/dl - indicates ascites due to portal HTN (related or non related to liver) - transudate.
 If SAAG is < 1.1 g/dl - indicates ascites due to serositis, peritoneal cancers - exudate.
 [SAAG: serum ascites albumin gradient - (serum albumin) minus (albumin level of ascitic fluid)].
○ Congestive splenomegaly (increased red pulp later becomes fibrotic) ----> Hypersplenism (overactive spleen)
 Spleen phagocytoses lots of RBCs due to blood pool up increasing the workload.
 Macrophages phagocytose normal blood cells inducing pancytopenia.
○ Portosystemic shunts disorders
 Esophageal varices
 Hemorrhoids
 Caput medusae
○ Hepatorenal syndrome: Acute kidney injury (failure) in individuals with cirrhosis.
 Rapidly developing renal failure secondary to cirrhosis explained by underfill theory.
[Portal HTN ---> splanchnic vasodialation (due to NO,PGs released by liver) --->
pooling of blood in splanchnic circulation ---> decreased effective circulatory volume
---> s.hypotension ---> JGA apparatus stimulation ---> RAAS activation ---> AGT-II --->
renal (e.arteriole) vasoconstriction [prolonged] ---> Hepatorenal syndrome].

5. What are presentations of decreased detoxification ?


○ ↑ serum ammonia leads to:
- Hepatic encephalopathy
- mental status change
- asterixis, clonus
- coma
○ ↑ estrogen (hyperestrinism) leads to: [liver converts estrogen to androgens]
- gynecomastia
- testicular atrophy
- spider angiomata
- palmar erythema
○ ↑ unconjugated bilirubin leads to:
- Jaundice

6. What are presentation of decreased protein synthesis ?


○ Hypoalbuminemia (leads to edema).
○ Coagulopathy due to decreased synthesis of clotting factors (high PT - due to
low function of vit k dependent epoxide reductase).
Liver: Alcohol related Liver disease

1. What is alcohol related Liver disease ?


- Damage to hepatic parenchyma due to consumption of alcohol
- Most common cause of liver disease in the West

2. What is the 1st step of progression of ARLD ?


- Fatty liver <--- Steatosis [insulin resistance --> defective FA metabolism -->
TGA vacuoles deposition --> droplets coalesce --> large droplets].
○ Fat accumulation in hepatocytes (reversible)
○ Results in a heavy, greasy liver
○ resolves with abstinence

2. What is the 2nd step of progression of ARLD ?


- Alcoholic hepatitis
○ generally seen with binge drinking
○ may result in death
○ Acetaldehyde (metabolite of alcohol) mediates chemical damage
to hepatocytes [mitochondrial toxin].
○ Histology:
=> Swelling of hepatocytes
=> Mallory bodies (damaged intermediate cytokeratin
filaments) - eosinophilic cytoplasmic inclusions
=> necrosis and acute inflammation
○ Presentation:
=> painful hepatomegaly Fig - circles showing balooning of hepatocyes and
=> elevated liver enzymes (AST > ALT) mallory bodies characterstic of alcoholic hepatitis

2. What is the 3rd step of progression of ARLD ?


- Cirrhosis
○ complication of long-term, chronic alcohol-induced liver damage.
○ occurs in 10-20% of alcoholics
○ Present with signs and symptoms of regular cirrhosis

Fig - white blob of fat in hepatocytes in


fatty liver disease.
Liver: Non-Alcoholic Fatty Liver disease

8. Describe non-alcohol fatty liver disease (NAFLD).


- Fatty liver, hepatitis, and/or cirrhosis that develop without exposure
to alcohol (or other known insult).
- Diagnosis of exclusion; ALT > AST
- Associated with obesity
Hemochromatosis

9. Describe hemochromatosis.
- Excess body iron leading to:
○ ↑ ferritin - iron deposition in tissues (Siderosis).
○ ↓ TIBC (total iron binding capacity).
○ ↑ serum iron
○ ↑ % Fe saturation.
○ Hemosiderosis - accumulation of hemosiderin.
- Tissue damage is mediated by generation of free radical by Fenton's reaction.

11. What are the causes of hemochromatosis ?


- Primary: HFE hereditary haemochromatosis or Haemochromatosis type 1.
- Secondary: Multiple frequent blood transfusions or Severe chronic haemolysis.

10. Describe the MOA of primary hemochromatosis.


- AR Mutations (transition point mutation ----> missense mutation) in the HFE
gene at 6p21.3 locus in hepatocyte.
- The HFE gene has two common mutations (loss of function):
○ C282Y - Cysteine-to-tyrosine substitution at amino acid position 282.
○ H63D - histidine-to-aspartic acid substitution at amino acid position 63.
- The HFE protein is a membrane modulator [interacts with the transferrin
receptor TFRC thus facilitating binding of Transferrin bound iron] --->
stimulates HAMP gene --> ↑ Hepcidin synthesis --> inhibits ferroportin.

12. What is the presentation of hemochromatotis ?


- Presents in late adulthood because it takes time for Fe to accumulate.
- Classic Triad:
○ bronze skin
○ cirrhosis
○ Secondary diabetes mellitus (aka bronze diabetes).
- Other presentations:
○ dilated cardiomyopathy
○ cardiac arrhythmias
○ gonadal dysfunction (due to testicular atrophy).

13. What is the diagnosis of hemochromatosis ?


- Liver biopsy reveals accumulation of brown pigment in hepatocytes.
- Prussian blue stain distinguishes iron (blue) from lipofuscin [brown pigment
that is a by-product from the turnover ('wear and tear') of peroxidized lipids;
it is commonly present in old hepatocytes].

14. What is the complication of hemochromatosis ?


- Increased risk of hepatocellular carcinoma

14. What is the Prognosis of hemochromatosis ?


- Phlebotomy [practice of drawing blood from patients and taking the blood
specimens to the laboratory to prepare for testing].
Fig - brown pigments in hepatocytes (maybe
hemochromatosis or lipofuscin) Fig - prussian blue stain showing Fe deposition in hepatocytes

Wilson's disease

15. What is the Pathophysiology of wilson's disease?


- Autosomal recessive defect in ATP7B gene leading to deficiency in ATP7B protein.
- Results in:
○ lack of copper transport into bile (main way of Cu excretion)
○ lack of copper incorporation into ceruloplasmin
- Copper thus:
○ builds up in hepatocytes (causing ROS mediated damage)
○ leaks into serum
○ deposits in other tissues.

16. What are the presentation of Wilson's disese ?


- Presents in childhood with:.
○ Cirrhosis
○ Neurologic manifestations [due to deposition of copper in basal ganglia]:..
- behavior changes
- dementia
- chorea
- parkinsonism
○ Kayser- Fleisher rings in cornea [Cu deposition in descement membrane]

16. What are the Lab values of Wilson's disese ?


○ ↑ serum & urine copper
○ ↓ serum ceruloplasmin
○ ↑ Cu on liver biopsy

14. What is the complication of Wilson's disese ?


- Increased risk of hepatocellular carcinoma

16. What is the treatment of Wilson's disese?


- D-penicillamine (chelates copper ---> excreted in urine).
Primary biliary cirrhosis Primary sclerosing cholangitis
- Autoimmune granulamotous destruction of - Inflammation and fibrosis of intrahepatic and
intrahepatic bile ducts extrahepatic bile ducts
- Classically arises in women (average age is 40 yr) - Periductal fibrosis w/ 'onion skinning' appearance.
- Uninvolved regions are dilated resulting in a
"beaded" appearance on contrast imaging.

- Associated with other autoimmune diseases - Associated with Ulceratice Colitis.


- Etiology is unknown - Etiology is unknown
- Anti-mitochondrial antibody positive. - p-ANCA positive

- Presentation: obstructive jaundice - Presentation: obstructive jaundice


- Late complication: cirrhosis - Late complication: cirrhosis
- Increased risk for hepatocellular carcinoma. - Increased risk for cholangiocarcinoma.

Fig - preiductal fibrosis with onion


Fig - string and pearl appearance of biliary tree in PSC
skinning seen in PSC

Reye syndrome
17. What is reye syndrome?
- Fulminant (severe and sudden onset) liver failure and encephalopathy in
children with viral illness who take aspirin
- Likely related to mitochondrial damage of hepatocytes

18. What is the presentation of reye syndrome ?


- Hypoglycemia
- Elevated liver enzyme
- Nausea w/ vomiting
- May progress to coma and death

α1-Antitrypsin deficiency
18. What is α1-Antitrypsin deficiency ?
- α1-Antitrypsin is a plasma glycoprotein synthesised by hepatocytes.
- α1-Antitrypsin deficiency is an autosomal recessive disorder.
- results in defective protein folding (defect in migration of protein from
ER to GA) marked by low levels of circulating α1AT.
- Hepatocellular accumulation of misfolded protein (cytoplasmic globular
inclusions) leads to liver disease.
HEPATIC MASS

a. Nodular hyperplasias
- Focal nodular hyperplasia.
- Nodular regenerative hyperplasia.
b. Benign Neoplasm - Hepatocellular adenoma
- Inflammatory (40%–50%)
- HNF1α-inactivated (35%–40%)
- β-catenin–activated
- β-Catenin–activated inflammatory
- Unclassified
c. Malignant tumors
- Hepatoblastoma
- Hepatocellular Carcinoma (HCC)
---> Fibrolamellar carcinoma Fig - multinodular liver is classic presentation of liver metastasis
d. Liver metastasis and can be felt on physical exam

Hepatocellular adenoma
19. Describe Hepatocellular adenoma.
- Tumors are subcapsular and grow with exposure to estrogen.
- Associated with oral contraceptive use; regresses upon cessation of drug.
- Risk of rupture (tumor is right below liver capsule) and intraperitoneal
bleeding, especially during pregnancy

Hepatocellular Carcinoma

20. What are risk factors ?


- Chronic hepatitis (usually caused by hep B and C)
- Cirrhosis (due to any cause)
- Aflatoxins derived from Aspergillus (induce p53 mutations).

20. What is the complication ?


- Increased risk for Budd-Chiari syndrome
○ Liver infarction secondary to hepatic vein obstruction.
○ Presents with painful hepatomegaly and ascites (SAAG > 1.1 g/dL).

22. What is prognosis ?


- Poor prognosis
- Tumor are often detected late as symptoms are masked by cirrhosis

23. What is the Diagnosis ?


- Serum tumor marker is alpha-fetoprotein

Liver metastasis
24. Describe cancer metastasis to liver.
- More common than primary liver cancers [coz of rich blood supply].
- Most common source: colon, pancreas, lung, breast carcinoma
- Presentation: multiple nodules in liver.
- Clinically may be detected as hepatomegaly with a nodular free edge of the liver
Chapter 12: Kidney and Urinary Tract
Pathology
Congenital

Horseshoe kidney (no. 1 congenital renal anomaly)


1. What is a finding in horseshoe kidney?
- Kidney abnormally located in lower abdomen as it gets caught in IMA (inferior
mesenteric artery) root during its ascent from pelvis to abdomen .
- Conjoined kidney joined at the lower pole.

Fig: horseshoe kidney getting stuck during ascent by IMA

Renal agenesis (failure of kidney to form)


1. Differentiate presentation of unilateral and bilateral renal agenesis.
Unilateral renal agenesis Bilateral renal agenesis (no kidney present) -
incompatible with life
- Hypertrophy of existing kidney - Oligohydramnios (no A.fluid; fetal urine → A. fluid)
- Risk of renal failure later in life due to - presents as Potter syndrome.
hyperfiltration • Lung hypoplasia (lung develops by stretching
as baby breathes amniotic fluid in and out)
•Flat face, low set ears, developmental extremity
defects (baby not floating, pressed against mother)

Dysplastic kidney
1. What is presentation of dysplastic kidney? Is it inherited?
- Mostly unilateral and non-inherited [congenital] malformation of renal parenchyma
- Characterized by cysts and abnormal tissue (ex-cartilage).

Fig: dysplastic kidney. Circle on left is cyst; circle on right shows growing cartilage
Polycystic kidney disease (PKD)
1. What is presentation of polycystic kidney disease?
- Bilateral enlargement of kidney with cysts in renal cortex and medulla
- Inherited defect.

Fig: polycystic kidney disease (note it's always bilateral)

2. Type of polycystic kidney disease.


Autosomal recessive (aka juvenile Autosomal Dominant (ADult) - cysts develop over time
form)
- Presents in infants as worsening - Presents in young adults with HTN (high renin level),
renal failure and HTN. hematuria, Azotemia and worsening renal failure.
- Newborns may have potter sequence - Presents as Dull loin pain [Palpable Renomegaly]
- Associated with: - Associated with:
• congenital hepatic fibrosis • berry aneurysm in brain (C. of Willis) - family
(leads to portal HTN) history of renal disease and brain related death
• hepatic cysts • hepatic cysts, and
• mitral valve prolapse
- Think as cyst in kidney and liver. - Think as cyst in kidney, liver and brain.
- Mutated polycystin-1 and - Mutation in APKD1 or APKD2 gene.
polycystin-2 proteins (APKD = adult polycystic kidney disease)

Medullary cystic kidney disease


1. What is medullary cystic kidney disease?
- Autosomal dominant cyst formation in medullary collecting ducts - Inherited disease.
- Parenchymal fibrosis leads to shrunken kidneys and worsening renal failure.
Acute Renal Failure
a) Classify the types of acute renal failure ?
- Pre-renal azotemia (common cause of ARF)
- Intra-renal azotemia
○ Acute tubular necrosis (most common cause of ARF)
○ Acute interstitial nephritis
○ Renal papillary necrosis
- Post-renal azotemia

b) What is the presentation of acute renal failure ?


○ Hallmark: Azotemia - increased BUN and creatinine [Cr] in blood
○ Oliguria (if we make < 500ml urine/day, we can't excrete solid waste)
○ Acute, severe decrease in renal function (develops within days)

b) What are come causes of increased serum BUN?


- Decreased kidney function
- High steroid dosage
- Upper GI bleed (metabolism of blood proteins can lead to nitrogenous waste build up).
- Hypercatabolic state (sepsis, critical illness)
- Dehydration, volume depletion.

Pre-renal azotemia (ARF)


b) Describe pre-renal azotemia and lab findings ?
- Azotemia caused due to decreased renal perfusion [GFR low].
- Causes - low output cardiac failure ----> hypotension
- Lab findings:
○ FENa < 1% (low): Na retention by the kidney to control hypovolemia.
○ Bun/Cr ratio > 15 (high): RAAS activation ---> Aldo. synthesis ---> More water is
reabsorbed and more BUN is reabsorbed with it.
○ High urine osmolality [osm] > 500 mOsm/kg.

c) What is Bun/Cr ratio ?


- Normally, Cr is excreted a little in PCT, and Bun is reabsorbed in PCT. Therefore,
BUN is higher in blood than Cr which makes normal BUN:Cr = 15
- BUN [mg/dL] = Urea [mg/dL] / 2.14
- Blood urea nitrogen (BUN) is a medical test that measures the amount of urea
nitrogen found in blood.

d) What is FENa [Fractional excretion of sodium] ?


- FENa = (urine Na/serum Na) / (urine Cr/serum Cr).
- It is the percentage of the sodium filtered by the kidney which is excreted in
the urine.

Post-renal azotemia

e) Describe post-renal azotemia ?


- Due to obstruction of urinary tract downstream from the kidney [GFR decreases
due to BACK PRESSURE].
- Lab findings:
○ Early stage - FENa <1% and urine osmolality > 500 Osm → normal tubular function.
- BUN:Cr ratio >15 → increased tubular backpressure forces BUN into blood.
○ Late stage - FENa >2% and urine osmolality >500 Osm → tubular damage.
- BUN:Cr ratio <15 → Tubular damage: unable to reabsorb BUN and excrete Cr.
Intra-renal azotemia (ARF)

a) What are the causes of intra-renal azotemia ?


- Acute tubular necrosis
- Acute interstitial nephritis (----> Renal papillary necrosis)

Acute tubular necrosis

b) Describe features of acute tubular necrosis ?


- Injury and necrosis of tubular epithelial cells.
- Necrotic tubular cells clog tubules [GFR decreases due to OBSTRUCTION].
- Hallmark - brown granular casts seen in urine (No inflammatory cells are seen).

Fig- ATN - T are tubules that are clogged up by dead cells. G is glomerulus.

c) What are lab values for acute tubular necrosis?


- Bun:Cr ratio < 15 - Dysfunctional tubular epithelium: BUN can't be reabsorbed and Cr can't be excreted.
- High Bun & Cr level in blood - increased tubular back pressure reduces their filtration.
- FENa > 2% - decreased reabsorption of sodium (sodium wasting).
- Urine osmolality < 500 Osm because kidney can't concentrate urine.
- Hyperkalemia (decreased K+ excretion).
- Metabolic acidosis due to increased anion gap (Cations minus Anions in serum)- low excretion of organic acids.

i) What are etiologies of acute tubular necrosis?


- Ischemic etiology - often preceded by prerenal azotemia. PCT and thick ascending limb
(medullary segment) are particularly susceptible to ischemic damage.
- Nephrotoxic etiology - PCT particularly susceptible
○ Aminoglycosides - most common cause
○ Myoglobinuria (crush injury to muscle)
○ Antifreeze (ethylene glycol) - pt develop oxalate crystal in their urine
○ Radiocontrast dye (iodine)
○ Heavy metal (lead)
○ Urate (chemotherapy kills lots of tumor cells damaging their nucleic acids making uric acid
as a byproduct which can cause ATN - Tumor Lysis Syndrome).

j) What is prognosis of ATN?


- Hydration (volume expansion by IV therapy) and allopurinol ( inhibitor of the enzyme xanthine
oxidase) are used prior to initiation of chemotherapy to decrease risk of urate-induced ATN.
- Tubular cells are stable cells (hence ATN is reversible) but often requires supportive dialysis
since electrolyte imbalances can be fatal.
- Oliguria can persist for 2- 3 weeks before recovery.
Acute interstitial nephritis

k) Describe acute interstitial nephritis ?


- Hypersensitivity reactions involving the interstitium and tubules
- Hallmark is presence of inflammatory cells in kidney interstitium.
- Causes -
○ drug induced AIN - penicillin, diuretics, NSAIDs
○ infectious AIN - Legionella (zinc melloprotease exotoxin).
- URINE - Eosinophils and monocytes are seen in drug induced AIN.
- Neutrophils are seen in infectious AIN.

l) What is presentation of acute interstitial necrosis?


- Oliguria, fever, and rash days-weeks after infection/drug start
- Resolves with cessation of drug/infection
- May progress to renal papillary necrosis

Renal papillary necrosis

m) What is renal papillary necrosis ?


- Necrosis of renal papillae
- Presents with gross hematuira and flank pain
- Causes - (mnemonic: POSTCARDS)
○ Pyelonephritis [severe acute inflammation of the kidney tissue].
○ Obstruction of the urogenital tract.
○ Sickle cell disease or trait.
○ Tuberculosis.
○ Cirrhosis of the liver.
○ Analgesic abuse [long-term phenacetin or aspirin use].
○ Renal vein thrombosis.
○ Diabetes mellitus.
○ Systemic vasculitis.

Renal Page 2.3


12.3 Nephrotic Syndrome
1. What are presentation of nephrotic syndrome?
 Hallmark is Proteinuria [>3.5g protein/day in urine] - glomerular disorder
 Hypoalbuminurea [pitting Edema] ---> Anasarca (generalized/widespread edema).
 Hypogammaglobulinemia [increase risk of infection].
 Hypercoagulable state - loss of antithrombin 3
 Hyperlipidemia and hypercholesteremia - produced by liver (to compensate for Proteinuria loss)

Minimal change disease [MCD]

2. Describe minimal change disease.


• Cause - mostly idiopathic or may associate with hodkin's lymphoma (HY) - due to massive
production of cytokines leading to effacement of foot process.
• Symptoms - Sudden onset of symptoms but renal function is normal. Not much symptoms
appear other than nephrotic syndrome (that's why called minimal change disease).
• Most common cause nephrotic syndrome in CHILDRENS.
• Selective Proteinuria : albumin is lost, no immunoglobulin loss.
• Diagnosis - diagnosis of exclusion
• Histology
○ Normal glomeruli seen on H&E (lipid may be seen in PCT).
○ EM shows effacement of foot process (a nonspecific finding seen in all nephrotic diseases).
○ No immune complex deposition - Negative IF.
• Treatment- excellent response to steroids because it reduces cytokines
production from active T cells.
• Prognosis - often recur

Fig - effacement of foot process. Bottom part is capillary; black blobs are RBC.

Focal segmental Glomerulosclerosis [FSGS]

3. Describe presentation of FSGS [Focal segmental Glomerulosclerosis].


• Most common cause of nephrotic syndome in hispanics and blacks.
• Risk factors - Usually idiopathic; may be associated with HIV, heroin use, and sickle cell disease.
• Treatment - Poor response to steroids; progresses to chronic renal failures.

4. Describe biopsy finding in FSGS .


• Focal (some glomeruli involved) and Segmental (involving only part of the glomerulus) and
Sclerosis (glomerulus hardened by hyalinosis - proteins and lipid deposition) is seen on H&E stain. .
• EM shows effacement of foot process.
• No immune complex deposition - Negative IF.
Fig - Kimmelstein-Wilson nodule Fig - tram track appearance of
Fig - Only bottom right glomerulus is affected membranoproliferative glomerulonephritis
segmentally.

Membranous nephropathy

5. Describe Membranous nephropathy.


• Most common primary nephrotic syndrome in Caucasian adults.
• Presents as very severe nephrotic syndrome.
• Treatment - Poor response to steroids. 20-30% ̀cases progress to CRF.
• Secondary causes are:
○ Hep B or C infections
○ Lupus [SLE] - autoimmune disease.
○ Lung and colon cancer [solid tumors]
○ Heavy metal or drugs [NSAIDs and penicillamine]

6. What are histologic findings of membranous nephropathy?


• Thick GBM seen on H&E stain (hematoxylin and Eosin staining).
• Immune complex deposition - Granular IF.
• Small and frequent subepithelial deposition (IgG or C3 deposits) seen on EM which
appears as 'spike and dome' pattern - Periodic acid–Schiff (PAS) or silver stain.

Membranoproliferative glomerulonephritis
7. Describe histology of Membranoproliferative glomerulonephritis
• Proliferative means mesangial cell (cytoplasm) proliferate.
• Thick GBM with 'tram track' appearance seen on H&E stain. Proliferation of mesangium cuts
the immune deposit [Granular IF] in half creating two lines (tram tracks).

7. What are two types of membranoproliferative glomerulonephritis ?


• Divided to two types based on location of immune deposit -
○ Subendothelial (type 1): associated with hepatitis B & hepatitis C infections.
Type 1 has more tram tracks association.
○ Within basement membrane (type 2) (dense deposit disease - intramembranous):
Pt have C3 nephritic factor (autoantibody). This antibody binds and stabilizes c3
convertase of Alternative pathway of complement system. This leads to
overactivation of complement, inflammation, and low levels of circulating C3.

8. What is the treatment of membranoproliferative glomerulonephritis ?


- Poor response to steroids; progresses to chronic renal failure
Fig - notice the small spikes and holes on PAS or silver stain. Holes are immune deposits as
they don't bind to silver. Spikes are new basement membrane laid on top of holes. They bind
to silver and stain dark.

Fig - capillary wall thickening in MN. FIg - the dark deposits are Fig - granular immunofluorescence
immune deposits seen in MN. seen in MN due to C3 and IgG
immune deposite

Diabetes Mellitus

8. Describe how Diabetes Mellitus leads to nephrotic disease.


• High blood glucose leads to non-enzymatic glycosylation of vascular basement membrane
which makes the GBM more leaky (protein leaks) -----> nephrotic syndrome.
• Leads to hyaline arteriolosclerosis.

9. What's histology finding of diabetic nephrotic syndrome. What's its treatment


• Efferent arteriole is affected more than afferent arteriole.
This causes high filtration pressure leading to
- Hyperfiltration injury ----> Microalbuminuria -----> nephrotic syndrome.
- Sclerosis of mesangium ----> Kimmelstein-Wilson nodules
These Mesangial nodules are IF negative and PAS positive.
• Treat by ACE-i because it relieves pressure in efferent arteriole [by reducing AGT II level].

Systemic amyloidosis

10. Describe how Systemic amyloidosis leads to nephrotic disease ?


• The most common organ affected in systemic amyloidosis is kidney.
• Amyloid deposits in mesangium
- Characterized by apple-green birefringence in polarized light by Congo red staining.
- Mesangial nodules are IF negative and PAS negative.
Nephritic Syndrome
1. What are features of nephritic syndrome?
• Hallmark is glomerular inflammation and bleeding - glomerular disorder
• RBC casts and dysmorphic RBC in urine.
• Limited proteinurea (< 3.5g/day).
• Oligurea ---> azotemia
• Salt retention with periorbital edema & HTN.

2. What are biopsy findings of nephritic syndrome?


• Hypercellular and inflamed glomeruli
• Immune deposits activate complement. C5a attracts neutrophil which cause damage.

Post- streptococcus glomerulonephrits (PSGN)

3. Describe etiology of post- streptococcus glomerulonephrits (PSGN).


• Occurs after group A Beta hemolytic strep infection (or Non strep organims infections as well).
• The nephritogenic strain carries M protein virulence factor.
• Causes Impetigo [highly contagious skin infection of kids] and Pharyngitis.

4. What is presentation of PSGN ?


- Appears 2-3 weeks after infection
- Usually seen in kids but may occur in adult.
- Presents as:
• Hematuria (cola colored urine)
• Oligurea
• Hypertension
• Periorbital edema

5. What are biopsy findings of PSGN.


• Hallmark - subepithelial 'humps' seen on EM.
• Granular IF - humps are made of immune complex deposition.
• Hypercellular inflamed glomeruli seen on H&E stain.

6. What is prognosis of PSGN.


- Treatment is Supportive. The disease heals by itself.
- Some adults (25%) develop RPGN.
- Children rarely (1%) progresses to Renal failure.

Fig - subepithelial hump seen in PSGN

Fig - cellular crescent seen in RPGN. Note crescent made of


fibrin and macrophage (inflammatory stuff - not collagen)
RPGN [Rapidly progressive glomerulonephritis]

7. What is RPGN [Rapidly progressive glomerulonephritis] ?


• RPGN is a nephritic syndrome that progress to renal failure in weeks to months.
• Hallmark - biopsy shows cresents in bowman space on H&E stain. Crescent is made up
of fibrin and macrophage.
8. What are etiologies of RPGN?
• Etiologies can be distinguished based on immunofluorescence pattern
IF Pattern Disease
Linear (anti-basement - Goodpasture syndrome
membrane Ab)
Granular (immune complex - Post Strep GN (most common)
deposition) - Diffuse proliferative GN
Negative IF (pauci-immune) - Wegner Granulomatosis (C-ANCA positive)
- Microscopic Polyangitis (P-ANCA positive)
- Churg-Strauss Syndrome (P-ANCA positive)

9. Describe Goodpasture syndrome


• Antibodies are made against alpha-3 subunit of type IV collagen in glomerular and alveolar BM.
• Pt presents with hematuria and hemoptysis
• Classically seen in young adult males

10. Describe Diffuse proliferative glomerulonephritis.


• Diffuse Ag-Ab complex deposition in sub-endothelial location.
• Most common type of renal disease in SLE (lupus) - most common cause of death in lupus
patients is renal failure.

11. Describe Wegner granulomatosis ?


• Imp. symptoms: Rhinitis and Sinusitis along with hematuria and hemoptysis.
• C-ANCA :Cytoplasmic Antineutrophil Cytoplasmic Antibodies.

12. Describe Churg-Strauss Syndrome.


• Characteristic :Granulomatous inflammation, eosinophilia & asthma.
• P-ANCA :Perinuclear Anti-Neutrophil Cytoplasmic Antibodies.

IgA nephropathy (Berger's disease)


13. Describe IgA nephropathy.
• Most common cause of nephropathy worldwide.
• IgA immune complex deposits in the mesangium - gives granular IF.
14. What is presentatin of IgA nephropathy
• Most commonly seen in kids after mucosal infection (ex- gastroenteritis)
• Following infection - Episodic gross or microscopic hematuria with RBC casts.
• This can slowly lead to renal failure.

Alport syndrome Fig - mesangial immune deposition seen


in IgA nephropathy
15. Describe Alport's syndrome
• Inherited defect of type IV collagen (most commonly X linked).
• BM of glomerulus, inner ear and eye becomes thin and splits
• Presents as hematuria, sensory hearing loss or ocular disturbances.
Urinary Tract Infection (UTI)

1. What are common features of UTI?


• UTI are infection of urethra (urethritis), u.bladder (cystisis) or kidney (pyelonephritis).
• Most UTI are ascending infections [organism ascend from urethra to kidney]- Systemic infection.
• Risk factors are - urinary stasis, sexual intercourse, catheters.
• There is increased incidence of UTI in females.

Cystitis

2. What is presentation of cystitis?


• Presentation-
○ Dysuria (pain with urination)
○ Urinary frequency and urgency
○ Suprapubic pain
○ Lack of systemic signs (ex- fever)

3. What are lab findings in cystitis?


• Urinalysis : cloudy urine with >10 WBCs/hpf (high power field)
• Dipstick: Positive leukocyte esterase (due to pyuria - pus in urine).
Positive nitrites (bacterias convert nitrates to nitrites in urine).
• Gold standard is culture. It yields >100,000 colony forming units.

4. What are etiology of cystitis?


• E. Coli - 80% of cases
• Staphylococcus saprophyticus - increased incidence in young, sexually active women
• Klebsilla pneumoniae
• Proteus mirabilis - produces alkaline urine with ammonia scent
• Enterococcus faecalis

5. What does sterile pyuria suggest?


• Sterile pyuria is pus in urine (>10 WBC/hpf and positive leukocyte esterase)
but negative bacterial culture of urine.
• It suggests urethritis with dysuria due to:
- Chlamydia trachomatis
- Neisseria gonorrhoeae
• Presenting sign of Urethritis:
- Dysuria (dominant sign)
Pyelonephritis

6. Describe presentation of pyelonephritis.


• Usually due to ascending infection
• Increased risk with vesicoureteral reflux (reflux of urine)
• Presentation-
○ Fever - systemic sign.
○ Flank pain (due to sensitization of nerves of renal capsule).
○ WBC casts and leukocytosis.
○ Symptoms of cystisis seen.

7. What are common pathogens for pyelonephritis?


• E coli (90% cases)
• Klebsiella species
• Enterococcus faecalis

8. Describe chronic pyelonephritis ?


• Interstitial fibrosis and atrophy of tubules due to multiple bouts of acute pyelonephritis.
• Seen due to:
- vesicoureteral reflux (in children)
- urinary obstruction (BPH or cervical carcinoma).

9. What are clinical features of chronic pyelonephritis?


• Cortical scarring with blunted calyces.
• Scarring at upper and lower pole [characterstic of vesicoureteral reflux].
• Histology
○ "Thyoidization of kidney" - atrophic tubules contain eosinophilic
proteinaceous mateiral resembling thyroid follicles.
○ Waxy casts may be present in urine.

Fig - thyroidization of kidney seen in chronic pyelonephritis


Nephrolithiasis
1. What are the risk factors for kidney lithiasis (stony concretions or calculi) ?
• High concentration of solute in urinary filtrate and low urine volume
-----> Precipitation of a urinary solute as a stone.

2. What is presentation of lithiasis?


• Colicky pain (pain that starts and stops suddenly); occurs due to muscular contraction of ureter.
• Hematuria
• Unilateral flank tenderness
• Stone is usually passed in hours; if not, may need surgery.

3. Explain different type of kidney stones.


Compositio Frequency Causes Treatment Comments
n
Calcium Most common - Most common cause is Hydrochlorothiazide
oxalate type idiopathic hypercalcuria (calcium sparing diuretic).
and/or (usually seen (exclude hypercalcemia as
Calcium in adults) a cause for hypercalcuria).
Phosphate - Also seen in Crohn's disease
(ileitis causes increased
absorption of oxalate).
Ammonium Second most Most common cause - - Surgically removal of the It forms
Magnesium common alkalization [urea --> NH3] of stone (due to size). Staghorn
Phosphate type urine by urease positive - Eradication of pathogen calculi in
(AMP) organism (ex-Proteus (to prevent recurrence). renal calyces -
vulgaris & Klebsiella). acts as a
Aka struvite
nidus for UTI
stone
by Proteus.
Uric acid -Third most - Most common in gout pt. - Hydration & alkalization
common (5%) - Hyperuricemia (seen in of urine (by KHCO3).
-Radiolucent leukemia or m.proliferative - Allopurinol is used for
(others are disorders) increases risk. for gout pt (xanthine
radiopaque) - Risk factors: acidic PH, hot oxidase inhibitor)
arid climite, low urine vol.
Cysteine Rare; most Associated with cystinuria Hydration and It May form
commonly (genetic defect of tubule alkalinization of urine staghorn
seen in resulting in low cysteine calculi .
children reabsorption).

Fig - staghorn AMP stone. If you see staghorn stone in adult, think uric acid stone; if you see
in kid, think cysteine stone.
Chronic Renal Failure

1. What are causes of ESRD [End-stage kidney failure]?


• Glomerular, tubular, inflammatory or vascular insults.
• Most common causes are:
- diabetes mellitus
- HTN.
- glomerular disease.

2. What are some presentations of ESRD?


• Uremia - pathological manifestations of severe azotemia.
• HTN - due to salt and water retention
• Hyperkalemia with metabolic acidosis.
• Hypocalcemia -
○ Decreased 1-alpha-hydroxylation of vitamin D in PCT.
○ Kidney can't excrete phosphate leading to hyperphosphatemia -
phosphate binds to free calcium in blood.
• Renal osteodystrophy - damage to bone due to kidney problem
○ Osteitis fibrosa cystica - hypocalcemia leads to high PTH (secondary hyperparathyroidism)
which decalcifies the bones; eventually leads to fibrosis and cyst formation.
○ Osteomalacia - when osteoid isn't mineralized (due to low calcium in blood)
○ Osteoporosis - bones become weak and brittle.
• Anemia (↓ erythropoietin production by renal peritubular interstitial cells).

2. What are the presentations of uremia?


- Abnormal Mental status, seizure, encephalopathy with Asterixis.
- Pericarditis
- platelet dysfunction.
- Metallic taste (due to parageusia).
- Urine-like odour on the breath (Uremic fetor)
- Crystallized urea deposits on skin (uremic frost)
- Pt have nausea leading to loss of apetite and causes anorexia.

3. What is prognosis / treatment of ESRD?


- Dialysis or renal transplant

5. What are classic indication of dialysis?


- A - acidosis (anion gap acidosis due to hyperkalemia)
- E - electrolyte imbalance (hyperkalemia)
- I - ingestion
- O - overload (volume) - due to salt and water retention
- U - uremia

4. What are some changes during dialysis ?


- ESRD kidneys becomes small and shrunken
- Cysts often develop
- Pt have high risk for renal cell carcinoma
- Pts get local amyloidosis in the joints.
Renal Neoplasia

Angiomyolipoma
1. What's angiomyolipoma? What's its associated with?
- Hamartoma made up of blood vessels, smooth muscle and adipose tissue.
- Increased frequency in tuberous sclerosis
- Syndrome with benign tumor primarily in brain, eyes, heart, kidney, skin and lungs.
- Common presentation: seizures, developmental delay, intellectual disability and autism.

Wilm's tumor (nephroblastoma)


1. What's histologic hallmark of Wilm's tumor?
- Malignant kidney tumor comprised of blastema (immature kidney mesenchyme),
primitive glomeruli and tubules, and stromal cells.
2. What is the presentation of Wilm's tumor ?
- Average age of presentation - 3 years : most common malignant renal tumor in children.
- Large unilateral flank mass, humaturia and HTN (due to renin production)

3. What are etiologies of Wilm's tumor?


- 90% of cases are sporadic. Syndromic tumors may be seen with:
WAGR syndrome Denys-Drash Beckwith-Wiedemann syndrome
syndrome
W = Wilm's tumor - Wilm's tumor - Wilm's tumor
A = Aniridia (lack of iris) - Progressive - Neonatal hypoglycemia
G = Genital abnormalities renal disease - Muscular hemihypertrophy
R = Retardation (mental & motor) - Male - Organomegaly (including tongue)
pseudo
hermaphroditism
- Deletion of WT1 tumor - Mutation of WT1 - Mutations in WT2 gene cluster
suppressor gene (located in gene (imprinted genes at 11p15.5),
11p13) particularly IGF-2 gene.
Renal cell carcinoma

1. What are presentations of renal cell carcinoma?


- Classic triad (rarely occurs together): hematuria (most common), palpable mass and
flank pain APART from fever and weight loss.
- Paraneoplastic syndromes :
○ Erythropoietin production (----> polycythemia vera)
○ Renin production (----> HTN)
○ Parathyroid hormone-related protein (PTHrP) ----> hypercalcemia.
○ ACTH production (----> cushings syndrome)
- Rare presentation: Left sided varicocele - left spermatic vein drainage to left renal
vein is blocked by tumor.
- Malignant ep. tumor arising from kidney tubules.

2. What is pathogenesis of renal cell carcinoma ?


- Loss of VHL (3p) tumor suppressor gene leads to:
○ ↑ IGF-1 (insulin like growth factor) - promotes growth.
○ ↑ HIF transcription factor - increases VEGF and PDGF synthesis.

3. What is staging of renal cell carcinoma?


- T - based on size and involvement of renal vein (occurs commonly and increases
risk of hematogenous spread to lung and bone)
- N - spread to retroperitoneal lymph nodes

4. Differentiate sporadic and hereditary versions of renal cell carcinoma.


Sporadic tumor Heriditary tumor
- Classically seen in males (avg age 60) as - Classically seen in young adults and are often
single tumor in upper pole of kidney bilateral
- Major risk factor is cigarette smoke. - Ex- Von Hippel-Lindau disease
• Autosomal dominant disorder
• Associated with inactivation of VHL gene
• Increased risk of:
=> hemangioblastoma of cerebellum
=> pheochromocytoma
=> renal cell carcinoma

5. What are gross and histologic features of RCC ?


- Microscopically, the most common variant
exhibits clear cytoplasm (clear cell type).
- Gross examination reveals a yellow mass.

Fig: gross feature (left) and microscopic feature (right) of RCC


Lower Urinary Tract Carcinoma
1. What constitutes the lower urinary tract?
- Renal pelvis, Ureter, Bladder, Urethra

2. What are 3 common malignant tumor of lower urinary tract ?


Cancer Risk factor Common
location
Urothelial (transitional cell) carcinoma - Cigarette smoking (major) - polycyclic Bladder.
(MOST COMMON CANCER OF LOWER URINARY aromatic hydrocarbon.
TRACT- urothelium lines entire lower urinary tract) - Naphthylamine.
- Azo dyes.
- Long term cyclophosphamide or
phenacetin use
Squamous cell carcinoma • Chronic cystitis (older woman) Bladder
- Arises after squamous metaplasia (urinary tract • Schistosoma hematobium infection
doesn't have squamous cells) (middle eastern male)
• Long standing nephrolithiasis

Adenocarcinoma • Arises from urachal remnant Bladder


- Normally no glandular ep. cells lines the bladder (tumor at dome of bladder)
- Malignant proliferation of glands • Cystitis glandularis - columnar
metaplasia due to chronic bladder
inflammation
• Exstrophy (congenital failure to form
caudal portion of anterior abdominal
and bladder wall) - bladder exposed to
outside world which increases risk in
future to have cancer

Fig: extrophy
- Urachal remnant - urachus is remnant of channel between bladder and umbilicus (urine drains this way in
fetus during 1st trimester of pregnancy). It's lined by glandular cells. From 12th week of gestation, it's
called median umbilical ligament.

2. What are clinical characters of urothelial carcinoma?


- Generally seen in older adults. Classically presents with painless hematuria
- Tumor are multifocal and recur (entire urothelium is mutated via mechanism called field defect).

3. What are two pathways of urothelial carcinoma ?


Flat Papillary
- Develops as high grade flat tumor and then - Develops as low grade papillary tumor.
invades. - Eventually becomes high grade papillary tumor and invades.
- Associated with early p53 mutation - Not associated with early p53 mutation
Chapter 13: Female Genital System
and Gestational Pathology

10. Pelvic inflammatory disease (PID) - infection of the upper female reproductive system
namely the uterus, fallopian tubes, ovaries
□ Untreated PID can lead to fibrosis that can lead to infertility and ectopic pregnancy.
13.1 Vulva

I. BASIC PRINCIPLES
A. Anatomically includes the skin and mucosa of the female genitalia external to the
hymen (labia majora, labia minora, mons pubis, and vestibule);
Lined by squamous epithelium.
B. One Bartholin gland is present on each side of the vaginal canal and produces
mucus-like fluid that drains via ducts into the lower vestibule;
Provide vulvar and vaginal (labial opening) lubrication.

1. Describe the following conditions about vulva.


Bartholin's A. Cystic dilation of the Bartholin gland
cyst B. Arises due to inflammation and obstruction of gland
- Usually occurs in women of reproductive age
C. Presents as a unilateral, painful cystic lesion at the lower vestibule
adjacent to the vaginal canal.
Condyloma A. Warty neoplasm of vulvar skin, often large & painless.
B. Types:
-> Condyloma acuminatum -Most commonly due to Low-risk-HPV types
6 & 11 [more commonly seen] aka Genital warts -benign mass.
-> Condyloma latum - due to secondary syphilis caused by Treponema
pallidum [less commonly seen].
C. It is sexually transmitted.
D. Histologically, HPV-associated condylomas are characterized by
koilocytes (hallmark of HPV-infected cells).
E. Condylomas rarely progress to carcinoma (HPV types 6 & 11 are low-
risk HPV types).
Lichen Lichen sclerosus Lichen simplex chronicus
- Vulvar involvement: thinning - Hyperplasia of vulvar squamous
of the epidermis and fibrosis epithelium.
(sclerosis) of dermis.
- Presents as a white patch - Presents as leukoplakia with
(leukoplakia) with parchment- thick, leathery vulvar skin .
like vulvar skin.
- Most commonly seen in - Seen at any age; usually in young
postmenopausal women. females.
- Benign; slightly increased risk - Benign; no increased risk of
for squamous cell carcinoma. squamous cell carcinoma.
- Possible autoimmune etiology. - Associated with chronic irritation
and scratching.
13.1 Vulvar Cancer

Vulvar carcinoma
Histology - Malignant proliferation of the squamous epithelium lining the vulva.
Epidemology - Relatively rare cancer -accounting for only a small % of female genital cancers.
Clinical - Vulvar carcinoma presents as a white lesion resembling Leukoplakia of vulva;
feature biopsy may be required to distinguish carcinoma from leukoplakia.
[Leukoplakia - dysplastic precancerous white lesion -benign condition.
Differential diagnosis is done by diagnosis of exclusion -via biopsy.
Cause of Leukoplakia: mainly tobacco].
- NB: here Leukoplakia is not/but may cause cancer.
Site - The labia majora are the most common site involved representing about 50%
of all cases, followed by the labia minora.

Etiology - HPV-related vulvar carcinoma: due to high-risk HPV types 16, 18, 31 & 33.
- Non-HPV related vulvar carcinoma: arises most often, from long-standing
Lichen sclerosus --> may also proceed to Differentiated VIN (dVIN).
HPV related 1. Risk factors are related to HPV exposure and include multiple partners and
early first age of intercourse; generally occurs in women of reproductive age.
2. Arises from Classic vulvar intraepithelial neoplasia (cVIN), a dysplastic precursor
lesion characterized by koilocytic change, disordered cellular maturation,
nuclear atypia, and increased mitotic activity -neoplasia limited to epithelium.
Non-HPV 1. Chronic inflammation and irritation eventually lead to carcinoma.
related 2. Generally seen in elderly women (average age is > 70 years).

Extramammary Paget disease


Histology - Characterized by malignant epithelial cells in the epidermis of the vulva.
Noninvasive intraepithelial neoplasia - carcinoma in situ.
- Malignant cells are spreading laterally within the epidermis along the
normal basal layers of vulvar epidermis -> extensive multicentric foci
involvement including the perianal region, genitocrural, & inguinal folds.
- It May be a primary lesion (malignant proliferation of Stratum basale
keratinocytes/Apocrine glands/Eccrine glands of vulvar epidermis -no
underlying malignancy) or associated with adenocarcinoma (malignant
proliferation of columnar/glandular cells -underlying carcinoma)
originating from local organs such as the Bartholin gland, the urethra,
or the rectum and thus be secondary lesion.

Presentation - Erythematous, pruritic, ulcerated vulvar skin.

Differential Must be distinguished from melanoma, which rarely can occur on the vulva:
Diagnosis(DDx) -> Paget cells: PAS+ve, keratin +ve and S100 -ve.
-> Melanoma: PAS-ve, keratin -ve and S100 +ve.
13.2 Vagina

I. BASIC PRINCIPLES
A. Canal leading to the cervix.
B. Mucosa is lined by non-keratinizing squamous epithelium.
C. During development, squamous epithelium from the lower 1/3 of the
vagina (derived from the urogenital sinus) grows upward to replace
the columnar epithelium lining of the upper 2/3 of the vagina (derived
from the Mullerian ducts).

Vaginal adenosis
Definition - Focal persistence of columnar epithelium in the upper vagina.
[presence of metaplastic cervical or endometrial epithelium
within the vaginal wall --> gland formation].
- Columnar ep. is a glandular tissue (normal component of
various exocrine glands) - benign abnormality in the vagina.
Cause - Increased incidence in females who were exposed to
diethylstilbestrol (DES) in utero- given to reduce pregnancy
complication (crosses placenta).
Complication - Progression to clear cell adenocarcinoma of vagina.
13.2 Vaginal Cancer

Clear cells adenocarcinoma


Histology - Malignant proliferation of glands with clear cytoplasm.
Cause B. Rare, but feared, complication of DES-associated
vaginal adenosis.
[Discovery of this complication (along with other DES-induced
abnormalities of the gynecologic tract such as abnormal shape
of the uterus) led to cessation of DES usage].

Sarcoma botryoides [a subtype of embryonal rhabdomyosarcoma]


Histology - Malignant proliferation of primitive/immature skeletal muscle of
embryo [mesenchymal stem cell derived tissue].
- Embryonal skeletal muscle = Myoblast -precursor cells.
Myogenesis is the formation of muscular tissue, particularly during
embryonic development. Skeletal muscle cell is a Permanent cell.
- Tumor is composed of malignant embryonal Rhabdomyoblasts
(resembles developing muscle cells of a 6–8 week old embryo);
This characteristic cell exhibits:
--> Spindle-shape and cytoplasmic cross-striations.
--> (+)ve immunohistochemical staining for desmin and myogenin.
- Beneath the vaginal epithelium, the tumor cells are crowded in a so-
called cambium layer, typically separated from mucosa by a myxoid
hypocellular zone. But in the deep regions they lie within a loose
fibromyxomatous stroma that is edematous and may contain many
inflammatory cells.
Clinical - Rare cancer
feature - Bleeding and grape like mass protruding from the vagina
(or nasopharynx or common bile duct or urinary bladder)
of an infant.
- Usually seen in < 5 yrs of age.

Vaginal carcinoma
Histology - Malignant proliferation of squamous epithelium lining the
vaginal canal.

Cause - Mostly HPV type 16, 18, 31, 33 - high-risk HPV.


Precursor lesion - Vaginal intraepithelial neoplasia (VAIN) -dysplastic changes.
Metastatis - When spread to regional lymph nodes occurs,
-> Cancer from lower 1/3rd of vagina - goes to inguinal LN.
-> Cancer from upper 2/3rd of vagina - goes to iliac LN.
13.3 Cervix
1. BASIC PRINCIPLES
A. Anatomically, comprises the "neck" of the uterus
B. Divided into the exocervix (visible on vaginal exam) and endocervix.
Exocervix Endocervix
Cervix that’s visible on vaginal exam Cervix that’s not visible on vaginal exam
Lined by nonkeratinizing squamous epithelium Lined by single layer of columnar cells
- Transformation zone - juction between the exocervix and endocervix.
13.3 Cervical Cancer

Cervical intraepithelial neoplasia (CIN)


Definition - Characterized by koilocytic change, disordered cellular maturation, nuclear atypia,
and increased mitotic activity within the cervical epithelium.
Site - HPVs infect basal stem cells of the immature metaplastic squamous cells present at the
squamocolumnar junction.
- HPVs cannot infect the mature superficial squamous cells that cover the ectocervix,
vagina, or vulva. Establishment of HPV infection in these sites requires damage to the
surface epithelium, which allows the virus access to the immature cells in the basal layer
of the epithelium.
Progression - CIN classically progresses in a stepwise fashion through CIN I, CIN II, CIN III, and
CIS to become invasive SCC of cervix.
1. Progression is not inevitable (e.g., CIN I often regresses).
2. The higher the grade of dysplasia, the more likely it is to progress to carcinoma
and the less likely it is to regress to normal.
3. CIN I, II and III are reversible (reversibility of I>II>III); CIS is irreversible.
Grades - Divided into grades based on the extent of epithelial involvement:
Thickness of entire epithelium = 'T' [Progression - basal --> surface ep.]
1. CIN I [mild dysplasia] involves < 1/3rd of T -LSIL.
2. CIN II [moderate dysplasia] involves < 2/3 of T -LSIL.
3. CIN III [severe dysplasia] involves > 2/3 of T -HSIL.
4. CIN III [Carcinoma in situ (CIS)] involves entire T -HSIL.
(Nb: LSIL: low-grade squamous intraepithelial lesion
HSIL: high-grade squamous intraepithelial lesion)
High risk HPVs
Histology A. Sexually transmitted DNA virus that infects the lower genital tract,
especially the cervix in the transformation zone.
B. Infection is usually eradicated by acute inflammation;
persistent infection leads to an increased risk for cervical dysplasia
(cervical intraepithelial neoplasia, CIN).
C. Most common cause of SCC of genito-urinal tract.
D. HPV infection --> Also leads to adenocarcinoma of cervix
(15% of cervical carcinoma cases)

Etiology - Risk of CIN depends on HPV type, which is determined by DNA sequencing.
1. High-risk-HPV types 16, 18, 31, and 33 --> CIN --> Invasive Cervical cancer
(80% of cervical carcinoma cases).
2. Low-risk-HPV types 6 and 11 --> No cervical cancer.

Patho - High-risk HPV produces proteins --> Loss of tumor suppressor proteins:
physiology • E6 -> inactivates p53 protein.
• E7 -> inactivates Rb protein.
Invasive Cervical carcinoma

Definition - Invasive carcinoma that arises from the cervical sq. epithelium.
Epidemology - Most commonly seen in middle-aged women
(average age is 40-50 years).
Clinical - Presents as vaginal bleeding, especially postcoital bleeding,
features or cervical discharge.
Secondary - immunodeficiency (-> AIDS-defining illness).
Risk factors - smoking.
- Oral contraceptives
- Multiple pregnancies
Complication F. Advanced tumors often invade through the anterior uterine
wall into the bladder, blocking the ureters.
--> Hydronephrosis with postrenal failure is a common cause
of death in advanced cervical carcinoma.
SCREENING AND PREVENTION OF CERVICAL CARCINOMA - Pap smear

Goal - To catch dysplasia (CIN) before it develops into carcinoma.


1. Progression from CIN to carcinoma, on average, takes 10-20 years.
Frequency - Screening begins at age 21 until the age of 65 and is performed every 3 to 5 years.
Epidemology - In the United States, it is the eighth-most common cancer of women.
The median age at diagnosis is 48.

Process B. Pap smear is the gold standard for screening.


1. Cells are scraped from the transformation zone using a brush and analyzed
under a microscope.
2. Dysplastic cells are classified as low grade (CIN I) or high grade (CIN II and III).
3. High-grade dysplasia is characterized by cells with hyperchromatic (dark)
pleomorphic nuclei and high nuclear to cytoplasmic ratios.

Significance C. Pap smear is the most successful screening test developed to date.
1. It is responsible for a significant reduction in the morbidity and mortality of
cervical carcinoma (cervical carcinoma went from being the most common to
one of the least common types of gynecologic carcinoma in the US).
2. Women who develop invasive cervical carcinoma usually have not undergone
screening.

Confirmatory D. An abnormal Pap smear is followed by confirmatory colposcopy


test (visualization of illuminated cervix with a magnifying glass) and
cervical biopsy.
Limitations E. Limitations of the Pap smear include inadequate sampling of the
transformation zone (false negative screening) and limited efficacy in
screening for adenocarcinoma.
l. Despite Pap smear screening, the incidence of adenocarcinoma has not
decreased significantly.
Immunization F. Immunization is effective in preventing HPV infections.
1. The quadrivalent vaccine covers HPV types 6, 11, 16, and 18.
2. Antibodies generated against types 6 and 11 protect against condylomas.
3. Antibodies generated against types 16 and 18 protect against ClN and carcinoma.
4. Protection lasts for 5 years.
5. Pap smears are still necessary due to the limited number of HPV types covered
by the vaccine.
Normal pap Endocervix - normal squamous epithelial cells present
smear --> [small round nuclei with abundant cytoplasm]
Exocervix - normal columnar epithelial cells present
--> [blue cytoplasm and cilia sometimes seen]
13.4 Endometrium and Myometrium

Endometrium - Mucosal lining of uterus


Myometrium - Smooth muscle underlying the endometrium
Hormone cycle of Proliferative Estrogen driven Endometrium grows
endometrium phase
Secretory phase Progesterone driven Endometrium prepares for
implantation
Menstrual Due to loss of Endometrium falls
phase progesterone

I. BASIC PRINCIPLES
A. Endometrium is the mucosal lining of the uterine cavity.
B. Myometrium is the smooth muscle wall underlying the endometrium.
1. Myometrium has 3 layers: - outer longitudinal smooth muscles.
- middle crisscrossing muscle fibre.
- inner circular fibres.
C. Endometrium is hormonally sensitive.
1. Growth of the endometrium is estrogen driven (proliferative phase).
2. Preparation of the endometrium for implantation is progesterone driven (secretory phase).
3. Shedding occurs with loss of progesterone support (menstrual phase).

A. In a woman of reproductive age, two layers of endometrium can be distinguished. These


two layers occur only in endometrium lining the cavity of the uterus, not in the lining of the
uterine (Fallopian) tubes:
1. The functional layer [Stratum Functionalis] is adjacent to the uterine cavity. This layer is
built up after the end of menstruation during the first part of the previous menstrual cycle.
Proliferation is induced by estrogen (follicular phase of menstrual cycle), and later changes in
this layer are engendered by progesterone from the corpus luteum (luteal phase). It is
adapted to provide an optimum environment for the implantation and growth of the
embryo. This layer is completely shed during menstruation.
2. The basal layer [Stratum Basalis] , adjacent to the myometrium and below the functional
layer, is not shed at any time during the menstrual cycle, and from it the functional layer
develops. This layer is composed of stem cells.

1. Dilation and curettage (D&C) refers to the dilation (widening/opening) of the cervix and
surgical removal of part of the lining of the uterus and/or contents of the uterus by scraping
and scooping (curettage). It is a therapeutic gynecological procedure as well as the most
often used method of first trimester miscarriage or abortion.
Asherman's - Secondary amenorrhea [absence of menstruation] due to intrauterine
syndrome adhesions and/or fibrosis(scarring) of the endometrium [---> loss of
Stratum Basalis].
- Result of overaggressive dilation and curettage (D&C) of the
intrauterine cavity.
Anovulatory - Varying degrees of menstrual intervals and the absence of ovulation
cycle and a luteal phase ---> infertility.
- But follicular phase is present.
- Normal menstrual bleeding in the ovulatory cycle is a result of a decline in
progesterone due to the demise of the corpus luteum. It is thus a
progesterone withdrawal bleeding. As there is no progesterone in the
anovulatory cycle, bleeding is caused by the inability of estrogen — to support
a growing endometrium. Anovulatory bleeding is hence termed estrogen
breakthrough bleeding.
A. Results in an estrogen-driven proliferative phase without a subsequent
progesterone driven secretory phase. Endometrium develops by estrogen
but fails to be maintained; so it falls off
1. Proliferative glands break down and shed resulting in uterine bleeding.
- Represents a common cause of dysfunctional uterine bleeding, especially
during menarche and menopause.

Acute endometritis Chronic endometritis


- Acute infection of the endometrium. - Chronic inflammation of endometrium
- Cause: Bacterial -[Mycoplasma genitalium]. - Cause - Bacterial - [Chlamydia trachomatis and
Neisseria gonorrhoeae].
FEATURE: FEATURE:
- Histologically, neutrophilic infiltration of the - Characterized by presence of plasma cells and
endometrial tissue is present during acute lymphocytes in biopsy
endometritis. - Plasma cells are necessary for the diagnosis of
chronic endometritis given that lymphocytes are
normally found in the endometrium.

ETIOLOGY: ETIOLOGY
- Usually due to retained products of - retained products of conception.
conception (after delivery or miscarriage) - - Intrauterine device.
serves as a nidus for infection. - miliary Tuberculosis.
- Chronic pelvic inflammatory disease (PID).
PRESENTATION: PRESENTATION:
Presents as fever, abnormal uterine bleeding, Presents as abnormal uterine bleeding, pain, and
and pelvic pain. infertility.
Endometriosis
Definition - Endometrial glands and stroma outside of the uterine endometrial lining.
[implantation of Endometrial tissue at other abnormal sites - follows normal
menstruation cycle just like normal endometrium of uterus].
Common - Most common site of involvement - which classically results in formation of
sites of hemorrhagic (aka 'chocolate') cyst.
involvement --> Ovary - results in Nontubal ectopic pregnancy.
- Other sites of involvement - implants classically appear as yellow-brown
'gun-powder' nodules.
--> Uterine ligament - cause pelvic pain
--> Pouch of Douglas - pain with defecation
--> Bladder wall (mucosal/serosal) - pain with urination
--> Bowel serosa - abdominal pain and adhesions (b/w loops of bowels).
--> Fallopian tube mucosa - scarring increases risk for ectopic tubal pregnancy.
[normal Hair-like cilia gets degenerated => fertilized eggs cannot reach uterus
=> gets implanted in Fallopian tube].
--> Uterine myometrium - Adenomyosis.
Cause It is Most likely due to:
---> retrograde menstruation.
---> coelomic metaplasia
Presentation - Presents as dysmenorrhea/menstrual cramps (pain that occurs prior to,
during or after menstruation); may cause infertility.
- Pain can occur during sexual intercourse, urination or bowel movement.
- Severe disabling Chronic pain in low back or pelvis.
Complication - There is an increased risk of carcinoma at the site of endometriosis,
especially in the ovary - aka endometoid tumor, which is a malignant
tumor of surface epithelium of ovary.

Endometrial polyp
Definition - Hyperplastic protrusion of endometrium.
- They are either sessile or pedunculated.
Presents - Menorrhagia [abnormal heavy uterine bleeding at menstruation].

Etiology - Can arise as a side effect of tamoxifen (which has anti-estrogenic


effects on the breast but weak pro-estrogenic effects on the
endometrium also) -Selective estrogen receptor modulators (SERMs).
Endometrial hyperplasia
Definition - Hyperplasia of endometrial glands MOSTLY -relative to stroma.
- resulting in an increased gland-to-stroma ratio when compared with
normal proliferative endometrium.
Etiology - Unopposed estrogen activity + insufficient levels of progesterone.
[Progestogens ordinarily counteract estrogen's proliferative effects
on this tissue].
Association • Obesity
• Polycystic ovary syndrome
• Estrogen replacement
• Granulosa cell tumour
Presentation - Classically presents as postmenopausal uterine bleeding.
Classification 1. Based on architectural growth pattern of glands
(histological) --> simple: Irregularity and cystic expansion of glands.
--> complex: crowding and budding of glands.
2. Based on appearance of individual gland cells
--> without atypia: no cellular atypia.
--> atypical: cell stratification, loss of nuclear polarity,
enlarged nuclei, and an increase in mitotic activity.
Prognosis 1. Most important predictor for progression to carcinoma (major
complication) is the presence of cellular atypia;
-> simple hyperplasia with atypia often progresses to cancer (30%).
-> complex hyperplasia without atypia rarely does (<5%).
Diagnosis - Endometrial biopsy
Treatment - continuous progestin therapy.
- Hysterectomy -surgical removal of the uterus.
13.4 Endometrial Carcinoma

Definition - Carcinoma of the endometrium.


Epidemiology - Most common invasive carcinoma of female genital tract.
Presentation - postmenopausal uterine bleeding
Causes - Arises via two distinct pathways: hyperplasia and sporadic
Hyperplasia pathway Sporadic pathway
Comprises: 75% of cases. Comprises: 25% of cases.
Origin: carcinoma arises from endometrial Origin: carcinoma arises in an atrophic
hyperplasia. endometrium with no evident precursor lesion.

- Average age of presentation is 60 years. - Average age at presentation is 70 years.

- Histology is endometrioid (i.e., normal - Histology is usually serous (i.e. referred to as


endometrium-like) and also show such because of morphologic and biologic
-adenocarcinoma (Malignant proliferation of overlap with ovarian serous carcinoma) and is
endometrial glands). characterized by papillary structures
with psammoma body formation [the
infarction and calcification of papillae tips].

- Risk factors are related to estrogen exposure - Tumor protein p53 (TRP53) mutation is
and include: common and the tumor exhibits aggressive
• Early menarche/late menopause. behavior [quickly growing, tending to spread
• Nulliparity -never having given birth. rapidly].
• Infertility with Anovulatory cycles.
• Obesity.
13.4 Myometrial Tumors

Uterine Leiomyoma (aka Uterine fibroids) aka Uterine leiomyomata

Definition - Benign neoplastic proliferation of smooth muscles of myometrium.


Epidem - Most common tumor in females.
- Common in premenopausal women.
Feature 1. Gross:
- Round, well circumscribed (but not encapsulated).
- May be single but often multiple in number -well defined.
- Size of gray-white solid Nodules/Mass is variable.
- Cut section: characteristic whorled(curly) meshwork appearance.
[formed by intersecting SM bundles & fibrous tissue].
2. Histology:
- Benign smooth muscle cells resemble normal SM cells.
- BSM cells are arranged in Whorl pattern -> Whorled bundles.
These Whorl patterns interlace -> Interlacing fascicles
- SM Bundles are directed in different directions.
[Composed of transverse + circular + oblique bundles].
Whorl = Concentric; Interlace = Cross; Fascicles = Bundles.
3. Site:
- Intramural - located within the myometrium.
- Submucosal - Beneath the endometrium or pedunculated.
- Subserosal - Beneath the serosa or pedunculated.

Relation - Dependent on estrogen and progesterone to grow.


- Enlarge during pregnancy; shrink after menopause.
Clinical - Usually asymptomatic.
feature - When present, symptoms include:
- abnormal uterine bleeding.
- infertility.
- pelvic mass -distort the uterus and impinge on pelvic structures.
Prognosis - DOES NOT PROGRESS TO LEIOMYOSARCOMA.

Uterine Leiomyosarcoma [LMS]


Definition - Malignant neoplastic proliferation of smooth muscles of myometrium.
Epidemiology - Often seen in postmenopausal women
Cause - Arise de novo; do not arise from leiomyoma
Presentation Gross Examination:
- Single lesion with area of necrosis and hemorrhage
Histological features:
- cellular atypia, mitotic activity and necrosis.
13.5 Ovary

Ovarian follicle
Cell type Location (+)ve Response
stimulus
Granulosa cell Located right around FSH Convert androgen from theca cells to
aka [follicular cell] follicle ESTROGEN (estradiol) -also secrete inhibin.
Theca cells Surround Granulosa LH Produce androgens (androstendione)
cells -lack the aromatase enzyme

Corpus luteum
Cell-type Feature Cell surface Hormones produced
receptor
Small luteal cells It is colored due FSH-R Produce mainly estrogen
(thecal-lutein cells) to lutein - - also secrete inhibin A [α + βA chain].
[a carotenoid].
Large luteal cells LH-R Produce PROGESTERONE (a Progestogen)
(granulosal-lutein and androgens.
cells)

I. BASIC PRINCIPLES
A. The functional unit of the ovary is the follicle.
B. A follicle consists of an oocyte surrounded by granulosa and theca cells.
1. LH acts on theca cells to induce androgen production.
2. FSH stimulates granulosa cells to convert androgen to estradiol (drives the
proliferative phase of the endometrial cycle).
3. Estradiol surge induces an LH surge, which leads to ovulation (marking the
beginning of the secretory phase of the endometrial cycle).
C. After ovulation, the residual follicle becomes a corpus luteum, which
primarily secretes progesterone (drives the secretory phase which prepares
the endometrium for a possible pregnancy).

Hemorrhagic corpus - Hemorrhage into a corpus luteum (Corpus luteal cysts are formed if
luteal cyst corpus leutem fails to disintegrate and instead persists).
- Corpus - Normal; commonly found during early phase of pregnancy
hemorrhagicum. - Temporary structure formed immediately after ovulation from the
ovarian follicle as it collapses and is filled with blood that quickly clots.
During ovulation small blood vessels rupture, and the cavity of the
ruptured follicle fills with a blood clot. After the trauma heals, the
subsequent structure is called the corpus luteum.
Primordial follicle
- contain immature oocytes surrounded by flat, squamous granulosa cells (the support cells)
that are segregated from the oocyte's environment by the basal lamina.
Primary follicle
- the granulosa cells now have a cuboidal structure.
- A glycoprotein polymer capsule called the zona pellucida forms around the oocyte,
separating it from the surrounding granulosa cells. It is secreted by both the oocyte and
the granulosa cells.
Secondary follicle
- Stroma-like theca cells are recruited by oocyte-secreted signals. They surround the follicle's
outermost layer, the basal lamina, and undergo cytodifferentiation to become the theca
externa and theca interna.
Antral follicle
- Formation of a fluid-filled cavity adjacent to the oocyte called the antrum
- The follicular fluid [FF] contains:
(i) sex steroids (i) glycoprotein hormones (i) plasma proteins
(i) mucopolysaccharides (i) enzymes (i) signalling proteins.
- FF is a product of both:
--> the transfer of blood plasma constituents that cross the BF barrier
Blood follicular (BF) barrier - tight junctions b/w the endothelial cells
of thecal arteriole + follicular basement membrane.
--> secretory activity of granulosa cells Mainly.
- granulosa cells begin to differentiate themselves into these layers:
1. Membrana granulosa - that's interior to the basal lamina.
2. Periantral granulosa - that's adjacent to the antrum.
3. Cumulus oophorus - regarded as extension of the membrana granulosa.
- connects layer1 & layer4 together.
4. Corona radiata - The innermost layer of layer3 that surrounds zona pellucida.

- Double-strand breaks accumulate in their primordial follicle reserve.


Expression of four key DNA repair genes [BRCA1, MRE11, RAD51 and
ATM] necessary for homologous recombinational repair [during
meiosis] declines in primary oocytes with age - Ovarian oocyte reserve
gets depleted due to degeneration of primordial follicles.
- % of non-growing follicles (NGFs) decreases with age until menopause
-(when no more primordial follicle reserve left).
[Dictyate - all P.oocytes in the primordial follicles are arrested in the
Prophase I (diplotene stage)- starts late in fetal life and is terminated
until puberty commences -halting of mRNA translation.
- likely an adaptation for removing DNA damages].
- A women ovulate approx around 400 times in her lifetime.
Typically around 20 follicles mature each month but only a single follicle is ovulated.
The rest undergo follicular atresia [granulosa cell APOPTOSIS - due to ↓ FSH signals].
- Normally 1 follicle is ovulated from both ovaries in a single ovarian cycle;
(exception may be present - sometimes >1 follicles are ovulated at same time).
Polycystic ovary syndrome (PCOS)

Definition - Multiple ovarian follicular cysts [<--- Follicular degeneration].


- These "cysts" are actually immature follicles [containing antral fluid].
Epidemiology - Affects roughly 5% of women of reproductive age.
Patho - Main finding is hormone imbalance - LH:FSH>2
physiology • ↑ LH by pitutary - induces ↑ androgen (testosterone) production
(from theca cells)
• Androgen is converted to Estrone (weak estrogen) in adipose tissue.
i. Estrone feedback - leads to ↓ FSH level.
- Insulin resistance and type 2 diabetes
• Compensatory hyperinsulinemia ---> ↑ insulin level -Insulin acts
synergistically with LH.

Stage - The follicles have developed from primordial follicles, but the development has
stopped ("arrested") at an early antral stage due to the disturbed ovarian function.
Diagnosis - The follicles may be oriented along the ovarian periphery, appearing as a
'string of pearls' on ultrasound examination.
Clinical • Central obesity - due to ↑ insulin circulating levels -[seen 10-15 years later].
feature • young infertile women - ↓ FSH results in cystic degeneration of follicles.
• oligomenorrhea - infrequent menstruation [occurring at intervals of > 35 days].
• hirsutism -(excess hair in a male distribution) - coz of excess androgens.
• endometrial carcinoma - ↑ levels of circulating estrone increases risk.
n.b) Small numbers of follicular cysts are common in women and have no
clinical significance -Follicular atresia.
13.6 Ovarian Tumors

I. BASIC PRINCIPLES
A. Ovary is composed of three cell types:
-> Surface epithelium/ Germinal epithelium (female)
Derived from coelomic epithelium that lines the ovary; coelomic epithelium
embryologically produces the epithelial lining of the fallopian tube (serous cells),
endometrium, and endocervix (mucinous cells).
-> Germ cells
-> Sex cord stroma.
B. Tumor can arise from any of these cell types or from metastases.

Tumor type Tumor subtype Remarks


Surface - Most common subtypes:. EPIDEMOLOGY:
epithelial • Serous tumor - filled with watery fluid. - Most common type of ovarian tumor (70% of cases).
tumors - usually cystic. PROGNOSIS:
• Mucinous tumor - filled with mucus-like fluid. 1. Prognosis is generally poor for surface epithelial carcinoma
- usually cystic. (worst prognois of all female genital tract cancers).
nb: Mucinous and serous tumors can be - 2. Epithelial carcinomas tend to spread locally, especially to
(i) benign (ii) borderline (iii) malignant. the peritoneum.
CLINICAL FEATURES:
- Less common subtypes:
Surface tumors clinically present late with vague abdominal
• Endometrioid tumor discomfort (pain and fullness) or signs of bladder compression
• Brenner tumor (urinary frequency).
DIAGNOSIS:
- CA-125 is a useful serum marker to monitor treatment
response and screen for recurrence.
- Distant metastasis - Peritoneum (via direct contact) - see
omental caking

Germ cells Germinomatous tumors: EPIDEMOLOGY:


tumors - 2nd most common type of ovarian tumor (15% of cases)
(GCT) Oocytes - Dysgerminoma - Usually occur in women of reproductive age.
Nongerminomatous tumors: FEATURES:
- GCTs are neoplasms derived from germ cells which normally
Placental tissue - Choriocarcinoma
occur inside the gonads - they resembles embryonic structures
Fetal tissue - Cystic teratoma histologically.
- Embryonal carcinoma - GCTs that originate outside the gonads may be birth defects
(congenital) resulting from errors during development of the
Yolk sac - Endodermal sinus tumor
embryo - abnormal migration of germ cells during
embryogenesis - initially thought to be isolated metastases.

Sex cord– Cell/tissue type Tumor location -Ovary (female) FEATURES:


gonadal -> SC-GSTs are derived from the stromal component of the
stromal Sex cord - Granulosa cell tumour gonads -characterized by their abnormal production of
tumour otherwise apparently normal cells or tissues.
Gonadal stroma - Thecoma
[SC-GST] -> Although each of the cell and tissue types normally occurs
- Fibroma
in just one sex (male or female), within a tumour they can
Mixed - Sertoli-Leydig cell tumour occur in the opposite sex. Consequently, depending on the
specific histology produced, these tumours can cause
virilization in women and feminization in men.
Surface epithelial tumors

Benign tumors [Cyst-adenomas] Borderline tumors Malignant tumors [Cyst-adeno-carcinomas]


- Composed of a single cyst with a - Have features in between benign - Composed of complex cysts with a thick,
simple, flat lining. and malignant tumors. shaggy lining.
- Better prognosis than clearly ---> BRCA1 mutation carriers have an
malignant tumors, but still carry increased risk for serous carcinoma of the
metastatic potential. ovary and fallopian tube.
---> BRCA1 carriers often elect to have a
prophylactic salpingo-oophorectomy
(along with prophylactic bilateral mastectomy
due to the increased risk for breast cancer).

- Most commonly arise in premenopausal - Most commonly arise in postmenopausal


women (30-40 years old) women (60-70 years old).

Endometroid tumors Brenner tumors


- Mostly malignant in nature. - Mostly benign in nature.
- Comprised of 10–20% of SC-GSTs of ovarian - Uncommon subtype of the surface epithelial-
neoplasms -Endometrioid type. stromal tumor [SC-GST].
HISTOLOGY: HISTOLOGY:
Tumor made of endometrial like glands -show Tumor made up of bladder like ep. -transitional
tubular glands. epithelial (urothelial) cells.
ORIGIN: ORIGIN:
1) 15%-20% cases - ovarian Endometrioid carcinoma 1) Arises from surface epithelial cells of ovary.
arise from Endometriosis of ovary.
2) 15%-30% cases - ovarian Endometrioid
carcinomas are accompanied by carcinoma of the
endometrium. The two arise independently rather
than by metastatic spread.
A B
Figs 18.22A and B: Gross appearance of mucinous cystadenoma of ovary. A. Outer smooth surface.
B. Cut section shows multiloculated cyst

A B

Figs 18.23A and B: Microscopic appearance of mucinous cystadenoma of ovary. A. Photomicrograph;


B. Diagrammatic representation shows cyst lined by columnar cells basally situated nuclei and apical mucin

A B C

Figs 18.21A to C: Microscopic appearance of papillary serous cystadenoma of ovary showing cyst wall and papilla lined by single layer of cuboidal
epithelium {A. (H&E) and B. (Diagrammatic)}. C. Papillary serous cystadenocarcinoma of ovary showing papillae lined by multilayered columnar
epithelium and one core of papilla with psammoma body (H&E)
Germ cell tumors

Dysgerminoma Choriocarcinoma Embryonal carcinoma Cystic teratoma Endodermal sinus (yolk


[Dermoid cyst] sac) tumor
- Most common malignant - Most common germ cell - Most common germ cell
GCT of ovary. tumor in females. tumor in children.
- Testicular counterpart is - Bilateral in 10% of cases.
called seminoma, which is a
relatively common GCT
in males. (histologically
indistinguishable)
- Malignant tumor - Malignant tumor Malignant tumor Benign Tumor [but presence Malignant tumor
- Good prognosis; responds (trophoblasts are very [Aggressive with early of immature tissue (usually - Good prognosis:
to radiotherapy invasive - early metastasis] neural) or somatic responds well to
hematogenous spread). - Poor prognosis malignancy (usually SCC of chemotherapeutic drugs.
- Poor prognosis. skin) indicates malignant
potential of tumor].

- Serum LDL maybe ↑. - ↑ β-hCG characterstic - - Monodermal teratoma - Serum AFP is often ↑.
may lead to Theca lutein [ex- Struma ovarii -
cyst in the ovary (Teratoma composed
[hCG acts on hCG- primarily of thyroid tissue
receptor on thecal-lutein --> Hyperthyroidism)].
cells & promotes the
survival of Corpus luteum
at early pregnancy].

- Tumor originates from - Tumor composed of Tumor resembles Tumor composed of fetal - Tumor histologically
primary oocyte within the cytotrophoblasts and totipotent stem cells - tissue derived from 2 or 3 mimics the yolk sac.
primordial follicle -(Oocyte syncytiotrophoblasts - histologically composed embryologic layers - Schiller-Duval bodies
escapes encapsulation of mimics placental tissue, of large primitive cells. (ectoderm, mesoderm and (glomerulus-like
follicle --> lack of contact but villi are absent. endoderm) -consists of structures) are
inhibition --> Rapid growth). - Small, hemorrhagic skin, hair, bone, teeth, classically seen on
- Tumor composed of large tumor on gross exam. cartilage, gut, and thyroid histology.
cells with clear cytoplasm and tissue.
central nuclei -fried egg - originates from germ cells.
apperance on biopsy.

A B C
Figs 18.25A to C: A. Opened mature cystic teratoma (dermoid cyst) of the ovary; B. Photomicrograph; C. Diagrammatic appearance of dermoid cyst of ovary
Sex cord-stromal tumors
Granulosa cell tumour Thecoma Fibroma Sertoli-Leydig cell tumor
- originates from & composed of - originates from & composed of - Benign tumor of fibrocytes - HISTOLOGY:
granulosa cells. thecal cells. (inactive mesenchymal cell). - Tumor is Composed of variable
proportions of Sertoli cells &
CLINICAL FEATURES: CLINICAL FEATURES:
Leydig cells -well differentiated
Presentation based on excess estrogen production by tumor cells. - On histologic examination, they
neoplasm.
a) Prior to puberty --> precocious puberty. are composed of well-differentiated
- Composed of tubules lined by
b) Reproductive age --> menorrhagia or metrorrhagia fibroblasts and a scant interspersed
Sertoli cells and interstitial
[Menorrhagia -abnormally heavy bleeding at menstruation collagenous stroma.
clusters of Leydig cells.
Metrorrhagia -abnormally heavy bleeding at irregular intervals]. ASSOCIATION:
- characterstic Reinke crystals
c) Postmenopause (most common setting for these tumors) Meigs syndrome:
(rod-like cytoplasmic inclusions)
--> endometrial hyperplasia with uterine bleeding. --> ovarian fibroma
found in Leydig cells.
PROGNOSIS: --> pleural effusions
CLINICAL FEATURES:
Good Prognosis -Malignant, but minimal risk for metastasis. --> ascites
- Presentation based on excess
- the syndrome resolves with
androgen [testosterone]
removal of tumor.
production by tumor cells.
- Associated with:
hirsutism and virilization.

Metastatic Tumors of ovary

Pathological Condition Source Presentation


Krukenberg tumor Gastric adenocarcinoma - Nature -Mucinous tumor of Ovary.
(diffuse type). - Involves both ovaries -Mostly bilateral (if unilateral,
think of primary mucinous carcinoma of the ovary,
i.e. Mucinous cystadenocarcinoma of ovary).
- Characterized by mucin-secreting signet-ring cells.

Pseudomyxoma peritonei (PMP) Mucinous Adenocarcinoma - Nature -Mucinous tumor of Ovary -cystic form.
[myxoma = benign tumour of of appendix -polyp form. - Massive amount of mucus in peritoneum - "jelly belly".
primitive connective tissue containing [caused by cancerous cells (mucinous adenocarcinoma)
mucus or gelatinous material] that produce abundant mucin or gelatinous ascites].
--> Rupture of the tumor intraperitoneally --> Spillage
of cells & mucus along the peritoneal fluid --> deposition
in peritoneal structures.
--> Here, Mucus is produced by both appendix & ovary.
13.7 Gestational Pathology

Teratogenic Diseases

- Effect of teratogens generally depends on:


---> the dose of teratogen.
---> the teratogenic agent.
---> the time of exposure to teratogen.

1. ↑ Dose of Teratogen ∝ ↑ Adverse effects

2. Time of exposure Adverse effect


0-2 weeks of Gestation Spontaneous abortion.
3-8 weeks of Gestation Risk of organ malformation.
3-9 months of Gestation Risk of organ hypoplasia.

3. Teratogen Associated Effects


Alcohol - Most common cause of mental retardation.
- Facial abnormalities and microcephaly.
Cocaine - Intrauterine growth retardation & placental abruption.
Thalidomide - Limb defects.
Cigarette smoke - Intrauterine growth retardation.
Isotretinoin - Spontaneous abortion.
- Hearing and visual impairment.
Tetracycline - Discolored teeth.
Warfarin - Fetal bleeding.
Phenytoin - Digit hypoplasia and cleft lip/palate.

Ectopic pregnancy
Definition - Implantation of fertilized ovum at site other than uterine wall.
- Most common site is the lumen of the fallopian tube.
Risk Factor - Scarring [secondary to Pelvic inflammatory(PID) disease or Endometriosis].
Presentation - Lower quadrant abdominal pain a few weeks after a missed period.
Prognosis - Surgical emergency -bad prognosis.
Complication - Bleeding into fallopian tube (hematosalpinx) and rupture.

Spontaneous abortion
Definition - Miscarriage of fetus <20 weeks of gestation (usually during 1st trimester).
Epidemiology - Common; occurs in up to 1/4th of recognizable pregnancies.
Presentation - Vaginal bleeding with passage of fetal tissue.
- Menstrual Cramp-like pain.
Etiology - Most common cause:
--> chromosomal anomalies (especially trisomy 16).
- Other causes:
--> hypercoagulable states (e.g., antiphospholipid syndrome).
--> Congenital infections (ex- bacterial and viral diseases).
--> Exposure to teratogens (especially during first 2 weeks of embryogenesis).
Placenta previa Placental abruption Placenta accreta
- Implantation of - Separation of the maternal - Abnormal Implantation of placenta into
placenta in the placenta (decidua basalis - the Myometrium.
Endometrium of lower modified functional layer of - Cause: Defective development of decidua
uterine segment; Endometrium) from the basalis --> Absence (partial/total) of
- Placenta overlies and uterus (stratum basalis of intervening decidua basalis [DB].
closes cervical os Endometrium) - prior to - 3 grades of abnormal placental
(opening). delivery of the fetus. attachment are defined according to the
depth of attachment and invasion into
Myometrium:
--> Accreta -chorionic villi attach to the
myometrium [does not penetrate]; rather
than being restricted within the decidua
basalis -CV invades stratum basalis.
--> Increta -chorionic villi [CV] invade deep
into the Myometrial layer.
--> Percreta -chorionic villi invade through
the perimetrium (uterine serosa) -placenta
grows completely through myometrium.
and causes damage to local organs
(ex- bladder or bowel).

- Presents as - Presents as painful 3rd- - Presents as difficult delivery of the


painless 3rd- trimester bleeding. placenta and postpartum bleeding
trimester bleeding. Placental insufficiency leads - Often requires hysterectomy -surgical
- Often requires to Fetal distress. removal of the uterus
delivery of fetus by - COMMON CAUSE OF
caesarian section. STILLBIRTH.

Pre-eclampsia Eclampsia HELLP syndrome


- Pregnancy induced: - Pre-eclampsia - Preeclampsia + thrombotic
= Severe hypertension --> headaches + seizures. microangiopathy (TMA) involving
and visual abnormalities. the liver.
= Proteinuria and Edema. - Clinical features:
- Usually arise in 3rd trimester. -> Hemolysis.
- Caused due to abnormality of the -> Elevated Liver enzymes.
maternal-fetal vascular interface in -> Low Platelets.
the placenta.

- Resolves with delivery - Warrant immediate delivery

Sudden infant death syndrome


Definition - Death of a healthy infant (1 month - 1 year old)
without any obvious cause.
Presentation - Infants often expire during sleep.
Risk factors - Sleeping on stomach.
- Exposure to cigarette smoke.
- Prematurity.
Hydatidiform mole [Molar pregnancy]
Neoplasm - It is a Gestational trophoblastic disease [GTD].
- Hydatidiform mole is a benign tumor.
Histology - Abnormal conception[fertilization of egg] characterized by swollen and
edematous villi [leaky FETAL BLOOD VESSELS] with benign proliferation of
trophoblastic cells.
Clinical - Uterus expands as if a normal pregnancy is present, but the uterus is much
feature larger due to benign mass.
- Classically presents in the 2nd trimester as passage of grape-like masses
through the vaginal canal.
Diagnosis - Tumor cells produce β-hCG --> ↑↑ β-hCG level in blood --> β-hCG is much
higher than expected for date of gestation.
- With prenatal care, moles are diagnosed by routine ultrasound in the early
1st trimester. Fetal heart sounds are absent, and a 'snowstorm' appearance is
classically seen on ultrasound.
Treatment - Tumor is removed by Suction curettage.
- Subsequent β-hCG monitoring is important to ensure adequate mole
removal and to screen for the development of choriocarcinoma.
Complication - Choriocarcinoma may arise as:
1. a complication of gestation: -Gestational trophoblastic disease [GTD].
--> Spontaneous abortion --> Hydatidiform mole
2. during normal term pregnancy -Gestational trophoblastic disease [GTD].
3. spontaneous germ cell tumor -Germ cell tumor.
- Choriocarcinoma that arise from germ cells do not respond well to
chemotherapy - Malignant tumors.
- Choriocarcinoma that arise from gestational pathway respond well to
chemotherapy - Benign tumors.

Classification Partial mole Complete mole


Genetic - Normal ovum (1N) fertilized by 2 normal - Empty ovum (0N) fertilized by 2 normal
Error sperm cell (1N + 1N) sperm cell (1N + 1N)
--> 69 [23(X)+23(X/Y)+23(X/Y)] triploid --> 46 [0+23(X/Y)+23(X/Y)] dipoid
chromosomes -XXX/XXY/XYY seen. chromosomes -XX/XY (YY not seen).
- Normal ovum (1N) fertilized by 1 sperm - Empty ovum (0N) fertilized by 1 sperm
cell with double chromosome (2N). cell with double chromosome (2N).
--> 69 [23(X)+46(XY)] triploid --> 46 [0+46(XY)] diploid
chromosomes -XXY seen. chromosomes -XY seen.
Fetal tissue Present Absent
Chorionic Villous Some villi are hydropic (edematous); and Most villi are hydropic
edema some are normal.
Trophoblastic - Focal proliferation present around - Diffuse, circumferential proliferation
proliferation hydropic villi around hydropic villi
- Cyto-trophoblast INVOLVED mainly. - Syncytio-trophoblast INVOLVED mainly.
Risk for Chorio- Minimal 2-3%
carcinoma
Uterine size Normal Increased ↑
hCG secretion Increased ↑ Increased ↑↑↑↑
Chapter 14: Male Genital System
Pathology

Lympho - Necrotizing granulomatous inflammation of the - Chlamydia trachomatis


granuloma inguinal lymphatics and lymph nodes -STD. (serotypes L1-L3)
venereum - Perianal involvement may result in rectal stricture
(coz it Eventually heals with fibrosis)
14.1 Penis
1. Describe the following penis conditions.
Pathologies Presentation Cause
Priaprism - Continuous penile erection, unrelated to sexual - Haematological
arousal. disorders
- Medical emergency as ischemia can lead to gangrene. - Neurological injury

Hypospadias - Opening of urethra on inferior surface of penis. - Due to failure of the


urethral folds to close.

Epispadias - Opening of urethra on superior surface of penis. - Due to abnormal


positioning of the
genital tubercle.
- Associated with bladder
exstrophy (bladder
located outside body
cavity).

Phimosis - Foreskin of the penis cannot be pulled back past the - The tip of the foreskin is
glans penis. too narrow
- The inner surface of the
foreskin is fused with
the glans.
- The frenulum is too
short to allow complete
retraction

Paraphimosis - Foreskin of a penis becomes trapped behind the glans - Usually caused by
penis. medical professionals or
parents who handle the
foreskin improperly

Condyloma - Benign warty growth on genital skin. - Due to HPV type 6 or 11


acuminatum - characterized by koilocytic change.
14.1 Penile cancer
Invasive Carcinoma of the Penis
Defn - Malignant proliferation of squamous cells of penile skin.

Risk factors - HPV type 16, 18, 31, 33 -(2/3rd of cases).


- Lack of circumcision - Foreskin acts as a nidus for inflammation
and irritation if not properly maintained.
Precursor A. Squamous hyperplasia.
lesions B. Low-grade penile intraepithelial neoplasia (PIN).
C. High-grade PIN (carcinoma in situ).
1. Bowen disease
- involves penile shaft (& scrotum).
- Presents as leukoplakia.
- seen in 5th decade of life.
2. Erythroplasia of Queyrat
- involves glans penis or inner prepuce.
- Presents as erythroplakia.
3. Bowenoid papulosis
- involves glans & shaft of penis.
- Presents as multiple reddish papules.
- Seen in younger patients (40s).
- Does not progress to invasive carcinoma.
14.2 Testicle

Cryptorchidism
Definition - Failure of testicle to descend into the scrotal sac [Testicles normally develop
in the abdomen and then "descend" into the scrotal sac as the fetus grows].
- Mostly unilateral
Epidemiology - MOST COMMON CONGENITAL MALE REPORDUCTIVE SYSTEM
ABNORMALITY
- Affects 1% of male infants.
Presentation - Histological exam: very prominent Leydig cells.
Treatment - Most cases resolve spontaneously.
- Orchiopexy is performed <2 years of age.
Complication - Testicular atrophy with infertility.
- Increased risk for seminoma.

Orchitis - Inflammation of the testicle


Cause Notes
External factor
- Chlamydia trachomatis - Seen in sexually active young adults.
(serotypes D-K) - Libido is not affected because Leydig
- Neisseria gonorrhoeae cells are spared
- ↑ risk of sterility.
- Escherichia coli - Seen in older adults.
- Pseudomonas - Caused by spread of UTI pathogen into
the reproductive tract
- Mumps virus - Seen in teenagers males (usually not
seen in children < 10 years old).
- ↑ risk of infertility.
Internal factor
Autoimmune orchitis - Characterized by granulomas
involving the seminiferous
tubules.

Testicular torsion
Definition - Twisting of spermatic cord ---> obstructs thin-walled spermatic vein --->
leading to congestion ---> hemorrhagic testicular infarction (in ~6 hrs).
Cause - Congenital failure of testes to attach to the inner lining of the scrotum
(via the processus vaginalis).
Epidem - Presents in adolescents.
Clinical - Sudden testicular pain.
features - absent cremasteric reflex.
Varicocele Hydrocele Spermatocele
Defn - Abnormal enlargement of the - Accumulation of serous - Head of the epididymis
pampiniform venous plexus in the fluid within the tunica distended with watery fluid
scrotum. vaginalis. that contains spermatozoa.
Cause - Usually left sided; left testicular vein - Incomplete closure of the - Originate as diverticulum
drains into the left renal vein, while the processus vaginalis from the epididymis --->
right testicular vein drains directly into leading to communication Sperm formation gradually
the IVC. with the peritoneal cavity causes the diverticulum to
I. Associated with left-sided renal cell --> (infants) increase in size --> cystic
carcinoma; RCC often invades the renal - Blockage of lymphatic dilation from epididymis
vein. drainage --> (adults). containing edema fluid -->
- Seen in a large percentage of infertile Spermatocele formed.
males.
Clinical - Presents as scrotal swelling with a - Presents as scrotal swelling - Presents as painless and
feature "bag of worms" appearance. that can be transilluminated. asymptomatic swelling.
14.3 Testicular Tumors

Cellular origin Arise from germ cells or sex cord-stromal cells.


Presentation - Firm, painless testicular mass that cannot be transilluminated.
Diagnosis - Usually not biopsied due to risk of seeding the scrotum
[tumor seeding -The spread of cancer cells to neighboring
tissues along the course of a biopsy needle].
Treatment - removed via radical orchiectomy.
Epidemiology - Most testicular tumors are malignant germ cell tumors.

Sex cord–Stromal tumour


- Tumors that originates from sex cord-stromal tissues of the testicle
- Usually benign tumors.
Leydig cell tumor - originates from & composed of Leydig cells.
- tumor cells produce androgens causing:
-> precocious puberty in children
-> gynecomastia in adults
- Histology: Reinke crystals -rod like cytoplasmic inclusions.
Sertoli cell tumor - originates from & composed of Sertoli cell.
- Closely packed solid or hollow Seminiferous tubules.
lined by well-differentiated cuboidal to columnar ep.
- Usually clinically silent

Cell/tissue type Tumor location -Testicle (male)


Sex cord - Sertoli cell tumour
Gonadal stroma - Leydig cell tumour

Mixed - Sertoli-Leydig cell tumour

Primary testicular lymphoma


- Most common cause of testicular mass in males >60 years
- testicular mass is often bilateral and malignant.
- Usually of diffuse large B-cell type.
Germ cell tumors
1. Describe the following features of germ cell tumors.
Epidem - Most common type of testicular tumor (>95% of cases).
- Mostly seen in b/w 15-40 year of age.
Risk factor - Cryptorchidism.
- Klinefelter syndrome.
Division Seminomatous tumors Non-seminomatous tumors Mixed germ cell tumors
55% of cases 45% of cases Common mixtures in
Excellent prognosis Poor prognosis Mixed germ cell tumor:
- Highly responsive to radiotherapy. - Variable response to treatment. – Teratoma, embryonal
- Metastasize late. - Metastasize early. carcinoma, and yolk
sac tumor.
Types: Types: – Seminoma with
- Classical or typical seminoma. - teratoma embryonal carcinoma.
- Spermatocytic seminoma. - embryonal carcinoma – Embryonal carcinoma
- endodermal sinus tumor with teratoma.
- choriocarcinoma

Mixed germ cell tumor


--> Germ cell tumors are usually mixed - ~ 60% of testicular tumors.
--> Prognosis is based on the most aggresive /worst component.
Tumor subtype based on tissue made by germ cells.
Germ cell -> Seminoma.
Placental tissue -> Choriocarcinoma.
Fetal tissue -> Teratoma & Embryonal carcinoma.
Yolk sac -> Endodermal sinus tumor

2. Describe some Tumor markers.


- AFP (Alpha-fetoprotein) is a major plasma protein produced by the yolk sac
and the liver during fetal development. It is thought to be the fetal form of
serum albumin.
- The function of AFP in adult humans is unknown.
- Plasma levels decrease rapidly after birth but begin decreasing prenatally
starting at the end of the first trimester.
- The normal serum range of AFP for for adults and childrens is under 5 ng/mL.
- hCG is a heterodimeric glycoprotein with - Human chorionic gonadotropin (hCG) and
an α subunit + β-subunit. Human placental lactogen (hPL) are produced by
1) α subunit of hCG - similar to the syncytiotrophoblasts of placenta.
α chains of FSH, LH, and TSH. 1) hCG - promotes corpus luteum survival.
2) free β-subunit hCG - unique to hCG. 2) hPL - stimulates mammary gland -> Lactation
2. Differentiate the following testicular tumors.
Seminoma Teratoma Embryonal Yolk sac (Endodermal Choriocarcinoma
carcinoma sinus) tumor
Most common Can occur at any Peak incidence Most common Incidence rates are
testicular tumor age from infancy occurs in persons testicular tumor in higher in white than
to adult life. aged 20-30 yrs. children nonwhite ♂.

Malignant Malignant tumor in Malignant tumor Malignant tumor Malignant tumor;


tumor ♂s & Benign tumor - spreads early spreads early via
in♀s. hematogenously. blood

- Large cells with clear - If Tumor is - Immature primitive - Tumor resembles yolk sac - Tumor resembles
cytoplasm & central composed of cells that may elements. placenta-like tissue,
nuclei (resemble Mature fetal produce glands. - Schiller-Duval bodies but villi are absent.
spermatogonia). tissue - Mature - Forms a hemorrhagic (glomerulus-like - Hyperthyroidism or
- Forms homogenous Teratoma. mass with necrosis. structures) are seen on gynecomastia may
mass with no - Chemotherapy may histology -> [perivascular also be seen.
- If Tumor is
hemorrhage or result in structure consists of a
composed of
necrois. differentiation into central vessel surrounded
immature fetal
another type of germ by tumor cells– the whole
- Resembles ovarian tissue - Immature
dysgerminoma. cell tumor structure being contained
Teratoma.
(e.g., teratoma). in a cystic space lined by
flattened tumor cells.

- Rare cases may - Increased AFP or - Increased AFP or - AFP is elevated. - β-hCG is elevated.
produce β-hCG. β-hCG is present. β-hCG is present. - α-hCG ↑ - maybe.

Tumor of Immature - Composed of - Tumor cells resemble - Derived from - Tumor histologically
precursor germ cells fetal tissue totipotent stem cells. Extraembryonal resembles the
undergoing Derived from 2 yolk sac. Syncytiotrophoblast
spermatogenesis or 3 embryonic and Cytotrophoblast.
[Spermatogonium --> layers (ectoderm,
Spermatid] mesoderm and
endoderm).
- Originates from
germ cells.

Tumor is sharply - Painless large - Tumor is Poorly Painless testicular - testicular tumor may
demarcated from testicular demarcated at masses, which are be small or
normal testis, which masses, the margins. typically bulky nonpalpable with /
may be compressed, - It has a solid - Tumor extends lesions. without pain radiating
and atrophic. heterogeneous into tunica - Yolk sac tumors pain to the groin and
– Usually, the tunica appearance, albuginea, (YSTs) of the testis abdomen
albuginea is not sometimes with epididymis or are located in the - Choriocarcinoma
penetrated. cystic areas. spermatic cord. testis parenchyma. occurs within the
testicular parenchyma
14.4 Prostate

1. Describe Anatomy & Histology of prostate gland.


A. Small, round organ that lies at the base of the bladder encircling the urethra.
B. Sits anterior to the rectum; posterior aspect of prostate is palpable by digital
rectal exam (ORE).
C. Consists of glands and stroma.
l. Glands are composed of:
○ inner layer of luminal cells.
○ outer layer of basal cells.
2. Glands secrete alkaline, milky fluid that is added to sperm and seminal
vesicle fluid to make semen.
3. Glands and stroma are maintained by androgens
[stromal cells make DHT by 5 alpha reductase from testosterone].

Acute bacterial prostatitis Chronic nonbacterial prostatitis


Defn - Acute inflammation of the prostate. - Chronic inflammation of prostate.

Causes • C.trachomatis & N.gonorrhoeae -young adults. • Neurological dysfunction.


• E coli & Pseudomonas -older adults. • Endocrine dysfunction.
• Immune dysfunction.

Clinical • Dysuria with fever and chills. • Dysuria with pelvic or low back pain.
features • Prostate is tender and boggy on digital rectal
exam [DRE].
Diagnosis - Prostatic secretion show: - Prostatic secretion show:
• WBC ↑ • WBC ↑
• +ve bacterial culture • -ve bacterial culture
Benign prostate hyperplasia
Defn - Hyperplasia of prostatic stroma and glands.
Epidem - Age-related change (present in most men by the age of 60 years).
- No increased risk for cancer.
Patho - Related to increased dihydrotestosterone (DHT)
physiology • DHT is made from testosterone by 5 alpha-reductase in stromal cells.
• DHT acts on androgen receptors of stromal and epithelial cells resulting in
hyperplastic nodules -->Occurs in the central periurethral zone of the prostate.
Clinical - Presentation based on compression of urethra by enlarged prostate:
features • Problems starting and stopping urine stream [dribbling urine].
• Impaired bladder emptying -> ↑ risk for infection and hydronephrosis.
• Nocturia/nocturnal polyuria.
• Hypertrophy of bladder wall smooth muscle -> ↑ risk for bladder diverticula.
• Microscopic hematuria maybe present.
Diagnosis - Prostate-specific antigen (PSA) is often slightly ↑ (usually < 10 ng/mL) in blood;
due to the increased number of glands.
- PSA is glycoprotein protease secreted by the epithelial cells of prostate gland;
it liquefies semen in the seminal coagulum (the sperm entrapping gel composed
of semenogelins and fibronectin -that encases ejaculated spermatozoa,
preventing capacitation in male RT) and allows sperm to swim freely in the
female RT.
Treatment - α1 antagonist (terazosin) to relax urethral smooth muscle.
• Also relaxes vascular smooth muscle lowering BP.
• α1A selective antagonist (tamsulosin) are used in normotensive
individuals to avoid a α1B effects on blood vessels.
- 5α-reductase inhibitor - reduce androgenic stimulation.
• Blocks conversion of testosterone to DHT.
• takes months to produce results.
• also useful for male pattern baldness.
• Side effects: gynecomastia & sexual dysfunction.
14.4 Prostate Cancer
Prostate adenocarcinoma
Defn - Malignant proliferation of prostate glands.
Epidem - Most common cancer in men.
- 2nd most common cause of cancer-related death.
Risk factors - Age
- Race (Blacks > Caucasians > Asians).
- Diet: high in saturated fats.
Clinical - Prostatic carcinoma is most often clinically silent.
features I. Usually arises in the peripheral posterior region of the prostate and hence,
does not produce urinary symptoms early on.
Screening - Screening begins at the age of 50 years with DRE and PSA.
• Normal PSA reference ranges for BPH & prostate cancer increases with age;
(2.5 ng/mL for ages 40-49 years vs 7.5 ng/mL for ages 70-79 years) ->BPH.
(163 ng/mL for ages < 50yrs vs 613 ng/mL for ages > 70yrs) ->prostate cancer.
• Serum PSA > 10 ng/mL is highly worrisome at any age.
• Decreased % of free-PSA is suggestive of cancer (cancer makes bound PSA);
PSA is bound to serum protein Alpha 1-antichymotrypsin.
Diagnosis - Prostatic biopsy is required to confirm the presence of carcinoma.
• Shows small invasive glands with large nuclei & prominent nucleoli.
• Cytoplasm ranges from pale-clear to amphophilic in appearance.
• Ducts are only 1 layer thick instead of having a basal & luminal epithelium.

Grading - Gleason grading system is based on the degree of glandular architectural


differentiation and the growth pattern of the tumor Only (and not on cellular/
nuclear atypia-abnormalities).
- Multiple regions of the tumor are assessed because architecture varies from
area to area.
i. A score of 1-5 is assigned for two distinct areas and then added to produce a
final score of 2-10.
ii. Higher score suggests worse prognosis.
Metastasis - Spread to lumbar vertebra or pelvic bone is common.
- Results in osteoblastic metastases -bone forming carcinomas.
• Presents as low back pain.
• ↑ serum Alkaline phosphatase (ALP).
↑ serum Prostate-specific antigen (PSA)
↑ serum Prostatic acid phosphatase (PAP).

Treatment - Prostatectomy is performed for localized disease -> when No Metastatis.


- Hormone suppression for advanced disease -(to reduce testosterone and DHT)
-> relieve symptoms of metastatis & reduces prostatic malignant tumor size.
• Continuous GnRH analogs (leuprolide) shut down the anterior pituitary
gonadotrophs (LH and FSH are reduced).
• Flutamide acts as a competitive inhibitor at the androgen receptor.
Chapter 15: Endocrine Pathology
Pituitary Gland

Hypothalamic hormones Anterior pituitary Hormones


1. Thyrotropin releasing hormone (TRH) 1. Thyrotropin (aka thyroid stimulating hormone)
2. Gonadotropin releasing hormone (GNRH) 2. Gonadotropin (FSH and LH)
3. Corticotropin releasing hormone (ACRH) 3. Adrenocorticotropin (ACTH)
4. Growth hormone releasing hormone 4. Growth hormone
5. Somatostatin 5. Prolactin (lactotropin)
6. Prolactin inhibiting hormone (dopamine)

Posterior Pituitary hormones


1. ADH (aka vasopressin)
2. Oxytocin
Anterior pituitary
Pituitary adenoma

1. What type of tumor is pituitary adenoma?


- Benign tumor of anterior pituitary cells.
- Can be functional (hormone producing) or non-functional (silent)

2. What are clinical presentation of non-functional tumor ?


- Bitemporal heminopsia - occurs due to compression (mass effect) of the optic chiasm.
- Hypopituitarism - occurs due to compression (mass effect) of normal pituitary tissue.
- Headache.

3. Mention some common functional tumor ?


- Prolactinoma: 50% of adenomas
- Growth hormone cell adenoma: 10-15% of adenomas
- ACTH cell adenoma: 10% of adenoma [leading to Cushing syndrome]
- TSH cell adenoma: rare
- LH producing adenoma: rare
- FSH producing adenoma: rare

4. Explain Prolactinoma.
○ Clinical presentation [prolactin inhibits GNRH synthesis and release] :
- In females - galactorrhea and amenorrhea.
- In males - decreased libido and headache.
○ Treatment:
- Dopamine agonists (e.g., bromocriptine or cabergoline) to suppress prolactin production
(shrinks tumor).
- Surgery for larger lesions.

4. Explain Growth hormone cell adenoma.


○ Clinical presentation:
- In children - gigantism
=> increased linear bone growth (epiphyses are not fused).
- In adults - acromegaly
=> Enlarged bones of hands, feet & jaw.
=> Growth of visceral organs leading to dysfunction (ex- cardiac failure).
=> Enlarged tongue.
- Secondary diabetes mellitus [GH mediated gluconeogenesis by liver and inhibited glucose
uptake by cells].
○ Diagnosis:
- Elevated GH level alongwith lack of GH suppression by oral glucose.
- Elevated Insulin growth factor- I (IGF-1) levels [IGF is made by liver in response to GH].
○ Treatment:
- Octreotide (somatostatin analog) - suppresses GH release
- GH receptor antagonists
- Surgery
Hypopituitary

8. What is hypopituitarism ?
- Insufficient production of hormones by the anterior pituitary gland.
- Symptoms arise when > 75% of the pituitary parenchyma is lost.

9. What are the causes of hypopituitarism?


- Pituitary adenoma (in adults) - mass effect or apoplexy (intratumoral beeding).
- Craniopharyngioma (in kids) - mass effect or apoplexy (intratumoral beeding).
- Sheehan syndrome:
○ pregnancy related infarction of the pituitary gland
○ Gland doubles in size during pregnancy (due to excessive hormone secretion) but blood supply
does not increase significantly
○ Blood loss during parturition precipitates infarction.
○ Presents as:
=> poor lactation
=> loss of pubic hair
=> fatigue
- Empty sella syndrome:
○ May occur due to congenital defect of the sella.
○ Herniation of the arachnoid and CSF into the sella compresses and destroys the pituitary gland.
○ Pituitary gland is "absent" (empty sella) on imaging.
Posterior pituitary
Central diabetes insipidus
10. Explain Central diabetic insipidus.
- ADH deficiency
- Presentation (based on loss of free water) - due to ADH deficiency.
○ Polyuria and polydipsia (excessive thirst) - risk of life-threatening dehydration.
○ Hypernatremia and high serum osmolality
○ Low urine osmolality and specific gravity
- Causes - pathology (tumor, trauma, infection or inflammation) of hypothalamus or post. pituitary.
- Diagnosis - Water deprivation test fails to increase urine osmolality.
- Treatment - Desmopressin (ADH analog).

Nephrogenic diabetic insipidus


11. Explain Nephrogenic diabetic insipidus.
- Impaired renal response to ADH
- Presentation - same as central DI but no response to desmopressin.
- Causes:
○ Acquired: amyloidosis, PKD, drugs (e.g., lithium and demeclocycline).
○ Hereditary: V2 receptor gene mutation / aquaporin 2 gene mutation.

Syndrome of inappropriate ADH (SIADH) secretion


12. Explain SIADH.
- Excessive ADH secretion
- Presentation
○ Hyponatremia and low serum osmolality
○ Mental status change and seizure [Hyponatremia ---> neuronal swelling and cerebral edema].
- Causes
○ Ectopic production (ex - small cell carcinoma of lung)
○ Pulmonary infection
○ CNS trauma
○ Drugs (cyclophosphamide)
- Treatment
○ Free water restriction.
○ Demeclocycline [inhibits AC activation after ADH binds to V2 receptor].
Thyroid

1. Differentiate b/w T3 and T4


T4 (thyroxine) T3 (triiodothyronine)
Both are made from tyrosine Most of it made from T4; 10x more potent
Thyroid secretes T4>>T3 T4--> T3 conversion reduced in severe illness
T1/2 = 7 days T1/2 = 7 days
Deiodinases:
- Iodotyrosine deiodinases: releases iodine from MIT & DIT.
- Iodothyronine deiodinase
- Type 1 & 2 deiodinases: peripheral deiodination of T4 to T3 (active form).
- Type 3 deiodinases: peripheral deiodination of T4 to rT3 (inactive form).

2. How are thyroid hormones transported in blood?


- Tyrosine binding protein (carries 70%)
- Albumin (carries 20%)
- Transthyretin (pre-albumin) (carries 10%)
- Free T4 is 0.04% and free T3 is 0.03%

3. How are T3 and T4 made in thyroid gland? What is the key enzyme?
- Key enzyme is thyroid peroxidase [mediates oxidation, organification, and coupling].
- Thyroglobulin are long chain of tyrosine that's stored as colloid in thyroid gland.
- Steps
○ Iodine is oxidized to oxidized iodine by thyroid peroxidase (oxidation step)
○ Oxidized iodine reacts with tyrosine to make monoiodotyrosine
○ Oxidized iodine reacts with monoiodotyrosine to make diiodotyrosine
○ The last two steps are called organification
○ Monoiodotyrosine + diiodotyrosine = triiodothyronine
○ Diiodotyrosine + diiodotyrosine = thyroxine
○ The last two steps are called coupling

4. What are functions of thyroid hormones?


a. Increase or decrease gene transcription by binding to nuclear receptor
b. Imp in development (Cretinism is mental retardation and dwarfism caused due to
impairment of brain and skeletal development due to hypothyroidism)
c. Increase BMR, O2 consumption and free radical formation
d. Exacerbate diabetes mellitus (increase glycogenolysis and increase hepatic gluconeogeneis)
(hyperthyroid = DM)
e. Increase LDL receptors (hyperthyroidism = low serum LDL)
f. Increase ionotropy (contractility), chronotropy (HR),
g. Increase sensitivity to catecholamine for B1 receptors(hyperthyroidism = hyperactive SANS)
h. Pulmonary - maintains hypoxic and hypocapnic drive (hypothyroid = hypoventilation
and respiratory arrest)
i. GI - increase gastric motility (hypothyroid= constipation, hyperthyroid = diarrhea)
j. Skeletal - increase bone reasborption and decrease bone formation (hyperthyroid =
osteopenia)
k. Neuromuscular - (hyperthyroid = increased reflex, anxiety, hyperactivity; hypothyroid =
decreased reflexes, fatigue and sluggish)
Thyroglossal duct cyst

5. What is Thyroglossal duct cyst ?


- Cystic dilation of thyroglossal duct remnant.
- Presents as an anterior neck mass.
- Thyroid develops at the base of tongue and then travels along the thyroglossal duct to the
anterior neck. Thyroglossal duct normally involutes; a persistent duct, however, may
undergocystic dilation.

Lingual thyroid

6. What is lingual thyroid?


- Persistence of thyroid tissue at the base of tongue.
- Presents as a base of tongue mass.

Hyperthyroidism (thyrotoxicosis)
7. What is the Basic principal of hyperthyroidism ?
- ↑ level of circulating thyroid hormone
- ↑ basal metabolic rate (due to increased synthesis of Na+-K+ ATPase)
- ↑ sympathetic nervous system activity (due to increased expression of β1-adrenergic R).

7. What are the clinical features of hyperthyroidism ?


- Weight loss despite increased appetite
- Heat intolerance and sweating
- Tachycardia with increased cardiac output
- Arrhythmia (e.g., atrial fibrillation), especially in the elderly
- Tremor, anxiety, insomnia, and heightened emotions
- Staring gaze with lid lag
- Diarrhea with malabsorption
- Oligomenorrhea
- Bone resorption with hypercalcemia (risk for osteoporosis)
- Decreased muscle mass with weakness
- Hypocholesterolemia
- Hyperglycemia (due to gluconeogenesis and glycogenolysis)

Multinodular goiter
14. What is multinodular goiter ?
- Enlarged thyroid gland with multiple nodules.
- Caused due to relative iodine deficiency
- Usually nontoxic (euthyroid).
- Rarely, regions become TSH-independent leading to T4 release and hyperthyroidism
('toxic goiter').

Fig - multinodular goiter


Graves disease

8. What is the cause of Grave's disease ?


- Autoantibody (IgG) that stimulates TSH receptor [type II hypersensitivity]
- Leads to increased synthesis and release of thyroid hormone

8. What is epidemiology of Grave's disease ?


- Most common cause of hyperthyroidism.
- Classically occurs in women of childbearing age (20-40 years).

9. What are the clinical presentation of grave's disease?


- Hyperthyroidism
- Diffuse goiter- Constant TSH stimulation leads to thyroid hyperplasia and hypertrophy
- Exophthalmos and pretibial myxedema - Fibroblasts behind the orbit and overlying the shin
express the TSH receptor. TSH activation results in:
○ glycosaminoglycan (chondroitin sulfate and hyaluronic acid) buildup.
○ inflammation, fibrosis, and edema.

11. What is the histology of Grave's disease ?


- Irregular follicle w/ scalloped colloid
- Chronic inflammation seen.

Fig - Thyroid in grave's. Notice irregular follicles. Also, the white space between colloid and thyroid
tissue is called scalloped and is classic occurrence in Grave's.

12.5. What are lab findings in Grave's?


- ↑ total and free T4.
- ↓ serum TSH (free T3 downregulates TRH receptors in the ant. pituitary to ↓ TSH release).
- Hypocholesterolemia
- Hyperglycemia

12. What is treatment of Grave's?


- β-blockers
- Propylthiouracil - inhibits thyroperoxidase and peripheral deiodinases (type 2).
- Radioiodine ablation (I-131) --> emits β particles --> follicles undergo pyknosis --> Fibrosis.
- Steroids
- Total thyroidectomy
13. Differentiate thyroid storm vs myxedema coma

Thyroid storm (hyperthyroid emergency) Myxedema coma


(hypothyroid emergency)
Presentation: Presentation:
- fever (>400C), sweating - altered mental status (confusion
- nausea, vomiting ---> coma)
- tachycardia, arrhythmia - Hypoglycemia
- delirium - Hypothermia
- hyperthermia - Hypoventilism
- hypovolemic shock.
Cause: Cause
- Trigger Stress (surgery, childbirth, MI) ----> - Stress (infection, MI stroke)
elevated catecholamines level & action.
Treatment Treatment
- Very high dose of Iodine salt (Wolff–Chaikoff effect) - - high dose of levothyroxine
increased iodine in blood leads to:
a] Inhibition of TPO enzyme ---> reduced synthesis of
thyroid hormone.
b] Decreased release of thyroid hormones from follicular
lumen into bloodstream.
- It is an autoregulatory phenomenon followed by an
escape phenomenon - down-regulation of Na+/I-
symporter (NIS) ---> decreased iodine uptake by thyroid
follicles --->normal thyroid functions.
Hypothyroidism

Cretinism

15. What is cretinism ?


- Hypothyroidism in neonates and infants (congenital) ---> impaired development.
- Classic findings
- Mental retardation (poor CNS developement)
- Dwarfism (poor skeletal developement)
- Coarse facial features
- Umbilical hernia
- Enlarged tongue (due to myxedema)

16. What are the causes of cretinism ?


- Maternal hypothyroidism during early pregnancy.
- Thyroid agenesis.
- Iodine deficiency
- Dyshormonogenetic goiter - inborn error of thyroid hormone production where one
or multiple steps maybe defective, most commonly involves thyroid peroxidase.

Myxedema

17. What is myxedema ?


- Hypothyroidism in older children or adults.
- Results in:
=> ↓ Basal Metabolic Rate.
=> ↓ sympathetic nervous system activity.

17. What are the classical findings ?


○ Myxedema - accumulation of glycosaminoglycans in the skin and soft tissue results
in a deepening of voice (larynx) and large tongue due to increased TSH level [lack
of negative feedback].
○ Weight gain despite normal appetite
○ Slowing of mental activity
○ Muscle weakness
○ Cold intolerance with decreased sweating
○ Bradycardia with decreased cardiac output
○ Hypoventilation and respiratory arrest
○ Oligomenorrhea
○ Hypoglycemia & Hypercholesterolemia
○ Constipation

18. What are the causes ?


- Hashimoto thyroiditis
- Iodine deficiency
- Drugs (lithium)
- Radioiodine ablation (I-131)
- Total thyroidectomy
Thyroiditis

Hashimoto thyroiditis

20. What is Hashimoto thyroditis ?


- Autoimmune destruction of the thyroid gland (follicular cell destruction).
- Autantibodies often present are:
○ Antithyroglobulin Ab
○ Antithyroid peroxidase Ab (Anti-microsomal Ab)
○ Anti Na+/ I− symporter Ab
- Associated with CTLA-4, PTPN-22 & HLA-DR5 gene polymorphism in
= CTLA-4 gene in T reg cell.
= PTPN-22 gene in T & B cells.
= HLA-DR5 gene in AP cell.
- Most common cause of hypothyroidism in regions where iodine levels are adequate.
- It has increased risk for developing other autoimmune disease like:
○ type 1 DM
○ pernicious anaemia
○ myasthenia gravis
○ lupus

19. What is the presentation ?


- Initially ↑ T4 level (due to epithelial damage)
- Later ↓ T4 & ↑ TSH (due to gland destruction)

20. What is histologic appearance of Hashimoto's thyroditis? (HY)


- Chronic inflammation with germinal cells ---> Increased risk for B-cell (marginal zone)
lymphoma ----> presenting as an enlarged thyroid gland late in disease course.
- Hurthle cells (eosinophilic metaplasia of cells that line follicles).

Fig - Hashimoto's thyroiditis. CI is chronic inflammation. GC is germinal center.


Little circles on left are herthel cells.
Subacute Granulomatous (De Quervain) Thyroiditis

22. Describe SUBACUTE GRANULOMATOUS (DE QUERVAIN) THYROIDITIS ?


- Granulomatous thyroiditis that follows a viral infection.
- Presents as a tender painful thyroid with transient hyperthyroidism
- Self-limited; rarely (15% of cases) may progress to hypothyroidism

Reidel Fibrosing thyroiditis


23. Describe Reidel Fibrosing thyroiditis ?
- Chronic inflammation of thyroid with extensive fibrosis.
- Presentation:
○ Hypothyroidism
○ 'Hard as wood' non tender thyroid gland.
○ Fibrosis may extend to involve local structures (ex - airway).
○ Clinically mimics anaplastic carcinoma [but in Reidel Fibrosing thyroiditis,
pts are usually younger (40s) females and malignant cells are absent]
Thyroid Neoplasia

24. Describe thyroid Neoplasia ?


- Usually presents as a distinct, solitary & benign nodule.
- I-131 radioactive uptake studies are useful to further characterize nodules:
○ ↑ uptake ('hot' nodule) is seen in Graves disease or nodular goiter
○ ↓ uptake ('cold' nodule) is seen in adenoma and carcinoma
- Biopsy is performed by fine needle aspiration (FNA).

27. What are the types of thyroid Neoplasia ?


a. Follicular adenoma
a. Follicular carcinoma
b. Papillary carcinoma
c. Medullary carcinoma
d. Anaplastic carcinoma

Follicular adenoma

26. Describe follicular adenoma.


- Benign proliferation of follicles surrounded by a fibrous capsule
- Mostly nonfunctional (less commonly may secrete thyroid hormone).

Follicular carcinoma

29. Describe Follicular carcinoma.


- Malignant proliferation of follicles surrounded by a fibrous capsule with invasion
through the capsule
- Diagnosis:
○ Entire capsule must be examined microscopically (to detect invasion).
○ FNA only examines cells and not the capsule (hence useless)
- Metastasis generally occurs hematogenously.

Papillary carcinoma

28. Describe Papillary carcinoma.


- Most common type of thyroid carcinoma (80% of cases).
- Often spreads to cervical lymph nodes.
- Major risk factor - exposure to ionizing radiation in childhood.
- Prognosis - excellent (10-year survival is > 95%).
- Histology:
○ Comprised of papillae lined by cells with 'Orphan Annie eye' nuclei
(cleared-out appearance) and Coffee bean nucleus (nuclear grooves).
○ Papillae are often associated with psammoma bodies (concentric
dystrophic calcification of papillae tips).
Medullary carcinoma

31. Describe medullary thyroid carcinoma (MTC)


- Malignant proliferation of parafollicular C cells [neuroendocrine cells
that secrete calcitonin].
- Comprises 5% of thyroid carcinomas.
- Diagnosis and Biopsy:
○ High levels of calcitonin produced by tumor may lead to hypocalcemia.
○ Calcitonin often deposits within the tumor as localized amyloid.
○ Biopsy reveals sheets of malignant cells in an amyloid stroma.

MEN 2A MEN 2B
Medullary carcinoma Medullary carcinoma
Pheochromocytoma Pheochromocytoma
Parathyroid adenomas Ganglioneuromas of the oral mucosa
"Marfanoid" body built

Cause: - AD (familial) disorder -----> activating point mutation of RET protooncogene


-----> overactive mutant tyrosine kinase protein.
Prophylaxis: Detection of the RET mutation warrants thyroidectomy.

Anaplastic carcinoma

33. Describe anaplastic carcinoma


- Undifferentiated carcinoma of thyroid gland.
- Classically seen in old people
- Tumor has +ve keratin stain
- Often invades local structures leading to dysphagia or respiratory compromise.
- Worst prognosis of all thyroid carcinomas
Fig - follicular adenoma. Red line is the capsule that divides adenoma (bottom half) from
normal thyroid (top half)

Fig - Papillary carcinoma. red circles indicate orphan eye annie nucleus. Turquoise circle
shows coffee bean nucleus.

Fig - Psammoma bodies in papillary carcinoma shown in black circle.


Fig - follicular carcinoma. Red line shows the capsule and the break in it.

Fig - MTC biopsy. All the pink stuff seen is calcitonin amyloid.

Fig - highly malignant cells in anaplastic carcinoma that don't resemble anything
seen in thyroid usually
Parathyroid

1. What is vitamin D? How do we get it?


- Two most imp vitamin D are vit D3 (cholecalciferol) and vit D2 (ergocalciferol)
- Cholecalciferol (D3) is made from 7-dehydrocholesterol by UV in skin
- Ergocalciferol (D2) is taken from food.
- Both of those are activated by liver in unregulated way, and kidney in regulated way

2. How does kidney activate vitamin D?


- Vit D itself is a prohormone. Kidney uses alpha 1 hydroxylase to make 1,25 OH D
(active vitamin D).

5. What are functions of Vit D?


- Main function is to maintain bone mineralization
- Increase Ca and PO4 reabsorption in kidney
- Increase Ca and PO4 absorption in gut
- Decrease PTH secretion

3. What are functions of PTH hormone?


- Chief cells make PTH that increase free (ionized) calcium
- PTH acts on 3 main tissue
○ Stimulate kidney to increase vit D activation
○ Increase Ca absorption from kidney and phosphate excretion (phosphate excretion key
because it increases free Ca in blood)
○ Increase Ca and PO4 absorption from gut - this action is via Vit D
○ Increase osteoclast activity (PTH activates osteoblast which secretes M-CSF
(macrophage colony stimulating factor) which increase osteoclast differentiation and
activation).It also stimulates Osteoblast activity of bone mineralization.

4. What regulates blood PTH hormone?


- PTH release is highly sensitive to serum ca.
- Vitamin D also reduces PTH release
- Increase Ca ---> Gq and Gi activation. Gq increase calcium release in parathyroid cells --->
Decreases PTH synthesis. Gi decreases cAMP which reduces PTH synthesis
- Low Ca ---> Gs activation. Gs increases cAMP which increases PTH synthesis

6. What are function of calcitonin ?


- Calcitonin: tone down serum Ca and PO4 levels.

6. What are function of calcitonin and phosphatonin?


- Phosphatonin [FGF23 secreted by osteocytes in response to elevated calcitriol]:
Regulate intestinal & renal absorption of phosphate ----> Maintains serum
Phosphate levels.
- Inhibits 1 alpha-hydroxylase enzyme in PCT.
- Decreases the expression of NPT2 Channel in PCT.
Primary hyperparathyroidism

7. Explain primary hyperparathyroidism.


- Excess PTH due to a disorder of the parathyroid gland.
- Most common cause: PT adenoma (80%) - benign neoplasm, usually involving one gland.
- Other causes: sporadic PT hyperplasia and PT carcinoma.

8. Describe presentation of PT adenoma ?


○ Asymptomatic hypercalcemia
○ Nephrolithiasis (calcium oxalate stones)
○ Nephrocalcinosis - metastatic calcification of renal tubules potentially leading to
renal insufficiency and polyuria.
○ CNS disturbances (e.g., depression and seizures)
○ Constipation, peptic ulcer disease, and acute pancreatitis
○ Osteitis fibrosa cystica - resorption of bone leading to fibrosis and cystic spaces

9. What are the lab findings ?


- ↑ serum PTH
- ↑ serum Ca
- ↓ serum phosphate
- ↑ urine cAMP (PTH stimulate Gs protein in PCT).
- ↑ serum alkaline phosphatase [PTH activate Osteoblast ----> ALP secreted by
Osteoblast helps in Bone mineralization].

7. What is the treatment ?


- Surgical removal of the affected gland.

Secondary hyperparathyroidism

7. Explain secondary hyperparathyroidism.


- Excess production of PTH due to a disease process extrinsic to the parathyroid gland.
- Most common cause - chronic renal failure [Renal insufficiency ---> ↓ phosphate excretion --->
↑ serum Po4 binds free Ca in blood ----> ↓ serum free Ca ----> lack of negative feedback on PT
gland ----> ↑ serum PTH ----> increased osteoclastic activity ----> Excess Bone resorption ---->
Renal osteodystrophy].
- Lab findings: ↑ serum alkaline phosphatase [PTH activate Osteoblast ----> ALP secreted by
Osteoblast helps in Bone mineralization].
Hypo-parathyroidism

12. What is Hypo-parathyroidism ?


- Low PTH level ----> low serum Ca2+
- Causes:
○ autoimmune damage to the parathyroids.
○ surgical excision
○ DiGeorge syndrome (failure to develop 3rd and 4th pharyngeal pouch
hence Parathyroid glands does not develop)

13. What are the presentation ?


- Numbness and tingling (particularly circumoral).
- Muscle spasms [may be elicited with filling of a blood pressure cuff (Trousseau sign)
or tapping on the facial nerve (Chvostek sign)].

Pseudo-hypoparathyroidism

14. What is pseudohypoparathyroidism?


- End organ resistance to PTH.
- Lab findings:
○ high PTH and low Ca2+ level.
○ normal serum ALP.

14. What is the cause of pseudohypoparathyroidism ?


- Autosomal dominant disorder (Type 1a) ----> mutant Gs alpha subunit.
- It is associated with skeletal defects (dystrophy) like:
○ short stature
○ short 4th and 5th digits
Endocrine Pancreas
3. What is the function of Insulin ?
- Major anabolic hormone
- Upregulates insulin-dependent glucose transporter protein (GLUT4) on skeletal muscle and
adipose tissue [glucose uptake by GLUT4 decreases serum glucose].
- Increased glucose uptake by tissues leads to increased glycogen synthesis, protein synthesis
and lipogenesis.

3. What is the function of Glucagon ?


- Major catabolic hormone
- Increases blood glucose levels via glycogenolysis, lipolysis and gluconeogenesis from AA.
Type 1 Diabetes Mellitus

2. Explain Type 1 DM.


- Insulin deficiency leading to a metabolic disorder characterized by hyperglycemia.

3. What is the cause of type 1 DM ?


- Autoimmune destruction of Beta cells.
- Associated with HLA-DR3, HLA-DR4, HLA-DQ8, HLA-DQ2 gene polymorphism in T reg cell.
- Characterized by inflammation of islets.
- AutoAb against insulin can be seen (sign of damage) years before disease develops.

3. What is the presentation of type 1 DM ?


- Presentation (of insulin deficiency) - classically seen in children
○ High blood glucose
○ Weight loss despite polyphagia (high hunger)
○ Low muscle mass - due to unopposed catabolic action of glucagon.
○ Glycosuria - Hyperglycemia exceeds renal ability to resorb glucose.
○ Polyuria & polydipsia - excess filtered glucose leads to osmotic diuresis.

4. What is the treatment of type 1 DM ?


- Treatment involves lifelong insulin.

4. What is the complication of type 1 DM ?


- Risk for diabetic ketoacidosis (DKA).

Diabetic ketoacidosis (DKA)


4. What is the characteristic of DKA ?
- Characterized by excessive serum ketones.
- Ketone bodies
=> beta-hydroxybutyric acid
=> acetoacetic acid
=> Acetone

4. Explain the cause of DKA ?


- Triggered by stress (ex-infection, trauma, drugs) ----> Epinephrine is released ----> stimulates glucagon secretion ---->
causes lipolysis in adipocyte ----> Free fatty acids generated migrate to liver ----> gets converted to ketone bodies by
beta oxidation ----> ketone bodies have low pKa enters blood ---> dissociates to release H+ (acidosis - low pH) ----> H+
acts on peripheral chemoreceptors ----> activates respiratory centre -----> Hyperventilation [Kussmaul respiration] ----->
washes out excess CO2 (CO2 + H2O ⇄ H2CO3 ⇄ HCO3- + H+).

5. What is presentation of DKA ?


- Hyperglycemia (>300 mg/dl) - due to unopposed glucagon action.
- Anion gap metabolic acidosis (due to keto-acidosis).
- Hyperkalemia [Insulin stimulates Na/K pump synthesis ----> redistribution of serum K+ into cells].
High serum K+ level ----> lots of K+ is lost in urine ----> total body K+ decreases.
- Kussmaul respiration (to compensate for acidosis)
- Dehydration [i) hyperglycemia ----> osmotic diuresis ii) Vomiting]
- Nausea, vomiting (Ketone bodies stimulate CT Zone of medulla).
- Mental status change.
- Fruity smelling breath (acetone diffuses from blood to alveoli to get expired).

6. How is the treatment of DKA?


- Give fluids (corrects dehydration from polyuria).
- Insulin therapy (----> K+ will go inside the cells----> abrupt Hypokalemia).
- Replacement of electrolytes (Intravenous K+ administration).
Type 2 Diabetes Mellitus
4. What is Type 2 DM ?
- End-organ insulin resistance (Target tissue fail to respond to insulin due to reduced sensitivity)
leading to a metabolic disorder characterized by hyperglycemia.
- Insulin levels are increased early in disease, but later, insulin deficiency develops due to beta
cell exhaustion. Histology reveals amyloid deposition in the islets.

4. What is the epidemology of Type 2 DM ?


- Most common type of diabetes (90% of cases).
- Affects 5- 10% of the US population; Incidence is rising.
- Arises in middle-aged and obese adults (---> decreased numbers of insulin receptors /
attenuation of insulin receptor signalling).
- Strong genetic predisposition exists (reduced tk phosphorylation of insulin receptor).

Fig - amyloid deposition in Islets in type 2 DM. Amyloid is made of


amylin - a protein produced with insulin.

9. What are the presentation (often clinically silent) of type 2 DM ?


- Polyuria
- Polydipsia
- Hyperglycemia

9. What are the diagnosis of type 2 DM ?


○ Random glucose (>200 mg/dl)
○ Fasting glucose (>126 mg/dl)
○ Glucose tolerance test (>200/dl two hours after glucose loading)

10. How do you treat type 2 DM?


- Diet and exercise (Weight loss) intially.
- May require drug therapy to counter insulin resistance (e.g., sulfonylureas or metformin).
- Exogenous insulin therapy after exhaustion of beta cells.

9. What is the complication of type 2 DM ?


○ Risk for hyperosmolar non-ketotic coma

Hyperosmolar non-ketotic coma


11. Describe hyperosmolar non-ketotic coma.
- High glucose levels (>500 mg/dl) leads to:
○ life threatening diuresis ---> hypotension
○ coma (due to osmolar effect in brain)
- Ketones are absent due to small amounts of circulating insulin (---> prevents lipolysis).
LONG-TERM CONSEQUENCES OF DIABETES

Nonenzymatic glycosylation (NEG) of vascular basement membranes

12. Describe NEG of vascular basement membranes.


○ NEG of large- and medium-sized vessels leads to atherosclerosis and its resultant
complications like:
=> Cardiovascular disease - leading cause of death among diabetics.
=> Peripheral vascular disease - leading cause of non traumatic amputations.
○ NEG of small vessels (arterioles) leads to hyaline arteriolosclerosis and its resultant
complications like:
=> Involvement of renal arterioles leads to glomerulosclerosis, resulting in
small scarred kidneys with a granular surface.
=> Preferential involvement of efferent arterioles leads to glomerular hyperfiltration
injury with microalbuminuria that eventually progresses to nephrotic syndrome;
characterised by Kimmelstiel-Wilson nodules in glomeruli. Involvement of renal artery
blocks renal perfusion ultimately leading to Chronic renal failure.
○ NEG of Hb produces glycated hemoglobin (HbA1c), a marker of glycemic control.

Osmotic damage

12. Describe Osmotic damage.


○ Some cells in the body can uptake sugars freely without insulin.
○ Aldose reductase converts glucose to sorbitol, resulting in osmotic damage.
 Schwann cells ---> peripheral neuropathy.
 Pericytes of retinal blood vessel ---> anurysm of retinal blood vessel which rupture
resulting in blindness (most common in developed world).
 Lens ---> cataract.
Pancreatic Endocrine neoplasms

13. What is pancreatic endocrine neoplasms ?


- Tumors of islet cells.
- Account for < 5% of pancreatic neoplasms.

13. Describe pancreatic endocrine neoplasms ?


- MEN 1
○ Components:
 Pancreatic Endocrine neoplasms
 Parathyroid hyperplasia
 Pituitary adenomas
- Insulinoma [β cell tumor]
○ Tumor makes excess insulin ---> presents as episodic hypoglycemia
○ Presentation - Mental status change that is relieved by glucose administration.
○ Diagnosis:
 Low serum glucose level (usually< 50 mg/dL).
 High insulin level
 High C peptide level (C peptide is made along with insulin).
- Gastrinoma [G cell tumor] ----> Zollinger Ellison syndrome
○ Gastrin induces parietal cells in stomach to make acid
○ Presentation:
 Treatment resistant peptic ulcers
 Ulcers may be multiple and can extend into the jejunum.
- Somatostatinoma [Delta cell tumor]
○ Somatostatin inhibits gastrin & cholecystokinin.
○ Presentation:
 Achlorhydria (low acid production in stomach)
 Cholelithiasis w/ steatorrhea (Lack of g.bladdder contraction ---> reduced bile release)
- VIPoma [non-β islet cell tumor]
○ Vasoactive intensinal peptide greatly stimulates secretion of water and electrolytes in
intestine and reduces gastric acid production.
○ Presentation
 Watery diarrhea
 Hypokalemia
 Achlorhydria
Adrenal Cortex

1. What are the 3 layers of adrenal cortex ?


Outer to inner:
Zona glomerulosa Mineralocorticoids (ex- Aldosterone)
Zona fasciculata Glucocorticoids (ex- Cortisol)
Zona reticularis Sex steroids [precursor Androgens (but no testosterone)]
Zr cells lack 17-Beta Hydroxysteroid dehydrogenase.

Cortex - hormones are made from cholesterol (cortex is yellow due to cholesterol)
Medulla- hormones made from tyrosine

2. Describe metabolism of cortisol.


 Transport
o 75% transported by transcortin (aka corticosteroid binding globulin - made by liver).
o 15% bound to albumin
o 10% in free form
 Receptor - nuclear receptor. Present in all body cell
 Can bind strongly to mineralocorticoid receptor.

3. How do you get hypokalemia from too much licorice ingestion?


 Cortisol is inactivated to cortisone in kidney tubules by 11-B-hydroxysteroid dehydrogenase
type 2. Licorice inactivates the type 2 enzyme, leading to active cortisol in kidney tubules.
 As cortisol has high mineralocorticoid action, it leads to potassium excretion in kidney
tubules leading to hypokalemia.

4. What are the functions of glucocorticoids ?


 Stimulation of gluconeogenesis in Liver with decreased glucose intake by muscle,
adipose tissues - increase serum glucose.
 Decreased insulin sensitivity in muscle cells - insulin resistance.
 Decreased immune activity via
o Decreased arachidonic acid production (due to inhibition of phospholipase A2)
o Low IL-2 production (IL2 important for proliferation of T cells)
o Low histamine production
• Increased α1 receptor (maitains vascular tone) production and sensitivity to catecholamines.
• Negative feedback to ACTH.
Hypercortisolism (Cushing syndrome) - excess cortisol

6. What are clinical features of Cushing’s syndrome ?


- Muscle weakness (cortisol mediated breakdown of AA in muscles for gluconeogenesis)
with thin extremities.
- Moon facies, buffalo hump & truncal obesity (high glucose in blood -----> insulin
release ----> insulin is anabolic hormone and leads to fat storage).
- Abdominal striae [cortisol prevents fibroblast function ----> impaired collagen and
elastin synthesis ---> tearing of dermis on stretching ----> scar formation].
- HTN (increased sensitivity and production of α 1 receptor)
- Osteoporosis [cortisol inhibits osteoblast and stimulates osteoclast activity].
- Immune suppression.

6. What are the diagnosis of Cushing’s syndrome ?


- Increased 24-hour urine cortisol levels.

7. What are the causes of Hypercortisolism ?


Cause Result ACTH level
Exogenously administered corticosteroid bilateral adrenal atrophy low
Primary adrenal adenoma (----> 1° hypercortisolism) atrophy of the uninvolved
low
adrenal gland
Cushing's disease:
ACTH secreting Pituitary adenoma (----> 2° hypercortisolism)
bilateral adrenal hyperplasia high
CRH secreting hypothalmic tumor (---> 3° hypercortisolism)
Paraneoplastic ACTH secretion bilateral adrenal hyperplasia high

8. What is dexamethasone suppression test ?


- High-dose dexamethasone (cortisol analog) suppresses ACTH production by a pituitary adenoma
(cortisol levels decrease), but fails to suppress ectopic (paraneoplastic) ACTH production by a
small cell lung carcinoma of lung (cortisol levels remain high).
Hyperadlosteronism - Excess aldosterone

9. What are the presentation of hyperadlosteronism ?


- Hypernatremia ----> HTN
- Hypokalemia ⇄ Metabolic alkalosis.
[Aldosterone increases Na+ reabsorption and K+ excretion from principal
cells (of collecting duct) while it increases H+ excretion from intercalated
cells (of collecting duct)].

10. Describe primary hyperaldosteronism (Conn syndrome).


○ Cause
- Most common cause - sporadic adrenal hyperplasia (bilateral).
- Rare cause - adrenal adenoma and adrenal carcinoma.
○ Diagnosis
- High aldosterone
- Low renin (high blood pressure downregulates renin synthesis via negative feedback).

11. Describe secondary hyperaldosteronism.


○ Cause
- Renovascular hypertension / CHF (hypotension) ----> overactivity of RAAS.
- Renovascular HTN is due to ra stenosis:
=> fibromuscular dysplasia of renal artery
=> atherosclerosis of renal artery
○ Diagnosis
- High aldosterone.
- High renin.
Congenital Adrenal Hyperplasia - Excess sex steroids

21-hydroxylase CAH
11. Describe the Pathogenesis of congenital adrenal hyperplasia.
- Most commonly seen due to inherited deficiency of 21- hydroxylase (95%).
- 21-hydroxylase is required for the production of mineralocorticoids and glucocorticoids.
- Steroidogenesis is predominantly shunted toward sex steroid production.

12. What are the presentation of congenital adrenal hyperplasia ?


- Due to Excess sex steroids:
o Clitoral enlargement (females)
o Precocious (early) puberty in males
- Due to Low Aldosterone:
o Salt wasting ---> Hyponatremia ---> hypovolemia ---> Life-threatening hypotension.
o Hyperkalemia ⇄ Metabolic acidosis.
- Due to Low Cortisol:
o leads to increased ACTH secretion (lack of negative feedback), which results in
bilateral adrenal gland hyperplasia.

11β-hydroxylase CAH
11. Describe the Pathogenesis of congenital adrenal hyperplasia.
- Least commonly seen due to inherited deficiency of 11β-hydroxylase (5%).
- Deficient production of Aldosterone and Cortisol.
- Very high level of 11-deoxycorticosterone (a weak mineralocorticoid) ----> causes effects
of mineralocorticoid excess.
- Very high level of 11-Deoxycortisol (a weak glucocorticoid) -----> causes hypertension
[enhances alpha-1 R activity on vascular smooth muscle].
Adreno-cortical insufficiency - lack of corticosteroids

Waterhouse-Friderichsen syndrome ----> Primary Acute insufficiency


14. Describe Waterhouse-Friderichsen syndrome ?
o Commonly seen in young kids.
o Due to N meningitidis infection ---> widespread endothelial damage ---> DIC ---->
Purpura of skin.
o Characterized by hemorrhagic necrosis of the adrenal glands (bilateral).
o MOA: Inflammatory response ----> Cytokines released ----> massive vasodilation ---->
decreased s. vascular resistance -----> low blood pressure (Septic shock) ----->
inadequate tissue perfusion ----> Necrosis.

Fig - sac of blood adrenals classically seen in waterhouse friderichsen syndrome

Addison's disease ----> Primary Chronic insufficiency


15. What are the causes of Addison's disease ?
- Autoimmune adrenalitis [most common cause in developed world]
=> Autoimmune polyendocrine syndrome type 1
=> Autoimmune polyendocrine syndrome type 2
- TB [most common cause in developing world].
- Metastatic carcinoma (arising from lung cancer).

16. What are the presentation of Addison's disease ?


- Hyponatremia ---> hypovolemia ---> Hypotension.
- Hyperkalemia ----> Metabolic acidosis
- Hypoglycemia.
- Weakness and Fatigue
- Vomiting and diarrhea
- Hyperpigmentation - [Pro-opiomelanocortin (POMC) polyprotein in the anterior pituitary cleaves
-----> ACTH is generated which is again cleaved -----> alpha-MSH is finally generated.
Lack of negative feedback from corticosteroids ----> ACTH synthesis increased from POMC ----->
hence alpha-MSH synthesis increases].
Adrenal Medulla
Pheochromocytoma
17. Describe pheochromocytoma.
- Tumor of chromaffin cell.
- Medulla is made of neural crest derived chromaffin cells.
- Classic finding: brown tumor (because chromaffin cells are brown).

Fig - pheochromocytoma. Yellow parts seen in left and right are adrenal cortex.

18. What are the presentation of pheochromocytoma ?


- Clinical features are due to increased serum catecholamines.
o Episodic hypertension, headache, palpitations, tachycardia, and sweating.
o Orthostatic hypotension may be seen - because alpha receptors are sensitized
to high levels of catecholamines.

18. What are the diagnosis of pheochromocytoma ?


o ↑ serum metanephrines
o ↑ 24hr urine metanephrines and VMA (vanillylmandelic acid)

18. What are the Treatment of pheochromocytoma ?


o Surgical excision - Catecholamines may leak into the bloodstream upon
manipulation of the tumor due to mechanical stress.
o Phenoxybenzamine (irreversible alpha-blocker) is administered
perioperatively to prevent a hypertensive crisis.

19. What is the rule of 10's?


- 10% bilateral
- 10% familial
- 10% malignant
- 10% located outside adrenal medulla
[common sites are:
=> urinary bladder wall
=> organ of Zuckerkandl at the IMA root]

20. What are the associations of pheochromocytoma ?


- MEN 2A (Multiple endocrine neoplasia type 2A) - Sipple syndrome.
- MEN 2B (Multiple endocrine neoplasia type 2B) - Wagenmann–Froboese syndrome
- von Hippei-Lindau disease
- neurofibromatosis type 1.
Chapter 16: Breast Pathology
16.1 Introduction

I. BREAST
A. Modified sweat gland embryologically derived from the skin
1. Breast tissue can develop anywhere along the milk line, which runs from the
axilla to the vulva (e.g., supernumerary nipples).
2. In females, lobules and ducts are present in highest density in upper lateral part
of breast. Females develop lobular unit after puberty.
3. In males, lobules and ducts are present in highest density in subaerolar area.
Males only have terminal duct but not lobular unit so can't make milk.

B. The terminal duct lobular unit is the functional unit of the breast; lobules
make milk that drains via ducts to the nipple.

C. Lobules and ducts are lined by two layers of epithelium.


1. Luminal cell layer-inner cell layer lining the ducts and lobules; responsible for
milk production in the lobules -columnar epithelial cell.
2. Myoepithelial cell layer-outer cell layer lining ducts and lobules; contractile
function propels milk towards the nipple.

D. Breast tissue is hormone sensitive.


1. Before puberty, male and female breast tissue primarily consists of large ducts
under the nipple.
2. Development after menarche is primarily driven by estrogen and progesterone
lobules and small ducts form and are present in highest density in the upper
outer quadrant.
3. Breast tenderness during the menstrual cycle is a common complaint, especially
prior to menstruation.
4. During pregnancy, breast lobules undergo hyperplasia.
i. Hyperplasia is driven by estrogen and progesterone produced by the corpus
luteum (early first trimester), fetus, and placenta (later in pregnancy).
5. After menopause, breast tissue undergoes atrophy.

E. Galactorrhea refers to milk production outside of lactation (during pregnancy).


1. It is not a symptom of breast cancer.
2. Causes include nipple stimulation (common physiologic cause), prolactinoma of
the anterior pituitary (common pathologic cause), and drugs.
16.2 Inflammatory Conditions

1. Describe the following inflammatory conditions of the breast.


Cause Presentation Treatment
Acute - Bacterial infection of the breast; - Erythematous breast. - Continued drainage
mastitis usually due to Staphylococcus aureus. - Purulet nipple discharge. (e.g., feeding).
- Associated with breast-feeding; - Abscess formation. - Antibiotics (e.g.,
fissures develop in the nipple providing dicloxacillin).
a route of entry for microbes.
Periductal - Inflammation of the subareolar ducts. - Subareolar mass.
mastitis - Usually seen in smokers. - nipple retraction - due
[Smoking --> Relative Vitamin A to fibrosis.
decifiency --> squamous metaplasia of
lactiferous ducts --> sq. cells produce
keratin --> duct blockage --> resulting in
inflammation].

Mammary - Chronic Inflammation due to dilation - Periareolar mass.


duct Ectasia (ectasia) of the subareolar ducts. - green-brown nipple
- Cause of dilation: discharge -inflammatory
-> stagnant milk/colostrum blocks the debris .
lactiferous duct -> dilation of duct. - Plasma cells is seen on
- Rarely seen; classically arises in biopsy.
multiparous postmenopausal women.
Fat necrosis - Necrosis of breast fat. - Mass on physical exam.
- Usually related to trauma (however, a - Abnormal calcification on
history of trauma may not always be mammography (due
evident). to saponification).
- Biopsy shows necrotic fat
with associated
calcifications and giant
cells.
16.3 Benign Tumors and Fibrocystic Changes

1. Describe the following benign tumor and fibrocystic changes in the breast.
Definition Epidemiology Presentation
Fibrocystic - Development of fibrosis and Most common change in the - Presents as vague irregularity
breast cysts in the breast. premenopausal breast; thought to of the breast tissue ('lumpy
changes [fibrocystic change presents be hormone mediated - seen in breast'), usually in the
as fibrosis of stromal cells 30-60% of women upper outer quadrant.
and cystically dilated ducts].

Benign fibrocystic-related RISK for Invasive Clinical


changes of Breast Breast carcinoma features
Fibrosis, cysts, and No increased risk Cysts have a blue-dome appearance on gross
apocrine metaplasia exam lined by a flattened atrophic epithelium
Ductal hyperplasia and 2x increased risk in Sclerosing = hardning; adenosis = too many
sclerosing adenosis both breasts glands; calcification maybe seen
Atypical hyperplasia 5x increased risk in Hyperplasia maybe lobular or ductal
both breasts

Definition Differential diagnosis Presentation


Intraductal - Papillary growth, usually into - Must be distinguished from - Classically presents as bloody/
papilloma a large duct. papillary carcinoma, which also serous nipple discharge in a
- Characterized by fibrovascular presents as bloody nipple premenopausal woman.
projections lined by epithelial discharge. - Usually present in one of the
(luminal) and myoepithelial main lactiferous duct below
cells. areola and may cause nipple
retraction.

Papillary - Fibrovascular projections lined - Must be distinguished from - Risk of papillary carcinoma
carcinoma by epithelial cells without Intraductal papilloma, which increases with age; thus, it is
underlying myoepithelial cells. also presents as bloody nipple more commonly seen
discharge. in postmenopausal women.

Definition Epidemology Presentation


Fibro- - Tumor of fibrous tissue and - Most common benign neoplasm - Well-circumscribed, mobile
adenoma glands of lobular unit. of the breast; usually seen in marble-like mass - move
- Growth of fibrous part premenopausal women. freely (in contrast to
squeezes the lumen of duct. - Benign, with no/mildly infiltrative ductal carcinoma
- Estrogen sensitive- grows increased risk of carcinoma. that's immobile).
during pregnancy and may be - gross speciman shows well
painful during the menstrual demarcated, capsulated tumor.
cycle.

Phyllodes - Fibroadenoma-like tumor but - Most commonly seen in - Characteristic 'leaf-like'


tumor much larger with overgrowth postmenopausal women projections are seen
of the fibrous component. - Most common in 5th decade on biopsy (stromal tumor).
- Maybe malignant in some cases
16.4 Breast Cancer

I. BASIC PRINCIPLES
A. Most common carcinoma in women by incidence (excluding skin cancer).
B. 2nd most common cause of cancer mortality in women.

1. What are the risk factors for breast cancer?


- Risk factors are mostly related to estrogen exposure.
○ Female gender (female:male = 100:1 for breast cancer incidence).
○ Age- Cancer usually arises in postmenopausal women, with the notable
exception of hereditary breast cancer.
○ Early menarche/late menopause (increases estrogen exposure).
○ Obesity (fat cells converts testosterone to estrogen).
○ Atypical hyperplasia.
○ First degree relative (mother, sister, or daughter) with breast cancer.
○ Race - African Americans at more risk.
○ BRACA +ve (BRACA 1 = risk of ovarian cancer and triple (-) breast cancer;
BRACA 2 = breast cancer in males)

VI. PROGNOSTIC AND PREDICTIVE FACTORS

A. Prognosis in breast cancer is based on TNM staging.


1. Metastasis is the most important factor, but most patients present before
metastasis occurs; hence not very useful.
2. Spread of tumor to axillary lymph nodes (N) is the most useful prognostic factor
(given that metastasis is not common at presentation);
Sentinel lymph node biopsy is used to assess axillary lymph nodes.
[The sentinel lymph node is the hypothetical first lymph node or group of nodes
draining a cancer].

B. Predictive factors predict response to treatment.


1. Most important factors are presence/absence of:
---> estrogen receptor (ER) gene amplification.
---> progesterone receptor (PR) gene amplification.
---> HER2/neu gene amplification.
-HER2 = human epidermal growth factor receptor 2.
-neu = proto-oncogene.

C. Treatment of Breast cancer:


1. ER and PR overexpression is associated with response to antiestrogenic agents (e.g.,
tamoxifen); both receptors are located in the nucleus
2. HER2/neu overexpression is associated with response to trastuzumab (Herceptin),
a designer antibody directed against the HER2 receptor; HER2/neu is a growth
factor receptor present on the cell surface -tk receptor.
3. No treatment: 'Triple-negative' tumors are negative for ER, PR, and HER2/ neu and
have a poor prognosis; African American women have an increased propensity to
develop triple-negative carcinoma.
Histology Biopsy
Ductal A. Malignant proliferation of cells in ducts and lobules with no invasion of the
carcinoma in basement membrane The term “ductal” was used to describe this
situ (DCIS) Moslty arises in Large ducts (/Medium and Small-sized ducts). lesion because when it involves lobules, the expanded
C. Histologic subtypes are based on architecture; acini take on an appearance resembling small ducts.
comedo type is characterized by high-grade cells with necrosis
(dead cancer cells) and dystrophic calcification in the center
of ducts.
D. Paget disease of the breast is DCIS that extends up the ducts to
involve the skin of the nipple.
1. Presents as nipple ulceration and erythema.
2. Paget disease of the breast is almost always associated with an
underlying carcinoma.
B. Often detected as calcification on mammography;
DCIS does not usually produce a mass.
1. Mammographic calcifications can also be associated with benign
conditions such as fibrocystic changes (especially sclerosing
adenosis) and fat necrosis.
2. Biopsy of calcifications is often necessary to distinguish between
benign and malignant conditions.

Lobular A. Malignant proliferation of cells in ducts and lobules with no The term “lobular” was used to describe this
carcinoma in invasion of the basement membrane lesion because the cells expand but do not distort
situ (LCIS) Mosty arises in TDLU [Often multifocal and bilateral]. involved spaces and, thus, the underlying lobular
B. LCIS does not produce a mass or calcifications and is usually architecture is preserved.
discovered incidentally on biopsy.
C. Characterized by dyscohesive cells lacking E-cadherin adhesion
protein

E. Treatment is tamoxifen (to reduce the risk of subsequent


carcinoma) and close follow-up; low risk of progression to invasive
carcinoma
Invasive • Invasive ductal carcinoma
(Infiltrating) aka (“not otherwise specified = NOS”)/(“no special type = NST”)
Carcinoma - MOST COMMON TYPE OF INVASIVE CARCINOMA OF BREAST
accounting for > 80% of cases.
- Biopsy usually shows Invasive carcinoma that classically forms duct-like
structures in a desmoplastic stroma.
- Rock hard immobile mass - grossly see classic 'stellate' appearance.
- Presents as a mass detected by physical exam or by mammography
1. Clinically detected masses are usually 2 cm or greater.
2. Mammographically detected masses are usually 1 cm or greater.
3. Advanced tumors may result in dimpling of the skin or retraction of the
nipple.

Special subtypes of Invasive (Infiltrating) Carcinoma


• Tubular carcinoma:
○ Characterized by well-differentiated tubules that lack
myoepithelial cells in desmoplastic stroma.
i. Relatively good prognosis
• Mucinous carcinoma:
○ Characterized by carcinoma with abundant extracellular
mucin ('tumor cells floating in a mucus pool').
i. Tends to occur in older women (avg. age is 70 years).
ii. Relatively good prognosis.
• Medullary carcinoma:
○ Characterized by large, high -grade cells growing in
sheets with associated lymphocytes and plasma cells.
i. Grows as a well-circumscribed mass that can mimic
fibroadenoma on mammography.
ii. Relatively good prognosis.
iii. Increased incidence in BRCA1 carriers.
• Inflammatory carcinoma:
○ Characterized by carcinoma in dermal lymphatics.
○ Presents classically as an inflamed, swollen breast
(tumor cells block drainage of lymphatics) with no
discrete mass w/ Peau d'orange appearance.
i. can be mistaken for acute mastitis
ii. Poor prognosis
• Invasive Lobular carcinoma:
○ Invasive carcinoma that characteristically grows in a
single-file pattern (aka Indian file) ; cells may exhibit
signet-ring morphology.
○ No duct formation due to lack of E-cadherin.
- Often bilateral and multiple lesions in same location
- Better prognosis than invasive ductal carcinoma
- Associated with LCIS in 90% of cases.
- Has diffuse invasive pattern of spread so difficult to
detect by physical or radiologic exam.
Hereditary breast cancer
Epidemiology - Represents 10% of breast cancer cases.
- Clinical features that suggest hereditary breast cancer include
--> tumor at an early age (premenopausal)
--> Presence of multiple tumor in a single pt.
--> multiple first degree relatives with breast cancer.
Genetics - BRCA1 and BRCA2 mutations are the most important single gene
mutations associated with hereditary breast cancer.
- BCAR mutations are autosomal dominant.
-BRCA1 = Breast cancer 1 gene; locus 17q -ts gene.
-BRCA2 = Breast cancer 2 gene; locus 13q -ts gene.
1. BRCA1 mutation involves:
--> female breast [medullary carcinoma].
--> ovaries [Serous carcinoma].
2. BRCA2 mutation is associated with Male breast carcinoma.
Prophylaxis - Women with a genetic propensity to develop breast cancer may
choose to undergo removal of both breasts (bilateral mastectomy)
to decrease the risk of developing carcinoma.
1. A small risk for cancer remains because breast tissue sometimes
extends into the axilla or subcutaneous tissue of the chest wall.

Male breast cancer


Epidemiology - Rare: Represents 1% of all breast cancers.
Types - Most common histological subtype: Invasive ductal carcinoma.
- Lobular carcinoma is rare (the male breast develops very few lobules).
Presentation - Usually presents as a subareolar mass in older males
1. Highest density of breast tissue in males is underneath the nipple.
2. May produce nipple discharge.
Genetic - Klinefelter syndrome (XXY).
associations - BRCA2 mutations.
Chapter 17: Central Nervous System
Pathology
17.1 Developmental Anomalies

Neural tube defects


Defn - Arise from incomplete closure of the neural tube.
Types Anencephaly Spina bifida
Defn - Absence of skull and brain - Failure of posterior vertebral arch to close
- disruption of the cranial end of the neural
tube --> Leads to a 'frog-like' appearance
of the fetus.
Result - Results inmaternal polyhydramnios since - Resulting in a vertebral defect (disruption of the
fetal swallowing of amniotic fluid is caudal end of the neural tube).
impaired.
Types a. Meroanencephaly a. Spina bifida occulta
- characterized by malformed cranial - The outer part of some of the vertebrae is not
bones, a median cranial defect, and a completely closed.The splits in the vertebrae are so
cranial protrusion filled with area small that the spinal cord does not protrude.
cerebrovasculosa. - Asymptomatic; presents as a dimple or patch of hair
b. Holoanencephaly overlying the vertebral defect.
- brain has entirely failed to form (except b. Spina bifida - presents with cystic protrusion of the
for the brain stem). underlying tissue through the vertebral defect.
c. Craniorachischisis
- entire spinal cord and brain are exposed. 1. Meningocele - meninges protrude.
- area cerebrovasculosa and area 2. Meningomyelocele - meninges and spinal
medullovasculosa fill both cranial cord protrude.
defects and the spinal column (the 3. Myelocele - spinal cord protrude.
vertebrae overlying the open portion of
the spinal cord do not fully form and
remain unfused and open, leaving the
spinal cord exposed).
Characterized by bony defects in the
spine and the exposure of neural tissue
as the vault of the skull fails to form.
Association - Associated with low fo late levels prior to conception

Detection - Detected during prenatal care by ↑ alpha-fetoprotein (AFP) levels in amniotic fluid and maternal blood.
[AFP most abundant protein in young fetus blood - made by fetal liver and yolk sac. AFP elevated in other
cases too - hepatocellular carcinoma, liver metastasis, yolk sac tumor, germ cell tumor.]
Anatomy 1. Neural plate invaginates early in gestation to form the neural tube,
which runs along the cranial-caudal axis of the embryo.
- The wall of the neural tube forms --> CNS tissue
- The hollow lumen forms --> the ventricles and spinal cord canal
- The neural crest forms --> the PNS.
CEREBRAL AQUEDUCT STENOSIS
Defn - Congenital stenosis of cerebral aqueduct (of sylvius) that drains cerebrospinal fluid
(CSF) from the 3rd ventricle into the 4th ventricle ----> Leads to accumulation of CSF in
the ventricular space [obstructive HYDROCEPHALUS].
Clinical - Presents with enlarging head circumference due to dilation of the ventricles (cranial
feature suture lines are not fused).
Epid - MOST COMMON cause of HYDROCEPHALUS in newborns.
Anat. 1. CSF is produced by the choroid plexus lining the ventricles.
2. Flows from the lateral ventricles into the 3rd ventricle via
the interventricular foramen of Monro.
3. Flows from the 3rd ventricle into the 4th ventricle via the
cerebral aqueduct.
4. Flows from the 4th ventricle into the subarachnoid space
via the foramina of Magendie and Luschka.

Dandy-Walker malformation
Defn - Congenital failure (partial or complete) of cerebellar vermis to develop ---> massively dilated
4th ventricle ---> The posterior fossa is enlarged and the tentorium is in high position.
- Congenital malformation of The foramina of Luschka and foramen of Magendie ----> often
leads to obstructive HYDROCEPHALUS [↑Intracranial pressure & Motor function defect].

Arnold-Chiari malformation [type II Chiari malformation (CM)]


Defn - Congenital downward displacement of cerebellar vermis[major extent] and cerebellar
tonsils[minor extent] along with brain stem (medulla) descent through the foramen
magnum ---> CSF flow obstruction ---> obstructive HYDROCEPHALUS.
- Classic Anatomic associations:
= Low lying torcular herophili (confluence of sinuses)
= tectal beaking
= clival hypoplasia
- Often occurs in associated with
--> meningomyelocele -most cases.
--> syringomyelia[cyst/cavity(syrinx) forms within spinal cord].
--> Colpocephaly -disproportionate enlargement of the occipital horns
of the lateral ventricles.
17.2 Spinal Cord Lesions

Syringomyelia
Defn - Cystic degeneration of the spinal cord.
Associa. - Arises with trauma or in association with an Arnold-Chiari malformation.
Clinical - Syrinx Usually occurs at C8-T1:
feature 1. Sensory loss of pain and temperature with sparing of fine touch and position
sense in the upper extremities ("cape like" distribution) -due to involvement of
the anterior white commissure of the spinothalamic tract with sparing of the
dorsal column
- Syrinx expansion results in involvement of other spinal tracts leading to:
2. Muscle atrophy and weakness with decreased muscle tone and impaired
reflexes -due to damage to lower motor neurons of the anterior horn.
3. Horner syndrome with ptosis (droopy eyelid), miosis (constricted pupil), and
anhidrosis (decreased sweating) -due to disruption of the lateral horn of the
hypothalamospinal tract.

Poliomyelitis
Defn - Damage to the anterior motor horn due to poliovirus infection.
Clinical - Presentation (lower motor neuron defective sign):
feature • Flaccid paralysis with muscle atrophy (wasting).
• Fasciculations/muscle twitch -small, local, involuntary
skeletal muscle contraction and relaxation. Spontaneous
depolarization of a lower motor neuron leading to the
synchronous contraction of all the skeletal muscle fibers
within a single motor unit.
• Weakness with decreased muscle tone (Hypotonia).
• Impaired reflexes involving skeletal muscle.
• Negative Babinski sign -no extension of the hallux.

Werding-Hoffman disease [Spinal muscular atrophy 1 (SMA1) -Infantile]


Defn - AR inherited degeneration of anterior motor horn.
- genetic defect in the SMN1 gene.
Clinical - Presents as "floppy baby syndrome".
feature - Usual age of onset upto 6mnts.
- death within a few years after birth due to pneumonia-induced
respiratory failure.
Amyotrophic lateral sclerosis(ALS) / motor neurone disease(MND) / Lou Gehrig's disease
Defn - Degenerative disorder of upper & lower motor neurons of the corticospinal tract.
Cause - Familial ALS (ALS type1) - Zinc-copper superoxide dismutase mutation (SOD1)
---> leads to free radical injury in neurons.
autosomal dominant disorder at 21q22.1 locus.
Epidem. - Most cases are sporadic.
- arises in middle age adults.
Clinical 1. Anterior motor horn degeneration leads to lower motor neuron signs:
feature [Early presentations]
- Flaccid paralysis with muscle atrophy (wasting).
- Fasciculations/muscle twitch.
- Weakness with decreased muscle tone (Hypotonia).
- Impaired reflexes.
- negative Babinski sign -no extension of the hallux.
2. Lateral corticospinal tract degeneration leads to upper motor neuron signs:
[Late presentations]
- spastic paralysis
- increased muscle tone (Hypertonia).
- hyperreflexia.
- positive Babinski sign -extension of the hallux.
Differential - Sensory nerves and the autonomic NS [Sympathetic + parasympathetic NS]
diagnosis are generally unaffected -distinguishes ALS from syringomyelia.

Friedreich's ataxia
Defn - Degenerative disorder of cerebellum and spinal cord.
Clinical - Degeneration of the cerebellum - Degeneration of multiple spinal cord tracts:
feature • saccade. • Loss of - vibratory sense.
• truncal ataxia. - proprioception.
• dysarthria. - deep tendon reflexes.
• dysmetria. • muscle weakness in lower extremities.
Epidem - Presents in early childhood
- patients are wheelchair bound within a few years.
Cause - Autosomal recessive disorder ---> expansion of an unstable trinucleotide repeat
(intronic -GAA) in the frataxin gene ---> ↓ Frataxin protein level ---> mitochondrial
iron overload ---> free radical mediated damage.
I. Frataxin is essential for mitochondrial iron regulation - Frataxin is a Fe-binding
protein responsible for forming Fe–S clusters.
Associa - hypertrophic cardiomyopathy
17.3 Meningitis

Defn - Inflammation of the leptomeninges (arachnoid and pia mater).


Cause - Most commonly due to an infectious agent.
BACTERIA - acute meningitis VIRUS - acute meningitis FUNGI - chronic meningitis
Neonates - Streptococcus agalactiae
Most - Enteroviruses Most - Cryptococcus neoformans
& infants - Escherichia coli. common - HIV common
- Listeria monocytogenes. - HSV-2
Children & - N meningitidis. Less - HSV-1 Less - Coccidioides immitis
teenagers - Streptococcus pneumoniae frequent - LCM virus frequent - Histoplasma capsulatum
- Haemophilus influenzae - Mumps virus - Blastomyces dermatitidis
Adults & - N meningitidis. - CMV ( HHV-5) - Candida sp.
elderly - Streptococcus pneumoniae - EBV (HHV-4)
- Listeria monocytogenes. - VZV (HHV-3)
BACTERIA - chronic meningitis PARASITE - chronic meningtis
- Mycobacterium tuberculosis - Toxoplasmosis gondii
- Borrelia burgdorferi - Naegleria fowleri
- Treponema pallidum - Schistosoma sp.
- Francisella tularensis - Echinococcus granulosus
- Nocardia sp.

Presentation - Classic triad of headache, fever and nuchal rigidity.


- Photophobia and vomiting.
- Altered mental status may be present.
Complication - Commonly seen in bacterial meningitis
• Hydrocephalus, hearing loss and seizures - sequelae related to fibrosis.
• Death - herniation secondary to cerebral edema.
Diagnosis - Diagnosis is made by lumbar puncture (sampling of CSF).
1. Performed by placing a needle between L4 and L5 (level of the iliac crest).
Spinal cord ends at L2, but subarachnoid space and cauda equina continue to S2.
2. Layers crossed include skin, ligaments, epidural space, dura, and arachnoid.
CSF findings
C. agent Cells Glucose Protein CSF pressure Appearance
Bacteria Neutrophils ↓ ↑ ↑ Turbid
Virus Lymphocytes Normal Normal Normal Clear
Fungal/TB Lymphocytes ↓ ↑ ↑ Fibrin web
Cerebrovascular Disease/ Stroke/ Brain attack

Ischemia (85% of cases)

Ischemic cascade
1. Lack of oxygen causes the neuron's normal process for making ATP for energy to fail.
2. The cell switches to anaerobic metabolism, producing lactic acid - irritant which could potentially destroy
cells since it is an acid and disrupts the normal acid-base balance in the brain.
3. ATP-reliant ion transport pumps fail, causing the cell to become depolarized, allowing ions, including
calcium (Ca2+), to flow into the cell.
4. The ion pumps can no longer transport calcium out of the cell, and intracellular calcium levels get too high.
5. The presence of calcium triggers the release of the excitatory amino acid neurotransmitter glutamate.
6. Glutamate stimulates AMPA receptors and Ca2+-permeable NMDA receptors, which open to allow more
calcium into cells [The concentration of glutamate outside the cells of the nervous system is normally kept
low by so-called uptake carriers, which are powered by the concentration gradients of ions (mainly Na+)
across the cell membrane].
7. Excess calcium entry overexcites cells and causes the generation of harmful chemicals like free radicals,
reactive oxygen species [In fact, many antioxidant neuroprotectants such as uric acid and NXY-059 work at
the level of the endothelium and not in the brain per se] and calcium-dependent enzymes such as calpain,
endonucleases, ATPases, and phospholipases in a process called excitotoxicity. Calcium can also cause the
release of more glutamate.
Release of matrix metalloproteases, which are zinc- and calcium-dependent enzymes that break down
collagen, hyaluronic acid, and other elements of connective tissue.
8. As the cell's membrane is broken down by phospholipases, it becomes more permeable, and more ions
and harmful chemicals flow into the cell.
9. Mitochondria break down, releasing toxins and apoptotic factors into the cell.
10. The caspase-dependent apoptosis cascade is initiated, causing cells to "commit suicide."
11. If the cell dies through necrosis, it releases glutamate and toxic chemicals into the environment around it.
Toxins poison nearby neurons, and glutamate can overexcite them.
12. If and when the brain is reperfused, a number of factors lead to reperfusion injury.
13. An inflammatory response is mounted, and phagocytic cells engulf damaged but still viable tissue.
14. Harmful chemicals damage the blood–brain barrier.
15. Cerebral edema (swelling of the brain) occurs due to leakage of large molecules like albumins from blood
vessels through the damaged blood brain barrier. These large molecules pull water into the brain tissue
after them by osmosis. This "vasogenic edema" causes compression of and damage to brain tissue.

Cerebral infarction
Defn - Neurologic deficit due to cerebrovascular compromise.
- Neurons are dependent on serum glucose as an essential energy source
and are particularly susceptible to ischemia (undergo necrosis within
3-5 minutes).
Epdm - No. 3 cause of death in USA - major cause of morbidity and mortality.
Types - Total Anterior Circulation Infarct (TACI) - of the Circle of Willis
- Partial Anterior Circulation Infarct (PACI) - of the Circle of Willis
- Lacunar infarct (LACI)/ Lacunar stroke
- Posterior Circulation Infarct (POCI) - of the Circle of Willis
Global Brain ischemia
Defn. - Global ischemia to the brain.
Major - Low perfusion (ex- atherosclerosis).
Etiology - Acute decrease in blood flow (ex - cardiogenic shock).
- Chronic hypoxia (ex- anemia).
- Repeated episodes of hypoglycemia (ex- insulinoma).
Clinical - Presentation is based on duration and magnitude of the insult.
features
Mild global ischemia Severe global ischemia Moderate global ischemia
- Results in transient - Results in diffuse necrosis; - Leads to infarcts in watershed areas (e.g., area lying
confusion with survival leads to a between regions fed by the anterior and middle cerebral
prompt recovery. 'vegetative state.' artery) and damage to highly vulnerable regions such as:
= Pyramidal neurons of the cerebral cortex (layers
3, 5, & 6) --> leads to laminar necrosis.
= Pyramidal neurons of the hippocampus (temporal
lobe) - important in long term memory.
= Purkinje layer of the cerebellum - integrates sensory
perception with motor control

Focal Brain ischemia


Defn - Regional ischemia to the brain that results in focal neurologic deficits lasting >24 hours.
- If symptoms last <24 hours, the event is termed a Transient ischemic attack (TIA).
Etiology Thrombotic stroke Embolic stroke Lacunar stroke
Cause - Rupture of an atherosclerotic plaque. - Thromboemboli from - Due to hyaline arteriolosclerosis
left side of the heart (complication of HTN and
(e.g., atrial fibrillation). diabetes mellitus).
Location - Mostly at branch points (ex- bifurcation - Usually involves the - Mostly involves the
of internal carotid artery & middle middle cerebral artery. lenticulostriate vessels
cerebral artery in the circle of Willis).
Clinical - Pale infarct at the periphery of the - Hemorrhagic infarct at - results in fluid filled Small
feature cortex. the periphery of the cystic areas of infarction.
cortex - If internal capsule is involved
--> pure motor stroke
- If thalamus is involved
--> pure sensory stroke

Outcome - Liquefactive necrosis


12 hrs - Eosinophilic change in the neuronal cytoplasm (red neurons).
24 hrs - Neuronal Necrosis.
1-3 days - Neutrophil infiltration
4-7 days - Microglial infiltration
2-3 wks - Gliosis (proliferation of astrocyte)
Hemorrhage (15% of cases)

Intra-Cerebral hemorrhage
Defn - Intraparenchymal hemorrhage ---> bleeding within the brain tissue.
↓ may spread to internal capsule & even progress to
- Intraventricular hemorrhage ---> bleeding within the brain's ventricular system.
Etiology - Classically due to rupture of Charcot-Bouchard microaneurysms of the
lenticulostriate vessels into --> Basal ganglia.
--> lateral ventricle.
- It is a Complication of hypertension;
treatment of HTN reduces incidence by half.
Clinical - Headache, nausea, vomiting, and eventual coma.
feature

Sub-arachnoid hemorrhage
Defn - Bleeding into subarachnoid space—the area between the arachnoid
membrane and the pia mater.
Cause 1. Rupture of berry aneurysm (85% of cases)
- thin-walled saccular outpouchings that lack a tunica media layer
--> increasing the risk for rupture.
- Associated with: --> Marfan syndrome
--> AD polycystic kidney disease.
- Most frequently located in the circle of Willis at branch points of:
= anterior communicating artery
= posterior communicating artery
= internal carotid artery
Other sites are: = branching pts. of Middle cerebral artery
= Tip of basilar artery
2. Arteriovenous malformation (AVM)
3. Anticoagulated state - aspirin use.
Clinical - Presents as a sudden headache ('thunderclap' headache) - a headache described
feature as "like being kicked in the head" or the "worst ever" developing over seconds to
minutes) -lone acute severe headache.
- Nuchal rigidity.
- Lumbar puncture [CSF is removed from the subarachnoid space of the spinal
canal] shows either:
Early- high RBCs count due to hemorrhage
Late- xanthochromia (yellow hue due to bilirubin breakdown).
17.5 Trauma

Epidural hematoma (EDH) Subdural hematoma (SDH)


Defn - Collection of blood b/w the outer periosteal layer of - Collection of blood b/w the inner meningeal layer of
Dura mater and inner table of the skull. Dura mater and arachnoid mater.
Patho- - Classically due to fracture of the temporal bone with - Trauma (Head injury) leads to tearing of bridging
physiology rupture of the middle meningeal artery; veins that lie b/w the dura and arachnoid.
---> bleeding separates the dura from the skull. ---> blood covers the surface of the brain.
- Other vessels involved are: - Depending on the speed of their onset:
- Frontal locus: anterior ethmoidal artery - acute: <3days
- Occipital locus: transverse or sigmoid sinuses - subacute: 3-14 days
- Vertex locus: superior sagittal sinus - chronic: >14 days

Present. - Lucid interval may precede neurologic sign. - Presents with progressive neurologic signs.
Imaging - Lens(biconvex) shaped lesion on CT. - Crescent shaped lesion on CT.
Lethal - Brain Herniation. - Brain Herniation.
Complica.
Treatment - Craniotomy - Craniotomy
Risk - Any age. - very old age - age related cerebral atrophy(shrinks)
factors --> subdural space enlarges --> old brittle veins that
traverse the space must travel over a wider distance
--> making them more vulnerable to tears/stretch.

Herniation - Displacement of brain tissue due to mass effect / ↑ intracranial pressure


Tonsillar herniation Subfalcine herniation Uncal herniation
Defn - displacement of the cerebellar - displacement of the cingulate - displacement of the temporal lobe
tonsils into the foramen magnum. gyrus under the falx cerebri. uncus under the tentorium cerebelli.
Clinical - Compression of brainstem leads - Compression of the anterior - Compression of cranial nerve III
feature to cardiopulmonary arrest. cerebral artery leads to infarction. (oculomotor) --> eye moving "down
and out" and a dilated pupil.
- Compression of posterior cerebral
artery --> infarction of occipital lobe
--> contralateral homonymous
hemianopsia.
- Rupture of the paramedian artery
--> Duret (brainstem) hemorrhage.

Herniation classification: The brain can shift across such structures as:
- Supratentorial herniation - the falx cerebri.
- Uncal (transtentorial -via tentorial notch) - the tentorium cerebelli.
- Central (transtentorial -via tentorial notch) - the foramen magnum.
- Cingulate (subfalcine/transfalcine) - fracture/surgical site in the skull
- Transcalvarial (external herniation)
- Tectal (herniates posteriorly)
- Infratentorial herniation
- Upward cerebellar/Upward transtentorial
- Tonsillar (downward cerebellar)
17.6 Demyelinating Disorders

Demyelinating Disorders
Defn - Disease where myelin is damaged; axons are generally preserved.
Myelin types - Oligodendrocytes - myelinate CNS; [single cell myelinate multiple neurons].
- Schwann cells - myelinate PNS; [single cell myelinate single neuron].
Myelin role - Myelin insulates axons, improving the speed and efficiency of conduction.

Multiple sclerosis
Defn - Autoimmune destruction of CNS myelin(CNS) -oligodendrocytes.
Epid - Most common chronic CNS disease of young adults (20-30 yrs age)
- More commonly seen in women & people living in regions away
from the equator.
Genetics - Associated with HLA-DR2 (HLA-DR15) & HLA-DQ6 polymorphism
(genetic variation) at locus 6p21.32 of AP cells.
Diagnosis - MRI reveals plaques (areas of white matter demyelination).
- Lumbar puncture shows:
• Increased lymphocytes and Igs
• oligoclonal IgG bands on high resolution electrophoresis
• Myelin basic protein
Clinical - Presents with Relapsing neurologic deficits with periods of
features remission (multiple lesions in time and space).
○ Optic nerve damage - Blurred vision in one eye
○ Medial longitudinal fasciculus damage - internuclear
opthalmoplegia
○ Brainstem damage - Vertigo and scanning speech
mimicking alcohol intoxication.
○ Periventricular cerebral white matter damage -
Hemiparesis or unilateral loss of sensation.
○ Spinal cord damage - Lower extremity loss of
sensation or weakness
○ Autonomic nervous system damage - damage
Bowel, bladder, and sexual dysfunction
Treatment - treat acute attacks with high dose steroids.
- Long term treatment with interferon beta - slows disease
progression of disease.
Leukodystrophies
Description - These are Group of disorders caused by mutations of genes whose
products are invoved in the generation, turnover or maintenance/
production of the myelin sheath ---> characterized by degeneration of
the white matter in the brain.

Metachromatic - MOST COMMON LEUKODYSTROPHY


leukodystrophy - ARSA gene mutation ---> Deficiency of arylsulfatase A (ASA) -
lysosomal enzyme ---> Sulfatides [myelin membrane component
-very long chain fatty acids (VLCFAs)] cannot be degraded and
accumulate in the lysosomes of oligodendrocytes -lysosomal
storage disease ---> toxic [triggers mitochondrial oxidative stress]
accumulation ---> oligodendrocytes gets damaged.
- Autosomal recessive
Krabbe disease - GALC gene mutation ---> Deficiency of beta-galactocerebrosidase
(GALC) - lysosomal enzyme. Galactocerebroside (myelin membrane
component) cannot be degraded[no catabolic activity] and
accumulate in the lysosomes of oligodendrocytes -lysosomal
storage disease ---> oligodendrocytes gets damaged.
- Autosomal recessive
X-linked adreno - Mutated ABCD1 gene ---> defective peroxisomal ATP-binding
leukodystrophy cassette transporters ---> halted transfer of fatty acids into
(X-ALD) peroxisomes (import) ---> No long-chain fatty acids inside the
peroxisomes -although the catabolic peroxisomal matrix enzymes
(Peroxins) are fully functional ---> Impaired addition of coenzyme A
to long-chain fatty acids -impaired peroxisomal fatty acid beta
oxidation ---> Hence no Plasmalogen [most abundant phospholipid
in myelin membrane] formation & accumulation of fatty acids in the
cytosol damages the adrenal cortex and Leydig cells in addition to
the myelin(CNS).
- X-linked defect.
Canavan disease - ASPA gene mutation ---> Deficiency of cytosolic Aspartoacylase
(aminoacylase 2) -it deacetylates N-acetyl-L-aspartate (NAA)
[NAA + Aspartoacylase = acetyl group + L-Aspartate.
- NAA is a Neuronal osmolyte hence regulates fluid balance.
- acetyl group is required for myelin(CNS) membrane lipid synthesis.
- L-Aspartate is a precursor of N-Acetylaspartylglutamic acid
neurotransmitter]
---> NAA is not metabolised ---> accumulation of excess NAA in the
cytosol of oligodendrocytes ---> oligodendrocytes gets damaged.
- Autosomal recessive
Alexander - GFAP gene mutation (gain of function) ---> accumulation of abnormal
disease Glial fibrillary acidic protein (GFAP) -[GFAP is a monomer of the
intermediate filament -cytoskeletal components of astrocytes] --->
formation of Rosenthal fibers ---> impairs cell functions [astrocytes
secrete cytokine leukemia inhibitory factor (LIF), a regulatory protein
that promotes the myelinating activity of oligodendrocytes] --->
Demyelination of myelin sheath.
- Spontaneous mutation
Subacute sclerosing panencephalitis (SSPE)
Cause - Slowly progressing, persistent infection of the brain by measles virus.
Epidem - Primary infection occurs in infancy;
neurologic signs arise years later (during childhood).
Diagnosis - viral inclusions within neurons (gray matter) and oligodendrocytes
(white matter).
- Rademecker complex is seen on EEG showing widespread cortical
dysfunction.
Clinical Progressive, debilitating encephalitis that leads to death.
feature - Initially presents as dementia, personality alterations, and loss of
movement control.
- Slowly progresses to speech loss of speech, loss of ability to walk
and dysphagia.
- Finally patient will be blind, mute, loss bodily functions and in
vegetative stage and/or comatose and evenually die.

Progressive multifocal leukoencephalopathy (PML)


Cause - JC virus infection of oligodendrocytes (white matter);
Immunosuppression (e.g., AIDS or leukemia or Rituximab drug) leads
to reactivation of the latent virus.
Diag - finding JC virus DNA in spinal fluid.
- consistent white matter lesions on brain MRI
- brain biopsy: bizarre astrocytes & enlarged oligodendroglial nuclei
Clinical - Presents with rapidly progressive neurologic signs (visual loss,
feature weakness, dementia) leading eventually to death in few mnths.

Central pontine myelinolysis (CPM)


Defn. - Focal demyelination of the pons (anterior brain stem).
Cause - Due to rapid intravenous correction of hyponatremia or
psychogenic polydipsia -inadequate thirst reception.
--->[neurons can regulate the electrolyte levels inside their cell
with changing levels of electrolyte in the extracellular space -
neurons adjust their osmolarities by changing levels of certain
osmolytes like inositol, betaine, and glutamine in response to
varying serum osmolality of Na+].
- Severely malnourished [serum ↓Na+ levels] patients.
- Alcoholics patients with liver disease (cirrhosis).
---> Solute poor alcohol prevents the free water excretion --->
thus diluting serum ---> serum↓Na+ conc.
---> Liver cirrhosis ---> Low arterial blood volume ---> triggers
nonosmotic hypersecretion of ADH ---> free water retention --->
serum↓Na+ conc.
Clin feature - Acute bilateral paralysis ("locked in" syndrome).
17.7 Dementia and Degenerative Disorders

1. What are some basic findings in neurodegenerative diseases ?


• Characterized by loss of neurons within the gray matter;
Accumulation of intra and/or extracellular proteins ---> damages neurons.
• Cortex degeneration => dementia;
Brainstem & basal ganglia degeneration => movement disorders.

Huntington's disease (HD)

14. What is the GENETICS of Huntington's disease?


• Autosomal dominant disorder (chromosome 4) characterized by expanded
trinucleotide repeats (CAG) in the huntingtin gene.
• Further expansion of repeats during spermatogenesis leads to anticipation.
- early onset of disease with each generation.

14. What is the finding in Huntington's disease?


• Degeneration of GABAergic neurons in the caudate nucleus of the basal ganglia.
---> loss of brain parenchyma (atrophy of caudate nucleus-CN) ---> compensatory
enlargement of lateral ventricles [LV] ---> HYDROCEPHALUS ex vaculo.
Anatomical boundaries:
1. Body of CN forms the floor of the Central part of LV.
2. Head of CN forms the floor of the anterior horn of LV.
3. Tail of CN forms the Roof & lateral wall of Inferior horn of LV.

15. What is presentation of huntington's disease?


• Average age at presentation is 40 years.
• Presents with chorea that can progress to dementia and depression.
• Suicide is a common cause of death.

Pick disease [PiD]

8. What is clinical presentation of frontotemporal dementia (Pick disease)?


• Early language (temporal) and behavioral (frontal) symptoms.
• Eventually progresses to dementia.
• Usually seen in 40-50 year old (can be confused with midlife crisis)
• Others - Hallucination, REM disturbances & Personality change.

9. What are biopsy findings of Pick disease?


• Degenerative disease of the frontal and temporal cortex -(spares the
parietal and occipital lobes) --> Frontotemporal dementia.
• Characterized by round aggregates of tau protein (Pick bodies) in
neurons of the cortex.

9. What is the PATHOPHYSIOLOGY of Pick disease?


• MAPT gene on c17 ---> Alternative splicing ---> Tau proteins isoforms.
• MAPT gene mutation ---> hyperphosphorylation of Tau proteins --->
aggregates into paired helical filaments ---> Tauopathy.
Pick disease: few Tau isoforms (1/2).
Alzheimer's disease: All Tau isoforms (6).
Alzheimer's disease (AD)

1. Define Alzheimer's disease (AD).


• Degenerative disease of cortex;
most common cause of dementia.

2. What is clinical presentation of Alzheimer's disease ?


• Slow-onset memory loss (begins with short-term memory loss and progresses
to long-term memory loss) and progressive disorientation.
• Loss of learned motor skills and language.
• Changes in behavior and personality.
• Patients become mute and bedridden; infection is a common cause of death.
• Focal neurologic deficits are seen in later stage of disease.

3. What are the etiologies of Alzheimer's disease?


Type Age of onset Genetics
Sporadic • Seen in elderly Gene(polymorphism) Inheritance:
(96.5% of cases) - Risk increases with age • ε4 allele of apolipoprotein E (APOE) = ↑ risk
-environmental (doubles every 5 years • ε2 allele of apolipoprotein E (APOE) = ↓ risk
after the age of 60).
Familial cases • Early onset Gene mutation:
(3.5% of cases) - usually b/w 50 and 65 • Presenilin 1 and 2 gene.
-AD disorder yrs of age (but can be • Amyloid precursor protein (APP) gene.
as early as 15)
Chromosomal • Early onset Associated with:
defect/ - commonly occurs by • Down's syndrome -> chromosome 21 trisomy
abnormality 40 years of age. -> Robertsonian translocation t(14q21q).

4. What are the Morphologic features of Alzheimer's disease?


• Gross:
○ Cerebral atrophy w/ narrowing of the gyri & widening of the sulci.
○ Dilation of the ventricles.
• Microscopic
○ Neuritic plaques - extracellular core comprised of Aβ amyloid with entangled
neuritic processes (dendrite/axon).
---> Aβ amyloid is derived from amyloid precursor protein (APP), which is coded
on chromosome 21. APP normally undergoes alpha cleavage to give Aα amyloid
(--> neurotrophic effects); beta cleavage results in Aβ amyloid.
○ Cerebral amyloid angiopathy - Aβ amyloid plaques may also deposit around vessels,
increasing the risk of hemorrhage.
○ Neurofibrillary tangles - intracellular aggregates of fibers composed of - tuft of
hyperphosphorylated tau protein - triangular shape
---> Tau is a microtubule-associated protein.
○ Loss of cholinergic neurons in nucleus basalis of Myenert.
---> Cholinergic hypothesis: ↓ synthesis of the neurotransmitter Ach is linked to
causative factor of Alzheimer's disease [Ach binds to M1-receptor ---> secondary
messenger system ---> dephosphorylation of tau proteins].

5. How is Alzheimer's disease diagnosed ?


• Clinical correlation: diagnosis of exclusion -Presumptive diagnosis.
• Histopathology: Confirmed by autopsy or biopsy (when possible).
WĂƌŬŝŶƐŽŶΖƐĚŝƐĞĂƐĞ;WͿ

10. What are biopsy finding of parkinson's disease?


• Degenerative loss of dopaminergic neurons in the substantia nigra of the basal ganglia
[Nigrostriatal pathway of basal ganglia uses dopamine to initiate movement].
• Histology reveals loss of pigmented neurons in the substantia nigra and round intracellular
eosinophilic inclusions of alpha-synuclein (Lewy bodies) in affected neurons.

11. What is the epidemology of Parkinsons?


• Common disorder related to aging; seen in 2% of older adults.

11. What is the ETIOLOGY of Parkinsons?


• Unknown etiology; Most commonly, parkinson's is idiopathic.
Historically, rare cases were related to MPTP exposure (a contaminant in
illicit drugs).

11. Describe the PATHOPHYSIOOGY of Parkinson's.


- Monomeric α-synuclein is natively unfolded in cytosol ⇄ bind to membranes in an α-helical form.
Unfolded monomer ⇄ β-sheet like oligomer species ---> higher molecular weight insoluble fibrils
- Polo-like kinase 2 (PLK2) phosphorylate α-synuclein at Ser129 - higher tendency to aggregate.
HMW fibrils + ubiquitin ---> Lewy body deposition in cytoplasm.
Missense mutations of the SNCA gene ---> triplications of the SNCA gene locus ---> Gain of
function mutation ---> ↑ expression of alpha-synuclein protein ---> leads to α-synuclein
toxicity ---> disrupts normal α-synuclein functioning ---> leading to:
- Blocked ER-golgi transport.
- Decreased synaptic vesicle release.
- Impaired Mitochondrial Energy production.
- Non functional Lysosomes.

12. What are clinical presentation of Parkinsons?


D. Clinical features ('TRAP')
1. Tremor - pill rolling tremor at rest; disappears with movement.
2. Rigidity - cogwheel rigidity in the extremities.
3. Akinesia/bradykinesia - slowing of voluntary movement;
expressionless face (Hypomimia).
4. Postural instability and shuffling gait.

10. What is Differential diagnosis of parkinson's disease?


F. Dementia is a common feature of late disease.
1. Early-onset (within 1 year) dementia is suggestive of Dementia with Lewy bodies (DLB),
which is characterized by dementia, hallucinations and parkinsonian features;
histology reveals cortical Lewy bodies.
Vascular dementia

7. Describe vascular dementia?


• 2nd most common cause of dementia.
• Occurs due to multifocal white matter infarction.
• Associated with HTN, artherosclerosis &/vasculitis -(they decrease
blood flow to brain resulting in infraction).

Normal pressure hydrocephalus

17. What is presentation of normal pressure hydrocephalus?


• Presents as triad of 3 W's:
○ Wet -urinary incontinence
○ Wobbly -gait instability
○ Wacky -dementia in adults

17. What is treatment of normal pressure hydrocephalus?


• Lumbar puncture improves symptoms.
• Treatment is ventriculo-peritoneal shunting.

18. What is normal pressure hydrocephalus ?


• Calculation of CSF Pressure -n.b. values are approximate
Normally CSF pressure is measured by Lumbar puncture.
Standard procedure: -Normal CSF pressure
- when patient lying on the side: 100-200 mm of H2O
[also, w/ the patient sitting up: 200-300 mm of H2O].
Pathological conditions:
1. Normal pressure hydrocephalus (NPH): 150 to 200 mm H2O.
2. Idiopathic intracranial hypertension (IIH): >250 mm H2O.

19. What is the PATHOPHYSIOLOGY of normal pressure hydrocephalus?


• Unknown etiology ---> abnormal accumulation of CSF in the ventricles --->
---> intracranial pressure rises ---> Ventriculomegaly ---> intracranial pressure
falls but still remains slightly elevated and reaches a high normal level.
Enlarged ventricles ---> stretches of corona radiata ---> symptoms triad.
• Possible etiology: Altered compliance (elasticity) of the ventricular walls, as
well as increased viscosity of the CSF.
Transmissible spongiform encephalopathies (TSEs)

18. What is the pathophysiology ?


• A. Prion protein is normally expressed in CNS neurons in an alpha-helical configuration (PrPc).
B. Disease arises with conversion to beta-pleated conformation (PrPsc).
Conversion can be sporadic(idiopathic), inherited(familial forms of disease), or transmitted.
C. Pathologic protein is not degradable and converts normal protein into the pathologic form,
resulting in a vicious cycle.
PrPC (Common or Cellular) - normal protein
- Human PRNP gene locus 20p13 ---> PrPC (Common or Cellular) protein.
- A GPI membrane anchor at the COOH-terminal tethers PrPC to cell membranes.
- PrPC is involved in the transport of ionic copper into cells from the outside.
PrPSc (scrapie) - infectious form
- PrPSc (scrapie) is an abnormal conformational isoform of PrPC.
- PrPSc (scrapie) is proteinase K-resistant -misfolded protein.
- Cellular protein (Protein X) enables the conversion of PrPC to PrPSc to form complex
via an elusive ligand-protein interaction.
- PrPSc formation:
--> polymorphism at codon 129 that encodes either (Met) or (Val)
Homozygous MM/VV; Heterozygous MV.
--> transmitted from external sources of PrPSc:
- ingestion of brain, spinal tissue of an infected cow suffering from Bovine SE.
- funerary cannibalism of CNS tissue from dead humans who suffered from TSEs.
- PrPSc can "spread" from from cell-to-cell which is a remarkable phenomenon that
allows a pathogenic protein to acquire many of the characteristics of an infectious
organism ---> "Transmissible" disease.

19. How to sterilize prion proteins ?


• In general, prions are quite resistant to proteases, heat, ionizing radiation, and formaldehyde
treatments, although their infectivity can be reduced.
• Effective prion decontamination relies upon protein hydrolysis or reduction or destruction of
protein tertiary structure. Examples include sodium hypochlorite, sodium hydroxide, and
strongly acidic detergents such as LpH.
• 134 °C (274 °F) for 18 minutes in a pressurized steam autoclave has been found to be
somewhat effective in deactivating the agent of disease.
• Ozone sterilization is currently being studied

19. What are biopsy findings ?


• Damage to neurons and glial cells is characterized by intracellular vacuoles
spongy degeneration [--> Degenerative disease].
The pathognomonic findings:
- Spongiform transformation of the cerebral cortex and, often, deep gray matter structures
(caudate, putamen); this multifocal process results in the uneven formation of small,
apparently empty, microscopic vacuoles of varying sizes within the neuropil and sometimes
in the perikaryon of neurons. Exception is Fatal Familial Insomnia (FFI).
- In advanced cases there is severe neuronal loss, reactive astrocytosis, and sometimes
expansion of the vacuolated areas into cystlike spaces (“status spongiosus”). Inflammation
is notably absent. Electron microscopy shows the vacuoles to be membrane-bound and
located within the cytoplasm of neuronal processes.
- Kuru plaques are extracellular deposits of aggregated abnormal protein; they are Congo
red- and PAS-positive and usually occur in the cerebellum, but are abundant in the cerebral
cortex in cases of variant CJD (vCJD). In all forms of prion disease, immunohistochemical
staining demonstrates the presence of proteinase K-resistant PrPsc in tissue.
20. Describe the presentation of Creutzfeldt-Jakob disease (CJD)?
• Most common spongiform encephalopathy.
• Presentation:
○ Rapidly progressive dementia (in wks-mnts).
○ Ataxia (due to cerebellar damage).
○ Startle myoclonus -(involuntary jerk to minor touch).
○ Periodic sharp waves are seen on EEG.
○ Results in death, usually in < 1 year.

19. What are its types ?


Creutzfeldt–Jakob disease types:
1. sporadic (sCJD): spontaneous mutation --> misfolding of prion-protein (85% of cases).
2. familial (fCJD): AD inherited mutation in the prion-protein gene (15% of cases).
3. iatrogenic (iCJD): caused by contamination with tissue from prion infected human:
- blood transfusion
- human-derived pituitary GH therapy
- corneal graft
- dural grafts /meningeal transplants
4. variant (vCJD): eating bovine spongiform encephalopathy (BSE)-infected beef
aka "Mad Cow Disease". Commonly Seen in younger people.

21. Describe the presentation of familial fatal insomnia?


○ Severe insomnia
○ Exaggerated Startle myoclonus.

19. What are its types ?


Fatal familial insomnia (FFI) type:
1. familial (fFFI): AD inherited prion disease of the brain -most common.
2. sporadic (sFFI): sporadic mutation in the prion-protein gene -rare.
17.8 CNS Tumors

Primary tumors (50%) - locally destructive, but rarely metastasize.


Cell type of origin Adult -(supratentorial) Kid -(infratentorial)
Astrocytes - Glioblastoma Multiforme (M) - most common 1° - Pilocytic astrocytoma (B) - most common 10
malignant CNS tumor in adults -high grade tumor. brain tumor in kids.
Oligodendrocytes - Oligodendroglioma (M)

Ependymal cells - Ependymoma (M)


Neuroectodermal - Medulloblastoma (M).
origin - Cerebellar
granule cell (neuron)
Meningothelial cells - Meningioma (B) - most common 1° benign CNS
- Arachnoid cells tumor in female adults - tumor cells expresses
estrogen receptors.
Schwann cells - Schwannoma (B)
Epithelial remnants - Craniopharyngioma (B) - also seen in young
of Rathke's pouch adults - Exception: Presents as a
- anterior pituitary supratentorial mass.
gland embryonic
tissue
B = benign; M = malignant

Metastatic tumors
Epidem - Represent 50% of brain tumors.
Source of - Most common are breast, lung & kidney (BLacK mnemonic).
metastatis - Others: skin, Gi tract.
Clinical - Characteristically lesion presents as:
features a. multiple location in brain.
b. well-circumscribed lesions.
c. present at the gray-white matter junction.
1. Glioblastoma multiforme (GBM)
Clinical - Usually arises in the cerebral hemisphere;
features characteristically crosses the corpus callosum ('butterfly' lesion).
- Characterized by regions of necrosis surrounded by tumor cells
(pseudopalisading) and endothelial cell proliferation.
Diagnosis - Tumor cells are GFAP positive (stains for glial cells).
Prognosis - Poor prognosis; one of rare brain tumors to metastasize easily.

2. Oligodendroglioma
Clinical - Imaging reveals a calcified tumor in the white matter, usually
features involving the frontal lobe. --> May present with seizures.
- Histology shows 'Fried-egg' appearance of tumor cells -Biopsy.

3. Meningioma
Clinical - Histology:
feature • Whorled pattern of tumor cells.
• Psammoma bodies may be present.
- Imaging reveals a round mass attached to the dura.
tumor compresses, but does not invade the cerebral cortex.
---> May present as seizures.

4. Schwannoma
Clinical - Involves cranial or spinal nerves;
features within the cranium, most frequently involves cranial nerve Vlll at the
cerebellopontine angle (--> presents as loss of hearing and tinnitus).
spinal nerve involvement -median nerve.
Diagnosis - Tumor cells are S-100 positive.
Association - Bilateral tumors are seen in neurofibromatosis type 2.
1. Pilocytic astrocytoma
Clinical - Imaging reveals a cystic lesion with a mural nodule;
features usually arises in the cerebellum.
- Biopsy shows Rosenthal fibers (thick eosinophilic corkscrew processes
of astrocytes) and eosinophilic granular bodies -histology.
Diagnosis - Tumor cells are GFAP positive.

2. Ependymoma
Clinical • Biopsy findings:
features - Ependymal rosettes [true rosettes]: Tumor cells may form gland like round or
elongated structures, which resemble the embryologic ependymal canal → show
long, delicate processes extending into a lumen. Less frequently noticed.
- Perivascular pseudorosettes: Tumor cells are arranged around vessels with an
intervening zone consisting of → thin ependymal processes directed toward the wall
of the vessel. More frequently seen.
[Ependymal cells: Shape ranges from squamous to columnar .
--> Luminal side - microvilli + cilia -propel CSF.
--> Basal side - tentacle-like extensions that attach to astrocytes].
• Morphological features:
- Most commonly arises from the floor of 4th ventricle;
may present with obstructive HYDROCEPHALUS.

3. Medullobastoma
Clinical - Biopsy → histology :
features • Homer-Wright rosettes: - pseudorosettes
- It is characterized by central neuropil (delicate pink material formed by neuronal
processes) surrounded by primitive/undifferentiated/anaplastic tumor cells.
- It may be characterized by Central areas of stromal response, marked by collagen
and reticulin deposition -contain abundant central fibrillary material.
[Neuropil (or "neuropile") is any area in the nervous system (most prevalent in brain)
composed of mostly unmyelinated axons, dendrites and glial cell processes that forms
a synaptically dense region containing a relatively low number of cell bodies - complex
dense feltwork net of interwoven cytoplasmic processes].
• Small, round, blue tumor cells.
Prognosis - Poor prognosis; tumor grows rapidly and spreads via CSF.
1. Metastasis to the cauda equina is termed 'drop metastasis'.

4. Craniopharyngioma -Adamantinomatous subtype.


Clinical - Tumor mass may compress the optic chiasm --> leading to bitemporal hemianopsia.
features Tumor is generally suprasellar in location.
- Calcifications are commonly seen on imaging (derived from "tooth-like" tissue).
[tumor specifically forms nests of odontogenic (tooth-forming) epithelium within the
suprasellar/diencephalic region]. Due to Activating CTNNB1 mutations.

Prognosis - tends to recur after tumor resection.


Virtigo

virtigo - room spinning around or you spinning in the room

CNS vertigo : originates in CNS


- no hearing loss
- vertical nystagmus
- focal neuro deficit
- associated CNS symptom (difficulty walking)

peripheral virtigo: originates in ears


- hearing loss
- no vertical nystagmus
- extremely nauseous (CNS vertigo has less nausea)
- head movement can trigger (wake up midnight because they moved their head)
- BPPV

Ddx for peripheral virtigo:


- ex - Meniere disease (vertigo that lasts 20 minutes)
- labyrinthitis (days
- ototoxicity

Ddx for central virtigo:


- MS
- tumor

age old male with PMH of ..... presents with .... most concerning with...
Chapter 18: Musculoskeletal
Pathology
18.1 Skeletal System

Achondroplasia
Defn. - Impaired cartilage proliferation in growth plate ---> dwarfism.
- Most common cause of dwarfism.
- Mental function, life span, and fertility are not affected.
Etiology - Activating AD mutation in fibroblast growth factor receptor 3 (FGFR3) ---->
Overexpression of FGFR3 inhibits growth [In normal development, FGFR3 has a
negative regulatory effect on bone growth].
- Most mutations are sporadic (80%) and related to increased parental age.
Clinical - PRESENTATION is based on impaired endochondral ossification; intramembranous
features bone formation not affected.
i. Endochondral bone formation is characterized by formation of a cartilage matrix,
which is then replaced by bone; it is the mechanism by which long bones grow.
• Short extremities.
ii. Intramembranous bone formation is characterized by formation of bone
without a preexisting cartilage matrix; it is the mechanism by which flat
bones (e.g., skull and rib cage) develop.
• Normal-sized head and chest.

Osteogenesis imprefecta
Defn - Congenital defect of bone formation ---> structurally weak bone.
Etiology - MOST COMMON - AD Mutation of COL1A1 or COL1A2 genes ---> defective collagen
type I synthesis ---> type I collagen deficiency.
[COL1A1 = Collagen, type I, alpha 1 and COL1A2 = Collagen, type I, alpha 2].
- Types (based on clinical features):
--> Type I-IV : autosomal dominant - COL1A1 or COL1A2 genes
--> Type IV-IX: autosomal recessive - IFITM5 gene
- SERPINF1 gene
- CRTAP gene
- LEPRE1 gene
- PPIB gene
Presen. - Multiple bone fractures (can mimic child abuse, but bruising is absent).
- Blue sclera - Thinning of scleral collagen reveals underlying choroidal veins.
- Hearing loss - Bones of the middle ear easily fracture.
- Dental imperfection - Lack of dentin (blue or brown teeth that easily rots)
Osteopetrosis
Defn. - Inherited defect of bone resorption --> abnormally thick, heavy bone that fractures easily.
- It of 2 types:
---> Autosomal recessive osteopetrosis (ARO) aka Malignant infantile osteopetrosis
---> Autosomal Dominant Osteopetrosis (ADO) aka Adult Osteopetrosis.

Etiology - Due to poor osteoclast function - Osteoclast dysfunction.


- CA2 gene mutation ---> Carbonic anhydrase II deficiency.
- [Carbonic anhydrase is required by osteoclasts for proton production --> acidification of the
osteoclast resorption pit ---> acidic microenvironment is created ---> dissociation of calcium
hydroxyapatite from the bone matrix ---> Bone resorption in progress].
Co2 + h20 + CA -------> HC03- + H+
- Multiple genetic variants exist:
OPTA1 LRP5 receptor gene NB: osteoclasts are derived from monocytes
OPTA2 CLCN7 chloride channel gene
OPTB1 TCIRG1 ATPase gene
OPTB2 RANKL gene
OPTB3 CA2 gene
OPTB4 CLCN7 chloride channel gene
OPTB5 OSTM1 ubiquitin ligase gene
OPTB6 PLEKHM1 adapter protein gene
OPTB7 TNFRSF11A (RANK receptor) gene
- The genes associated with osteopetrosis are involved in the development and/or
function of osteoclasts. Mutations in these genes can lead to abnormal osteoclasts,
or having too few osteoclasts.
Presen. - Defective Bone remodelling ---> Bone fractures
- Myelophthisic process: Bony replacement of the hemopoietic bone-marrow tissue.
---> Pancytopenia [Anemia + leukopenia +thrombocytopenia].
---> displaced hematopoietic cells migrate to liver & spleen --->
extramedullary hematopoiesis --> Hepatosplenomegaly.
- Skull involvement:
i. Impingement on cranial nerves --> Vision and hearing impairment.
ii. Narrowing of the foramen magnum ---> Hydrocephalus.
- Renal tubular acidosis:
i. CA2 gene mutation ---> Carbonic anhydrase II [cytosolic] deficiency ---> No tubular
reabsorption of HC03-/H+ secretion ---> Metabolic acidosis.
ii. It is of 4 types
--> distal RTA: Failure of α intercalated cells to secrete H+ into tubule.
--> proximal RTA: Failure of proximal tubular cells to reabsorb HCO3− in blood.
--> combined proximal and distal RTA
--> absolute hypoaldosteronism or aldosterone insensitivity.
Treat. - Bone marrow transplant
Rickets/Osteomalacia
Defn. - Defective mineralization of osteoid [Osteoblasts normally produce osteoid, which
is then mineralized with Ca2+ and PO4 to form bone] --> ↑ fracture risk.
Etiology - ↓ vitamin D level ----> ↓ serum Ca2+ and PO4.
- Vitamin D deficiency is seen with:
--> ↓ sun exposure (e.g., northern latitudes).
--> poor diet, malabsorption.
--> liver failure and renal failure.
Vit D Active vitamin D raises serum Ca2+ and PO4 by acting on:
function i. Intestine --> ↑ absorption.
ii. Kidney --> ↑ reabsorption.
iii. Bone --> ↑ resorption.
Vit D - Vitamin D is normally derived from:
biochem => skin upon exposure to sunlight (85%)
=> diet (15%) in form of --> Vit D2 (ergocalciferol).
--> Vit D3 (cholecalciferol).
- Activation of Vitamin D requires:
=> 7 dehydrocholesterol + UV rays(sun) ---> Vit D3 (Cholecalciferol)
=> 25-hydroxylation by the hepatocyte ---> 25-OH-vit D3 (Calcidiol).
=> 1-alpha-hydroxylation by the PCT cells ---> 1,25-OH-vit D (Calcitriol)- active form
Rickets PRESENTATION:
(children - Pigeon-breast deformity [Pectus craniatum] - inward bending of the ribs with
<1yr) anterior protrusion of the sternum.
- Frontal bossing (enlarged forehead)- due to osteoid deposition on the skull.
- Rachitic rosary- osteoid deposition at the costochondral junction.
- Bowing of the legs - may be seen in ambulating children.
- Harrison sulcus - strong diaphragm pull the last ribs inward.
Osteo LAB FINDINGS:
malacia • ↓ serum Ca2+ and PO4.
(adults) • ↑ serum PTH
• ↑ serum ALP (alkaline phosphatase) - due to an increase in compensatory
osteoblast activity.
[ALP is a membrane-anchored protein that is a characteristic marker expressed in
large amounts at the apical (secretory) face of active osteoblasts. Function:
RO-PO4 (phosphate monoester) + h2o + ALP ----> R-OH + PO4 (phosphate).
functions best under alkaline pH environments and has the physiological role of
dephosphorylating compounds].
Osteoporosis
Defn. -Reduction in trabecular [Spongy] bone mass (bone density) ---> Results in
porous bone with an increased risk for fracture.
- Most common forms of osteoporosis are senile and postmenopausal.

Risk - Risk of osteoporosis is based on peak bone mass (attained in early adulthood) and
factors rate of bone loss that follows thereafter.
• Peak bone mass is achieved by 30 years of age and is based on
(1) genetics (e.g.,vitamin D receptor variants), (2) diet, and (3) exercise.
• Thereafter, slightly < 1% of bone mass is lost each year; bone mass is lost more
quickly with lack of weight-bearing exercise (e.g., space travel), poor diet, or
decreased estrogen (e.g., menopause).

Clinic. - Bone pain and fractures in weight-bearing areas such as:


features • the vertebrae (--> loss of height & kyphosis)
• hip, and distal radius.
- Reduced bone density on DEXA scan
Lab diag. - Serum Ca2+ = normal
- Serum PO4 = normal
- Serum PTH = normal
- Serum ALP (alkaline phosphatase) = normal
NB: labs help to exclude osteomalacia [which has a similar clinical presentation].
Treatment Therapy:
- Exercise, vit.D, & Ca2+ ---> limits bone loss.
- Bisphosphonates ---> induce apoptosis of osteoclasts.
Contraindication:
- Estrogen replacement therapy is debated (currently not recommended) -
increases risk of breast and endometrium .
- Glucocorticoids are contraindicated (worsen osteoporosis) - speeds up bone loss.

Paget disease of bone


Defn. - Imbalance between osteoclast and osteoblast function.
- Localized process involving one or more bones; does not involve the entire skeleton.
- Stages of Paget disease of bone:
• Osteoclastic
• Mixed osteoblastic-osteoclastic.
• Osteoblastic.
- End result is thick, sclerotic bone that fractures easily -> micro fractures -> Bone pain.
- Biopsy reveals a mosaic pattern of lamellar bone.
Clinical - Skull is commonly affected
features • impingement on cranial nerve - Hearing and vision loss.
• involvement of craniofacial bones - Lion-like facies and increased hat size.

Epid. - Mostly seen in late adulthood (average age> 60 years)


- Most common cause of isolated ↑ alkaline phosphatase [ALP] in patients > 40yr old.

Etiology - Unknown; possibly viral.


Treatment - Bisphosphonate - induce apoptosis of osteoclasts.
- Calcitonin - inhibit osteoclast function.
Complicati - Risk of Osteosarcoma during osteoblast overactivity phase
on - High output cardiac failure - due to formation of AV shunt in bone.
Osteomyelitis
Defn - Infection of marrow and bone.
- Usually occurs in children.
- Most commonly bacterial; arises via hematogenous spread.
Types - Transient bacteremia affects metaphysis --> Seen in kids.
- Open wound bacteremia affects epiphysis --> Seen in adults.
Causes Most common cause (90% cases) - Staph aureus
Sexually active young adult - Neisseria gonorrhoeae
Sickle cell disease - Salmonella
Diabetics and IV drug users - Pseudomonas
Dog or cat bite/scratch association - Pasteurella
Involves vertebrae (Pott disease) - Mycobacterium TB

Present. - Bone pain with systemic signs of infection (e.g., fever and leukocytosis).
- Lytic focus (abscess) surrounded by sclerosis of bone on x-ray;
lytic focus is called sequestrum, and sclerosis is called involucrum.
Diagnosis - Blood culture
Treat - Antibiotics

Avascular/ Aseptic necrosis


Defn - Ischemic necrosis of bone and bone marrow
Causes - Fracture/Trauma & joint dislocation (most common) - ex: AVN of femoral head.
- High dose steroids & alcoholism.
- Sickle cell anemia - vasoocclusive crisis --> Dactylitis.
- Caisson disease (decompression sickness) - gas bubbles form in blood.
Complica. - Osteoarthritis.
- Fracture.
18.2 Bone Tumors
Benign tumors
Osteoma - Benign tumor of bone
- Most commonly arises on the surface of facial bones
- Associated with Gardner syndrome:
--> Osteoma.
--> FAP (familial adenomatous polyposis).
--> Retroperitoneal fibrosis.

Chondroma - Benign tumor of cartilage.


- Usually arises in the
--> medulla of small bones of the hands and feet - enchondroma.
--> seen on bone surface - juxtacortical chondroma

Osteoid - Benign tumor of osteoblasts (that produce osteoid) surrounded by a rim of reactive bone.
osteoma - Occurs in young adults < 25 years of age (more common in males).
- Arises in cortex of long bones (e.g., femur) - diaphysis
- Presents as localized bone pain (most severe at night) that resolves with aspirin
- Imaging reveals a bony mass(< 2 cm) with a radiolucent core (osteoid).

Osteo - Similar to Osteoid osteoma


blastoma - Differences:
• Size >2cm
• Most commonly arises in vertebrae.
• Bone Pain not relieved by asprin

Osteo - Most common benign tumor of bone.


chondroma - Tumor of bone with an overlying cartilage cap.
- Arises from a lateral projection of the growth plate (metaphysis); bone is continuous
with the marrow space.
- Overlying cartilage can transform (rarely) to chondrosarcoma

Giant cell - Tumor comprised of multinucleated giant cells and stromal cells
tumor - Occurs in young adults
- Arises in the epiphysis of long bones - usually the distal femur or proximal tibia
(region of the knee)
- 'Soap-bubble' appearance on x-ray
- Locally aggressive tumor; may recur
Malignant tumors
Osteo Defn - Malignant proliferation of osteoblasts
sarcoma - Peak incidences - most common in teenagers, less commonly seen in the elderly.
Risk - Familial retinoblastoma
factors - Paget disease (during the osteolbastic hyperactivity phase)
- Radiation exposure
Presentati - Occurs mostly in metaphysis of long bones (distal femur or proximal tibia mainly)
on - Imaging shows destructive mass with 'sunburst appearance' due to periosteal reaction
and 'codman triangle' due to lifting of periosteum by adjacent tumor.
- Presents as a pathologic fracture or bone pain with swelling
Biopsy - Pleomorphic cells that produce osteoid
Chondro - Malignant cartilage-forming tumor.
sarcoma - Arises in medulla that can grow out through cortex to make sessile paracortical mass.
- Seen mostly in central skeleton or pelvis

Ewing - Malignant proliferation of poorly-differentiated cells derived from neuroectoderm.


Defn
sarcoma - Arises in the diaphysis of long bones; usually in male children (< 15 years of age).

Imaging - Onion skin appearance on X-ray


- Tumor arises in medullary cavity and comes outside the cortex to make
soft tan-white mass.
Biopsy - Small round blue cells that resemble lymphocytes (can be confused with lymphoma or
chronic osteomyelitis). Presence of homer wright rosettes indicate neural differentiation.

Cause - Often associated with 11;22 translocation [ewing sarcoma gene (EWS gene) on ch22
merges with transcripton factor gene (TF gene) on ch11].
Prognosis - Often metastasizes but responsive to chemotherapy

Metastatic - Most common cause of malignant bone tumors --> even More common than primary tumors.
tumors - Classic site of origin (lead kettle - PB KTL) - Prostate>breast>kidney>thyroid>lung
- Usually causes osteolytic (punched out) lesion except Prostatic carcinoma which classically produces
osteoblastic (sclerotic) lesion
18.3 Joint

BASIC PRINCIPLES
A. Connection between two bones
B. Solid joints are tightly connected to provide structural strength (e.g., cranial sutures).
C. Synovial joints have a joint space to allow for motion.
l. Articular surface of adjoining bones is made of hyaline cartilage (type II
collagen) that is surrounded by a joint capsule.
2. Synovium lining the joint capsule secretes fluid rich in hyaluronic acid to
lubricate the joint and facilitate smooth motion.

DJD RA Spondyloarthropathy
+ve Rheumatic factor Negative rheumatic factor
Affects both DIP and PIP Affects mainly PIP symmetrically Affects mainly DIP (sausage
(heubordin-bouchard nodes) fingers)- for psoriatic only
Pain worsens during day Pain gets better during day
Associated with HLA-DR4 Associated with HLA-B27
Degenerative joint disease / Osteoarthritis
Defn - Progressive degeneration of articular cartilage - most common type of arthritis
- aka Non-inflammatory arthritis - no redness or swelling in joints plus lymphocyte infiltrate are
not seen in the Synovial fluid.
- Most often due to 'wear and tear'.
Risk factors - Major risk factor: Age (common after 60 years).
- Additional risk factors: Obesity and trauma.
Clinical - Affects a limited number of joints (oligoarticular); commonly affects:
features ○ hips, lower lumbar spine, knees.
○ Distal interphalangeal joints (DIP) and Proximal interphalangeal joints (PIP) of fingers.
- Joint stiffness in the morning that worsens during the day.
Pathologic - Disruption of the cartilage that lines the articular surface --> fragments of cartilage floating in
features the joint space are called 'joint mice'.
- Eburnation (bone rub) of the subchondral bone --> Subchondral bony sclerosis.
- Osteophyte formation (reactive bony outgrowths/Bony spur); classically arises in the DIP
[Heberden nodes] and PIP [Bouchard nodes] joints of the fingers --> Enlarged joints.

Rheumatoid arthritis
Defn. - Chronic, systemic autoimmune disease.
- Characterized by involvement of joints.
- Classically arises in women of late childbearing age.
Genetics - Associated with HLA-DR4
Pathogen. ○ Synovitis leading to formation of pannus (inflamed granulation tissue) ----->
Contraction of pannus by myofibroblast ----> which leads to
• Ankylosis (fusion) of the joint - joint distortion.
• Destruction/loss of cartilage.
• Osteopenia (bone mineral density is lower than normal).
• Joint-space narrowing.
Presentati - Arthritis with morning stiffness that improves with activity.
on - Characteristic symmetric involvement of:
=> PIP (proximal interphalangeal) joints of the fingers -> (swan-neck deformity).
=> MCP (metacarpophalangeal) joints of wrists -> (radial deviation).
=> Elbows, ankles, and knees.
- Systemic autoimmune signs --> Fever, malaise, myalgia, wt loss.
- Rheumatoid nodules- central zone of necrosis surrounded by epithelioid
histiocytes; arise in skin and visceral organs.
- Vasculitis - Multiple organs may be involved.
- Baker cyst - swelling of bursa behind the knee
- Pleural effusions
- lymphadenopathy
- interstitial lung fibrosis
Labs find. - Positive rheumatoid factor -> IgM autoantibody against Fc portion of IgG to form i.complexes.
IgM autoantibody is rheumatoid factor - marker of tissue damage and disease activity.
- Presence of neutrophils and high protein in synovial fluid.
Complica. - Anemia of chronic disease
○ Chronic inflammatory state produces hepcidin (acute phase protein). Hepcidin blocks ability
to use iron that's stored in macrophage. Results in anemia
- Secondary amyloidosis
○ Liver makes acute phase protein SAA that gets converted to AA amyloid deposition.
SERONEGATIVE SPONDYLO-arthropathies/arthritis

7. What are the Characteristics of SERONEGATIVE SPONDYLO-arthropathies/arthritis ?


- Lacks of rheumatoid factor (aka seronegative).
- Axial skeleton involvement.
- Associated with HLA-B27.

7. What are the Types of SERONEGATIVE SPONDYLO-arthropathies/arthritis ?


○ Ankylosing spondyloarthritis
 Seen in young adult males
 Presents with:
--> Involves the sacroiliac joints - Sacroiliitis.
--> Low back pain - ankylosis of spine ('bamboo spine'). Involvement of
vertebral bodies leads to fusion of the vertebrae.
--> Extra-articular manifestations include:
=> Uveitis - eye inflammation.
=> Aortitis (weak aorta can lead to aortic regurgitation).
○ Reiter syndrome (reactive arthritis)
 Usually seen in young males weeks after genital infection with C.trachomatis
or gastrointestinal infection with Shigella.
 Presents with:
--> Can't see (conjuctivitis)
--> Can't pee (urethritis - pains with urination)
--> Can't climb tree (arthritis)
○ Psoriatic arthritis
 Affects 10% of psoriatic patients
 Affects axial and peripheral joints
 Affects distal interphalangeal joints of hands and feet (--> leading to
"sausage" fingers or toes).

Infectious arthritis or Septic arthritis


 Arthritis due to an infectious agent, usually bacterial like:
--> N gonorrhoeae - young adults; most common cause.
--> S aureus - older children and adults; 2nd most common cause.
 Classically involves a single joint, usually the knee.
 Presents as (Systemic signs):
- warm joint with limited range of motion
- fever
- ↑ white count
- ↑ ESR.
 Can be easily treated with antibiotics

Musculoskeletal Page 3.3


Gout

8. What causes gout?


- Deposition of MSU (monosodium urate) crystals in joint.
- Crystals arise due to hyperuricemia.
○ Overproduction of Uric acid - derived from purine metabolism.
○ Decreased excretion of Uric acid by kidney - more common.

8. What are some etiologies of Primary gout?


- The most common form.
- Etiology of hyperuricemia is unknown.

9. What are some etiologies of Secondary gout?


Overproduction Decreased excretion
Leukemia and myeloproliferative disorders ---> Renal insufficiency
Increased cell turnover leads to hyperuricemia. Competition for excretion by:
Lesch-Nyhan syndrome- X Iinked deficiency of - Drugs (thiazides, salicylate)
hypoxanthine-guanine phosphoribosyl transferase - EtOH (alcohol consumption)
(HGPRT) ---> presents with mental retardation and - Lactic acid
self mutilation. - Ketosis
- Glycogen storage disease
Psoriasis (increased skin turnover)
Strenous exercise
Excess meat consumption (Meat have lots of DNA/
RNA molecules)

9. Acute Gout vs Chronic gout presentations.


Acute Gout Chronic gout.
- Presents as exquisitely painful monoarticular - Development of tophi-white, chalky aggregates of
arthritis of the great toe (podagra). uric acid crystals with fibrosis and giant cell reaction
- MSU crystals deposit in the joint, triggering an in the soft tissue and joints.
acute inflammatory reaction. - Renal failure: Urate crystals may deposit in kidney
- Alcohol or consumption of meat may precipitate tubules (urate nephropathy).
arthritis.

8. Laboratory findings of Gout?


- Blood test: hyperuricemia.
- Synovial fluid shows needle-shaped crystals with negative birefringence
under polarized light.

13. What is pseudogout?


- Pseudogout resembles gout clinically, but is due to deposition of calcium
pyrophosphate dihydrate (CPPD).
- Knee is to pseudogout (some involvement of wrist) as big toe is to gout.
- Synovial fluid shows rhomboid-shaped crystals with weakly positive
birefringence under polarized light
18.4 Skeletal Muscle

Dermatomyositis Polymyositis
Defn - Inflammatory disorder of the skin and skeletal muscle - Inflammatory disorder of skeletal muscle.
Cause - Unknown etiology; some cases are associated with
carcinoma (e.g., gastric carcinoma).
Presentati - Bilateral proximal muscle weakness; - Resembles dermatomyositis clinically,
on distal muscles can be affected in later stages but skin is not involved.
- Rash on upper eyelids (heliotrope rash);
malar rash may also be seen.
- Red papules on elbows, knuckles and knees
(Gottron papules).
Lab find. - ↑ creatine kinase
- positive ANA (Anti-nuclear Ab) subtype
i.e. anti-Jo-1 Ab.
Biopsy - Perimysial inflammation (CD4+ T cells) with - Endomysial inflammation (CD8+ T cells) with
perifascicular atrophy necrotic muscle fibers
Treat. - Corticosteroids.

X-linked muscular dystrophy


Defn. - Degenerative disorder characterized by muscle wasting and replacement of skeletal muscle by
adipose tissue
Pathoph. - Dystrophin is important for anchoring the muscle cytoskeleton to the underlying basal lamina
(extracellular matrix) through a protein complex containing many subunits.
- The absence of dystrophin permits excess calcium to penetrate the sarcolemma ---> Alterations in
calcium and signalling pathways cause water to enter into the mitochondria, which then burst --->
mitochondrial dysfunction gives rise to an amplification of stress-induced cytosolic calcium signals
and an amplification of stress-induced ROS production ---> Necrosis & cell death ---> replaced with
adipose and connective tissue.
Genetics - Dystrophin gene (largest human gene) mutations ---> abnormal Dystrophin protein.
- Mutations are often spontaneous; large gene size predisposes to high rate of mutation.
- X-linked recessive disorder (Xp21.2 locus).

Types Duchenne muscular dystrophy [DMD] Becker muscular dystrophy [BMD]


- Small out of frame deletion - Large in frame deletion
- Clinically Severe Signs andsymptoms are seen - Clinically milder Signs & symptoms seen
- More common incidence. - Less common incidence.
PRESENTATION: COMPLICATIONS:
- Presents as proximal muscle weakness at 1 year of age; - Cardiac arrythmias
progresses to involve distal muscles. - Mental impairment
• Pseudohypertrophy (enlarging) of tongue & calf muscles. - Pulmonary failure
• Muscle contractures of Achilles tendon and hamstrings. - Pneumonia
• Positive Gowers' sign - Use of Gower's Maneuver. - Death due to cardiac or respiratory
• Lumbar hyperlordosis -> hip-flexor muscles shorten. failure; myocardium commonly
• Scoliosis - skeletal deformity involved.
• Elevated serum creatine kinase (CPK-MM) level
18.5 Neuromuscular junction
Myasthenia gravis Lambert-Eaton syndrome
Defn - Auto-Ab against postsynaptic Ach receptor - Auto-Ab against presynaptic Ca2+ channel at
at NM junction. NM junction.
- Leads to impaired Ach release - Firing of
presynaptic calcium channels is required for
acetylcholine release.
Risk - More commonly seen in women. - Arises as a paraneoplastic syndrome, most
factor - 15% of cases are associated with thymic commonly due to small cell carcinoma of
hyperplasia or thymoma. the lung.
Presentati - Muscle weakness that worsens with use and - Proximal muscle weakness that improves
on improves with rest. with use and worsens with rest (EYES ARE
- Early eye involvement - ptosis and diplopia (MOST USUALLY SPARED).
MUSCULAR DISEASE SPARE EYES).
Treatment - Thymectomy improves symptoms - Resolves with resection of the cancer.
- Use anti-cholinesterase agents to increase Ach in - Anti-cholinesterase agents don't improve
synaptic region. symptoms.

Musculoskeletal Page 4.2


18.6 Soft Tissue Tumor

Lipoma - Benign tumor of adipose tissue


- Most common benign soft tissue tumor in adult

Liposarcoma - Malignant tumor of adipose tissue


- Most common malignant soft tissue tumor in adults
- Lipoblast is characteristic cell - immature fat cell with multiple fat vacuole that create
scalloped appearance of nucleus
Rhabdomyoma - Benign tumor of skeletal muscle
- Cardiac rhabdomyoma is associated with tuberous sclerosis
Rhabdo - Malignant tumor of skeletal muscle.
myosarcoma - Most common malignant soft tissue tumor in children.
- Rhabdomyoblast is the characterstic cell - desmin positive.
- Most common site is head & neck; vagina is classic site in young girls.
Chapter 19: Skin Pathology
INTRODUCTION

1. Describe the histology of epidermis.


Layer Character
Stratum basalis Stem cell layer
Statum spinosum Appears spinous due to desmosomes between keratinocytes
Stratum granulosum See granules in keratinocytes
Stratum corneum See keratin and anucleate cells

I. SKIN
A. Functions as a barrier against environmental insults and fluid Joss
B. Composed of an epidermis and dermis
C. Epidermis is comprised of keratinocytes and has four layers
l. Stratum basalis-regenerative (stem cell) layer
2. Stratum spinosum-characterized by desmosomes between keratinocytes
3. Stratum granulosum-characterized by granules in keratinocytes
4. Stratum corneum- characterized by keratin in anucleate cells
D. Dermis consists of connective tissue, nerve endings, blood and lymphatic vessels,
and adnexal structures (e.g., hair shafts, sweat glands, and sebaceous glands).
19.1 Inflammatory Dermatoses

Atopic (Eczematous) dermatitis Contact dermatitis


- Pruritic, erythematous, oozing rash with vesicles - Pruritic, erythematous, oozing rash with vesicles
and edema. and edema.
- Usually start in face in infancy; - Type IV hypersensitivity.
then seen mostly on flexor surface afterwards. - Arises upon exposure to:
- Type I hypersensitivity reaction ○ Chemical irritant: Detergents, surfactants.
- Associated with --> asthma ○ Allergens: Poison ivy and nickel jewelry.
--> allergic rhinitis. : Drugs --> (Penicillin,Neomycin).
- TREATMENT: - removal of the offending agent.
- topical glucocorticoid (if needed).

Acne vulgaris
Presentation - Comedones (whiteheads and blackheads).
- Pustules (pimples) and Nodules.
Epidemology - Extremely common, especially in adolescents.
Cause - Chronic inflammation of hair follicle and associated sebaceous gland
Pathophy. - Hormone [androgen]- associated hair follicle blockage:
↑ sebum production by sebaceous glands.
↑ keratin production.
----> comedones.
- Propionibacterium acnes infection produces lipases that break down sebum,
releasing proinflammatory fatty acids.
----> pustule or nodule formation.

Treatment - Benzoyl peroxide (antimicrobial) for P.acnes.


- Vit A derivatives (e.g.,isotretinoin) - reduces keratin production.
Psoriasis
Presentation - Well-circumscribed, salmon-colored plaques with silvery scale;;
usually on extensor surfaces and the scalp.
- Pitting of nails may also be present.
Pathophy. - Excess keratinocyte proliferation
Etiology - Autoimmune: associated with HLA-C.
- Environmental trigger: Lesions often arise in areas of trauma.
Histology - Acanthosis -[epidermal hyperplasia].
- Parakeratosis -[hyperkeratosis with retention of keratinocyte nuclei in the
stratum corneum].
- Munro microabscess -[Collections of neutrophils in the stratum corneum].
- Auspitz sign -[Thinning of the epidermis above elongated dermal papillae;
results in bleeding when scale is picked off].
Treatment - Corticosteroids
- UV-A light with psoralen
- Immune modulating therapy

Lichen planus
Presentation - Pruritic, planar, polygonal, purple papules ---> Cutaneous lichen planus;
involves wrists, elbow, ankles.
- Reticular white lines on oral mucosa surface ---> Mucosal lichen planus;
[Wickham striae].
Histology - Inflammation of dermal-epidermal junction with a 'saw tooth' appearance.
Etiology - Unknown
- Associated with chronic Hep C virus infection .
19.2 Blistering Dermatoses

Pemphigus vulgaris Bullous pemphigoid


• IgG mediated attack to desmosomes • IgG mediated attack to hemidesmosome
• Cell separate somewhere in stratum • Cells separate in dermal-epidermal junction -
spinosum - vesicles rupture easily vesicles don't rupture easily
• Fish net immunofluorecence • Linear immunofluorescence
• Histology - tombsonte cells, acantholysis • Histology -
(suprabasal vesicle), Nikolky sign
• Oral mucosa involved Oral mucosa spared

Pemphigus vulgaris
Etiology - Type II hypersensitivity reaction ---> IgG antibody against desmoglein 1&3 --->
Autoimmune destruction of desmosomes b/w keratinocytes .
Presentation - Skin and oral mucosa bullae
Acantholysis Seperation of stratum spinosum keratinocytes (normally
connected by desmosomes) --> results in suprabasal bullae.
Tombstone Basal layer cells remain attached to basement membrane
appearance via hemidesmosomes.
Nikolsky's Thin-walled bullae (filled with Transudative fluid + cytolytic
sign fluid) rupture easily making shallow erosions with dried crust.

Immuno - Highlights IgG surrounding keratinocytes in a 'fish net' pattern.


fluorecence

Bullous pemphigoid
Etiology - Type II hypersensitivity ---> IgG antibody against Dystonin (BPAG1) and/or
Collagen, type XVII, alpha 1 (aka BP180) ---> Autoimmune destruction of
TYPE 1 hemidesmosome b/w basal cells and the underlying basement
membrane.
Presentation - Blisters of the skin, oral mucosa is spared.
1. Basal cell layer is detached from the basement membrane.
2. Tense bullae do not rupture easily; clinically milder than pemphigus
vulgaris.
3. Usually in the elderly people.
Immuno - Highlights IgG along basement membrane (linear pattern).
fluorescence
Dermatitis herpetiformis
Etiology - Autoimmune deposition of IgA at the tips of
dermal papillae.
Presentation - Pruritic papules, vesicles and bullae that
are grouped (herpetiform).
Association Strong association with celiac disease
Treatment Gluten free diet
Erythema Presentation - Considered to be a type IV hypersensitivity reaction - deposition of IgM
multiforme (EM) bound immune complexes in the superficial microvasculature of the
skin and oral mucosa/lip involvement
- HISTOLOGY:
a) superficial perivascular lymphocytic infiltrate --> dermal Edema.
b) dermoepidermal junction infiltrate ---> Epidermal necrosis.
c) With time, there is upward migration of T cells into epidermis.
- Characterized by keratinocyte injury mediated by skin-homing:
--> CD8+ Tcells - central portion of lesion.
--> CD4+ Tcells & Langerhans cells - peripheral portion of lesion.
- Begins peripherally (toes & fingers) --> spreads centrally (trunk & Face).
Features ----> Target lesion/Targetoid lesion - due to central epidermal necrosis
(epidermal detachment <<10%TBSA) surrounded by erythema.
----> EM lesions are multiform (polymorphous) and include:
i.macules ii.papules iii.vesicles iv.bullae
----> Target lesion consists of three zones:
=> Dark centre of small Erythematous papule, vesicle, or bulla.
=> Pale(pink) intermediate raised zone - due to Edema.
=> Peripheral rim of erythema (due to micro-haemorrhage - RBC
extravasation and possibly hyperaemia) -macule.
Types • Erythema multiforme minor - Cutaneous without mucous involvement
• Erythema multiforme major - Cutaneous with mucous involvement
Cause - Herpes simplex virus (MOST COMMON).
- Mycoplasma.
- Drugs (penicillin and sulfonamides).
- Autoimmune disease (SLE).
- Malignancy.
COMMON FEATURES:
• Begins centrally (trunk & face) ---> spreads peripherally (hand & foot).
• Flat atypical targets (only 2 zones) - irregular lesion tend to coalesce:
=> Red dark central macule.
=> Outer Pale/pinkish area.
• Cutaneous + mucous involvement. Also involves the Eye -> Conjunctivitis.
Genital and anal region involved too.
• EARLY SYMPTOMS include fever and flu-like symptoms.
• DIAGNOSIS:
Positive Nicolsky's sign: dislodgement of intact superficial epidermis from
the papillary dermis by a shearing force;,indicating a plane of cleavage in
the skin at the dermal-epidermal junction.
Positive Asboe-Hansen sign - lateral extension of bullae with pressure
• COMPLICATION include dehydration, sepsis, pneumonia& MO failure.

Presentation • Usual onset Age <30yrs.


• Widespread Lesion: ---> erythematous or purpuric macules.
• Confluent Epidermal necrosis --> Skin loss [epidermal detachment
Stevens-Johnson
<10%TBSA]; with minimal associated inflammation.
syndrome (SJS)
Cause - Drugs - Allopurinol, Amoxicillin, Barbiturates, Diclofenac.
- Infections - Mycoplasma, EBV, CMV.
- Genetics - strongly associated with HLA-B*1502 (HLA-B75).

Toxic epidermal Presentation • Usual onset Age >40yrs.


necrolysis (TEN) • Widespread: ---> purpuric macules.
• Diffuse full thickness epidermal necrosis and sloughing of skin [epidermal
detachment >30%TBSA]; separation occurs at the dermo-epidermal
junction (subepidermal separation) resembling a large burn.

Cause - Most commonly due to adverse drug reaction


Drugs - antibiotics,NSAIDs, corticosteroids.
- HIV infection, Lupus.
- HLA-types such as, HLA-B*1502, HLA-A*3101,[13] and
HLA-B*5801.
19.3 Epithelial Tumors

Seborrheic keratosis
Defn. - Benign squamous proliferation;
common tumor in the elderly.
Presentation - Presents as:
--> raised, discolored plaques
--> Seen on the extremities or face
--> often has a coinlike, waxy, 'stuck-on' appearance.
Leser-Trelat sign:
- sudden onset of multiple seborrheic keratoses
- suggests underlying carcinoma of the GI tract
Histology - Characterized by keratin pseudocysts
Keratin filled cysts (horn cysts).

Acanthosis nigricans
Presentation - Epidermal hyperplasia w/ symmetric darkening of skin ('velvet-like' skin).
- Often presents in axilla, neck, groin.
Association - Type I – familial - autosomal dominant trait.
- Type II – endocrine
- Diabetes mellitus type 2 ---> Hyperinsulinemia.
- Addison's disease & Cushing's disease.
- hypothyroidism,acromegaly.
- polycystic ovary disease.
- Type III – obesity and pseudoacanthosis nigricans
- Type IV – drug-related - nicotinic acid, glucocorticoid, combined OCP.
- Type V – malignancy (occurring as a paraneoplastic syndrome)
- especially gastrointestinal adenocarcinomas.
- as well as genitourinary cancers.
Basal cell carcinoma Squamous cell skin cancer
Defn. - Malignant proliferation of the basal cells - Malignant proliferation of supra-basal cell
of the epidermis. of epidermis [squamous cells].
- UVB-induced DNA damage. - UVB-induced DNA damage.
Epidem - Most common cutaneous malignancy; - 2nd most common skin cancer.
ology locally invasive but rarely metastasize - metastasis is uncommon.
- 75% of non-melanoma skin cancer - The usual age at diagnosis is around 66 yrs.
- Seen in people w/ 50-70 yrs of age.
Risk - prolonged exposure to sunlight - prolonged exposure to sunlight
factors - Albinism - Albinism
- Xeroderma pigmentosum [AR disorder: - Xeroderma pigmentosum.
nucleotide excision repair (NER) enzymes - immunosuppressive therapy
are mutated]. - arsenic exposure
- chronic inflammation (e.g., scar from
burn or draining sinus tract).

Progres 1. Actinic keratosis


sion --> precursor lesion of SCC and presents as a
hyperkeratotic, scaly plaque.
--> often on the face, back, or neck.
--> 0.07% of AK progress to SCC per year.
2. Keratoacanthoma
--> well-differentiated SCC that develops rapidly
and regresses spontaneously
--> presents as a cup-shaped tumor filled with
keratin debris

Present I. Classic location is the upper lip. I. Usually on the face (classically involving the
ation - Elevated nodule with a central ulcerated lower lip).
crater surrounded by dilated (telangiectatic) - Ulcerated, Crusty, nodular mass.
vessels ---> 'pink, pearl-like papule'
Histo - Nodules of basal cell with peripheral - Keratin pearls
logy palisading.

Treatm - Excision by surgery. - Surgical Excision.


ent - Chemotherapy & Immunotherapy - Electrodessication and curettage or EDC.
19.4 Disorders of Pigmentation and Melanocytes

Melanocytes basic
Melanocyte location Basal layer
Embryogenic development From neural crest cells
Melanin production - Make melanin from tyrosine in melanosomes
- Pass melanosomes to keratinocytes

I. BASIC PRINCIPLES
A. Melanocytes are responsible for skin pigmentation and are present in the basal layer
of the epidermis.
I. Derived from the neural crest
2. Synthesize melanin in melanosomes using tyrosine as a precursor molecule
3. Pass melanosomes to keratinocytes
Vitiligo Albinism Freckle (Ephelis) Melasma
Defn. - Localized loss of skin - Congenital lack of - Small, tan to brown - Mask-like
pigmentation pigmentation. macule; darkens when hyperpigmentation
exposed to sunlight. of the skin.
Etiology - Autoimmune destruction - Enzyme defect - Due to ↑ number of - Stimulation of
of melanocytes. (tyrosinase) that impairs melanosomes; melanocytes by E & P
melanin production. No ↑ in number of to produce more
melanocytes. melanin pigments.
Location - Typically both sides of the - May involve the: - Predominantly found on - Mostly seen on
body are affected. -> eyes (ocular form) or the face. cheeks, nose and
- More noticeable in -> both the eyes and skin - Most easily noticed in fair forehead.
people with dark skin (oculocutaneous form) complexion people.

Association - Hashimoto's thyroiditis - Complications: - Associated with red - Pregnancy women on


- Rheumatoid arthritis SC skin carcinoma. haired people (rare alleles oral contraceptives or
- Type 1 DM. Basal cell carcinoma of the MC1R gene) HRT medication.
Melanoma.

Melanocytic nevus
Definition - Benign neoplasm of melanocytes
Types - Congenital nevus - present at birth.
---> often associated with localized Hypertrichosis.
- Acquired nevus - arises later in life.
---> Types:
=> Junctional nevus - most common mole in children.
--> Begins as nests of melanocytes at the dermal-epidermal junction.
=> Compound nevus
--> Grows by extension into the dermis.
--> Mixture of junctional and intradermal proliferation.
=> Intradermal nevus - most common mole in adults.
--> Junctional component is eventually lost; located in the dermis only.
- Dysplastic nevus - precursor to melanoma.
Presentation - Small diameter (< 6 mm).
- Flat macule or raised papule.
- Symmetric with sharp borders.
- Evenly distributed color.
Prognosis - Hormonal changes during pregnancy and diabetics are often contributing to
mole formation - good Prognosis.
Melanoma
Defn. - Malignant neoplasm of melanocytes Genetics • BRAF-V600E kinase gene mutation
- ETIOLOGY: UVB-induced DNA damage. ----> ↑ MAP kinase pathway ----> ↑
- Most common cause of death from skin cancer. mitosis ----> cellular proliferation.
- De novo melanoma is more common form of Melanoma. (40% of human melanomas).
Risk factors - Prolonged exposure to sunlight Treat. • Vemurafenib [BRAF kinase inhibitor].
- Albinism for BRAF (+)ve tumors.
- Xeroderma pigmentosum • Surgical excision
- Dysplastic nevus syndrome [AD hereditary disorder].
--> Clinical Features:
1. ≥ 2 dysplastic nevi.
2. > 100 melanocytic nevi - pts b/w 20-50 yrs of age.
3. > 50 melanocytic nevi - pts <20 yrs age or >50 yrs age.
4. > 1 melanocytic nevi - in buttocks or instep.
5. melanocytic nevi - on the anterior scalp.
6. ≥ 1 pigmented lesions in the iris.
Diagnosis S-100 protein marker.
Presentation - Mole-like growth with "ABCDE" pattern.
• A = Asymmetrical skin lesion.
• B = Border of the lesion is irregular - uneven, ragged, or notched.
• C = Colour is non-uniform - different shades of brown, black, or tan.
• D = Diameter > 6mm [with/without ulceration].
• E = Enlarging or Evolving over time.
- Nodular melanomas have their own "EFG" pattern.
• E = Elevated - lesion is raised above the surrounding skin.
• F = Firm - nodule is solid to the touch.
• G = Growing- nodule is increasing in size.

Growth types Characterized by two growth phases:


1. Radial growth:
--> grow horizontally along the epidermis and superficial dermis.
--> low risk of metastasis.
2. Vertical growth
--> grow into the deep dermis.
--> increased risk of metastasis (regional/distant).
Staging Clark's level [describes the level of anatomical invasion of the melanoma Breslow's depth [depth of extension is
in the skin. Higher levels have worsening prognostic implications]. the most important prognostic factor
- Level 1 : Melanoma confined to the epidermis (melanoma in situ) in predicting metastasis].
- Level 2 : Invasion into the papillary dermis - Stage I: less or equal to 0.75mm
- Level 3 : Invasion to the junction of the papillary and reticular dermis - Stage II: 0.76 mm - 1.50mm
- Level 4 : Invasion into the reticular dermis - Stage III: 1.51 mm - 2.25mm
- Level 5 : Invasion into the subcutaneous fat. - Stage IV: 2.26 mm - 3.0mm
- Stage V: greater than 3.0 mm
Classification 1. Superficial spreading melanoma Other Classification:
---> most common subtype. - Mucosal melanoma
---> dominant early radial growth results in GOOD prognosis. - Polypoid melanoma
2. Lentigo maligna (melanoma in situ) ---> - Desmoplastic melanoma
Lentigo maligna melanoma (radial growth): GOOD prognosis. - Melanoma with small nevus-like cells
3. Nodular melanoma - early vertical growth : POOR prognosis - Melanoma with features of a Spitz nevus
4. Acral lentiginous melanoma - Uveal melanoma
--> arises on the palms or soles.
--> often seen in dark-skinned people.
--> not related to UV light exposure.
19.5 Infectious Disorders

Impetigo Cellulitis
Clinical - Erythematous macules ---> pustules ---> - Red, tender, swollen painful rash.
feature rupture of pustules ---> moist erosions - Fever & exhaustion.
---> dry, honey crusted skin.
Cause - Superficial bacterial skin infection most - Deeper (dermal and subcutaneous)
often due to: infection usually due to:
--> Staph aureus --> Staph aureus
--> Strep pyogenes --> Strep pyogenes
Epidem - Commonly affects children. - It Affects at any age.
ology - Usually on the face. - Can be seen anywhere on body.

Risk - Poor nutrition. - Recent surgery, trauma, or insect bite.


factors - diabetes mellitus.
- contact sports.
Complica - Poststreptococcal glomerulonephritis - Necrotizing fasciitis
tion - necrosis of subcutaneous tissues.
- infection with anaerobic 'flesh-eating'
bacteria like: --> Staph aureus
--> Strep pyogenes
- C02 production ---> Crepitus.
- Treatment: surgical debridement.

Staphylococcal scalded skin syndrome


Presentation - Sloughing of skin with erythematous rash
- leads to significant skin loss.
- Fever (systemic sign).
Pathophysiology - Due to Staph aureus infection.
- Exfoliative A and B (exo)toxin ---> These proteases cleave desmoglein-1
(component of desmosome) ---> epidermolysis of stratum granulosum
(and partly stratum lucidum) ---> intraepidermal clefts between the
stratum corneum and stratum spinosum [intraepidermal separation].
Differential diagnosis - Toxic epidermal necrolysis (TEN).
- Psoriasis.
Verruca (Plantar wart)
Clinical feat. - Flesh-colored papules with a rough surface.
Location - soles of feet and toes are common locations.
Histology - keratinocytes undergo koilocytic change.
Cause - Human papillomavirus (HPV) infection.
Mainly by HPV type 1, 2, 4, 63.

Molluscum contagiosum (MC)


Clinical feat. - Firm, pink, umbilicated papules.
Histology - affected keratinocytes show cytoplasmic inclusions
(molluscum bodies).
Cause - Viral infection.
Family: Poxviridae
Species: Molluscum contagiosum virus
Classic - Children.
patients - Sexually active adults.
- immunocompromised individuals.

Skin pathology Page 5.2


Ophthalmo-LOGY
Oto-rhino-laryngo-LOGY

Vous aimerez peut-être aussi